You are on page 1of 333

YASHWANTRAO CHAVAN MAHARASHTRA OPEN UNIVERSITY

SCHOOL OF SCIENCES
(FORMERLY SCHOOL OF ARCHITECTURE, SCIENCE & TECHNOLOGY)

V154: M.Sc. Chemistry


2023 {As per NEP 2020}

Pattern

CHE509
INORGANIC CHEMISTRY - II

(4 Credits)

Semester - II

Email: director.ast@ycmou.ac.in
Website: www.ycmou.ac.in
Phone: +91-253-2231473
Yashwantrao Chavan CHE509
Maharashtra Open University Inorganic Chemistry-II

Brief Contents
Vice Chancellor’s Message ........................................................................................ 3

Foreword By The Director ........................................................................................ 4

Credit 01 ........................................................................................................................ 5
Credit 01 -Unit 01: Coordination Chemistry .................................................... 6
Credit 01-Unit 02: Bonding ................................................................................. 28
Credit 01-Unit 03: Molecular orbital theory .................................................. 57
Credit 01-Unit 04: octahedral & square planar symmetries ...................... 72

Credit 02 .................................................................................................................... 101


Credit 2 Unit 01: Electronic spectra ............................................................... 102
Credit 02 -Unit 02: Orgel diagram and Tanabe – Sugano energy level
diagrams ................................................................................................................ 121
Credit 02 -Unit 03: Metal carbonyls ............................................................... 146
Credit 02 -Unit 04: Nitrosyl complexes and dinitrogen complexes ....... 169

Credit 03 .................................................................................................................... 183


Credit 03 -Unit 01: Carbocyclic pi complexes and Reaction of
coordination compounds................................................................................... 184
Credit 03 -Unit 02: The Structure and Reactivity of molecules .............. 205
Credit 03 -Unit 03: Applications of VSEPR theory ..................................... 218
Credit 03 -Unit 04: Applications of MOT ...................................................... 236

Credit 04.................................................................................................................... 254


Credit 04 -Unit 01: Symmetry elements and symmetry operations ....... 255
Credit 04 -Unit 02: Principalaxisandsubsidiaryaxes ................................. 272
Credit 04 -Unit 03: Ionic compounds............................................................. 291
Credit 04 -Unit 04: Lattice energy of ionic solids ....................................... 307

Feedback Sheet for the Student .......................................................................... 329

CHE509: Inorganic Chemistry – II Page 1


CHE509: Inorganic Chemistry-II
Yashwantrao Chavan Maharashtra Open University
Vice-Chancellor: Prof. Dr. P. G. Patil
School of Architecture, Science and Technology
Director of the School: Dr. Sunanda More
Programme Advisory Committee
Dr. Sunanda More Dr. Chetana Kamlaskar
Director, Associate Professor,
School of Architecture, Science & Technology, School of Architecture, Science & Technology,
YCMOU, Nashik-422 222 YCMOU, Nashik-422 222

Prof. Dr. S. D. Delekar Prof. Dr. Nandkishor N. Prof. Dr. S. D. Delekar


Professor, Department of Karade, Department of Professor, Department of
Chemistry, Shivaji University Chemistry, Rashtrasant Chemistry, Shivaji University
Kolhapur-416 004 Tukadoji Maharaj Nagpur Kolhapur-416 004
University,
Nagpur-440 033

Dr. Tukaram. S. Thopate Prof. Shamrao Golekar Dr. Tukaram. S. Thopate


Vice Principal and Head Deptt Associate Professor and Head Vice Principal and Head Deptt
of Chemistry, New Arts, Deptt of Chemistry of Chemistry, New Arts,
Commerce Science College, Jamkhed College Jamkhed, Commerce Science College,
Parner, Taluka Parner, Dist: Dist: Ahmednagar-413 201 Parner, Taluka Parner, Dist:
Ahmednagar-414 302 Ahmednagar-414 302
Dr. Amol Kategaonkar Dr. Bharat More
Associate Prof. in Chemistry, Academic Coordinator, School of
KSKW College, Cidco, Nashik-422 008 Architecture, Science and Technology, YCMOU

Development Team
Instructional Course Coordinators Book Writer Book Editor
Technology
Editor

Dr Sunanda 1.Dr. Bharat More Dr. Pankaj D. Baviskar Dr. Deepak V. Nighot
More 2.Ghanshyam Patil Assistant Professor, Associate Professor &
Director, Academic First Year Engineering HOD,
School of Coordinator, Department, First Year Engineering
Architecture, School of All India Shri Shivaji Department,
Science & Architecture, Science Memorial Society's All India Shri Shivaji
Technology, & Technology, College of Engineering Memorial Society's
YCMOU, YCMOU, (A+ NAAC Accredited), College of Engineering
Nashik-422 222 Nashik-422 222 Kennedy Road, (A+ NAAC Accredited),
Pune-411 001. Kennedy Road, Pune

This work by YCMOU is licensed under a Creative Commons Attribution-


NonCommercial-ShareAlike 4.0 International License.
 Book Publication : Date : 10 th Feb 2023 Publication No: 2615
 Publisher : Mr. B. P. Patil, Registrar(I/C), YCMOU, Nashik- 422 222, MS
 ISBN: 978-93-95855-72-3
 This SLM V142: M.Sc. Chemistry {2022 Pattern}, dtd. 10/02/2023
Book used in V154: M.Sc. Chemistry {2023 Pattern}, dtd. 31/08/2023

CHE509: Inorganic Chemistry – II Page 2


V ICE C HANCELLOR ’ S M ESSAGE

Dear Students,
Greetings!!!

I offer cordial welcome to all of you for the Master’s degree programme of Yashwantrao
Chavan Maharashtra Open University.

As a postgraduate student, you must have autonomy to learn, have information and
knowledge regarding different dimensions in the field of Chemistry and at the same time intellectual
development is necessary for application of knowledge wisely. The process of learning includes
appropriate thinking, understanding important points, describing these points on the basis of
experience and observation, explaining them to others by speaking or writing about them. The
science of Education today accepts the principle that it is possible to achieve excellence and
knowledge in this regard.

The syllabus of this course has been structured in this book in such a way, to give you
autonomy to study easily without stirring from home. During the counseling sessions, scheduled at
your respective study centre, all your doubts will be clarified about the course and you will get
guidance from some qualified and experienced counsellors/ professors. This guidance will not
only be based on lectures, but it will also include various techniques such as question-answers,
doubt clarification. We expect your active participation in the contact sessions at the study
centre. Our emphasis is on ‘self study’. If a student learns how to study, he will become
independent in learning throughout life. This course book has been written with the objective of
helping in self-study and giving you autonomy to learn at your convenience.

During this academic year, you have to give assignments, complete laboratory activities, field
visits and the Project work wherever required. You have to opt for specialization as per programme
structure. You will get experience and joy in personally doing above activities. This will enable
you to assess your own progress and thereby achieve a larger educational objective.

We wish that you will enjoy the courses of Yashwantrao Chavan Maharashtra Open
University, emerge successful and very soon become a knowledgeable and honorable
Master’s degree holder of this university.

I congratulate “Development Team” for the development of this excellent high quality
“Self- Learning Material (SLM)” for the students. I hope and believe that this SLM will be
immensely useful for all students of this program.

Best Wishes!
- Prof. Dr. P. G. Patil
Vice-Chancellor, YCMOU

CHE509: Inorganic Chemistry – II Page 3


F OREWORD B Y T HE D IRECTOR

Dear Students,
Greetings!!!
This book aims at acquainting the students with conceptual and applied
fundamentals about Chemistry required at degree level.
The book has been specially designed for Science students. It has a comprehensive
coverage of concepts and its application of Chemistry in practical life. The book
contains numerous examples to build understanding and skills.
The book is written with self- instructional format. Each chapter is prepared with
articulated structure to make the contents not only easy to understand b ut also
interesting to learn.
Each chapter begins with learning objectives, which are stated using Action Verbs
as per the Bloom’s Taxonomy. Each Unit is started with introduction to arouse or
stimulate curiosity of learner about the content/ topic. Thereafter the unit contains
explanation of concepts supported by tables, figures, exhibits and solved
illustrations wherever necessary for better effectiveness and understanding.
This book is written in simple language, using spoken style and short sentences.
Topics of each unit of the book presents from simple to complex in logical sequence.
This book is appropriate for low achiever students with lower intellectual capacity
and covers the syllabus of the course.
Exercises given in the chapter include conceptual questions and practical questions
so as to create a ladder in the minds of students to grasp each and every aspect of a
particular concept.
I thank the students who have been a constant motivation for us. I am grateful to the
writers, editors and the School faculty associated in this SLM development of the
Programme.
Best Wishes to all of you!!!

Dr. Sunanda More


Director,
School of Arch., Science &Technology,
YCMOU

CHE509: Inorganic Chemistry – II Page 4


C REDIT 01

CHE509: Inorganic Chemistry – II Page 5


CREDIT 01 -UNIT 01: C OORDINATION CHEMISTRY

LEARNING OBJECTIVES
After successful completion of this unit, you will be able to
 Understand the nomenclature of inorganic coordination compounds
 Apply the concept of isomerism in coordination compounds
 Understand the concept of methods of preparation
 Understand the concept of ligands and types of legands
INTRODUCTION

Alfred Werner (1866 –1919)


A coordination complex consists of a central atom or ion, which is usually
metallic and is considered as coordination centre, and a surrounding array of bound
molecules or ions that are known as ligands or complexing agents. Many metal -
containing compounds, especially those that include transition metals (Mostly
elements from d block) are coordination complexes. Coordination complexes have
been known since the beginning of modern chemistry. It was not until 1893 that the
most widely accepted Alfred Werner published version of the theory today. Werner's
work included two important changes to the Blomstrand theory. The first was that
Werner described the two possibilities in terms of location in the coordination
sphere. He claimed that if the ions were to form a chain, this would occur outside of

CHE509: Inorganic Chemistry – II Page 6


the coordination sphere while the ions that bound directly to the metal would do so
within the coordination sphere. In one of his most importan t discoveries, however
Werner disproved the majority of the chain theory. Werner discovered the spatial
arrangements of the ligands that were involved in the formation of the complex
hexacoordinate cobalt. His theory allows one to understand the difference between a
coordinated ligand and a charge-balancing ion in a compound, for example the
chloride ion in the cobalt ammine chlorides and to explain many of the previously
inexplicable isomers.

01-01: NOMENCLATURE OF COORDINATION COMPLEXES


The naming system of coordination complexes are depends on type of metal
and ligands, and charges on metal. According to the Lewis base theory, ligands are
Lewis bases since they can donate electrons to the central metal atom. The metals, in
turn, are Lewis acids since they accept electrons. Coordination complexes consist of
a ligand and a metal center cation. The overall charge can be positive, negative, or
neutral. Coordination compounds are complex or contain complex ions, for example
Complex Cation: [Co (NH3) 6 ] 3+
Complex Anion: [CoCl 4 (NH 3 ) 2 ] −
Neutral Complex: [CoCl 3 (NH 3 ) 3 ]
Coordination Compound: K 4 [Fe (CN) 6 ]
A ligand can be an anion or a neutral molecule that donates an electron pair to
the complex for e.g NH 3 , H 2 O, Cl - . The number of ligands that bonded to a metal
depends on whether the ligand is monodentate or polydentate. To begin naming
coordination complexes, here are some things should be considered
 Ligands are named first in alphabetical order.
 The name of the metal comes next.
 The oxidation state of the metal follows, noted by a Roman numeral in
parentheses (II, IV).

Rule 1: Anionic Ligands

Ligands that act as anions which end in "-ide" are replaced with an ending "-
o" (e.g., Chloride replaced with Chloro). Anions ending with "-ite" and "-ate" are
replaced with endings "-ito" and "-ato" respectively (e.g., Nitrite replaced with
Nitrito, Nitrate replaced with Nitrato).

CHE509: Inorganic Chemistry – II Page 7


Molecular Ligand Name Molecular Ligand Name
Formula Formula
F- Fluoro OH - Hydroxo
Cl- Chloro SO 4 2- Sulfato
Br- Bromo S 2 O 3 2- Thiosulfato
I- Iodo NO 2 - Nitrito-N - ; Nitro
O 2- Oxo ONO - Nitrito-O - ; Nitrito
CN - Cyano SCN - Thiocyanato-S - ;
Thiocyanato NC -
NC - Isocyano NCS - Thiocyanato-N - ;
Isothiocyanato
Rule 2: Neutral Ligands
Most neutral ligands (without charge) that are ligands carry their normal
name. The few exceptions are the first four on the chart: ammine, aqua, carbonyl,
and nitrosyl.
Molecular Ligand Name Molecular Ligand Name
Formula Formula
NH 3 Ammine NO Nitrosyl
H2O Aqua CH 3 NH 2 Methylamine
CO Carbonyl C5H5 N Pyridine
Rule 3: Ligand Multiplicity
The number of ligands present in the complex can represent with the prefixes
di, tri, etc. The exceptions are polydentates that have a prefix (di, tri, tetra etc)
already in their name (e.g. EDTA). When indicating how many of these are present
in a coordination complex, put the ligand's name in parentheses and use bis (for two
ligands), tris (for three ligands), and tetrakis (for four ligands).

Number of Ligands Monodentate Ligands Polydentate Ligands


1 Mono -

2 Di bis

3 Tri tris

4 Tetra tetrakis

5 Penta pentakis

6 Hexa hexakis

Prefixes always go before the ligand name; they are not taken into account
when putting ligands in alphabetical order. Note that "mono" often is not used. For

CHE509: Inorganic Chemistry – II Page 8


example, [FeCl(CO) 2 (NH 3 ) 3 ] 2+ would be called triamminedicarbonylchloroiron(III)
ion. Remember that ligands are always named first, before the metal is.

Rule 4: The Metals

When naming the metal center, you must know the formal metal name and the
oxidation state. To show the oxidation state, we use Roman numerals inside
parenthesis. For example, in the problems above, chromium and cobalt have the
oxidation state of +3, so that is why they have (III) after them. Copper, with an
oxidation state of +2, is denoted as copper (II). If the overall coordination complex
is an anion, the ending "-ate" is attached to the metal center. Some metals also
change to their Latin names in this situation. Copper +2 will change into cuprate(II).
The following change to their Latin names when part of an anion complex.

Transition Metal Latin


Iron Ferrate
Copper Cuprate
Tin Stannate
Silver Argentate
Lead Plumbate
Gold Aurate

The rest of the metals simply have -ate added to the end (cobaltate, nickelate,
zincate, osmate, cadmate, platinate, mercurate, etc. Note that the -ate tends to
replace -um or -ium, if present). Finally, when a complex has an overall charge,
"ion" is written after it. This is not necessary if it is neutral or part of a coordination
compound. Here are some examples with determining oxidation states, naming a
metal in an anion complex, and naming coordination compounds.

Rule 5: Oxidation state or charge of metal


Oxidation state or charge of metal should be writing in front of metal in roman
number and oxidation state or charge on metal can be calculate with the help of
number of electrons lost and number of electrons gain by metal atom. For e.g.
1) [Mn(CO) 5 ] CO (Carbonyl) is a neutral ligand and complex is also neutral.
We need to find oxidation state of Mn so let it be x then x +5(0)=0 ie, x=0

CHE509: Inorganic Chemistry – II Page 9


2) [NiCl 4 ] 2- Cl is a negatively charged ligand and complex is anionic as it it
is negatively charged. So to find oxidation state of Ni again let oxidation
state of Ni be x; so x+4(-1)=-2 ie, x=+2
3) [Ti(H 2 O) 6 ] 3+ H 2 O is a neutral ligand and the complex is positively charged
hence a cationic complex. Again, x+6(0)=+3 ie, x=+3
4) [Ni(en) 2 ] +2 en (ethylenediamine) is a bidentate neutral ligand a nd the
complex is cationic. In this case, it would be x+2 (0) =+2 ie, x=+2.
SOLVED PROBLEMS 01
1] Describe any two rules for IUPAC nomenclature of coordination compounds with
suitable examples.
Solution: Rule 1: Anionic Ligands
Ligands that act as anions which end in "-ide" are replaced with an ending "-
o" (e.g., Chloride replaced with Chloro). Anions ending with "-ite" and "-ate" are
replaced with endings "-ito" and "-ato" respectively (e.g., Nitrite replaced with
Nitrito, Nitrate replaced with Nitrato).
Rule 2: Neutral Ligands
Most neutral ligands (without charge) that are ligands carry their normal
name. The few exceptions are the first four on the chart: ammine, aqua, carbonyl,
and nitrosyl.
For e.g. Potassium hexacyanoferrate (III) is be written as K 3 [Fe (CN) 6 ]
Amminetetraaquachromium (II) ion could be written as [Cr (H 2 O) 4 (NH 3 )] +2

SHORT ANSWER QUESTIONS WITH MODEL ANSWER 01


1] Give name to the following compound according to the IUPAC rules
[Co (NH 3 ) 6 ] Cl 3
Solution: hexaamminecobalt (III) chloride
2] Give name to the following compound according to the IUPAC rules
[Cr(H 2 O) 4 Cl 2 ] NO 2
Solution: tetraaquadichloridochromium (III) nitrate
3] Give name to the following compound according to the IUPAC rules
[Co (NH 3 ) 4 Cl(NO 2 )] NO 3
Solution: tetraamminechloridonitrocobalt (III) nitrate
4] Give name to the following compound according to the IUPAC rules
K 3 [Fe(C 2 O 4 ) 3 ]
Solution: potassium trioxalatoferrate (III)

CHE509: Inorganic Chemistry – II Page 10


01-02: ISOMERISM IN COORDINATION COMPOUNDS
The presence of coordination compounds with the same formula but different
arrangements of the ligands were necessary in the development of coordination
chemistry. Two or more compounds with the same formula but different
arrangements of the atoms are called isomers. As long as isomers usually have
different physical and chemical properties, it is important to know which isomer we
are dealing with if more than one isomer is possible. As we will see, coordination
compounds show the same types of isomers as organic compounds, as well as several
kinds of isomers that are unique. Isomers are compounds with the same molecular
formula but different structural formulas and do not necessarily share common
properties. There are multiple different classes of isomers, like stereoisomers,
enantiomers, and geometrical isomers. There are two main forms of isomerism:
structural isomerism and stereoisomerism.
I] Structural Isomers
Structural Isomers are the isomers that contain the same number of atoms of
each kind but differ in structural arrangement of atoms are called structural isomers .
For coordination complexes, there are three types of structural isomers: ionization,
coordination and linkage. Structural isomers, as their name indicates, differ in their
structure or bonding, which are different from stereoisomers that differ in the
arrangement of the ligands are attached, however still have the bonding properties.
The different chemical formulas in structural isomers are caused either by a
difference in what ligands are bonded to the central atoms or how the individual
ligands are bonded to the central atoms. When determining a st ructural isomer, you
look at the ligands that are bonded to the central metal and which atom of the
ligands attach to the central metal.
a) Ionization Isomerism:
Ionization isomers occur when a ligand that is bound to the metal center
exchanges places with an anion or neutral molecule that was originally outside the
coordination complex. The geometry of the central metal ion and the identity of
other ligands are identical. For example, an octahedral isomer will have five ligands
that are identical, but the sixth will differ. The non -matching ligand in one
compound will be outside of the coordination sphere of the other compound.
Because the anion or molecule outside the coordination sphere is different, the

CHE509: Inorganic Chemistry – II Page 11


chemical properties of these isomers are different. A hydrate isomer is a specific
kind of ionization isomer where a water molecule is one of the molecules that
exchanges place

For e.g.[Co(NH 3 ) 5 Cl]Br and [Co(NH 3 ) 5 Br]Cl are ionization isomers of each other. In
coordination compounds, atoms mentions in square bracket are not ionisable but
atoms outside square bracket are ionisable.
The difference between the ionization isomers can be view within the context
of the ions generated when each are dissolved in solution. For example, when
pentaamminebromochlorocobalt (II) chloride is dissolved in water, Cl − ions are
generated
[Co (NH 3 ) 5 Br]Cl → [Co (NH 3 ) 5 Br] + + Cl -
Whereas when pentaamminechlorochlorocobalt (II) bromide is dissolved, Br− ions
are generated
[Co (NH 3 ) 5 Cl]Br → [Co (NH 3 ) 5 Cl] + + Br -
b) Coordination Isomerism:
Coordination isomerism observed in compounds containing complex anionic
and complex cationic parts and can be viewed as an interchange of some ligands
from the cation to the anion. Therefore two complex compounds bound together, one
with a negative charge and the other with a positive charge. In coordination isomers,
the anion and cation complexes of a coordination compound exchange one or more
ligands. For example, the [Zn(NH 3 ) 4 ][Cu(Cl 4 )] and [Cu(NH 3 ) 4 ][Zn(Cl 4 )]
compounds are coordination isomers.

In a coordination isomer the total ratio of ligand to metal remains the same,
but the ligands attached to a specific metal ion change. Examples of a complete
series of coordination isomers require at least two metal ions and sometimes more
for example, a solution containing ([Co (NH 3 ) 6 ] 3+ and [Cr (CN) 6 ] 3− ) is a coordination
isomer with a solution containing [Cr(NH 3 ) 6 ] 3+ and [Co(CN) 6 ] 3− .

CHE509: Inorganic Chemistry – II Page 12


c) Linkage Isomerism:

Linkage isomerism is a form of isomerism in which certain coordination


compounds have the same composition but differ in their metal atom's connectivity

to a ligand. Typical ligands that give rise to linkage isomers are: thiocyanate, SCN
and isothiocyanate, NCS –

Linkage isomerism occurs with ambidentate ligands that are capable of


coordinating in more than one way. The best known cases involve the monodentate
ligands: SCN − /NCS − and NO −2 /ONO − . The only difference is what atoms the
molecular ligands bind to the central metal ion. The ligands must have more than one
donor atom, but bind to ion in only one place. For example, the (NO −2 ) ion is a
ligand that can bind to the central atom via the nitrogen or the oxygen atom, but
cannot bind to the central atom with both oxygen and nitrogen simultaneously, in
which case it would be called a polydentate ligand rather than an ambidentate ligand .

When donation is from nitrogen to a metal center, the complex is known as a


nitro- complex and when donation is from one oxygen to a metal center, the complex
is known as a nitrito- complex. An alternative formula structure to emphasize the
different coordinate covalent bond for the two isomers.

[Co(ONO)(NH3) 5 ]Cl: the nitrito isomer -O attached


[Co(NO 2 )(NH3) 5 ]Cl: the nitro isomer - N attached
Another example of an ambidentate ligand is thiocyanate, SCN − , which can
attach to the transition metal at either the sulfur atom or the nitrogen atom. Such
compounds give rise to linkage isomerism. Other ligands that give rise to link age
isomers include selenocyanate, SeCN − , isoselenocyanate, NCSe − and sulfite, SO 2−
2] Geometric Isomers
The existence of coordination compounds with the same formula but a
different arrangement of the ligands was important in the development of
coordination chemistry. Two or more compounds with the same formula but different

CHE509: Inorganic Chemistry – II Page 13


arrangements of the atoms are called isomers. Because isomers usually have
different physical and chemical properties, it is important to know which isomer we
are dealing with if more than one isomer is possible. Remember that in many cases
more than one structure is possible for organic compounds with the same molecular
formula; examples discussed previously include n -butane versus isobutane and cis-2-
butene versus trans-2-butene. As we will see, coordination compounds exhibit the
same types of isomers as organic compounds, as well as several kinds of isomers
that are unique.
In coordination chemistry, its coordination number, the number of ligands
attached to the metal, first describes a structure (more specifically, the number of
donor atoms). Usually one can count the ligands attached, but sometimes even the
counting can become ambiguous. Coordination numbers are normally between two
and nine, but large numbers of ligands are not uncommon for the lanthanides and
actinides. The number of bonds depends on the size, charge, and electron
configuration of the metal ion and the ligands. Metal ions may have more than one
coordination number.
Geometrical isomers are of different types and out of that some important
types in coordination compounds are here
a) Planar Isomers
Metal complexes that differ only in which ligands are adjacent to one another
(cis) or directly across from one another (trans) in the coordination sphere of the
metal are called geometrical isomers. They are most important for square planar and
octahedral complexes. Because all vertices of a square are equivalent, it does not
matter which vertex is occupied by the ligand B in a square planar MA 3 B complex;
hence only a single geometrical isomer is possible in this case (and in the analogous
MAB 3 case). All four structures shown here are chemically identical because they
can be superimposed simply by rotating the complex in space:

CHE509: Inorganic Chemistry – II Page 14


For an MA 2 B 2 complex, there are two possible isomers: either the A ligands
can be adjacent to one another (cis), in which case the B ligands must also be cis, or
the A ligands can be across from one another (trans), in which case the B ligands
must also be trans. Even though it is possible to draw the cis isomer in four different
ways and the trans isomer in two different ways, all members of each set are
chemically equivalent:

Because there is no way to convert the cis structure to the trans by rotating or
flipping the molecule in space, they are fundamentally different arrangements of
atoms in space. Probably the best-known examples of cis and trans isomers of an
MA 2 B 2 square planar complex are cis-Pt(NH 3 ) 2 Cl 2 , also known as cisplatin, and
trans-Pt(NH 3 ) 2 Cl 2 , which is actually toxic rather than therapeutic.

b) Octahedral Isomers

Octahedral complexes also exhibit cis and trans isomers. Like square planar
complexes, only one structure is possible for octahedral complexes in which only
one ligand is different from the other five (MA 5 B). Even though we usually draw an
octahedron in a way that suggests that the four “in-plane” ligands are different from
the two “axial” ligands, in fact all six vertices of an octahedron are equivalent.
Consequently, no matter how we draw an MA 5 B structure, it can be superimposed on
any other representation simply by rotating the molecule in space. Two of the many
possible orientations of an MA 5 B structure are as follows:

CHE509: Inorganic Chemistry – II Page 15


SOLVED PROBLEMS 02
1] Explain the ionization isomer and coordination isomers in details.
Solution: Ionization Isomerism:
Ionization isomers occur when a ligand that is bound to the metal center exchanges
places with an anion or neutral molecule that was originally outside the coordination
complex. The geometry of the central metal ion and the identity of other ligands are
identical. For example, an octahedral isomer will have five ligands that are identical,
but the sixth will differ. The non-matching ligand in one compound will be outside
of the coordination sphere of the other compound. Because the anio n or molecule
outside the coordination sphere is different, the chemical properties of these isomers
is different. A hydrate isomer is a specific kind of ionization isomer where a water
molecule is one of the molecules that exchanges place

For e.g. [Co(NH 3 ) 5 Cl]Br and [Co(NH 3 ) 5 Br]Cl are ionization isomers of each other.
In coordination compounds, atoms mention in square bracket is not ionisable but
atoms outside square bracket are ionisable.
The difference between the ionization isomers can be view within the context
of the ions generated when each are dissolved in solution. For example, when
pentaamminebromocobalt (II) chloride is dissolved in water, Cl − ions are generated
[Co (NH 3 ) 5 Br]Cl → [Co (NH 3 ) 5 Br] + + Cl -
Whereas when pentaamminechlorocobalt (II) bromide is dissolved, Br − ions are
generated
[Co (NH 3 ) 5 Cl]Br → [Co (NH 3 ) 5 Cl] + + Br -

b) Coordination Isomerism:
Coordination isomerism observed in compounds containing complex anionic
and complex cationic parts and can be viewed as an interchange of some ligands
from the cation to the anion. Therefore two complex compounds bound together, one

CHE509: Inorganic Chemistry – II Page 16


with a negative charge and the other with a positive charge. In coordination isomers,
the anion and cation complexes of a coordination compound exchange one or more
ligands. For example, the [Zn(NH 3 ) 4 ][Cu(Cl 4 )] and [Cu(NH 3 ) 4 ][Zn(Cl 4 )]
compounds are coordination isomers.
In a coordination isomer the total ratio of ligand to metal remains the same,
but the ligands attached to a specific metal ion change. Examples of a complete
series of coordination isomers require at least two metal ions and sometimes more
for example, a solution containing ([Co (NH 3 ) 6 ] 3+ and [Cr (CN) 6 ] 3− ) is a coordination
isomer with a solution containing [Cr(NH 3 ) 6 ] 3+ and [Co(CN) 6 ] 3− .

2] Describe geometrical isomerism in details


Solution: Geometric Isomers
The existence of coordination compounds with the same formula but a
different arrangement of the ligands was important in the development of
coordination chemistry. Two or more compounds with the same formula but different
arrangements of the atoms are called isomers. Because isomers usually have
different physical and chemical properties, it is important to know which isomer we
are dealing with if more than one isomer is possible. Remember that in many cases
more than one structure is possible for organic compounds with the same molecular
formula; examples discussed previously include n-butane versus isobutane and cis-2-
butene versus trans-2-butene. As we will see, coordination compounds exhibit the
same types of isomers as organic compounds, as well as several kinds of isomers
that are unique.

In coordination chemistry, its coordination number, the number of ligands


attached to the metal, first describes a structure (more specifically, the number of
donor atoms). Usually one can count the ligands attached, but sometimes even the
counting can become ambiguous. Coordination numbers are normally between two
and nine, but large numbers of ligands are not uncommon for the lanthanides and
actinides. The number of bonds depends on the size, charge, and electron
configuration of the metal ion and the ligands. Metal ions may have more than one
coordination number.

Geometrical isomers are of different types and out of that some important
types in coordination compounds are here

CHE509: Inorganic Chemistry – II Page 17


a) Planar Isomers

Metal complexes that differ only in which ligands are adjacent to one another
(cis) or directly across from one another (trans) in the coordination sphere of the
metal are called geometrical isomers. They are most important for square planar and
octahedral complexes. Because all vertices of a square are equivalent, it does not
matter which vertex is occupied by the ligand B in a square planar MA 3 B complex;
hence only a single geometrical isomer is possible in this case (and in the analogous
MAB 3 case). All four structures shown here are chemically identical because they
can be superimposed simply by rotating the complex in space:

For an MA 2 B 2 complex, there are two possible isomers: either the A ligands
can be adjacent to one another (cis), in which case the B ligands must also be cis, or
the A ligands can be across from one another (trans), in which case the B ligands
must also be trans. Even though it is possible to draw the cis isomer in four different
ways and the trans isomer in two different ways, all members of each set are
chemically equivalent:

Because there is no way to convert the cis structure to the trans by rotating or
flipping the molecule in space, they are fundamentally different arrangements of
atoms in space. Probably the best-known examples of cis and trans isomers of an
MA 2 B 2 square planar complex are cis-Pt (NH 3 ) 2 Cl 2 , also known as cisplatin, and
trans-Pt (NH 3 ) 2 Cl 2 , which is actually toxic rather than therapeutic.
b) Octahedral Isomers

CHE509: Inorganic Chemistry – II Page 18


Octahedral complexes also exhibit cis and trans isomers. Like square planar
complexes, only one structure is possible for octahedral complexes in which only
one ligand is different from the other five (MA 5 B). Even though we usually draw an
octahedron in a way that suggests that the four “in-plane” ligands are different from
the two “axial” ligands, in fact all six vertices of an octahedron are equiv alent.
Consequently, no matter how we draw an MA 5 B structure, it can be superimposed on
any other representation simply by rotating the molecule in space. Two of the many
possible orientations of an MA 5 B structure are as follows:

SHORT ANSWER QUESTIONS WITH MODEL ANSWER 02


1] Define geometrical isomerism.
Solution: Geometric isomers are two or more coordination compounds, which
contain the same number and types of atoms, and bonds (i.e., the connectivity
between atoms is the same), but which have different spatial arrangements of the
atoms.
2] Define linkage isomerism.
Solution: Linkage isomerism is a form of isomerism in which certain coordination
compounds have the same composition but differ in their metal atom's connectivity
to a ligand.
01-03: METHODS OF PREPARATION OF COORDINATION COMPLEXES
Coordination compounds are generally prepared by common methods such as
substitution reactions, redox reactions and by the direct combination of the reactants.

1] By Substitution Reactions
Most conjugates are made by substitution reactions. A common synthetic
procedure is to replace the water molecules surrounding the metal ion with other
ligands. For example: Most conjugates are made by substitution reactions. A
common synthetic procedure is to replace the water molecules surrounding the metal
ion with other ligands. For example:

CHE509: Inorganic Chemistry – II Page 19


(a) Heating an aqueous CuSO4 solution with excess ammonia forms the dark blue
[Cu (NH 3 ) 4 ] 2+ species.
[Cu (H 2 O) 4 ] 2+ + 4 NH 3 → [Cu(NH 3 ) 4 ] 2+ + 4 H 2 O

(b) Cobalt (II) nitrate reacts with concentrated aqueous ammonia and NH4NO3 in
the presence of H2O2 to give the complex, pentaamminenitrocobalt (III) nitrate as
2[Co (H 2 O) 6 ](NO 3 ) 2 +8NH 3 +2NH 4 NO 3 +H 2 O 2 → 2[Co(NO 2 )(NH 3 ) 5 ](NO 3 ) 3
+14H 2 O
Some other examples are:

[Co (NO3)3]3-+ 2en → [Co(en)2(NO2)2]+ +4NO2

[Ni (H2O)6]2++ 6NH3 → [Ni(NH3)6]2+ +6 H2O

2] By Redox Reactions

Hexamminecobalt (III) chloride can be prepared by oxidizing an aqueous solution of


cobalt (II) chloride made alkaline with ammonia in the presence of NH 4 Cl and
hydrogen peroxide.

2COCL2 + 2NH4C L + 10NH3 + H2O2 → 2[CO(NH3)6]CL3 + 2H2O

An aqueous solution of oxalic acid and potassium oxalate reduces K 2 Cr 2 O 7 to the


complex ion trioxalatochromate (III) ion.

K 2 Cr 2 O 7 + 7H 2 C 2 O 4 + 2K 2 C 2 O 4 → 2K 3 [Cr(C 2 O 4 ) 3 ] + 6CO 2 + 7H 2 O

3] By Direct Combination of Reactants

Metal amines can be prepared by the direct addition of a metal salt to liquid ammonia.

NiCl2 + 6NH3 → [Ni(NH3)6]Cl2

CHE509: Inorganic Chemistry – II Page 20


CoCl3 + 6NH3 → [Co(NH3)6]Cl3
AgCl + 2NH3 → [Ag(NH3)2]Cl

Similarly, addition of ethylenediamine (en) to PtCl2 gives [Pt(en)2]Cl2

PtCl2 + 2en → [Pt(en)2]Cl2

SOLVED PROBLEMS 03

1] Give methods of preparation of coordination compounds

Solution: Coordination compounds are generally prepared by common


methods such as substitution reactions, redox reactions and by the direct
combination of the reactants.

1] By Substitution Reactions

Most conjugates are made by substitution reactions. A common synthetic


procedure is to replace the water molecules surrounding the metal ion with other
ligands. for example: Most conjugates are made by substitution reactions. A common
synthetic procedure is to replace the water molecules surrounding the metal ion wi th
other ligands. for example:
Heating an aqueous CuSO4 solution with excess ammonia forms the dark blue
[Cu(NH 3 ) 4 ] 2+ species.
[Cu(H 2 O) 4 ] 2+ + 4 NH 3 → [Cu(NH 3 ) 4 ] 2+ + 4 H 2 O
Cobalt (II) nitrate reacts with concentrated aqueous ammonia and NH4NO3 in
the presence of H2O2 to give the complex, pentaamminenitrocobalt (III) nitrate as
2[Co(H 2 O) 6 ](NO 3 ) 2 +8NH 3 +2NH 4 NO 3 +H 2 O 2 → 2[Co(NO 2 )(NH 3 ) 5 ](NO 3 ) 3 +14H 2 O
Some other examples are:

[Co(NO3)3]3-+ 2en → [Co(en)2(NO2)2]+ +4NO2


[Ni(H2O)6]2++ 6NH3 → [Ni(NH3)6]2+ +6 H2O

2] By Redox Reactions
Hexamminecobalt (III) chloride can be prepared by oxidizing an aqueous solution of
cobalt (II) chloride made alkaline with ammonia in the presence of NH 4 Cl and
hydrogen peroxide.

CHE509: Inorganic Chemistry – II Page 21


2COCL2 + 2NH4C L + 10NH3 + H2O2 → 2[CO(NH 3)6]CL3 + 2H2O
An aqueous solution of oxalic acid and potassium oxalate reduces K 2 Cr 2 O 7 to the
complex ion trioxalatochromate (III) ion.
K 2 Cr 2 O 7 + 7H 2 C 2 O 4 + 2K 2 C 2 O 4 → 2K 3 [Cr(C 2 O 4 ) 3 ] + 6CO 2 + 7H 2 O
3] By Direct Combination of Reactants
Metal amines can be prepared by the direct addition of a metal salt to liquid ammonia.

NiCl2 + 6NH3 → [Ni(NH3)6]Cl2


CoCl3 + 6NH3 → [Co(NH3)6]Cl3
AgCl + 2NH3 → [Ag(NH3)2]Cl

Similarly, addition of ethylenediamine (en) to PtCl2 gives [Pt(en)2]Cl2


PtCl2 + 2en → [Pt(en)2]Cl2

SHORT ANSWER QUESTIONS WITH MODEL ANSWER 03

1] Give redox method for synthesis of coordination compounds.


Solution: Hexamminecobalt(III) chloride can be prepared by oxidizing an aqueous
solution of cobalt(II) chloride made alkaline with ammonia in the presence of NH 4 Cl
and hydrogen peroxide.

2COCL2 + 2NH4C L + 10NH3 + H2O2 → 2[CO(NH3)6]CL3 + 2H2O

An aqueous solution of oxalic acid and potassium oxalate reduces K 2 Cr 2 O 7 to the


complex ion trioxalatochromate (III) ion.

K 2 Cr 2 O 7 + 7H 2 C 2 O 4 + 2K 2 C 2 O 4 → 2K 3 [Cr(C 2 O 4 ) 3 ] + 6CO 2 + 7H 2 O

2] Give direct combination method for synthesis of coordination compounds.


Solution:
Metal amines can be prepared by the direct addition of a metal salt to liquid ammonia.
NiCl2 + 6NH3 → [Ni(NH3)6]Cl2
CoCl3 + 6NH3 → [Co(NH3)6]Cl3
AgCl + 2NH3 → [Ag(NH3)2]Cl

Similarly, addition of ethylenediamine (en) to PtCl2 gives [Pt(en)2]Cl2

CHE509: Inorganic Chemistry – II Page 22


PtCl2 + 2en → [Pt(en)2]Cl2

01-04: TYPES OF LIGANDS

Neutral molecules or ions that are directly bonded to a central metal ion or
atom by coordinate bonds in a complex are called ligands or ligands. In other words,
any species that can donate an electron pair to a metal is called a ligand. Ligands can
be negatively or positively charged ions or neutral molecules.

There are several requirements for an atom or group of atoms or ions to behave as a
ligand. A ligand must have at least one lone pair. The ligand must have the ability to donate
a lone pair of electrons to the central metal atom or ion, thereby forming a coordinative
covalent bond. Ligands behave like Lewis bases, metal atoms or ions behave as Lewis
acids, and Lewis acid-base reactions occur between them to form coordination compounds.

Examples: Cl-, Br-, SO42-, NH2NH3+, NH3, H2O, NH2CH2CH2NH2, etc.


Ligands can be classified based on many things. The most common
classification of ligands is based on their binding sites with a central metal atom or
ion. Based on the number of sites, ligands can be classified into monodentate,
bidentate, multidentate, etc. ligands.
Monodentate Ligands: These are ligands that coordinate to a single site on the
metal ion. That is, only one pair of electrons can be donated to the metal ion. Examples: Cl -
, SO 4 2- , Br - , NH 3 , NH 2 NH 3+ , H 2 O
Bidentate ligand: These are the ligands that occupy two sites of a metal ion. That
is, it can be attached to two metal ion positions, e.g. NH2CH2CH2NH2 etc.
Polydentate Ligands: These are ligands that occupy many sites on the same metal
ion. For example: EDTA. B. Tridentate, tetradentate, pentadentate, hexadentate, etc.
Another classification of ligands is based on the molecular complex structures
formed upon the formation of coordinate bonds. According to this classification, ligands are
divided into two types: chelating ligands and ambient ligands.
Chelating agents: These are ligands that bind to the same central metal atom or ion,
forming a ring-like structure. Bidentate or polydentate ligands usually fall into this
category. Chelating ligands generally form a ring structure around a central metal atom or
ion. The best-known example of this type of ligand is EDTA (ethylenediaminetetraacetic
acid).
Ambient Ligands: Ambient ligands are ligands that bind to a central metal atom or
ion through multiple sites. Monodentate ligands usually fall into this category of ligands.

CHE509: Inorganic Chemistry – II Page 23


The most common examples of this type of ligand are cyanides (M –CN) and isocyanides
(M–NC).
SOLVED PROBLEMS 04
1] Define legands and explain different types of legands
Solution: Neutral molecules or ions that are directly bonded to a central metal ion or
atom by coordinate bonds in a complex are called ligands or ligands. In other words,
any species that can donate an electron pair to a metal is called a ligand. Ligands can
be negatively or positively charged ions or neutral molecules.
There are several requirements for an atom or group of atoms or ions to behave as a
ligand. A ligand must have at least one lone pair. The ligand must have the ability to donate
a lone pair of electrons to the central metal atom or ion, thereby forming a coordinative
covalent bond. Ligands behave like Lewis bases, metal atoms or ions behave as Lewis
acids, and Lewis acid-base reactions occur between them to form coordination compounds.
Examples: Cl-, Br-, SO42-, NH2NH3+, NH3, H2O, NH2CH2CH2NH2, etc.
Ligands can be classified based on many things. The most common
classification of ligands is based on their binding sites with a central metal atom or
ion. Based on the number of sites, ligands can be classified into monodentate,
bidentate, multidentate, etc. ligands.
Monodentate Ligands: These are ligands that coordinate to a single site on the
metal ion. That is, only one pair of electrons can be donated to the metal ion. Examples: Cl -
, SO 4 2- , Br - , NH 3 , NH 2 NH 3+ , H 2 O
Bidentate ligand: These are the ligands that occupy two sites of a metal ion. That
is, it can be attached to two metal ion positions, e.g. NH2CH2CH2NH2 etc.
Polydentate Ligands: These are ligands that occupy many sites on the same metal
ion. For example: EDTA. B. Tridentate, tetradentate, pentadentate, hexadentate, etc.

SHORT ANSWER QUESTIONS WITH MODEL ANSWER 04


1] Explain the term chelating agent
Solution:
Chelating agents: These are ligands that bind to the same central metal atom or ion,
forming a ring-like structure. Bidentate or polydentate ligands usually fall into this
category. Chelating ligands generally form a ring structure around a central metal atom or
ion. The best-known example of this type of ligand is EDTA (ethylenediami netetraacetic
acid).
2] Explain the term Ambient Ligands

CHE509: Inorganic Chemistry – II Page 24


Solution:
Ambient Ligands: Ambient ligands are ligands that bind to a central metal atom or
ion through multiple sites. Monodentate ligands usually fall into this category of ligands.
The most common examples of this type of ligand are cyanides (M –CN) and isocyanides
(M–NC).
CHECK POINT 01-01
1] Which of the following ligands will be listed last in a coordination sphere?
a) Cl
b) CO
c) CO 3

d) C 2 O 4

Solution: d

Because Ligands shown are chlorine (Cl), carbonyl (CO), carbonate (CO 3 ), oxalate
(C 2 O 4 ). Of these, oxalic acid is the last one to occur in alphabetical order, so it is
listed last in the unit of adjustment.
2] The prefix di is used twice in the naming of [NiCl 2 (PPh 3 ) 2 ].
a) True
b) False
Solution: b
The prefix di is used only once for Chlorido. For the PPh 3 ligand, PPh 3
(triphenylphosphine) already has a numeric prefix in its name, so the bis prefix
should be used instead.
3] Which of the following compounds has enantiomers?
a) K 3 [Fe(CN) 6 ]
b) K 3 [Al(C 2 O 4 ) 3 ]
c) K 2 [Zn(OH) 4 ]
d) K 2 [PdCl 4 ]
Solution: b
K 3 [Al(C 2 O 4 ) 3 ]exhibits optical isomerism due to its octahedral structure and is bound
to the bidentate ligand oxalate. On the other hand, K 3 [Fe(CN) 6 ], K 2 [Zn(OH) 4 ], and
K 2 [PdCl 4 ] have nonidentical ligands and CN=6 (octahedral), 4, and 4, respectively.
is a compound.
4] Identify the correct naming for K 3 [Fe(CN) 6 ].
a) Tripotassium hexacyanidoferrate(III)

CHE509: Inorganic Chemistry – II Page 25


b) Potassium hexacyanoferrate (III)
c) Tripotassium hexacyanoferrate (III)
d) Potassium hexacyanidoferrate(III)
Solution: d
While doing nomenclature of any compound the number of cations and anions are
never denoted in its naming. Here, Potassium will be the name of the counter ion and
tripotassium is not at all valid here. And the name of CN should end with –‘o’,
cyanido being the correct naming.

SUMMARY

Coordination complexes are so pervasive that their structures and reactions


are described in many ways, sometimes confusingly. The atom within a ligand that is
bonded to the central metal atom or ion is called the donor atom. In a typical
complex, a metal ion is bonded to several donor atoms, which can be the same or
different. A polydentate (multiple bonded) ligand is a molecule or ion that bonds to
the central atom through several of the ligand's atoms; ligands with 2, 3, 4 or even 6
bonds to the central atom are common. These complexes are called chelate
complexes; the formation of such complexes is called chelation, complexation, and
coordination. The number of donor atoms bonded to the central atom or ion is called
the coordination number. The most common coordination numbers are 2, 4 and
especially 6. Hydrated ions are a type of complex ion, a species formed between a
central metal ion and one or more surrounding ligands, molecules, or at least one. It
contains a lone pair of electrons. In coordination chemistry, a ligand is an ion or
molecule (functional group) that binds to a central metal atom to form a coordination
complex. The bonding with the metal generally involves formal donation of one or
more of the ligand's electron pairs, often through Lewis bases.

KEY WORDS

Coordination, Isomerism, Legands.

REFERENCES
MOOCS
1) https://archive.nptel.ac.in/noc/courses/noc20/SEM2/noc20-cy19/

2) https://www.lunduniversity.lu.se/lubas/i-uoh-lu-KEMM52

CHE509: Inorganic Chemistry – II Page 26


YOUTUBE VIDEOS
1) https://www.youtube.com/watch?v=REJPwUQDjxA

2) https://www.youtube.com/watch?v=PpuPUMfr91Y

3) https://www.youtube.com/watch?v=kgiyurcr5XI&list=PLdZcCa6mtW22HTEHF
8-rqOyXD-fNB7OKD

4) https://www.youtube.com/watch?v=LKM_p1w_--s

5) https://www.youtube.com/watch?v=7lGOM8G4TaE

WIKIPEDIA
1) https://en.wikipedia.org/wiki/Coordination_complex

2) https://en.wikipedia.org/wiki/Ligand

OER
1) https://chem.libretexts.org/Bookshelves/Inorganic_Chemistry/Supplemental_Mo
dules_and_Websites_(Inorganic_Chemistry)/Coordination_Chemistry/Structure
_and_Nomenclature_of_Coordination_Compounds/Coordination_Numbers_and
_Geometry

2) https://www.vedantu.com/iit-jee/ligands-and-its-types

3) https://chemed.chem.purdue.edu/genchem/topicreview/bp/ch12/complex.php

REFERENCE BOOKS
1) Huheey, J. E., Keitler, E. A., & Keitler, R. L. (2012). Inorganic Chemistry -
Principles of Structure and Reactivity (IV Edition). Singapore: Pearson Education.

2) Coordination chemistry Textbook Binding – 1 January 2016 by Ajai Kumar

3) Concise Coordination Chemistry Paperback – 1 January 2008 by R. Gopalan, V.


Ramalingam

CHE509: Inorganic Chemistry – II Page 27


CREDIT 01-UNIT 02: B ONDING
LEARNING OBJECTIVES
After successful completion of this unit, you will be able to
 Understand the Valence bond theory of bond
 Understand the crystal field theory of bond
 Apply the concept of crystal field stabilization energy
 Apply the concept of strong field and weak field of legands
 Apply the concept of spectrochemical series
INTRODUCTION

Walter Heinrich Heitler (1904–1981)


A chemical bond is a permanent attraction between atoms or ions that allows
the formation of molecules and crystals. Bonding can result from electrostatic forces
between oppositely charged ions, as in ionic bonding, or through sharing of
electrons, as in covalent bonding. The strength of chemical bonds varies greatly.
There are "strong bonds" or "primary bonds" such as covalent, ionic and metallic
bonds, and "weak" or "secondary" bonds such as dipole -dipole interactions, London
dispersion forces and hydrogen bonds. Strong chemical bonds are fo rmed by sharing
or transferring electrons between participating atoms.
Neither chemistry nor quantum mechanics have mathematical formulas for the
arrangement of electrons in atoms, but the hydrogen atom can be described by the
Schrödinger equation, derived in 1925, and the equations of matrix mechanics. But
for hydrogen alone, his Heitler-London theory was formulated in 1927, making it

CHE509: Inorganic Chemistry – II Page 28


possible for the first time to calculate the bonding properties of the hydrogen
molecule H 2 based on quantum mechanical considerations.
02-01: THEORIES OF BONDING
1] Valence bond theory (VBT)
In chemistry, valence bond (VB) theory, along with molecular orbital (MO)
theory, is one of two fundamental theories developed to explain chemical bonds
using methods of quantum mechanics. It focuses on how the atomic orbitals of
dissociated atoms connect to form individual chemical bonds as molecules are
formed. In contrast, molecular orbital theory has orbital’s that cover the entire
molecule.
In 1916, G.N. Lewis proposed that a chemical bond is formed by the
interaction of two covalent electrons, representing the molecule as a Lewis structure.
Chemist Charles Rugeley Berry proposed in 1921 that 8 and 18 electrons form stable
configurations in the shell. Berry proposed that the electron configuration of
transition elements depends on the outer shell valence electrons. In 1916 Cossel
published his theory of ionic chemical bonding (the octet rule) and Gilbert N. Lewis
Walther Kossel presented a similar theory to Lewis, but his model assumed perfect
transfer of electrons between atoms and was therefore a model of ionic bonding.
Both Lewis and Cossel constructed attachment models according to the rules of
Abegg (1904).
Valence bond theory based on following Assumptions/Postulates
 A covalent bond is formed by the superposition of two half-filled valence
orbital’s of two different atoms. The overlap increases the electron density
between the two bonded atoms and stabilizes the molecule.
 Multiple bonds can form if there are multiple unpaired electrons in an atomic
orbital, and electrons paired in the valence shell cannot participate in such
bond formation.
 Covalent bonds are directional and run parallel to the region where the atomic
orbital’s overlap. Based on the overlap pattern, there are two types of covalent
bonds: sigma bonds and pi bonds.
 Covalent bonds formed by lateral overlap of atomic orbitals are known as pi
bonds, and bonds formed by overlap of atomic orbitals along the internuclear
axis are known as sigma bonds.
Formation of H 2 molecule based on valence bond theory.

CHE509: Inorganic Chemistry – II Page 29


Suppose two hydrogen atoms (A and B) with nuclei (N A and N B) and
electrons (e A and e B) react in the hydrogen molecule. To do. To construct. If A and
B are far apart, there is no interaction between them. As they approach each other,
attractive and repulsive forces begin to act.
 An attractive force occurs between the nucleus and its own elec trons. H. N A
– e A and n d N B – e B.
 The nucleus of one atom and the electrons of another atom, namely H . N A –
e B and N B – e A.
 The electrons of two atoms. H. e A – e B.
 The nuclei of two atoms, namely N A – N B. Attraction pulls two atoms
together, while repulsion tends to push them apart.
The magnitude of the attractive force is greater than the magnitude of the
repulsive force. The two atoms are therefore close to each other. This reduces the
potential energy. Eventually the attractive force will balance the repulsive force and
the system will reach a state where minimal energy is conserved. This forms a
dihydrogen molecule.

The number of new attractive and repulsive forces is the same, but the
magnitude of the attractive force is greater. So when two hydrogen atoms get closer
to each other, the total potential energy decreases, thus forming a stable hydrogen
molecule.
Since the energy gets released when the bond is formed between two hydrogen
atoms, the hydrogen molecule is more stable than that of isolated hydrogen atoms.
The energy so released is called as bond enthalpy, which is corresponding to
minimum in the curve depicted in following Figure Conversely, 435.8 kJ of energy is
required to dissociate one mole of H2 molecule.

CHE509: Inorganic Chemistry – II Page 30


Crystal Field Theory
An important feature of transition metals is their tendency to form complexes.
Complexes can be viewed as consisting of a central metal atom or ion surrounded by
a set of ligands. The interaction of these ligands with the central metal atom or ion is
governed by crystal field theory. Crystal field theory was established in 1929 and
treats the interaction of metal ions with ligands as a purely electrostatic
phenomenon, where the ligands are viewed as point charges near the atomic orbitals
of the central atom. The development and extension of crystal field theory took into
account the partial covalent nature of the bond between ligands and metal atoms,
mainly through the application of molecular orbital theory.
A clear understanding of the crystal field interactions of transition metal
complexes requires knowledge of the geometric or spatial arrangement of the d-
orbital. The d orbitals are five-fold degenerate for free gaseous metal ions. When a
spherically symmetric field of negative ligand field charge is imposed on the central
metal ion, the d orbitals remain degenerate, but some change in free ion energy
occurs.
A crystal field theory was proposed to describe the metal -ligand bond as an ionic
bond arising solely from the electrostatic interaction between the metal ion and the
ligand. Crystal field theory considers anions as point charges and neutral molecules
as dipoles. When transition metals are not bound to ligands, their d orbitals are
degenerate. That is, they have the same energy. When they start to associate with
other ligands, the d-orbital are split and non-degenerate due to the different
symmetries of the d-orbital and the inductive effect of the ligands on the electrons.

CHE509: Inorganic Chemistry – II Page 31


SOLVED PROBLEMS 01
1] Describe in detail Valence bond theory and give postulates.
Solution:
In chemistry, valence bond (VB) theory, along with molecular orbital (MO)
theory, is one of two fundamental theories developed to explain chemical bonds
using methods of quantum mechanics. It focuses on how the atomic orbi tals of
dissociated atoms connect to form individual chemical bonds as molecules are
formed. In contrast, molecular orbital theory has orbital’s that cover the entire
molecule.
In 1916, G.N. Lewis proposed that a chemical bond is formed by the
interaction of two covalent electrons, representing the molecule as a Lewis structure.
Chemist Charles Rugeley Berry proposed in 1921 that 8 and 18 electrons form stable
configurations in the shell. Berry proposed that the electron configuration of
transition elements depends on the outer shell valence electrons. In 1916 Cossel
published his theory of ionic chemical bonding (the octet rule) and Gilbert N. Lewis
Walther Kossel presented a similar theory to Lewis, but his model assumed perfect
transfer of electrons between atoms and was therefore a model of ionic bonding.
Both Lewis and Cossel constructed attachment models according to the rules of
Abegg (1904).
Valence bond theory based on following Assumptions/Postulets
 A covalent bond is formed by the superposition of two half-filled valence
orbitals of two different atoms. The overlap increases the electron density
between the two bonded atoms and stabilizes the molecule.

 Multiple bonds can form if there are multiple unpaired electrons in an atomic
orbital, and electrons paired in the valence shell cannot participate in such
bond formation.

 Covalent bonds are directional and run parallel to the region where the atomic
orbital’s overlap. Based on the overlap pattern, there are two types of covalent
bonds: sigma bonds and pi bonds.

 Covalent bonds formed by lateral overlap of atomic orbitals are known as pi


bonds, and bonds formed by overlap of atomic orbitals along the internuclear
axis are known as sigma bonds.
2] Explain the formation of Hydrogen molecule by using VBT.

CHE509: Inorganic Chemistry – II Page 32


Solution:
Formation of H 2 molecule based on valence bond theory.
Suppose two hydrogen atoms (A and B) with nuclei (NA and NB) and
electrons (e A and e B) react in the hydrogen molecule. To do. To construct. If A and
B are far apart, there is no interaction between them. As they approach each other,
attractive and repulsive forces begin to act.
 An attractive force occurs between the nucleus and its own electro ns. H. N A
– e A and n d N B – e B.
 The nucleus of one atom and the electrons of another atom, namely H . N A –
e B and N B – e A.
 The electrons of two atoms. H. eA – e B.
 The nuclei of two atoms, namely N A – N B. Attraction pulls two atoms
together, while repulsion tends to push them apart.
The magnitude of the attractive force is greater than the magnitude of the
repulsive force. The two atoms are therefore close to each other. This reduces the
potential energy. Eventually the attractive force will balance the repulsive force and
the system will reach a state where minimal energy is conserved. This forms a
dihydrogen molecule.

The number of new attractive and repulsive forces is the same, but the
magnitude of the attractive force is greater. So when two hydrogen atoms get closer
to each other, the total potential energy decreases, thus forming a stable hydrogen
molecule.
Since the energy gets released when the bond is formed between two hydrogen
atoms, the hydrogen molecule is more stable than that of isolated hydrogen atoms.
The energy so released is called as bond enthalpy, which is corresponding to
minimum in the curve depicted in following Figure Conversely, 435.8 kJ of energy is
required to dissociate one mole of H2 molecule.

CHE509: Inorganic Chemistry – II Page 33


3] Describe Crystal field theory in detail.
Solution:
Crystal field Theory (CFT)
An important feature of transition metals is their tendency to form complexes.
Complexes can be viewed as consisting of a central metal atom or ion surrounded by
a set of ligands. The interaction of these ligands with the central metal atom or ion is
governed by crystal field theory. Crystal field theory was established in 1929 and
treats the interaction of metal ions with ligands as a purely electrostatic
phenomenon, where the ligands are viewed as point charges near the atomic orbitals
of the central atom. The development and extension of crystal field theory took into
account the partial covalent nature of the bond between ligands and metal atoms,
mainly through the application of molecular orbital theory.
A clear understanding of the crystal field interactions of transition metal
complexes requires knowledge of the geometric or spatial arrangement of the d-
orbital. The d orbitals are five-fold degenerate for free gaseous metal ions. When a
spherically symmetric field of negative ligand field charge is imposed on the central
metal ion, the d orbitals remain degenerate, but some change in free ion energy
occurs.
A crystal field theory was proposed to describe the metal -ligand bond as an ionic
bond arising solely from the electrostatic interaction between the metal ion and the
ligand. Crystal field theory considers anions as point charges and neutral molecules
as dipoles. When transition metals are not bound to ligands, their d orbital are
degenerate. That is, they have the same energy. When they start to associate with

CHE509: Inorganic Chemistry – II Page 34


other ligands, the d orbitals are split and non-degenerate due to the different
symmetries of the d orbitals and the inductive effect of the ligands on the electrons.

SHORT ANSWER QUESTIONS WITH MODEL ANSWER 01


1] Give any two postulates of VBT
Solution:
Postulates of VBT
 A covalent bond is formed by the superposition of two half-filled valence
orbitals of two different atoms. The overlap increases the electron density
between the two bonded atoms and stabilizes the molecule.
 Multiple bonds can form if there are multiple unpaired electrons in an atomic
orbital, and electrons paired in the valence shell cannot participate in such
bond formation.
2] Explain crystal field theory in short
Solution:
A clear understanding of the crystal field interactions of transition metal
complexes requires knowledge of the geometric or spatial arrangement of the d-
orbital. The d orbitals are five-fold degenerate for free gaseous metal ions. When a
spherically symmetric field of negative ligand field charge is imposed on the central
metal ion, the d orbitals remain degenerate, but some change in free ion energy
occurs.

02-02: CRYSTAL FIELD SPLITTING

High Spin and Low Spin


Complexes with a large number of unpaired electrons are called high -spin
complexes, while low-spin complexes contain a less number of unpaired electrons.
High spin complexes are expected with weak field ligands while the crystal field
splitting energy is small Δ (Δ is energy difference between eg and t 2 g orbital’s which
obtain after splitting). The opposite is true for low-spin complexes, where strong
electric field ligands induce maximal electron pairing in the set of three t 2 atomic
orbitals due to the large Δo.
Ligands that cause large Δ d orbital splitting are called high -field ligands,
such as CN - and CO in the spectrochemical series. In complexes with these ligands,
placing electrons in high-energy orbital’s is unfavorable. Therefore, according to the

CHE509: Inorganic Chemistry – II Page 35


aufbau principle, the low-energy orbital’s are completely filled before the occupation
of the upper set begins. Such complexes are called "low-spin". For example, NO 2- is
a strong field ligand and produces a large Δ. An octahedral ion [Fe(NO 2 ) 6 ] 3− with 5 d
electrons has the octahedral splitting diagram shown on the figure if all 5 electrons
are in the t 2 g level. Therefore, this low spin state does not obey Hund's rule.

Low spin

High spin
Tips for determining high-spin or low-spin configurations
 Determining the shape of the complex (i.e. octahedral, tetrahedral, square
planar)
 Determining the oxidation state of the metal center d electrons at the metal
center
 Determine the configuration Crystal field diagram of the complex in terms of
Draw geometry
 Determine if the splitting energy is greater than the pairing energy Determine
the strength of the ligand (i.e. spectrochemical series)
Crystal field splitting in Octahedral Complex
 For octahedral coordination compounds with six ligands surrounding a metal
atom/ion, repulsion between d-orbital electrons and ligand electrons is
observed.
 This repulsion is felt stronger for the dx2-y2 and dz2 orbitals, since they are
oriented with their axes aligned with the direction of the ligand.

CHE509: Inorganic Chemistry – II Page 36


 They therefore have energies higher than the average energy of the spherical
crystal field.
 On the other hand, the dxy, dyz, and dxz orbitals are less repulsive because
they are oriented between the axes.
 Therefore, these three orbitals have less energy than the average energy of the
spherical crystal field.

Fig: - Crystal Field Splitting in Octahedral Complex


This degenerate level splitting in the presence of ligands is known as crystal
field splitting. The difference between the energy of t 2 g and the eg level is denoted
by 'Δo' (the index o represents the octahedron). Some ligands tend to generate strong
fields, causing large crystal field splittings, while some ligands tend to generate
weak fields, causing small crystal field splittings.

Crystal Field Splitting in Tetrahedral Complex


The splitting of the pentafold degenerate metal ion d orbitals i nto two levels
in the tetrahedral crystal field is the two sets of orbitals denoted as Td. Electrons in
the dx 2 -y2 and dz 2 orbitals are less repelled by the ligand than electrons in the dxy,
dyz, and dxz orbitals. As a result, the dxy, dyz, and dxz orbital sets increase in
energy and the dx 2 -y2 and dz 2 orbitals sets decrease in energy.
 Since there are only four ligands in the Td complex, the total negative charge
of four ligands, and thus the ligand field, is smaller than for six ligands.
 The orientation of the orbital does not match the direction in which the ligand
approaches the metal ion.

CHE509: Inorganic Chemistry – II Page 37


Therefore, the repulsion of tetrahedral coordination compounds gives
rise to two energy levels.

 t 2 - set of three high energy orbital’s (dxy, dyz, and dxz)

 e - set of two low energy orbital’s (dx 2 -y2 and dz 2 )

The crystal field splitting of tetrahedral complexes is intrinsically smaller in


the octahedral field. This is because there are only two-thirds the number of ligands
and less direct influence on the d-orbital’s. The relative stabilizing effect of the e set
is -6 Dq and the destabilizing effect of the t2 set is +4 Dq .

Crystal field splitting in Square Planar Complexes

The square planar complex also has a coordination number of 4. The structure
of the complex differs from a tetrahedron because the ligands form simple squares
on the x and y axes. For this reason, the crystal field splitting is also different. Due
to the absence of ligands along the z-axis in square planar complexes, electron
repulsion in the dxz, dyz, and dz 2 orbitals is much less than in octahedral complexes
(dz 2 is a "donut" in x, on the y-axis). The dx 2 -y2 orbitals have the highest energy,
then he is followed by the dxy orbitals, followed by the remaining orbital tails (but
dz 2 is slightly more energetic than the dxz and dyz orbitals). This pattern of orbital
splitting remains constant for all geometries. Orbitals in direct contact with the
ligand field are higher in energy than orbitals that slip through the ligand field and
make more indirect contact with the ligand field. So if you're confused about which
geometry leads to which splitting, think about how the ligand field interacts with the
electron orbitals of the central atom.

CHE509: Inorganic Chemistry – II Page 38


For square planar complexes, Δ is almost always large, even in the presence
of weak field ligands. Since the pairing energy is usually much smaller than Δ,
electrons are more likely to be paired than unpaired. Therefore, square planar
complexes typically have lower spin.

SOLVED PROBLEMS 02
1] Explain the concept of high spin and low spin complexes

Solution:

Complexes with a large number of unpaired electrons are called high -spin
complexes, while low-spin complexes contain a less number of unpaired electrons.
High spin complexes are expected with weak field ligands while the crystal field
splitting energy is small Δ (Δ is energy difference between eg and t2g orbitals which
obtain after splitting). The opposite is true for low-spin complexes, where strong
electric field ligands induce maximal electron pairing in the set of three t 2 atomic
orbitals due to the large Δo.

Ligands that cause large Δ d orbital splitting are called high -field ligands,
such as CN - and CO in the spectrochemical series. In complexes with these ligands,
placing electrons in high-energy orbitals is unfavorable. Therefore, according to the
aufbau principle, the low-energy orbitals are completely filled before the occupation
of the upper set begins. Such complexes are called "low-spin". For example, NO 2- is
a strong field ligand and produces a large Δ. An octahedral ion [Fe(NO 2 ) 6 ] 3− with 5 d
electrons has the octahedral splitting diagram shown on the figure if all 5 electrons
are in the t 2 g level. Therefore, this low spin state does not obey Hund's rule.

CHE509: Inorganic Chemistry – II Page 39


Low spin

High spin
2] Explain the concept of octahedral splitting and tetrahedral splitting
Solution:
Crystal field splitting in Octahedral Complex
 For octahedral coordination compounds with six ligands surrounding a metal
atom/ion, repulsion between d-orbital electrons and ligand electrons is
observed.
 This repulsion is felt stronger for the dx2-y2 and dz2 orbitals, since they are
oriented with their axes aligned with the direction of the ligand.
 They therefore have energies higher than the average energy of the spherical
crystal field.
 On the other hand, the dxy, dyz, and dxz orbitals are less repulsive because
they are oriented between the axes.
 Therefore, these three orbitals have less energy than the average energy of the
spherical crystal field.

Crystal Field Splitting in Octahedral Complex

CHE509: Inorganic Chemistry – II Page 40


This degenerate level splitting in the presence of ligands is known as crystal
field splitting. The difference between the energy of t 2 g and the eg level is denoted
by 'Δo' (the index o represents the octahedron). Some ligan ds tend to generate strong
fields, causing large crystal field splittings, while some ligands tend to generate
weak fields, causing small crystal field splittings.
Crystal Field Splitting in Tetrahedral Complex
The splitting of the pentafold degenerate metal ion d orbitals into two levels
in the tetrahedral crystal field is the two sets of orbitals denoted as Td. Electrons in
the dx 2 -y2 and dz 2 orbitals are less repelled by the ligand than electrons in the dxy,
dyz, and dxz orbitals. As a result, the dxy, dyz, and dxz orbital sets increase in
energy and the dx 2 -y2 and dz 2 orbitals sets decrease in energy.
 Since there are only four ligands in the Td complex, the total negative charge
of four ligands, and thus the ligand field, is smaller than for six ligands.
 The orientation of the orbital does not match the direction in which the ligand
approaches the metal ion.

Therefore, the repulsion of tetrahedral coordination compounds gives rise to


two energy levels.
 t 2 - set of three high energy orbitals (dxy, dyz, and dxz)
 e - set of two low energy orbitals (dx 2 -y2 and dz 2 )
The crystal field splitting of tetrahedral complexes is intrinsically smaller in
the octahedral field. This is because there are only two-thirds the number of ligands
and less direct influence on the d-orbitals. The relative stabilizing effect of the e set
is -6 Dq and the destabilizing effect of the t2 set is +4 Dq .
3] Write a note on crystal field splitting in square planer complexes
Solution: Crystal field splitting in Square Planar Complexes

CHE509: Inorganic Chemistry – II Page 41


The square planar complex also has a coordination number of 4. The structure
of the complex differs from a tetrahedron because the ligands form simple squares
on the x and y axes. For this reason, the crystal field splitting is also different. Due
to the absence of ligands along the z-axis in square planar complexes, electron
repulsion in the dxz, dyz, and dz 2 orbitals is much less than in octahedral complexes
(dz 2 is a "donut" in x, on the y-axis). The dx 2 -y2 orbitals have the highest energy,
then he is followed by the dxy orbitals, followed by the remaining orbital tails (but
dz 2 is slightly more energetic than the dxz and dyz orbitals). This pattern of orbital
splitting remains constant for all geometries. Orbitals in direct contact with the
ligand field are higher in energy than orbitals that slip through the ligand field and
make more indirect contact with the ligand field. So if you're co nfused about which
geometry leads to which splitting, think about how the ligand field interacts with the
electron orbitals of the central atom.

For square planar complexes, Δ is almost always large, even in the presence
of weak field ligands. Since the pairing energy is usually much smaller than Δ,
electrons are more likely to be paired than unpaired. Therefore, square planar
complexes typically have lower spin.

SHORT ANSWER QUESTIONS WITH MODEL ANSWER 02


1] Write a note on low spin complexes
Solution:
Complexes with a less number of unpaired electrons are called low-spin
complexes, Ligands that cause large Δ d orbital splitting are called high -field
ligands, such as CN - and CO in the spectrochemical series. In complexes with these
ligands, placing electrons in high-energy orbitals is unfavorable. Therefore,

CHE509: Inorganic Chemistry – II Page 42


according to the aufbau principle, the low-energy orbitals are completely filled
before the occupation of the upper set begins. Such complexes are called "low -spin".
For example, NO 2- is a strong field ligand and produces a large Δ.

2] Write short note on High spin complexes


Solution:
Complexes with a large number of unpaired electrons are called high-spin
complexes. E.g. following d 5 high spin complex configuration.

02-03: CRYSTAL FIELD STABILIZATION ENERGY (CFSE)


In chemical environments, energy levels are commonly shared, as determined
by the symmetry of the local fields surrounding metal ions. The energy difference
between eg and t 2 g levels is given as 10Dq. They state that each electron moving to a
lower t 2 g level stabilizes the system by an amount of -4Dq, while an electron moving
to an eg level destabilizes the system by +6Dq. That is, t 2 g is lowered by 4Dq and
the eg level is raised by +6Dq.
For example, the d5 and d10 systems have zero net energy change, as shown
below.
d 5 : 3(-4D q ) + 2(+6D q ) = -12D q + 12D q = 0
d 10 : 6(-4D q ) + 4(+6D q ) = -24D q + 24D q = 0
The decrease in energy due to energy level splitting is called the "ligand field
stabilization energy (LFSE)".
Therefore, the crystal field splitting depends on the fields generated by the
ligands and the charges of the metal ions. Experimentally determined series based on
the absorption of light by coordination compounds with different ligands have
become known as spectrochemical series. The spectrochemical series orders the
ligands in order of electric field strength as follows:

CHE509: Inorganic Chemistry – II Page 43


I– < Br – < Cl – < SCN – < F – < OH – < C 2 O 4 2- < H 2 O < NCS – < EDTA 4- < NH 3 <
en < CN – < CO
The filling of d orbitals is done in the following way. The first three electrons
of her are placed in the t2g level according to Hund's rule. The fourth electron can
enter the t2g level, giving the configuration t2g4eg0, or enter the eg orbital, giving
the configuration t2g3eg1. It depends on two parameters: the crystal field splitting
magnitude Δo and the pairing energy P. Two possible cases can be explained as
follows.
Δo > P - the electron enters the t 2 g level and assumes the t 2 g 4 eg 0
configuration. Ligands that produce this configuration are known as high-field
ligands and form low-spin complexes.
Δo < P - the electron enters the eg plane and has the configuration t 2 g3 eg 1 .
Ligands that produce this configuration are known as weak-field ligands and form
high-spin complexes.
Calculation of CFSE in octahedral field
The splitting of d orbitals into different energy levels in transition metal
complexes has important consequences for their stability, reactivity, and magnetic
properties. Let us first consider the simple case of th e octahedral complex
[M(H 2 O) 6 ] 3+ where M = Ti, V, Cr. Since the complexes are octahedral, they all have
the same energy level diagram.

CSFE for Octahedral complex =


[(no. of electrons in t2g X −0.4) + (no. of electrons in eg X 0.6)]
E.g. CFSE for the complexes [V (H 2 O) 6 ] 3+ is
In [V(H 2 O) 6 ] 3+ , the ground state V has the 3d 2 ,4s 0 system, Since this is a
low-spin complex with the H 2 Oas neutral ligand, two electrons are present in the the
t 2 g orbital’s and there are 0 electrons in orbital eg applying the formula,

CHE509: Inorganic Chemistry – II Page 44


High spin V +3 Configuration
CFSE for Oh = [( t2g X −0.4 ) + ( eg X 0.6) ]
= 2(−0.4)+0(0.6)
= −0.8 Δ 0
Calculation of CFSE in tetrahedral field
The resolution of tetrahedral complexes is exactly the opposite of that of
octahedral complexes. Note that in the octahedral complex, the dz 2 and dx 2 -y2
orbitals have higher energies than the dxz, dxy, and dyz orbitals. The opposite
happens in the tetrahedral complex, as the dxz, dxy, and dyz orbitals have higher
energies than the dz 2 and dx 2 -y2 orbitals. Again, this is because the contacts between
ligands and orbitals are opposite to those in octahedral complexes. In contrast to
octahedral complexes, the ligands of tetrahedral complexes make direct contacts
with the dxz, dxy, and dyz orbitals. Therefore, due to the direct contact, these
orbitals have high electron-electron repulsion and high energy. The dz 2 and dx 2 -y2
orbitals are not in direct contact because ligands can crush or slide these orbitals,
reducing the electron-electron repulsion and orbital energy.

CFSE for Th = [(t2g X −0.6) + ( eg X 0.4) ]

CHE509: Inorganic Chemistry – II Page 45


Limitations of Crystal Field Theory

 This theory only considers the d orbitals of the central atom. The s and p
trajectories were not considered in this study.

 This theory cannot explain the behavior of certain metals that exhibit high
fission and metals that exhibit little fission. For example, theory cannot
explain why H 2 O is a stronger ligand than OH – .

 This theory excludes the possibility of p-bonding. This is a significant


drawback, as it is found in many complexes.

 The trajectories of the ligands are theoretically irrelevant. As a result, the


properties of ligand orbitals and their interactions with metal orbitals cannot
be explained.

Advantages of Crystal Field Theory


 This theory can be used to explain the stability of the complex. The higher the
crystal field splitting energy, the higher the stability.
 The color and spectrum of the complex can be explained using this theory.
 This theory explains the magnetic properties of the complex.
SOLVED PROBLEMS 03
1] Explain the crystal field stabilization energy with factores affecting on it.
Solution:

CHE509: Inorganic Chemistry – II Page 46


In chemical environments, energy levels are commonly shared, as determined
by the symmetry of the local fields surrounding metal ions. The energy difference
between eg and t 2 g levels is given as 10Dq. They state that each electron moving to a
lower t 2 g level stabilizes the system by an amount of -4Dq, while an electron moving
to an eg level destabilizes the system by +6Dq. That is, t 2 g is lowered by 4Dq and
the eg level is raised by +6Dq.
For example, the d5 and d10 systems have zero net energy change, as shown
below.
d 5 : 3(-4D q ) + 2(+6D q ) = -12D q + 12D q = 0
d 10 : 6(-4D q ) + 4(+6D q ) = -24D q + 24D q = 0
The decrease in energy due to energy level splitting is called the "ligand field
stabilization energy (LFSE)".
Therefore, the crystal field splitting depends on the fields generated by the
ligands and the charges of the metal ions. Experimentally determined series based on
the absorption of light by coordination compounds with different ligands have
become known as spectrochemical series. The spectrochemical series orders the
ligands in order of electric field strength as follows:
I– < Br – < Cl – < SCN – < F – < OH – < C 2 O 4 2- < H 2 O < NCS – < EDTA 4- < NH 3 <
en < CN – < CO
The filling of d orbitals is done in the following way. The first three electrons
of her are placed in the t2g level according to Hund's rule. The fourth electron can
enter the t2g level, giving the configuration t2g4eg0, or enter the eg orbital, giving
the configuration t2g3eg1. It depends on two parameters: the crystal field splitting
magnitude Δo and the pairing energy P. Two possible cases can be explained as
follows.
Δo > P - the electron enters the t 2 g level and assumes the t 2 g 4 eg 0
configuration. Ligands that produce this configuration are known as high -field
ligands and form low-spin complexes.
Δo < P - the electron enters the eg plane and has the configuration t 2 g3 eg 1 .
Ligands that produce this configuration are known as weak-field ligands and form
high-spin complexes.
2] How to calculate CFSE in Octahedral and Tetrahedral field explain with examples
Solution:

CHE509: Inorganic Chemistry – II Page 47


The splitting of d orbitals into different energy levels in transition metal
complexes has important consequences for their stability, reactivity, and magnetic
properties. Let us first consider the simple case of the octahedral complex
[M(H 2 O) 6 ] 3+ where M = Ti, V, Cr. Since the complexes are octahedral, they all have
the same energy level diagram.

CSFE for Octahedral complex =


[(no. of electrons in t2g X −0.4) + (no. of electrons in eg X 0.6)]
E.g. CFSE for the complexes [V (H 2 O) 6 ] 3+ is
In [V(H 2 O) 6 ] 3+ , the ground state V has the 3d 2 ,4s 0 system, Since this is a
low-spin complex with the H 2 O as neutral ligand, two electrons are present in the the
t 2 g orbital’s and there are 0 electrons in orbital eg applying the formula,

High spin V +3 Configuration

CFSE for Oh = [ ( t2g X −0.4 ) + ( eg X 0.6) ]

= 2(−0.4)+0(0.6)

= −0.8 Δ 0
Calculation of CFSE in tetrahedral field
The resolution of tetrahedral complexes is exactly the opposite of that of
octahedral complexes. Note that in the octahedral complex, the dz 2 and dx 2 -y2
orbitals have higher energies than the dxz, dxy, and dyz orbital’s. The opposite
happens in the tetrahedral complex, as the dxz, dxy, and dyz orbitals have higher
energies than the dz 2 and dx 2 -y2 orbital’s. Again, this is because the contacts
between ligands and orbital’s are opposite to those in octahedral complexes. In
contrast to octahedral complexes, the ligands of tetrahedral complexes make direct

CHE509: Inorganic Chemistry – II Page 48


contacts with the dxz, dxy, and dyz orbital’s. Therefore, due to the direct contact,
these orbital’s have high electron-electron repulsion and high energy. The dz 2 and
dx 2 -y2 orbital’s are not in direct contact because ligands can crush or slide these
orbitals, reducing the electron-electron repulsion and orbital energy.

CFSE for Th = [ ( t2g X −0.6 ) + ( eg X 0.4) ]


For E.g. complex having metal with d6 valence configuration

SHORT ANSWER QUESTIONS WITH MODEL ANSWER 03


1] Calculate CFSE for following complex
[CoF 6 ] 3−
Solution:
[CoF 6 ] 3− , cobalt has the 3d 7 ,4s 2 system in the ground state, but in the excited state it loses
three electrons in the formation of ions, and 4s to 2 lose one electron. And one from the 3D
orbit, so Cobalt gets the 3D placement 6 . Now, because of the high spin, 4 electrons enter
orbitals t 2 g and 2 electrons enter orbitals eg . Δ in the formula = no. Number of electrons in
t 2g ⋅ (−0.4) + no. 0
Δ = no. of electrons in t 2 g ⋅ (−0.4)+ no. of electrons in e g (0.6)
= 4(−0.4) +2(0.6)

CHE509: Inorganic Chemistry – II Page 49


=−1.6+1.2
= −0.4 Δ 0
2] Calculate CFSE of the following complex: [Fe (CN) 6 ] 4−
Solution:
Tip: To find the CFSE, we need to fill the t 2 g and zg orbitals according to CFT (Crystal
Field Theory). In the complex [Fe (CN) 6 ] 4− , the iron ( Fe ) is in the +2 oxidation state,
hence the 3d 6 system. Since CN is a strong electric field ligand, the complex becomes a
low-spin complex, with all six electrons buried in the t 2 g orbitals.

Iron (Fe) has an electron configuration of 3d 6 4s 2 in the ground state. However, in the
[Fe(CN) 6 ] 4− complex, the iron is in the +2 oxidation state. So the configuration will be
3d 6 4s 0 . Also, since iron has a coordination number of 6, the complex has an octahedral
geometry.

According to CFT (Crystal Field Theory), the five degenerate d orbitals split into three t 2 g
and two eg orbitals in the presence of ligands. This splitting of degenerate levels due to the
presence of ligands in a particular geometry is called crystal field splitting, and the energy
splitting is denoted by Δo. So here we have the formula for the crystal field splitting
energy, CFSE, which is

Δo= number of electrons in t 2 g× (-0.4Δo) + number of electrons eg×(0.6Δo)

Therefore, Δo=6×(−0.4Δo)+0×(0.6Δo)

=−2.4Δo

02-04: SPECTROCHEMICAL SERIES


Another factor that plays an important role in whether a transition metal
complex is high or low spin is the nature of the ligand. The energy splitting of the d
orbitals is affected by how strongly the ligand interacts with the metal. Weakly
interacting ligands produce small changes in d-orbital energy levels, whereas
strongly interacting ligands produce large changes in d-orbital energy levels. A
spectrochemical series is a list of ligands based on their strength of interaction with
the metal ion. They are often listed in order from weakest to strongest, as follows:
I− < Br− < S2− < SCN− < Cl− < NO3− < N3− < F− < OH− < C2O42− < H2O < NCS− < CH3CN <
py < NH3< en < bipy < phen < NO2− < PPh3 < CN− < CO
Some of the trends seen in the spectrochemical series are attributed to pi -
donating and pi-accepting effects of the ligands. Ligands with additional lone pairs
(other than those sigma donors) are π donors. The π-donor raises an otherwise
unbonded t2g orbital as the lone pair on the ligand forms a π-bond with the metal.

CHE509: Inorganic Chemistry – II Page 50


The t2g orbital and the ligand lone pair orbital form two new orbitals. The
antibonding orbitals are energetically close to the higher energy d orbitals. The bond
orbitals are energetically close to the low-energy ligand orbitals.
 A π-donation increases the energy of the t 2 g set of d orbitals.
 This results in smaller d-orbital splitting.
 Also, as a result, complexes with π-donations are slightly less stable than
those without π-donations.
On the other hand, a ligand whose donor atom is already π-bonded to another
atom can accept π-donation from the metal. This is done by donating an electron pair
from the metal's t 2 g orbital to the ligand's π* orbital. π∗ is an anti-bonding orbital, so
in this case it has higher energy than the metal's d orbitals (or t2g orbitals). The
resulting bonding orbital more closely resembles the lower-energy metallic orbital,
while the resulting anti-bonding orbital more closely resembles the higher-energy π*
orbital of the ligand.
 π-acceptor ligands lower the energy of the t 2 g set of d-orbitals.
 This increases the d-orbital splitting.
 Furthermore, complexes containing π-acceptor ligands are slightly more
stable than complexes without π-acceptor ligands.
The spectrochemical series gets its name from the band shift in the UV-Vis
spectrum when comparing two similar complexes with two different ligands. The
effect of ligands on d-orbital splitting affects the wavelength of light associated with
electronic transitions from d-orbitals (filled) to d-orbitals (empty). This transition is
actually not related to major absorption. However, since they often occur in the
visible spectrum, they are often associated with colored transition metal complexes.
Strong-field ligands and weak-field ligands.
The spectrochemical series ranks the ligands according to the energy difference
Δ O between the t 2 g and eg orbitals of the octahedral complexes. This energy
difference is measured at the spectral transition between these levels. This is often in
the visible part of the spectrum and responsible for the color of complexes with
partially filled d orbitals. Ligands that produce large splits are called strong -field
ligands, and those that produce small splits are called weak-field ligands.
Why is CFT superior over VBT?

CHE509: Inorganic Chemistry – II Page 51


Orbital mixing during bond formation is explained by valence bond theory
(VBT). The explanation was mainly based on the concept of hybridization. Crystal
field theory explains how orbitals split when ligands approach metal ions, but VBT
could not adequately explain the magnetic behavior. The formation of outer and
inner orbital complexes could not be explained. CFT, on the other hand, explained
everything.

SOLVED PROBLEMS 04
1] Write a note on Spectrochemical series
Solution:
Another factor that plays an important role in whether a transition metal
complex is high or low spin is the nature of the ligand. The energy splitting of the d
orbitals is affected by how strongly the ligand interacts with the metal. Weakly
interacting ligands produce small changes in d-orbital energy levels, whereas
strongly interacting ligands produce large changes in d-orbital energy levels. A
spectrochemical series is a list of ligands based on their strength of interaction with
the metal ion. They are often listed in order from weakest to strongest, as follows:
I− < Br− < S2− < SCN− < Cl− < NO3− < N3− < F− < OH− < C2O42− < H2O < NCS− < CH3CN <
py < NH3< en < bipy < phen < NO2− < PPh3 < CN− < CO
Some of the trends seen in the spectrochemical series are attributed to pi-
donating and pi-accepting effects of the ligands. Ligands with additional lone pairs
(other than those sigma donors) are π donors. The π-donor raises an otherwise
unbonded t 2 g orbital as the lone pair on the ligand forms a π-bond with the metal.
The t 2 g orbital and the ligand lone pair orbital form two new orbitals. The
antibonding orbitals are energetically close to the higher energy d orbitals. The bond
orbitals are energetically close to the low-energy ligand orbitals.
 A π-donation increases the energy of the t 2 g set of d orbitals.
 This results in smaller d-orbital splitting.
 Also, as a result, complexes with π-donations are slightly less stable than
those without π-donations.
On the other hand, a ligand whose donor atom is already π-bonded to another
atom can accept π-donation from the metal. This is done by donating an electron pair
from the metal's t 2 g orbital to the ligand's π* orbital. π∗ is an anti-bonding orbital, so
in this case it has higher energy than the metal's d orbitals (or t2g orbitals). The
resulting bonding orbital more closely resembles the lower-energy metallic orbital,

CHE509: Inorganic Chemistry – II Page 52


while the resulting anti-bonding orbital more closely resembles the higher-energy π*
orbital of the ligand.
 π-acceptor ligands lower the energy of the t2g set of d-orbitals.
 This increases the d-orbital splitting.
 Furthermore, complexes containing π-acceptor ligands are slightly more
stable than complexes without π-acceptor ligands.
The spectrochemical series gets its name from the band shift in the UV-Vis
spectrum when comparing two similar complexes with two different ligands. The
effect of ligands on d-orbital splitting affects the wavelength of light associated with
electronic transitions from d-orbitals (filled) to d-orbitals (empty). This transition is
actually not related to major absorption. However, since they often occur in the
visible spectrum, they are often associated with colored transition metal complexes.

SHORT ANSWER QUESTIONS WITH MODEL ANSWER 04


1] Explain the concept of strong field and weak field legands

Solution:

The spectrochemical series ranks the ligands according to the energy difference
Δ O between the t 2 g and eg orbitals of the octahedral complexes. This energy
difference is measured at the spectral transition between these levels. This is often in
the visible part of the spectrum and responsible for the color of complexes with
partially filled d orbitals. Ligands that produce large splits are called strong -field
ligands, and those that produce small splits are called weak-field ligands.
2] Why is CFT superior over VBT?
Solution:
Orbital mixing during bond formation is explained by valence bond theory
(VBT). The explanation was mainly based on the concept of hybridization. Crystal
field theory explains how orbitals split when ligands approach metal ions, but VBT
could not adequately explain the magnetic behavior. The formation of outer and
inner orbital complexes could not be explained. CFT, on the other hand, explained
everything.
CHECK POINT 01-02
1] The series with correct order of legands in increasing Δo in their complexes is-
a. I - < PR 3 < CH 3 - < CO
b. PR 3 < CH 3 - < I - < CO

CHE509: Inorganic Chemistry – II Page 53


c. CH 3 - <PR 3 <I - < CO
d. I - <CH 3 - <PR 3 < CO
Solution: d (According to the sequence in spectrochemical series)
2] Which of the following ligands is most likely to form high spin complexes in
octahedral fields?
a. Cl -
b. OH -
c. C 2 O 4 -
d. CN -
Solution: a (Cl - is the weakest electric field ligand among the given options. The splitting
field Δo is small and the energy gap between eg orbitals and t2g orbitals is narrow.
Therefore, electrons tend to occupy eg orbitals, and half -filled orbitals tend to increase.)
3] Tetrahedral fields can easily form low spin complexes.
a. True
b. False
Solution: b (The splitting energy of the tetrahedral field is almost half that of the
octahedral field. Because of this, the orbital splitting energy is not very large, making
electron pairing difficult and rarely forming low-spin complexes.)
4] For a high spin d4 octahedral complex the crystal field splitting energy will be
a. -1.6 Δo
b. -0.8 Δo
c. -0.6 Δo
d. -1.2 Δo
Solution: c
We have d 4 in octahedral field
Octahedral d orbital is split as follows

CFSE For Octahedral = [No of t 2 g electrons (0.6) + No of eg electrons (-0.4)]Δo


= [1 (0.6) + 3 (-0.4)] Δo
= [(0.6) + (-1.2)] Δo
= [-0.6] Δo

CHE509: Inorganic Chemistry – II Page 54


SUMMARY
Several theories have been proposed to explain the nature of coordination compound
bonding. Valence bond theory was developed to explain chemical bonds using quantum
physics. This theory is mainly concerned with the formation of single bonds from the
atomic orbitals of atoms involved in the formation of molecules. Crystal field theory is one
of the most widely accepted theories describing bonding in coordination complexes. We
treat atoms as hard spheres and their interactions are purely electrostatic. The c entral metal
atom is positively charged and the surrounding ligands are negatively charged. When this
negatively charged ion approaches a positively charged ion, the electrostatic attraction
changes the energy level of the metal ion, eventually forming a b ond. Crystal field theory
can explain the coupling of most complexes. However, there were some limitations. This
chapter also discusses the limitations of crystal field theory.

KEY WORDS
VBT, CFT, CFSE, Splitting of atomic orbitals, Strong field, Weak field
REFERENCES

MOOCS
--
YOUTUBE VIDEOS
1) https://www.youtube.com/watch?v=8B__xDUKqbM
2) https://www.youtube.com/watch?v=z9CmSAV1wcY
3) https://www.youtube.com/watch?v=c2O7hJwaYoU
4) https://www.youtube.com/watch?v=p27zVpmEtkk
5) https://www.youtube.com/watch?v=TOckIJ-2FZc
6) https://www.youtube.com/watch?v=Llf_G8dMS0k

WIKIPEDIA
1) https://en.wikipedia.org/wiki/Valence_bond_theory#:~:text=Valence%20bond%
20theory%20considers%20that,coupled%20orbitals%20(small%20overlap) .
2) https://en.wikipedia.org/wiki/Crystal_field_theory
3) https://en.wikipedia.org/wiki/Crystal_field_theory#:~:text=The%20crystal%20f
ield%20stabilization%20energy,by%20a%20set%20of%20ligands .

OER
1) https://byjus.com/chemistry/valence-bond-theory/
2) https://chem.libretexts.org/Bookshelves/Physical_an d_Theoretical_Chemistry_T
extbook_Maps/Supplemental_Modules_(Physical_and_Theoretical_Chemistry)/
Chemical_Bonding/Valence_Bond_Theory

CHE509: Inorganic Chemistry – II Page 55


3) https://chemed.chem.purdue.edu/genchem/topicreview/bp/ch12/crystal.php
4) https://www.embibe.com/exams/crystal-field-theory/
REFERENCE BOOKS
1) Principles of Inorganic Chemistry 2020 by B.R. Puri, L.R. Sharma, K.C. Kalia
2) Coordination chemistry textbook binding , 2016 by Ajai K umar
3) Concise Coordination Chemistry Paperback – 1 January 2008 by R. Gopalan , and V.
Ramalingam
4) Coordination Chemistry Paperback – 1 January 2022 by Kamlodvab Jha

CHE509: Inorganic Chemistry – II Page 56


CREDIT 01-UNIT 03: M OLECULAR ORBITAL THEORY

LEARNING OBJECTIVES

After successful completion of this unit, you will be able to


 Understand molecular orbitals and interactions of them
 Understand why molecules are formed
 Understand the concept of bonding in atoms
 Apply the concept of MOT to understand properties of molecules
 Understand the nature and reactivity of molecules

INTRODUCTION

Robert Mulliken, Chicago 1929


Molecular orbital theory (often abbreviated to MOT) is a theory of chemical
bonding developed by F. Hund and R.S. Mulliken in the early 20th century to
explain the structure and properties of various molecules. Valence bond theory fails
to adequately explain how certain molecules contain two or more equivalent bonds
whose bond order is between single and double bonds, such as th ose in resonance-
stabilized molecules. This is where molecular orbital theory proved to be stronger
than valence bond theory (because the orbitals described by MOT reflect the shape
of the molecule to which it is applied).
03-01: FEATURES OF MOLECULAR ORBITAL THEORY (MOT) FOR METAL
COMPLEXES

Crystal field theory fails to account for many physical properties of transition metal
complexes, as the interactions between metal orbitals and ligand orbitals are not

CHE509: Inorganic Chemistry – II Page 57


considered. Molecular orbital theory is very well applied to transition metal complexes and
can explain both covalent and ionic nature in metal -ligand bonds. Transition metal ions
have nine valence atomic orbitals, consisting of five nd, three (n+1)p, and one (n+1)s
orbitals. These orbitals have the proper energy to form binding interactions with the ligand.
Molecular orbital theory relies heavily on the geometry of the complex and can be used to
successfully describe octahedral, tetrahedral and square planar complexes. The main
features of the molecular orbital theory of metal complexes are:
 Atomic orbitals at the metal center and surrounding ligands combine to form
new orbitals known as molecular orbitals.
 The number of molecular orbitals formed is the same as the numb er of
bonded atomic orbitals.
 Additive overlap leads to bonding molecular orbitals and subtractive overlap
leads to antibonding overlaps.
 The energy of the bonding molecular orbitals is lower than the energy of the
non-bonding molecular orbitals, and the energy of the anti -bonding
molecular orbitals is higher than the energy of the non-bonding orbitals.
 The energy of non-bonding orbitals remains the same.
 The ionic character of covalent bonds arises from the energy difference
between the bonding orbitals.
 If the molecular orbitals are comparable in energy to the atomic orbitals,
they are essentially not much different from the atomic orbitals.

The polarity of a bond can be explained by the superposition of two atomic orbitals
of different energies. Supposing φA and φB are the two atomic orbitals of atoms A and B
respectively, these two atomic orbitals each have one electron and together form the
bonding (σ) and antibonding (σ*) molecular orbitals form the After molecular orbital
formation, both electrons occupy σ orbitals. Here, when the energy of the σ orbital
approaches ϕA, it has more ϕA nature, so the electron densities of both electrons are more
concentrated in atom A than in B. Similarly, when the energy of the σ orbital approaches
ϕB, it has more ϕB features, and the electron densities of both electrons are more
concentrated in atom B than in A. The same explanation applies to ionic features in metal -
ligand bonds. The main concern here is that the total number of molecular orbitals formed
by bond-antibond interactions is very high. Therefore, understanding symmetry energies
can lead to many complexities, all of which require a very sophisticated treatment of
chemical bonds. Therefore, without extensive knowledge of chemical applications of group
theory, it is very difficult to explain the whole concept. Nevertheless, we can give a
primitive explanation for σ-bonds in transition metal complexes of different geometries.

CHE509: Inorganic Chemistry – II Page 58


SOLVED PROBLEMS 01
1] Explain the important features of molecular orbital theory.
Solution: Crystal field theory fails to account for many physical properties of
transition metal complexes, as the interactions between metal orbitals and ligand orbitals
are not considered. Molecular orbital theory is very well applied to transition metal
complexes and can explain both covalent and ionic nature in metal-ligand bonds. Transition
metal ions have nine valence atomic orbitals, consisting of five nd, three (n+1)p, and one
(n+1)s orbitals. These orbitals have the proper energy to form binding interactions with the
ligand. Molecular orbital theory relies heavily on the geometry of the complex and can be
used to successfully describe octahedral, tetrahedral and square planar complexes. The main
features of the molecular orbital theory of metal complexes are:
 Atomic orbitals at the metal center and surrounding ligands combine to form
new orbitals known as molecular orbitals.
 The number of molecular orbitals formed is the same as the numb er of
bonded atomic orbitals.
 Additive overlap leads to bonding molecular orbitals and subtra ctive overlap
leads to antibonding overlaps.
 The energy of the bonding molecular orbitals is lower than the energy of the
non-bonding molecular orbitals, and the energy of the anti -bonding
molecular orbitals is higher than the energy of the non-bonding orbitals.
 The energy of non-bonding orbitals remains the same.
 The ionic character of covalent bonds arises from the energy difference
between the bonding orbitals.
 If the molecular orbitals are comparable in energy to the atomic orbitals,
they are essentially not much different from the atomic orbitals.

SHORT ANSWER QUESTIONS WITH MODEL ANSWER 01


1] Give any two important features of molecular orbital theory.
Solution:
 Atomic orbitals at the metal center and surrounding ligands combine to form
new orbitals known as molecular orbitals.
 The number of molecular orbitals formed is the same as the numb er of
bonded atomic orbitals.
2] Why MOT is more informative than CFT?
Solution: Crystal field theory fails to account for many physical properties of transition
metal complexes, as the interactions between metal orbitals and ligand orbitals are not

CHE509: Inorganic Chemistry – II Page 59


considered. Molecular orbital theory is very well applied to transition metal complexes and
can explain both covalent and ionic nature in metal -ligand bonds. Transition metal ions
have nine valence atomic orbitals, consisting of five nd, three (n+1)p, and one (n+1)s
orbitals.
03-02: MOT FOR OCTAHEDRAL COMPLEXES
In octahedral complexes, the molecular orbitals generated by coordination of the
metal center can be viewed as the result of the donation of two electrons to the metal d -
orbitals by each of the six σ-donor ligands. The metal orbitals involved in this type of bond
are nd, (n+1)p, and (n+1)s. Note that only dz 2 and dx 2 -y 2 orbitals can participate in the σ
overlap, not all nd orbitals. The dxy, dxz, and dyz orbitals remain non -bonding orbitals. The
ligand approaches the metal center along the x, y, and z axes, and its σ sym metry orbitals
forms bonding and antibondingcombines with s, px, py, pz, pz 2 and px 2 -y 2 orbitals of the
metal. By the combination of A total of 6 bonding and 6 antibonding molecular orbitals
were formed. The symmetry designations for the various metal orbit als involved in
octahedral overlap are:
Metalorbitals Symmetrydesignation
2 2 2
dz and dx -y eg
s a1g
px, py, pz t1u
d xy d xz d yz t2g

The following exact properties of Mulliken symbols, B., a1g, etc., can only be
understood at a core level of group theory. However, we can say that a, e, and t represent
the 1-fold, 2-fold, and 3-fold degenerate orbitals, respectively. The subscripts g and u
denote the even and odd nature of the orbitals. Orbitals are even if they all have the same
sign in opposite directions from the center. These also need a center of symmetry. On the
other hand, an orbital is said to be odd if it has opposite signs all in opposite directions
from the center. These must also lack a center of symmetry.
The symmetry-matched linear combination of symmetry-matched atomic orbitals for
direct metal-ligand overlap can be obtained simply by solving a reduced expression based
on the bond vector along the axis of σ overlap. A metal -ligand σ bond has a total of 6 bond
vectors, resulting in a 6-dimensional reduced representation. Because the six ligands of the
octahedral complex approach the central metal ion along three Cartesian axes (two along
the x-axis, two along the y-axis, and two along the z-axis) , the six σ-based coupling
vectors can also be chosen along the x-, y-, and z-axes.

CHE509: Inorganic Chemistry – II Page 60


SOLVED PROBLEMS 02
1] Describe the molecular orbital theory for octahedral complex

Solution: In octahedral complexes, the molecular orbitals generated by coordination of the


metal center can be viewed as the result of t he donation of two electrons to the metal d-
orbitals by each of the six σ-donor ligands. The metal orbitals involved in this type of bond
are nd, (n+1)p, and (n+1)s. Note that only dz 2 and dx 2 -y 2 orbitals can participate in the σ
overlap, not all nd orbitals. The dxy, dxz, and dyz orbitals remain non -bonding orbitals. The
ligand approaches the metal center along the x, y, and z axes, and its σ symmetry orbitals
forms bonding and antibondingcombines with s, px, py, pz, pz 2 and px 2 -y 2 orbitals of the
metal. By the combination of A total of 6 bonding and 6 antibonding molecular orbitals
were formed. The symmetry designations for the various metal orbitals involved in
octahedral overlap are:

Metalorbitals Symmetrydesignation
dz 2 and dx 2 -y 2 eg
s a1g
px, py, pz t1u
d xy d xz d yz t2g
The following exact properties of Mulliken symbols, B., a1g, etc., can only be
understood at a core level of group theory. However, we can say that a, e, and t represent
the 1-fold, 2-fold, and 3-fold degenerate orbitals, respectively. The subscripts g and u
denote the even and odd nature of the orbitals. Orbitals are even if they all have the same
sign in opposite directions from the center. These also need a center of symmetry. On the
other hand, an orbital is said to be odd if it has opposite signs all in opposite directions
from the center. These must also lack a center of symmetry.
The symmetry-matched linear combination of symmetry-matched atomic orbitals for
direct metal-ligand overlap can be obtained simply by solving a reduced e xpression based
on the bond vector along the axis of σ overlap. A metal -ligand σ bond has a total of 6 bond

CHE509: Inorganic Chemistry – II Page 61


vectors, resulting in a 6-dimensional reduced representation. Because the six ligands of the
octahedral complex approach the central metal ion along three Cartesian axes (two along
the x-axis, two along the y-axis, and two along the z-axis) , the six σ-based coupling
vectors can also be chosen along the x -, y-, and z-axes.

SHORT ANSWER QUESTIONS WITH MODEL ANSWER 02


1] Explain the formation of molecular orbitals from atomic orbitals
Solution:
In octahedral complexes, the molecular orbitals generated by coordination of the metal
center can be viewed as the result of the donation of two electrons to the metal d -orbitals by
each of the six σ-donor ligands. The metal orbitals involved in this type of bond are nd,
(n+1)p, and (n+1)s. Note that only dz 2 and dx 2 -y 2 orbitals can participate in the σ overlap,
not all nd orbitals. The dxy, dxz, and dyz orbitals remain non -bonding orbitals. The ligand
approaches the metal center along the x, y, and z axes, and its σ symmetry orbitals forms
bonding and antibondingcombines with s, px, py, pz, pz 2 and px 2 -y 2 orbitals of the metal
03-03: COMBINATION OF ATOMIC ORBITAL’S IN OCTAHEDRAL COMPLEX ACCORDING TO
MOT
A pictorial representation of the various symmetry -matched linear combination
orbitals of octahedral complexes that can overlap metal orbitals with σ (plane symmetry)
can be specified as follows:
1. If the six atomic orbitals of the six ligands point their positive lobes towards the
s orbitals of the metal, then the resultant orbitals are
φ 1 =σ 1 +σ 2 +σ3 +σ 4 + σ5 +σ 6
Here positive sign (addition) because of central metal orbitals and combining
legands orbitals are of same sign (both are having positive sign)

CHE509: Inorganic Chemistry – II Page 62


A combination of ligand orbitals that overlap with the metal -centered s-orbitals.
2. The px orbitals of the metal center can overlap with the atomic orbitals of
the ligand, making them closer together along the x axis. So the compound
trajectory looks like this:
φ2 = σ1 – σ2
Here negative sign is because of central metal orbitals and combining legands
orbitals are of apposite sign

Combinations of ligand orbitals that overlap metal-centered s-orbitals


3. The py orbital of the metal center may overlap with the atomic orbital of
the approaching ligand along the y-axis. So the compound trajectory looks
like this:
φ 3 = σ 3 –σ 4

CHE509: Inorganic Chemistry – II Page 63


A combination of ligand orbitals that overlap with the py orbitals of the central metal.

4. The pz orbital of the metal center may overlap with the atomic orbital of the
approaching ligand along the z-axis. So the compound trajectory looks like this:

φ 4 = σ 5 –σ 6

A combination of ligand orbitals overlapping the pz orbitals of the central metal.


5. The d x 2- y 2 orbitals of the metal center can overlap with the atomic orbitals of
approaching ligands along the x and y axes. So the compound trajectory looks
like this:
φ 5 = σ 1 +σ 2 –σ 3 –σ 4

CHE509: Inorganic Chemistry – II Page 64


Combinations of ligand orbitals that overlap the dx2 y2 orbitals of the metal
center
6. The dz2 orbitals of the metal center can overlap with the atomic orbitals of the
ligands, making them close along the x, y, and z axes. So the compound
trajectory looks like this:
φ 6 = σ 5 +σ 6 –σ 1 –σ 2 –σ 3 –σ 4

A combination of ligand orbital’s that overlap with the dz 2 orbital’s of the central metal.
SOLVED PROBLEMS 03
1] Describe the combination of atomic orbital’s Px, Py and Pz of legands with S and Py
Orbital’s of metal in octahedral complex
Solution:
Solution: If the six atomic orbital’s of the six ligands point their positive lobes towards the
s orbital’s of the metal, then the resultant orbitals are

CHE509: Inorganic Chemistry – II Page 65


φ 1 =σ 1 +σ 2 +σ3 +σ 4 + σ5 +σ 6
Here positive sign (addition) because of central metal orbitals and combining
legands orbitals are of same sign (both are having positive sign)

A combination of ligand orbitals that overlap with the metal -centered s-orbitals.
1. The px orbitals of the metal center can overlap with the atomic orbitals of
the ligand, making them closer together along the x axis. So the compound
trajectory looks like this:
φ2 = σ1 – σ2
Here negativesignis because of central metal orbitals and combining legands orbitals
are of apposite sign

Combinations of ligand orbitals that overlap metal-centered s-orbitals

CHE509: Inorganic Chemistry – II Page 66


SHORT ANSWER QUESTIONS WITH MODEL ANSWER 03
1] Explain in short signs in combination of atomic orbitals in wave equation
Solution: If the six atomic orbitals of the six ligands point their positive lobes towards the
s orbitals of the metal, then the resultant orbitals are
φ 1 =σ 1 +σ 2 +σ3 +σ 4 + σ5 +σ 6
Here positive sign (addition) because of central metal orbitals and combining
legands orbitals are of same sign (both are having positive sign)

03-04: MOLECULAR ORBITAL DIAGRAM FOR OCTAHEDRAL COMPLEX


The overall picture of molecular orbital energy levels for σ bonds in octahedral complexes
can be expressed as

Formation of σ molecular orbitals (bonding, antibonding, and nonbonding) in octahedral


complexes of transition metals.
Compound φ1 orbitals with a1g symmetry interact with a set of metal ions from the
1g orbitals to create bonding (a1g) and antibonding (a*) molecular orbitals. A composite
φ2, φ3, and φ4 orbital set with t1u symmetry interacts with the t1u orbital set of the metal
ion to produce triple degenerate bond (t1u) and triple degenerate anti bond (t*) molecular
orbital sets . A compound φ and φ orbital set of e -symmetry interacts with z. B. Combines
with the orbital set of metal ions to produce a doubly degenerate bond (eg) and a doubly
degenerate antibonding molecular orbital set (e*). The me tal-centered t-orbital set remains
essentially unbonded. The filling of electrons into different orbitals occurs according to the
composition principle, Hund's law, and Pauli's exclusion principle.
SOLVED PROBLEMS 04
1] Describe Molecular orbital diagram for octahedral complex and explain it

CHE509: Inorganic Chemistry – II Page 67


Solution: The overall picture of molecular orbital energy levels for σ bonds in octahedral
complexes can be expressed as

Formation of σ molecular orbitals (bonding, antibonding, and nonbonding) in octahedral


complexes of transition metals.
Compound φ1 orbitals with a1g symmetry interact with a set of metal ions from the
1g orbitals to create bonding (a1g) and antibonding (a*) molecular orbitals. A composite
φ2, φ3, and φ4 orbital set with t1u symmetry interacts with th e t1u orbital set of the metal
ion to produce triple degenerate bond (t1u) and triple degenerate antibond (t*) molecular
orbital sets . A compound φ and φ orbital set of e -symmetry interacts with z. B. Combines
with the orbital set of metal ions to produce a doubly degenerate bond (eg) and a doubly
degenerate antibonding molecular orbital set (e*). The metal -centered t-orbital set remains
essentially unbonded. The filling of electrons into different orbitals occurs according to the
composition principle, Hund's law, and Pauli's exclusion principle.
CHECK POINT 01-03
1] The number of molecular orbitals formed is the same as the numb er of bonded atomic
orbitals.
a) True
b) Fals
Solution: A
2] dz 2 and dx 2 -y 2 metal orbitals belong to …….. Symmetry designation.
a) eg
b) a1g
c) t1g

CHE509: Inorganic Chemistry – II Page 68


d) t2g
Solution: A
3] The energy of non-bonding orbitals are…...
a) Different
b) Zero
c) Highest
d) Same
Solution: d
4] The filling of electrons into different orbitals occurs according to …..
a) Pauli's exclusion principle
b) Newton’s law of motion
c) Hund's law
d) A and C
Solution: d
5] ψMO = ψA + ψB.
a) True
b) Fals
Solution: a, Linear combinations such as addition and subtraction of the wave functions of
individual atomic orbitals indicate the formation of molecules mathematically as given, i.e.
ψMO = ψA + ψB. Where ψ represents the wave functions of the atomic orbitals.
6] Which statement is false? A sigma molecular orbital ?
a) may result from overlap of p atomic orbitals perpendicular to the molecular axis
(side-on).
b) may result from overlap of p atomic orbitals along the molecular axis (head -on).
c) may result from overlap of two s atomic orbitals.
d) may result from overlap of one s and one p atomic orbitals.
Solution: a
7] Which one of the following statements is false?
a) Valence bond theory and molecular orbital theory can be described as two
different views of the same thing.
b) When one considers the molecular orbitals resulting from the overlap of
any two specific atomic orbitals, the bonding orbitals ar e always lower in
energy than the antibonding orbitals.
c) Molecular orbitals are generally described as being more delocalized than
hybridized atomic orbitals.
d) One of the shortcomings of molecular orbital theory is its inability to
account for a triple bond in the nitrogen molecule, N2.
Solution:d
8] Which of the following cannot be a ligand?
a) Ni 2+

CHE509: Inorganic Chemistry – II Page 69


b) Cl –
c) H 2 O
d) NH 3
Solution: a, The ions/molecules bound to the central atom/ion is called a ligand. Ni 2+ is a
metal ion, and according to Werner the secondary valences can be satisfied only by neutral
molecules or negative ions. Cl – , H 2 O and NH 3 are all possible ligands.
SUMMARY
The characteristics of the transition metal-ligand bonds will be clarified by
analyzing the molecular orbitals of the 3d metal coordinated by six identical ligands in
octahedral complexes [ML 6 ]. Bonding, non-bonding and anti-bonding complex molecular
orbitals are formed as a result of the interaction between the metal d orbitals and the ligand.
In general, the energy levels of ligand orbitals are lower than those of metal orbitals,
bonding orbitals have more ligand character, and nonbonding and antibonding orbitals have
more metal character.
KEY WORDS
Octahedral complexes, MOT, Bonding, Legands

REFERENCES

WIKIPEDIA
1) https://en.wikipedia.org/wiki/Octahedral_molecular_geometry
2) https://en.wikipedia.org/wiki/Octahedral_molecular_geometry#Splitting_
of_d-orbital_energies
3) https://en.wikipedia.org/wiki/Octahedral_cluster
4) https://en.wikipedia.org/wiki/Molecular_orbital

OER
1) https://chem.libretexts.org/Bookshelves/General_Chemistry/Chemistry_1e_(Ope
nSTAX)/08%3A_Advanced_Theories_of_Covalent_Bonding/8.4%3A_Molecular
_Orbital_Theory#:~:text=The%20mathematical%20process%20of%20combinin
g,of%20atomic%20orbital%20wave%20functions.
2) https://chem.libretexts.org/Bookshelves/Inorganic_Chemistry/Book%3A_Introd
uction_to_Inorganic_Chemistry_(Wikibook)/02%3A_Molecular_Orbital_Theor
y/2.02%3A_Constructing_Molecular_Orbitals_from_Atomic_Orbitals
3) https://www.toppr.com/guides/chemistry/chemical-bonding-and-molecular-
structure/molecular-orbital-theory/

CHE509: Inorganic Chemistry – II Page 70


REFERENCE BOOKS
1) Inorganic Chemistry Fourth Edition By Pearson: Principles of Structure and
Reactivity Paperback January 2006 by James E. Huheey
2) Applications of Molecular Orbital Theory in Organic Chemistry” by I G
Csizmadia

CHE509: Inorganic Chemistry – II Page 71


CREDIT 01-UNIT 04: O CTAHEDRAL & SQUARE PLANAR SYMMETRIES
LEARNING OBJECTIVES
After successful completion of this unit, you will be able to
 Understand geometries of octahedral and square planer complexes
 Understand magnetic behavior of of tetrahedral and square planer complexes
 Describe pi bonding in tetrahedral and square planer complexes
 Understand magnetic moment in coordinated complexes

INTRODUCTION

Square planar molecular geometry in chemistry describes the stereochemistry


(spatial arrangement of atoms) adopted by certain chemical compounds. As the name
suggests, molecules of this geometry have their atoms located at the corners.
In chemistry, octahedral molecular geometry, also called square bipyramidal,
describes the shape of compounds with six atoms or groups of atoms or ligands
symmetrically arranged around a central atom, defining the vertices of t he
octahedron. An octahedron has eight faces, hence the prefix vinegar. An octahedron
is one of the Platonic solids, although octahedral molecules typically have an atom
at their center and no bonds between ligand atoms. A perfect octahedron belongs to
the point group Oh. Examples of octahedral compounds are sulfur fluoride SF 6 and
molybdenum hexacarbonyl Mo(CO) 6 . The term "octahedral" is used somewhat
loosely by chemists, focusing on the geometry of the bonds to the central atom and
not taking into account the differences between the ligands themselves. For example,
[Co (NH3) 6 ] 3+ , which is not octahedral in the mathematical sense due to the
orientation of the N−H bonds, is referred to as octahedral.

CHE509: Inorganic Chemistry – II Page 72


04-01: IDENTIFICATION OF PLANES OF SYMMETRY IN OCTAHEDRAL
COMPLEXES
The symmetry of a molecule is determined by the existence of symmetry
operations performed with respect to symmetry elements. An element of symmetry is
a line, plane, or point in or through an object about which rotation or reflection
leaves the object in an orientation indistinguishable from the original. A plane of
symmetry and reflection operation is the matching of atoms on one side of the plane
with corresponding atoms on the other side, as if reflected in a mirror.
Nine Planes of Potential Symmetry
Identifying symmetry planes in an octahedral geometry can be daunting when
evaluating whether a molecule is chiral or not. There is a simple approach that
requires identifying and testing the six possible planes of symmetry that may exist
for this geometry. To demonstrate this, we will consider the [Cr(CO) 6 ] complex in
the figure

While you can no doubt recognize at least one plane of symmetry to confirm
that this is an achiral complex, let's identify all six of them. First we have three
planes that include the Cr metal center (as they all do) and the four attached ligands.
These planes are often easily identifiable.

Three potential planes of symmetry for an octahedral complex. These planes have
four in-plane and two out-of-plane ligands. Perspective makes it difficult to distinguish, but

CHE509: Inorganic Chemistry – II Page 73


the atoms behind the plane are brighter, and the slope of the plane can be identified by
projection through the central Cr atom.
The next six planes are less identifiable and can be seen as a 45° rotation of the
planes in the image. These planes bisect one angle between the ligands but still have two
ligands in the plane

Three potential planes of symmetry for an octahedral complex. These planes have
two in-plane and four out-of-plane ligands. Perspective makes it difficult to distinguish, but
the atoms behind the plane are brighter, and the slope of the plane can be identified by
projection through the central Cr atom.
The next three planes are even harder to see in the same perspective, but they are
also 45° rotations of the planes in the perpendicular direction. These planes bisect the angle
between the ligands but still have two ligands in the plane.

Three potential planes of symmetry for an octahedral complex. These planes have
two in-plane and four out-of-plane ligands. Perspective makes it difficult to distinguish, but
the atoms behind the plane are brighter, and the slope of the plane can be identified by
projection through the central Cr atom.
All nine planes, in each of the planes, are combined in the i mages below.

Now that you can identify these nine planes that may be mirror planes in the
octahedral complex, you need to evaluate whether they are indeed mirror planes. This is

CHE509: Inorganic Chemistry – II Page 74


simply a question of whether the ligand field above the plane is similar to the ligand field
below it. For the [Cr(CO)] complex discussed above, all nine planes are indeed mirror
planes. However, this may not be the case in other complexes. Fortunately, only one plane
needs to be identified to determine whether a complex is chiral (optically active).
Square planar molecular geometry
Square planar molecular geometry in chemistry describes the stereochemistry
(spatial arrangement of atoms) adopted by certain chemical compounds. As the name
suggests, molecules of this geometry have their atoms located at the corners.

Many compounds adopt this geometry, examples being particularly numerous


for transition metal complexes. The noble gas compound XeF 4 adopts this structure
as predicted by VSEPR theory. The geometry predominates for trans ition metal
complexes with the d 8 configuration, which includes Rh(I), Ir(I), Pd(II), Pt(II), and
Au(III). Notable examples include the anticancer drugs cisplatin [PtCl 2 (NH 3 ) 2 ] and
carboplatin. Many homogeneous catalysts are square planar in their resting state,
such as Wilkinson catalyst and Crabtree catalyst. Other examples include Vask's
complex and Zeise's salt. Certain ligands (such as porphyrins) stabilize this
geometry.
In the square planar case, strongly π-donor ligands can cause the dxz and dyz
orbitals to be higher in energy than the dz2 orbital, while in the octahedral case, the
π-donor ligands only affect the magnitude of the d -orbital splitting and the relative
arrangement. of orbitals is preserved
In square planar complexes, the central atom is surrounded by four base
atoms forming the corners of a square in the same plane. The coordination number of
the central atom for the square planar complex is 4.
In square planar complexes, the central atom is surrounded by four base
atoms forming the corners of a square in the same plane. The coordination number of
the central atom for the square planar complex is 4.

CHE509: Inorganic Chemistry – II Page 75


As in the given structure, all four basic groups attached to the central atom
are the same, so it is an achiral molecule, and the structure consists of a plane of
symmetry, so it is an optically inactive compound.

As in the given structure, the three basic groups attached to the central atom
are the same, so it is an achiral molecule, and the structure consists of a plane of
symmetry, so it is an optically inactive compound.

As in the given structure, two constituent groups connected to the central


atom are the same, so it is an achiral molecule and the structure consists of a plane
of symmetry, therefore it is an optically inacti ve compound.

Although a molecule consists of four different basic groups, a molecule is


achiral when a rotation, which is a combination of in -plane rotation and reflection,
results in the formation of the same molecule, making the compound or molecule
optically inactive. Therefore, it is also an optically inactive molecule.Thus, square
planar complexes do not show optical isomerism because optical isomerism in the

CHE509: Inorganic Chemistry – II Page 76


structures is due to the absence of symmetry elements, but all structures of square
planar complexes have an element of symmetry.
SOLVED PROBLEMS 01
1] Describe nine plane of symmetries in octahedral complexes
Solution: Nine Planes of Potential Symmetry
Identifying symmetry planes in an octahedral geometry can be daunting when
evaluating whether a molecule is chiral or not. There is a simple approach that
requires identifying and testing the six possible planes of symmetry that may exist
for this geometry. To demonstrate this, we will consider the [Cr(CO) 6 ] complex in
the figure

While you can no doubt recognize at least one plane of symmetry to confirm
that this is an achiral complex, let's identify all six of them. First we have three
planes that include the Cr metal center (as they all do) and the four attached ligands.
These planes are often easily identifiable.

Three potential planes of symmetry for an octahedral complex. These planes have
four in-plane and two out-of-plane ligands. Perspective makes it difficult to distinguish, but
the atoms behind the plane are brighter, and the slope of the plane can be identified by
projection through the central Cr atom.

The next six planes are less identifiable and can be seen as a 45° rotation of the
planes in the image. These planes bisect one angle between the ligands but still have two
ligands in the plane

CHE509: Inorganic Chemistry – II Page 77


Three potential planes of symmetry for an octahedral complex. These planes have
two in-plane and four out-of-plane ligands. Perspective makes it difficult to distinguish, but
the atoms behind the plane are brighter, and the slope of the plane can be identified by
projection through the central Cr atom.
The next three planes are even harder to see in the same perspective, but they are
also 45° rotations of the planes in the perpendicular direction. These planes bisect the angle
between the ligands but still have two ligands in the plane.

Three potential planes of symmetry for an octahedral complex. These planes have
two in-plane and four out-of-plane ligands. Perspective makes it difficult to distinguish, but
the atoms behind the plane are brighter, and the slope of the plane can be identified by
projection through the central Cr atom.
2] Describe the plane of symmetries present in square planar complexes.
Solution: Square planar molecular geometry
Square planar molecular geometry in chemistry describes the stereochemistry
(spatial arrangement of atoms) adopted by certain chemical compounds. As the name
suggests, molecules of this geometry have their atoms located at the corners.

CHE509: Inorganic Chemistry – II Page 78


Many compounds adopt this geometry, examples being particularly numerous
for transition metal complexes. The noble gas compound XeF 4 adopts this structure
as predicted by VSEPR theory. The geometry predominates for transition metal
complexes with the d8 configuration, which includes Rh(I), Ir(I), Pd(II), Pt(II), and
Au(III). Notable examples include the anticancer drugs cisplatin [PtCl 2 (NH 3 ) 2 ] and
carboplatin. Many homogeneous catalysts are square planar in their resting state,
such as Wilkinson catalyst and Crabtree catalyst. Other examples include Vask's
complex and Zeise's salt. Certain ligands (such as porphyrins) stabilize this
geometry.
In the square planar case, strongly π-donor ligands can cause the dxz and dyz
orbitals to be higher in energy than the dz2 orbital, while in the octahedral case, the
π-donor ligands only affect the magnitude of the d-orbital splitting and the relative
arrangement. of orbital’s is preserved
In square planar complexes, the central atom is surrounded by four base
atoms forming the corners of a square in the same plane. The coordination number of
the central atom for the square planar complex is 4.
In square planar complexes, the central atom is surrounded by four base
atoms forming the corners of a square in the same plane. The coordination number of
the central atom for the square planar complex is 4.

As in the given structure, all four basic groups attached to the central atom
are the same, so it is an achiral molecule, and the structure consists of a plane of
symmetry, so it is an optically inactive compound.

As in the given structure, the three basic groups attached to the central atom
are the same, so it is an achiral molecule, and the structure consists of a plane of
symmetry, so it is an optically inactive compound.

CHE509: Inorganic Chemistry – II Page 79


As in the given structure, two constituent groups connected to the central
atom are the same, so it is an achiral molecule and the structure consists of a plane
of symmetry, therefore it is an optically inactive compound.

Although a molecule consists of four different basic groups, a molecule is


achiral when a rotation, which is a combination of in-plane rotation and reflection,
results in the formation of the same molecule, making the compound or molecule
optically inactive. Therefore, it is also an optically inactive molecule.Thus, square
planar complexes do not show optical isomerism because optical isomerism in the
structures is due to the absence of symmetry elements, but all structures of square
planar complexes have an element of symmetry.

SHORT ANSWER QUESTIONS WITH MODEL ANSWER 01


1] Explain any three planes of symmetries in octahedral complexes

Solution: three planes of symmetries in octahedral complexes can be seen as a 45°


rotation of the planes in the image. These planes bisect one angle between the ligands but
still have two ligands in the plane

2] Describe the condition for squire planer complexes to become optically inactive

CHE509: Inorganic Chemistry – II Page 80


Solution:

Although a molecule consists of four different basic groups, a molecule is


achiral when a rotation, which is a combination of in -plane rotation and reflection,
results in the formation of the same molecule, making the compound or molecule
optically inactive. Therefore, it is also an optically inactive molecule.Thus, square
planar complexes do not show optical isomerism because optical isomerism in the
structures is due to the absence of symmetry elements, but all structures of square
planar complexes have an element of symmetry.
04-02: Π-B ONDING BETWEEN METALS AND LIGANDS
An important factor contributing to the high ligand field intensity of ligands
such as CO, CN- and phosphines is the π-bonding between the metal and the ligand.
There are three types of pi-bonds in metal complexes. The most common situation is
when a ligand such as carbon monoxide or cyanide donates its sigma (non -bonding)
electrons to the metal while accepting electron density from the metal through
overlap of the metal t2g orbital and the ligand π* orbital. This situation is called
"back-bonding" because the ligand gives the metal the σ-electron density and the
metal the π-electron density of the ligand. Thus, the ligand acts as a σ-donor and a π-
acceptor. In π-backbonding, the metal donates π electrons to the ligand's π* orbital
and adds electron density to the antibonding molecular orbital. This results in a
weakening of the C–O bond, which is observed experimentally as a lengthening of
the bond (relative to free CO in the gas phase) and a decrease in the C –O infrared
stretching frequency..

CHE509: Inorganic Chemistry – II Page 81


d-d π bonding occurs when an element such as phosphorus, which has σ -
symmetry lone pair and an empty metal 3d orbital, bonds to a metal that has
electrons in the t 2 g orbital. This is a common situation for phosphine complexes (eg,
triphenylphosphine) bound to low-power late transition metals. The basic bond in
this case is analogous to the CO example, except that the acceptor orbital is a
phosphorus 3d orbital rather than a ligand π* orbital.
Here, the phosphine ligand acts as a σ-donor and π-acceptor and forms a dπ-
dπ bond.

A third kind of metal-ligand π-bond occurs when a π-donor ligand—an


element with a σ-symmetric electron pair and a filled orthogonal p -orbital—bonds to
a metal, as shown above right for O 2- ligand. This occurs in early transition metal

CHE509: Inorganic Chemistry – II Page 82


complexes. In this example, O 2- acts as both a σ-donor and a π-donor. This
interaction is typically depicted as a metal-ligand multiple bond, eg the V=O bond in
the vanadyl cation [VO] 2+ . Typical π-donor ligands are oxide (O 2- ), nitride (N 3- ),
imide (RN 2- ), alkoxide (RO - ), amide (R 2 N - ) and fluoride (F - ). In late transition
metals, strong π-donors form antibonding interactions with filled d-levels, with
consequences for spin state, redox potentials, and ligand exchange rates. π -donor
ligands are low in the spectrochemical series.

Carbon-containing ligands that are π-donors and their complexes with


transition metal ions are very important in olefin metathesis, a reaction in which
carbon-carbon double bonds are exchanged. Using these catalysts, cyclic olefins can
be transformed into linear polymers in high yield via ring -opening metathesis
polymerization (ROMP). Catalysts of this kind were developed by the groups of
Richard Schrock and Robert Grubbs, who shared the 2005 Nobel Prize in Chemistry
with Yves Chauvin for their discoveries. Schrock catalysts are based on early
transition metals such as Mo; are more reactive but less tolerant to various organic
functional groups and protic solvents than Grubbs catalysts, which are based on Ru
complexes.

Experimental evidences for pi binding:

Crystallography – The greater the extent of pi -backbonding, the more M=C


characters there will be be and the more CO will resemble C=O. The difference in
bond lengths CO and C=O is approx 0.1 Å and should be usable for quant.
Unfortunately, this was not observed. in in contrast, the M -C bond lengths change.
Consider the complexes Cr(CO) 6 and Cr(CO) 5 (PR 3 ). In the absence of pi-back
bonding Cr-C bond lengths should be equal. If this happens, then bond lengths

CHE509: Inorganic Chemistry – II Page 83


should be shorter Cr(CO) 5 (PMe 3 ). Why? Two reasons: PMe 3 is a very poor pi-
acceptor at best, so there are only 5 of them competes for electron density from the
metal, not 6; and PR 3 is a very good pi-donor, CO is not. Therefore, Cr has more
electron density to share with fewer acceptors. Another trend is expected. The Cr -
C(O) bond trans to PR 3 should be shorter than cis This is because trans CO will be
bond to the same d-orbital as PR 3 and that is where the effect will be greatest.
Infrared spectroscopy Evidence of C=O character is most clearly seen in
infrared spectroscopy.  (CO) for C=O is approx 2150 cm-1, while in R2C=O,
 (C=O) is about 1700 cm-1. So the bigger the range backbonding the lower the
expected  (C=O). This is dramatically seen in two series of complexes 11 M(CO) 6 n+/-
and M(CO) 4 n+/- . This can also be seen when CO is substituted or by another ligan d as
seen in crystallography. The only problem with using this technique is that the CO
tension band is almost always divided into several components, which allows for
interpretation difficult.

SOLVED PROBLEMS 02
1] Describe π bonding back bonding in between legand and metal in complexes .
Solution: An important factor contributing to the high ligand field intensity of
ligands such as CO, CN- and phosphines is the π-bonding between the metal and the
ligand. There are three types of pi-bonds in metal complexes. The most common
situation is when a ligand such as carbon monoxide or cyanide donates its sigma
(non-bonding) electrons to the metal while accepting electron density from the metal
through overlap of the metal t2g orbital and the ligand π* orbita l. This situation is
called "back-bonding" because the ligand gives the metal the σ -electron density and
the metal the π-electron density of the ligand. Thus, the ligand acts as a σ -donor and
a π-acceptor. In π-back bonding, the metal donates π electrons to the ligand's π*
orbital and adds electron density to the antibonding molecular orbital. This results in
a weakening of the C–O bond, which is observed experimentally as a lengthening of
the bond (relative to free CO in the gas phase) and a decrease in th e C–O infrared
stretching frequency.

CHE509: Inorganic Chemistry – II Page 84


d-d π bonding occurs when an element such as phosphorus, which has σ -
symmetry lone pair and an empty metal 3d orbital, bonds to a metal that has
electrons in the t 2 g orbital. This is a common situation for phosphine complexes (eg,
triphenylphosphine) bound to low-power late transition metals. The basic bond in
this case is analogous to the CO example, except that the acceptor orbital is a
phosphorus 3d orbital rather than a ligand π* orbital.
Here, the phosphine ligand acts as a σ-donor and π-acceptor and forms a dπ-
dπ bond.

A third kind of metal-ligand π-bond occurs when a π-donor ligand—an


element with a σ-symmetric electron pair and a filled orthogonal p -orbital—bonds to
a metal, as shown above right for O 2- ligand. This occurs in early transition metal
complexes. In this example, O 2- acts as both a σ-donor and a π-donor. This
interaction is typically depicted as a metal-ligand multiple bond, eg the V=O bond in

CHE509: Inorganic Chemistry – II Page 85


the vanadyl cation [VO] 2+ . Typical π-donor ligands are oxide (O 2- ), nitride (N 3- ),
imide (RN 2- ), alkoxide (RO - ), amide (R 2 N - ) and fluoride (F - ). In late transition
metals, strong π-donors form antibonding interactions with filled d-levels, with
consequences for spin state, redox potentials, and ligand exchang e rates. π-donor
ligands are low in the spectrochemical series.

Carbon-containing ligands that are π-donors and their complexes with


transition metal ions are very important in olefin metathesis, a reaction in which
carbon-carbon double bonds are exchanged. Using these catalysts, cyclic olefins can
be transformed into linear polymers in high yield via ring -opening metathesis
polymerization (ROMP). Catalysts of this kind were developed by the groups of
Richard Schrock and Robert Grubbs, who shared the 2005 Nobel Prize in Chemistry
with Yves Chauvin for their discoveries. Schrock catalysts are based on early
transition metals such as Mo; are more reactive but less tolerant to various organic
functional groups and protic solvents than Grubbs catalysts, which ar e based on Ru
complexes.

SHORT ANSWER QUESTIONS WITH MODEL ANSWER 02


1] Explain the experimental evidences for pi-bonds in metal complexes

Solution: Experimental evidences for pi binding

Crystallography – The greater the extent of pi -backbonding, the more M=C


characters there will be be and the more CO will resemble C=O. The difference in
bond lengths CO and C=O is approx 0.1 Å and should be usable for quant.
Unfortunately, this was not observed. in in contrast, the M-C bond lengths change.
Consider the complexes Cr(CO) 6 and Cr(CO) 5 (PR 3 ). In the absence of pi-

CHE509: Inorganic Chemistry – II Page 86


backbonding Cr-C bond lengths should be equal. If this happens, then bond lengths
should be shorter Cr(CO) 5 (PMe 3 ). Why? Two reasons: PMe 3 is a very poor pi-
acceptor at best, so there are only 5 of them competes for electron density from the
metal, not 6; and PR 3 is a very good pi-donor, CO is not. Therefore, Cr has more
electron density to share with fewer acceptors. Another trend is expected. The Cr-
C(O) bond trans to PR 3 should be shorter than cis This is because trans CO will be
bond to the same d-orbital as PR 3 and that is where the effect will be greatest.

Infrared spectroscopy Evidence of C=O character is most clearly seen in


infrared spectroscopy.  (CO) for C=O is approx 2150 cm-1, while in R2C=O,
 (C=O) is about 1700 cm-1. So the bigger the range backbonding the lower the
expected  (C=O). This is dramatically seen in two series of complexes 11 M(CO) 6 n+/-
and M(CO) 4 n+/- . This can also be seen when CO is substituted or by another ligand as
seen in crystallography. The only problem with using this technique is that the CO
tension band is almost always divided into several components, which allows for
interpretation difficult.

04-03: MAGNETIC MOMENTS OF TRANSITION METALS


Magnetic moments are often used in conjunction with electronic spectra to
obtain information about the oxidation number and stereochemistry of the central
metal ion in coordination complexes. A common laboratory procedure for
determining the magnetic moment for a complex is the Gouy method, which involves
weighing a sample of the complex in the presence and absence of a magnetic field
and observing the difference in weight. A template is provided for the rel evant
calculations. For the first row of transition metal ions in the state of free ions, i.e.
isolated ions in vacuum, all 5 of 3d orbitals are degenerate. A simple crystal field
theory approach to bonding in these ions assumes that when they form octahed ral
complexes, the energies of the d orbitals are no longer degenerate but are split such
that two orbitals, dx2-y2 and dz2 (e.g. a subset) are at higher energy than the dxy,
dxz, dyz (a subset of t2g). For octahedral ions with between 4 and 7 d electrons, this
gives rise to 2 possible configurations called either high spin/weak field or low
spin/strong field. The energy gap is dependent on the position of the coordinated
ligands in the spectrochemical series.

CHE509: Inorganic Chemistry – II Page 87


Square planar complexes are less common than tetrahedral and d8 eg Ni (II) ,
Pd (II) , Pt (II) etc. have a strong tendency to form square planar complexes. As with
octahedral complexes, the energy gap between dxy and dx 2 −y2 is Δoct and these are
considered strong field / low spin, therefore all are diamagne tic, μ=0 Bohr Magneton
(B.M.) The formula used to calculate the spin-only magnetic moment can be written
in two forms; the first is based on the number of unpaired electrons, n, and the
second based on the electron spin quantum number, S. Because for each unpaired
electron, n=1 and S=1/2, then the two formulas are clearly related and the answer
obtained must be identical. The unit of mangetic moment is bohr magneton (BM)

Were µ is magnetic moment and n is the number of unpaired electrons in atom

μso / μobs /
Ion Configuration
B.M. B.M.

√3 =
Ti(III) d1 (t2g1) 1.6-1.7
1.73

√8 =
V(III) d2 (t2g2) 2.7-2.9
2.83

√15 =
Cr(III) d3 (t2g3) 3.7-3.9
3.88

d4 high spin √24 =


Cr(II) 4.7-4.9
(t2g3 eg1) 4.90

The difference in energy between the eg and t2g orbitals is called the crystal field
splitting and is symbolized by Δoct, where oct means octahedral.

The magnitude of Δoct depends on many factors, including the nature of the six
ligands placed around the central metal ion, the charge on the metal, and whether t he metal
uses 3d, 4d, or 5d orbitals. Different ligands produce different crystal field splittings. The
increasing splitting of the crystal field produced by the ligands is expressed in a
spectrochemical series, a short version of which is given here:

I− <Br − <Cl − <F − <H2O<C2O4 2− <NH 3 <en<NO −2 <CN −

CHE509: Inorganic Chemistry – II Page 88


−−−−−−−−−−−−−−−−−−−−−−−−−−−−−−−−−−−→
A few ligands of the spectrochemical series, in order of increasing field
strength of the ligand

In this series, the ligands on the left cause a small crystal field splitting and are
weak field ligands, while those on the right cause more splitting and are strong field
ligands. Thus, the Δoct value for the octahedral complex with iodide ligands (I – ) is much
smaller than the Δoct value for the same metal with cyanide ligands (CN – ).

Electrons in d orbitals follow the aufbau ("filling") principle, which states that the
orbitals will be filled to give the lowest total energy, just as in main group chemistry. When
two electrons occupy the same orbital, like charges repel each other. The ene rgy required to
pair two electrons in a single orbital is called the pairing energy (P). Electrons will always
occupy every orbital in the degenerate set before pairing. P is similar in magnitude to Δoct.
When electrons fill d orbitals, the relative magnit udes of Δoct and P determine which
orbitals will be occupied.
4−
in [ Fe(CN) 6 ] , a strong field of six cyanide ligands produces a large Δoct. Under
these conditions, the electrons need less energy to pair than they need to be excited to e.g.
orbitals (Δoct > P). Six 3d electrons of the Fe 2+ ion pair in three t 2 g orbitals. Complexes in
which the electrons are paired due to a large splitting of the crystal field are called low -spin
complexes because the number of unpaired electrons (spins) is minimized.

On the other hand, the weak field of water molecules produces only a small
distribution of the crystal field (Δoct < P). Because it takes less energy for electrons to
occupy eg orbitals than to pair together, there will be an electron in each of the five 3d
2+
orbitals before pairing. For the six d electrons in the iron(II) center. [Fe(H 2 O) 6 ] , there
will be one pair of electrons and four unpaired electrons. Complexes such as [Fe(H 2 O) 6 ] 2+
on in which the electrons are unpaired because the crystal field splitting is not large enough

CHE509: Inorganic Chemistry – II Page 89


to cause them to pair are called high -spin complexes because the number of unpaired
electrons (spins) is maximized.
SOLVED PROBLEMS 03
1] Explain the magnetic moment for transition metal complexes
Solution: Magnetic moments are often used in conjunction with electronic
spectra to obtain information about the oxidation number and stereochemistry of the
central metal ion in coordination complexes. A common laborat ory procedure for
determining the magnetic moment for a complex is the Gouy method, which involves
weighing a sample of the complex in the presence and absence of a magnetic field
and observing the difference in weight. A template is provided for the relev ant
calculations. For the first row of transition metal ions in the state of free ions, i.e.
isolated ions in vacuum, all 5 of 3d orbitals are degenerate. A simple crystal field
theory approach to bonding in these ions assumes that when they form octahedra l
complexes, the energies of the d orbitals are no longer degenerate but are split such
that two orbitals, dx2-y2 and dz2 (e.g. a subset) are at higher energy than the dxy,
dxz, dyz (a subset of t2g). For octahedral ions with between 4 and 7 d electrons, t his
gives rise to 2 possible configurations called either high spin/weak field or low
spin/strong field. The energy gap is dependent on the position of the coordinated
ligands in the spectrochemical series.
Square planar complexes are less common than tetrahedral and d8 eg Ni (II) ,
Pd (II) , Pt (II) etc. have a strong tendency to form square planar complexes. As with
octahedral complexes, the energy gap between dxy and dx 2 −y2 is Δoct and these are
considered strong field / low spin, therefore all are diamagneti c, μ=0 Bohr Magneton
(B.M.) The formula used to calculate the spin-only magnetic moment can be written
in two forms; the first is based on the number of unpaired electrons, n, and the
second based on the electron spin quantum number, S. Because for each un paired
electron, n=1 and S=1/2, then the two formulas are clearly related and the answer
obtained must be identical. The unit of mangetic moment is bohr magneton (BM)

Were µ is magnetic moment and n is the number of unpaired electrons in atom

μso / μobs /
Ion Configuration
B.M. B.M.

CHE509: Inorganic Chemistry – II Page 90


μso / μobs /
Ion Configuration
B.M. B.M.

√3 =
Ti(III) d1 (t2g1) 1.6-1.7
1.73

√8 =
V(III) d2 (t2g2) 2.7-2.9
2.83

√15 =
Cr(III) d3 (t2g3) 3.7-3.9
3.88

d4 high spin √24 =


Cr(II) 4.7-4.9
(t2g3 eg1) 4.90

2] Explain the effect type of legands on magnetic moment.


Solution: The difference in energy between the eg and t 2 g orbitals is called the crystal field
splitting and is symbolized by Δoct, where oct means octahedral.
The magnitude of Δoct depends on many factors, including the nature of the six
ligands placed around the central metal ion, the charge on the metal, and whether the metal
uses 3d, 4d, or 5d orbitals. Different ligands produce different crystal field splittings. The
increasing splitting of the crystal field produced by the ligands is expressed in a
spectrochemical series, a short version of which is given here:
I− <Br − <Cl − <F − <H2O<C2O4 2− <NH 3 <en<NO −2 <CN −

−−−−−−−−−−−−−−−−−−−−−−−−−−−−−−−−−−−→
A few ligands of the spectrochemical series, in order of increasing field
strength of the ligand
In this series, the ligands on the left cause a small crystal field splitting and are
weak field ligands, while those on the right cause more splitting and are strong field
ligands. Thus, the Δoct value for the octahedral complex with iodide ligands (I – ) is much
smaller than the Δoct value for the same metal with cyanide ligands (CN – ).
Electrons in d orbitals follow the aufbau ("filling") principle, which states that the
orbitals will be filled to give the lowest total energy, just as in main group chemistry. When
two electrons occupy the same orbital, like charges repel each other. The energy required to
pair two electrons in a single orbital is called the pairing energy (P). Electrons will always

CHE509: Inorganic Chemistry – II Page 91


occupy every orbital in the degenerate set before pairing. P is similar in magn itude to Δoct.
When electrons fill d orbitals, the relative magnitudes of Δoct and P determine which
orbitals will be occupied.
4−
in [ Fe(CN) 6 ] , a strong field of six cyanide ligands produces a large Δoct. Under
these conditions, the electrons need less energy to pair than they need to be excited to e.g.
orbitals (Δoct > P). Six 3d electrons of the Fe 2+ ion pair in three t 2 g orbitals. Complexes in
which the electrons are paired due to a large splitting of the crystal field are called low -spin
complexes because the number of unpaired electrons (spins) is minimized.

on the other hand, the weak field of water molecules produces only a small
distribution of the crystal field (Δoct < P). Because it takes less energy for electrons to
occupy eg orbitals than to pair together, there will be an electron in each of the five 3d
2+
orbitals before pairing. For the six d electrons in the iron(II) center. [Fe(H 2 O) 6 ] , there
will be one pair of electrons and four unpaired electrons. Complexes such as [Fe(H 2 O) 6 ] 2+
on in which the electrons are unpaired because the crystal field splitting is not large enough
to cause them to pair are called high -spin complexes because the number of unpaired
electrons (spins) is maximized.

SHORT ANSWER QUESTIONS WITH MODEL ANSWER 03


1] Predict the number of unpaired electrons and magnetic moment for following complex

K3[CrI6]

Solution: K3[CrI6]

Above complex is octahedral (as it has six legands). Cr 3+ has a d 3 configuration.

These 3 electrons will all be unpaired.

Magnetic moment for above complex is

CHE509: Inorganic Chemistry – II Page 92


µ= √ 3(3+2)
µ=3.87 BM

2] Predict the number of unpaired electrons and magnetic moment for following
[Cu(en)2(H2O)2]Cl2
Solution: Cu2+ is d9, so there will be one unpaired electron.

µ= √ 1(1+2)
µ=1.73 BM

3] Predict the number of unpaired electrons and magnetic moment for following
Na3 [Co(NO2)6]
Solution: Co3+ has d6 valence electrons, so the crystal field splitting will determine how
many are paired. Nitrite is a strong-field ligand, so the complex will be low spin. Six
electrons will go in the t2g orbitals, leaving 0 unpaired.

µ= √ 0(0+2)
µ=0 BM
04-04: PARAMAGNETIC AND DIAMAGNETIC BEHAVIOR OF COMPLEXES
Experimental evidence from magnetic measurements supports the theory of
high- and low-spin complexes. Remember that molecules like O 2 that contain
unpaired electrons are paramagnetic. Paramagnetic substan ces are attracted by
magnetic fields. Many transition metal complexes have unpaired electrons and are
therefore paramagnetic. Molecules such as N 2 and ions such as Na + and [Fe(CN) 6 ] 4− ,
which contain no unpaired electrons, are diamagnetic. Diamagnetic subst ances have
a slight tendency to be repelled by magnetic fields. When an electron in an atom or
ion is unpaired, the magnetic moment due to its spin makes the entire atom or ion
paramagnetic.
The magnitude of the magnetic moment of a system containing unpa ired
electrons is directly related to the number of such electrons: the greater the number
of unpaired electrons, the greater the magnetic moment. Magnetic susceptibility
measures the force exerted by a substance in a magnetic field. When we compare the
weight of the sample with the weight measured in the magnetic field, paramagnetic

CHE509: Inorganic Chemistry – II Page 93


samples that are attracted to the magnet will appear heavier due to the force exerted
by the magnetic field. Based on the increase in mass, we can calculate the number of
unpaired electrons.

An interesting property of transition metals is their ability to form magnets.


Metal complexes that have unpaired electrons are magnetic. Since the last electrons
reside in d orbitals, this magnetism must be due to unpaired d electrons. If w e
consider only monometallic complexes, unpaired electrons arise because the
complex has an odd number of electrons or because the electron pairing is
destabilized. For example, a Ti +3 ion has one d electron and must be (weakly)
paramagnetic, regardless of the geometry or nature of the ligands. However, the Ti +2
ion with two d-electrons sometimes forms complexes with two unpaired electrons
and sometimes forms complexes with no unpaired electrons.

For example, Fe prefers to exist as Fe 3+ and is known to have a coordination


number of six. Since the configuration of Fe 3+ has five d electrons, we would expect
to see five unpaired spins in complexes with Fe. This is true for [FeF 6 ] 3− however,
[Fe(CN) 6 ] 3− has only one unpaired electron, making it a weaker magnet. This trend
can be explained based on the properties of the ligands. Based on the
spectrochemical series, we expect the CN − ligands to have a stronger electric field
than the F − ligands, so the energy differences in the d-orbitals should be larger for
the cyanide complex.

For this to make sense, the paired spins must have some energetic
contribution to our cyanide complex (spin pairing energy). This means that the
energy level difference must be greater than the repulsive energy of pairing the
electrons together. As the systems try to reach the lowest possible energy, the
electrons pair up before moving to higher orbitals. This is called low spin and the
electron moving up before pairing is known as high spin.

CHE509: Inorganic Chemistry – II Page 94


SOLVED PROBLEMS 04
1] Write a note on Diamagnetism and Paramagnetic complexes
Solution: Experimental evidence from magnetic measurements supports the theory of
high- and low-spin complexes. Remember that molecules like O 2 that contain
unpaired electrons are paramagnetic. Paramagnetic substances are attracted by
magnetic fields. Many transition metal complexes have unpaired electrons and are
therefore paramagnetic. Molecules such as N 2 and ions such as Na + and [Fe(CN) 6 ] 4− ,
which contain no unpaired electrons, are diamagnetic. Diamagnetic substances have
a slight tendency to be repelled by magnetic fields. When an electron in an atom or
ion is unpaired, the magnetic moment due to its spin makes the entire atom or ion
paramagnetic.
The magnitude of the magnetic moment of a system containing unpaired
electrons is directly related to the number of such electrons: the greater the number
of unpaired electrons, the greater the magnetic moment. Magnetic susceptibility
measures the force exerted by a substance in a magnetic field. When we compare the
weight of the sample with the weight measured in the magnetic field, paramagnetic
samples that are attracted to the magnet will appear heavier due to the force exerted
by the magnetic field. Based on the increase in mass, we can calculate the number of
unpaired electrons.
An interesting property of transition metals is their ability to form magnets.
Metal complexes that have unpaired electrons are magnetic. Since the last electrons
reside in d orbitals, this magnetism must be due to unpaired d electrons. If we
consider only monometallic complexes, unpaired electrons arise because the
complex has an odd number of electrons or because the electron pairing is
destabilized. For example, a Ti +3 ion has one d electron and must be (weakly)
paramagnetic, regardless of the geometry or nature of the ligands. However, the Ti +2
ion with two d-electrons sometimes forms complexes with two unpaired electrons
and sometimes forms complexes with no unpaired electrons.
For example, Fe prefers to exist as Fe 3+ and is known to have a coordination
number of six. Since the configuration of Fe 3+ has five d electrons, we would expect
to see five unpaired spins in complexes with Fe. This is true for [FeF 6 ] 3− however,
[Fe(CN) 6 ] 3− has only one unpaired electron, making it a weaker magnet. This trend
can be explained based on the properties of the ligands. Based on the
spectrochemical series, we expect the CN − ligands to have a stronger electric field

CHE509: Inorganic Chemistry – II Page 95


than the F − ligands, so the energy differences in the d-orbitals should be larger for
the cyanide complex.

For this to make sense, the paired spins must have some energetic
contribution to our cyanide complex (spin pairing energy). This means that the
energy level difference must be greater than the repulsive energy of pairing the
electrons together. As the systems try to reach the lowest possible energy, the
electrons pair up before moving to higher orbitals. This is called low spin and the
electron moving up before pairing is known as high spin.

SHORT ANSWER QUESTIONS WITH MODEL ANSWER 04


1] Which ligand is most likely to form high-spin complexes when bound to transition
metals: en, F - , or CN - ?

Solution: F − are more likely to form high spin complexes, since is a weak-field ligand on
the spectrochemical series.

2] Which one of the following coordination compounds would you expect to be


paramagnetic?

a) [Zn (NH 3 ) 4 ]Cl 2

Solution:

Let's consider each compound separately. Since each compound has only one
complex ion, we only need to consider how the d electrons are distributed in the
ligand field of that ion. We can ignore the other parts (counterions) of the compound.

The zinc ion in [Zn(NH 3 ) 4 ]Cl 2 has an oxidation state of +2, so it is a d10 ion
because the zinc atom loses two 4s electrons and no 3d electrons to form the ion. We
don't need to fill the ions with electrons in the corresponding tetrahedral d -orbital
diagram because we know they will all be paired. It is a diamagnetic compound.
a) K[FeCl 4 ] (low spin, tetrahedral)
Solution:

CHE509: Inorganic Chemistry – II Page 96


The iron ion in K[FeCl 4 ] (low spin, tetrahedral) has an oxidation state of 3+, so it is a
d5 ion because the iron atom loses two 4s electrons and one 3d electron. Adding 5 electrons
to the tetrahedral d-orbital diagram identifies that one electron will be unpaired. This
compound will be paramagnetic (but weakly since it is only unpaired on the electron).
b) [Cd(H 2 O) 6 ]SO 4
Solution:
The cadmium ion in [Cd(H 2 O) 6 ]SO 4 has an oxidation state of +2, so it is a d10
ion because the cadmium atom loses two 5s electrons and no 4d electrons to form the
ion. We don't need to fill the ions with electrons in the corresponding octahedral d -
orbital diagram because we know they will all be paired. It is a diamagnetic
compound.
CHECK POINT 01-04
1] The magnetic moment of Co 2+ in tetrahedral field is:
a) 3.87 MB
b) 1.73 MB
c) 4.87 MB
d) 5.87 MB
Solution: a
Co +2 contain 3d 7 electrons in the outermost orbit. To calculate magnetic moment
in tetrahedral geometry, the electronic configuration will be d 2 x2 −y 2 , d z 2 , dxy1 ,
d yz 1 , d xz 1 . Hence, the number of unpaired electrons (n) will be 3. By
substituting this value in the formula of magnetic moment, μ= n(n+2) ; μ will be 3.87
B.M.

2+
2] The structure of [Pt(NH 3 ) 4 ] is ___________.

a) Square Planer

b) Tetrahedral

c) Pyramidal

CHE509: Inorganic Chemistry – II Page 97


d) Pentagonal

Solution: a
2+
In [Pt(NH 3 ) 4 ] , Pt is in +2 oxidation state. Pt in +2 state always exhibit square
planar.

3] Calculate the magnetic moment of high spin Cu 2+ ion in Octahedral field.

Solution: Configuration of d orbitals for Cu 2+ is 3d 9 so the number of unpaired


electrons are two therefore the magnetic moment of Ni 2+ given by

Hence, only 1 unpaired electron found

Magnetic moment can be calculated as

µ= √ 1(1+2)

µ=1.73 BM

4] How meny unpaired electrons are in last d orbital of Sc +2

a) 1

b) 2

c) 3

d) 4

Solution : a, Because it lost all s orbital electrons (two electrons) and only one
unpaired electron is remain in last d orbital

SUMMARY
Square planar geometries are much less common than octahedral, but square
planar complexes are asserting their importance due to their frequent occurrence in

CHE509: Inorganic Chemistry – II Page 98


key catalytic processes and other environments. Furthermore, having learned
something about the bonds in octahedral complexes, we can make some educated
guesses about the metal-orbital interactions in square planar complexes. Both
geometries are nicely described by Cartesian coordinates, so it is relatively easy to
compare them. We can imagine how we could arrive at a square planar geometry
simply by taking the octahedral geometry and removing the two axial ligands. The
four remaining equatorial ligands form a square planar complex. en electron in an
atom or ion is unpaired, the magnetic moment due to its spin makes the whole atom
or ion paramagnetic. The magnitude of the magnetic moment of a system containing
unpaired electrons is directly related to the number of such electrons: the greater the
number of unpaired electrons, the greater the magnetic moment. Magnetic
susceptibility measures the force exerted by a substance in a magnetic field. When
we compare the weight of the sample to the weight measured in the magnetic field,
paramagnetic samples that are attracted to the magnet will appear heavier due to the
force exerted by the magnetic field. Based on the increase in mass, we can calculate
the number of unpaired electrons.

KEY WORDS
Magnetic moment, Plane of symmetry, CFT

REFERENCES

MOOCS
1) https://archive.nptel.ac.in/content/storage2/courses/downloads_new/104101121/
noc20_cy03_assessment_id_Week_3.pdf

YOUTUBE VIDEOS
1) https://www.youtube.com/watch?v=TOckIJ-2FZc

2) https://www.youtube.com/watch?v=Q81ajAgyBAc

3) https://www.youtube.com/watch?v=Y1WliIALt-I

4) https://www.youtube.com/watch?v=n9PCa2N9LjQ

5) https://www.youtube.com/watch?v=mkZO9dWVoZY

WIKIPEDIA
1) https://en.wikipedia.org/wiki/Spin_states_(d_electrons)

2) https://en.wikipedia.org/wiki/Magnetic_moment

CHE509: Inorganic Chemistry – II Page 99


OER
1) https://chem.libretexts.org/Courses/University_of_California_Davis/UCD_Chem
_002C/UCD_Chem_2C_(Larsen)/Text/03%3A_Coordination_Chemistry/3.11%3
A_Magnetic_Behavior_of_Complex_Ions#:~:text=Many%20transition%20metal
%20complexes%20have,be%20repelled%20by%20magnetic%20fields.

2) https://chem.libretexts.org/Bookshelves/General_Chemistry/Map%3A_General_
Chemistry_(Petrucci_et_al.)/24%3A_Complex_Ions_and_Coordination_Compo
unds/24.06%3A_Magnetic_Properties_of_Coordination_Compounds_and_Crys t
al_Field_Theory

REFERENCE BOOKS
1) Magnetism and Transition Metal Complexes (Dover Books on Chemistry) ., 2008 by
F E Mabbs , D J Machin, Lord Lewis, Dante Gatteschi

2) Principles of INORGANIC CHEMISTRY , 2014 by Puri.sharma. Kalia

CHE509: Inorganic Chemistry – II Page 100


C REDIT 02

CHE509: Inorganic Chemistry – II Page 101


CREDIT 2 UNIT 01: ELECTRONIC SPECTRA
LEARNING OBJECTIVES
After successful completion of this unit, you will be able to
 Understand types of electronic excitations
 Understand the concept of electronic Spectra
 Understand the concept of selection rules for determination of ground state terms
 Determination of term symbol for metal ions.
INTRODUCTION

Friedrich Hermann Hund (4 February 1896 – 31 March 1997)


The ultraviolet and visible spectra of transition metal coordination compounds involve
transitions between the d orbitals of the metals. Therefore, we will have to look in detail at
the energies of these orbitals and at the possible ways in which electrons can be raised from a
lower to a higher energy level. Obviously, the colors produced are closely related to the size
of the spacing between these levels. Since this distance depends on factors such as the
geometry of the complex, the nature of the ligands present, and the oxidation state of the
central metal atom, electronic spectra of complexes can provide valuable information
regarding bonding and structure.
01-01: ELECTRONIC SPECTRA OF COMPLEX I ONS
This is also known as UV-Vis spectroscopy and covers the region from 200 to 800 nm
of the electromagnetic spectrum. The region from 200 to 400 nm is called ultraviolet (UV)
and that from 400 to 800 nm is known as the visible region. A substance appeared colored
when the emitted light falls in this visible region. This spectroscopy is used to find out the

CHE509: Inorganic Chemistry – II Page 102


geometry of complexes because the number of bands seen in the visible region due to d-d
transitions depends on the geometry of the complex.

Example (1): [Ti(H 2 O) 6 ] 3+ ; its electronic spectrum is given in Figure


below, which shows only one max

There is only one absorption near 500nm, which indicates that there is only one
transition taking place in this complex. It means that the d orbitals are spilt into only
two levels.

**Note: To express the value of the crystal field splitting parameter (Δ) for this
ion in cm -1 : we know that:
Wave number=1/ Wavelength
= 1/500nm
[1nm =10 -7 cm]
=0.002x10 7 = 20000cm -1
**Note: To obtain the crystal field splitting energy (Δ) of this complex compound
From its absorption spectrum:
Thewavenumberof20000cm -1 isassociatedwithenergywhichisequalto
239.23KJmol -1 as shown below:
We know that since 83.6cm -1 = 1KJ mol -1
20000/83.6KJmol -1 =239.23KJmol -1

This energy is the energy difference between t 2g and e g sets of orbitals(Figurebelow)

We know that since


83.6cm -1 = 1KJ mol -1

CHE509: Inorganic Chemistry – II Page 103


20000/83.6KJmol -1 =239.23KJmol -1

**Note: The higher is the value of the frequency in cm -1 (wave number) of the
peak of maximum absorption in the absorption spectrum of a given complex
ion, greater is the magnitude of
Example (2): [Cr(en) 3 ] 3+ , its electronic spectrum (Figure below) shows two max

indicating two transitions. This shows that d-orbitals are split into more than
one energy level.
Example (3): [CrF 6 ] 3- , the electronic spectrum of this complex (Figure below)
Shows three absorptions indicating three transitions. This shows the splitting o f
d orbital’s in a different way

These examples show that the number of absorptions for the complexes varies even
though two of them contain chromium and in the oxidation state +3. Moreover, they
are octahedral complexes. Hence, the difference in the number of absorptions is due

CHE509: Inorganic Chemistry – II Page 104


to the nature of the ligands. Similarly, it can be shown that the number of
absorptions will also vary depending upon the geometry of the complex as well as
the oxidation state.
SHORT ANSWER QUESTIONS WITH MODEL ANSWER 01
1] Explain the absorption spectra of Cr +3 in short
3-
Solution: [CrF 6 ] , the electronic spectrum of this complex (Figure below) Shows
three absorptions indicating three transitions. This shows the splitting of d orbitals in
a different way

These examples show that the number of absorptions for the complexes varies
even though two of them contain chromium and in the oxidation state +3. Moreover,
they are octahedral complexes. Hence, the difference in the number of absorptions is
due to the nature of the ligands. Similarly, it can be shown that the number of
absorptions will also vary depending upon the geometry of the complex as well as
the oxidation state.
01-02: TYPES OF ELECTRONIC SPECTRA OF COMPLEXES
1) Ligand Spectra
2) Charge-transfer Spectra
3) d-d Spectra
4) Counter-ion Spectra
1) LIGAND SPECTRA
Most of the organic ligand has absorption bands that usually appear in UV region,
especially when these molecules contain π electron system.

CHE509: Inorganic Chemistry – II Page 105


All the (UV-Vis) spectra of the organic ligands show two absorption peaks:
*
1- → these transitions found in the molecules that have double or triple
bonds and without atoms having nonbonding electrons. Examples olefins and
aromatic system; benzene and diene.
*
2- → These transitions found in the molecules that have a lone pair and
bond

e.g. Ketones C=O, Aldehydes-CHO, C≡N and pyridine

2) CHARGE-TRANSFER SPECTRA

There are cases where the absorption bands in the visible or ultraviolet
regions ranges between ε values of 1000 to 55,000 Lmol -1 cm -1 . For these cases such
high value of absorption has been suggested due to charge transfer bands which are
so much intense since they are allowed transitions that transmit exceptionally deep
colors to the respective transition metal complex. Examples include KMnO 4 ,
K 2 CrO 4 , [Fe(bipy) 3 ] 2+, Cr(CO) 6 , [Ir(Br) 6 ] 2- , [Ni(Cl) 4 ] 2- ,etc. Incharge transfer
transition either electron is donated from the low lying orbitals of the ligand to the
metal or from orbitals of the metal to the ligand.

CHE509: Inorganic Chemistry – II Page 106


Origin of charge transfer bands
TYPES OF CHARGE TRANSFER SPECTRA
There are chiefly three types of charge transfer spectrum

(a) Ligand to metal charge transfer spectrum(LMCT)


(b) Metal to ligand charge transfer spectrum(MLCT)
(c) Metal to metal charge transfer spectrum(MMCT)

a) LIGAND TO METAL CHARGE TRANSFER SPECTRUM (LMCT)


In these types of transitions, the transfer of electron occurs from the orbital’s
that are ligand based to the orbital’s that are metal based. An example of these type
of complexes includes [Cr (NH 3 ) 6 ] 3+ , [Cr (Cl)(NH 3 ) 5 ] 2+ . The LMCT charge transfer
spectrum has been shown in Figure below.

CHE509: Inorganic Chemistry – II Page 107


Mostly in these cases the ligands are good σ or π donors. Generally, the
transitions lead to metal reduction takes place. Thus metal which is easily reduced
combines with the ligand that is easily oxidized giving rise to a transition low in
energy. Therefore, anions that are easily oxidized like I - often form complexes where
charge transfer absorption in the visible region is quite appreciable. The examples
include TiI 4 which is bright violet,HgI 2 red and AgI that is vivid yellow in color.

b) METAL TO LIGAND CHARGE TRANSFER SPECTRUM (MLCT)


In these type of transitions basically the metal orbitals are involved that can
easily supply their electrons present in the low lying molecular orbital to the empty
π* orbitals of the ligand. Example of complexes showing these type of transitions
is[Fe(CO) 3 (bipy)], [Ru(bipy) 3 ] 2+ , [W(CO) 4 (phen)] etc. in all these cases the π*
empty orbital present on the ligand becomes the receptor of electrons with the
introduction of light and the absorption process.
In the process of charge relocation, the metal is oxidized and the ligand is reduced ,
therefore for this type of charge transfer phenomenon, it is important that the
metaloxidation as well as ligand reduction is quite feasible. Easily reducible lig ands
are thosewhich have a low lying, vacant π* orbital, such as pyridine, which then
forms stable colored complexes with the metal ions that are easily oxidized such
asFe 2+ and Cu

c) METAL TO METAL CHARGE TRANSFER SPECTRUM (MMCT)


Somecompoundspossessmetalionsintwodifferentoxidationstates.Inthesecompo
unds, a charge transfer transition may occur when the electron moves from one metal
ion to the other, with one metal ion acting as the reducing agent and the other acting
as the oxidizing agent. Compounds of this nature are generally very intensely
colored, such as Prussian Blue, KFe III [Fe II(CN) 6 ].

3) d-d Spectra
In case of transition metal complexes with octahedral geometry the most
important type of electronic transition taking place is the d-d transition where
transition of an electron takes place from the lower t 2g level to the upper e g level.
Mostly it is the transition which imparts the color to the complex since it occurs in
the visible or ultraviolet part of the spectrum. But the value of molar extinction
coefficient, ε for these transitions is quite low since, these are Laporte forbidden

CHE509: Inorganic Chemistry – II Page 108


transitions. Hence for d-d transitions, the value of molar extinction coefficient, ε-
ranges from0.5 upto 20Lmol -1 cm -1 .
4) COUNTER-ION SPECTRA
Counterionistheion(negativeorpositive)whichboundtothecomplexiontobalancet
hecharge.TherearemanyionsthathavehighintensityabsorptionbandsinUVregion(250 -
390nm)likeOxyanions(NO 3 - ,NO 2 -
).MostofionsdonothaveabsorptionpeaksinUVregionlikeSO 4 2- ,Cl - andClO 4 -
thusthereisnointerferencewithcomplexionspectra,thereforetheseionsarepreferredascou
nterions.
Example: Identify all the expected electronic transitions in UV-Visible for the
following ions:
1)[Cr(C 2 O 4 ) 3 ] 3-
For [Cr(C 2 O 4 ) 3 ] 3-
Cr 3+ [ 18 Ar]3d 3 4s 0
a) Weak absorption bands in the visible region 400-800 nm attributed to d-d
transitions of Cr 3+ ion.
b) Veryintensityabsorptionbandsofchargetransferintheregion300 -380nmareattributed
to LMCT.
c) Absorptionbandsintheregion200-350nmareassignedtotheligandspectra
* *.
→ and n→
2)[Ti(H 2 O) 6 ] 3+
Ti 3+ [ 18 Ar]3d 1 4s 0
a) There are no transitions of electrons in UV region.
b) Electronicabsorptioninthevisibleregion400-800nmisattributedtod-dofTi 3+ ion.

SOLVED PROBLEMS 02
1] Explain the concept of ligand to metal and metal to ligand charge transfer spectra
Solution :
Ligand to Metal Charge Transfer Spectrum (LMCT)

In these types of transitions, the transfer of electron occurs from the orbitals
that are ligand based to the orbital’s that are metal based. An example of these type
of complexes includes [Cr (NH 3 ) 6 ] 3+ , [Cr(Cl)(NH 3 ) 5 ] 2+ . The LMCT charge transfer
spectrum has been shown in Figure below.

CHE509: Inorganic Chemistry – II Page 109


Mostly in these cases the ligands are good σ or π donors. Generally, the transitions
lead to metal reduction takes place. Thus metal which is easily reduced combines
with the ligand that is easily oxidized giving rise to a transition low in energy.
Therefore, anions that are easily oxidized like I - often form complexes where charge
transfer absorption inthe visible region is quite appreciable. The examples include
TiI4 which is bright violet,HgI 2 red and AgI that is vivid yellow in color.

Metal to Ligand Charge Transfer Spectrum (MLCT)

In these type of transitions basically the metal orbitals are involved that can
easily supply their electrons present in the low lying molecular orbital to the empty
π*orbital’s of the ligand. Example of complexes showing these type of transitions is
[Fe(CO) 3 (bipy)], [Ru(bipy) 3 ] 2+ , [W(CO) 4 (phen)] etc. in all these cases the π* empty
orbital present on the ligand becomes the receptor of electrons with the introduction
of light and the absorption process.

In the process of charge relocation, the metal is oxidized and the ligand is
reduced, therefore for this type of charge transfer phenomenon, it is important that
the metal oxidation as well as ligand reduction is quite feasible. Easily reducible
ligands are those which have a low lying, vacant π* orbital, such as pyridine, which
then forms stable colored complexes with the metal ions that are easily oxidized
such asFe 2+ and Cu

2] Describe the d-d spectra and Counter-ion spectra


Solution: d-d Spectra

CHE509: Inorganic Chemistry – II Page 110


In case of transition metal complexes with octahedral geometry the most important
type of electronic transition taking place is the d-d transition where transition of an
electron takes place from the lower t 2g level to the upper e g level. Mostly it is the
transition which imparts the color to the complex since it occurs in the visible or
ultraviolet part of the spectrum. But the value of molar extinction coefficient, ε for
these transitions is quite low since, these are Laporte forbidden transitions. Hence
for d-d transitions, the value of molar extinction coefficient, ε-ranges from 0.5up to
20 Lmol -1 cm -1 .

Counter-ion Spectra

Counter ion is the ion (negativeorpositive) which bound


tothecomplexiontobalancethecharge. There are many ions that have high intensity
absorption bands in UVregion (250-390nm) like Oxyanions (NO 3 - ,NO 2 - ). Most of
ions do not have absorption peaks in UV region like SO 4 2- , Cl - and ClO 4 - thus there is
no interference with complex ion spectra, and therefore these ions are preferred as
counter ions.

SHORT ANSWER QUESTIONS WITH MODEL ANSWER 02


1] Write a short note on Counterion spectra.

Solution: Counter ion is the ion (negativeorpositive) which bound to the complex
ion to balance the charge. There are many ions that have high intensity absorption
bands in UVregion (250-390nm) like Oxyanions (NO 3 - ,NO 2 - ). Most of ions do not
have absorption peaks in UV region like SO 4 2- , Cl - and ClO 4 - thus there is no
interference with complex ion spectra, and therefore these ions are preferred as
counter ions.

2] Write a short note on Counterion spectra.


Solution: In case of transition metal complexes with octahedral geometry the most
important typeof electronic transition taking place is the d-d transition where
transition of an electron takesplace from the lower t 2g level to the upper e g level.
Mostly it is the transition which impartsthe color to the complex since it occurs in
the visible or ultraviolet part of the spectrum. Butthe value of molar extinction
coefficient, ε for these transitions is quite low since, these are Laporte forbidden
transitions. Hence for d-d transitions, the value of molar extinction coefficient, ε-
ranges from 0.5upto20Lmol -1 cm -1 .

CHE509: Inorganic Chemistry – II Page 111


01-03: SELECTION RULESFORELECTRONICTRANSITIONS
The relative intensities of absorption are governed by a series of selection rules. On
the bases of the symmetry and spin multiplicity of ground and excited electronic
states, two of these rules may be stated; these rules help us in finding out the
possible transitions between the different energy levels. Transitions taking place
obeying the selection rules are called allowed transitions and those taking place
without obeying the selection rules are called forbidden transitions.
RULE1: LAPORTE SELECTION RULE
Transitions will be allowed only when there is a change in parity. That is, from
gerade to ungerade( g→u) and vice versa ( u→g); s→p , p →dtransitions are
allowed, but( g →g ) and(u→u);d→d ,p→p are forbidden.
According to this selection rule ΔL= 1for an allowed transitions.
In centrosymmetric molecules, that is, molecules with center of symmetry, the d
orbitalshave gerade symmetry (g symmetry) and hence, d-d transitions are forbidden
(not allowed).In other words, if an octahedral complex has a center of symmetry, the
d-d transitions in thatcomplex are forbidden.
**Note: However, in complexes, d-d transitions are taking place in electronic
spectra and they not only give important clues about the structure of the
complex but also proof the theories of bonding in complexes.
D-P MIXINGAND THEBREAKDOWN OFLAPORTESELECTIONRULE

In tetrahedral complexes, there is no center of symmetry and hence, this selection


rule isnot applicable. Moreover, the MOs formed have the character of both d and p
orbitals fromwhich they are formed. This is calledd-p mixing because the p orbitals
have (u) characterand thedorbitalshave (g) character.
**Note: In a molecule which is noncentro symmetric (e.g. tetrahedral), p -d mixing
can occur to a greater extent and so the probability of d-d transitions is greater
than in acentro symmetric complex. This leads to tetrahedral complexes being
more intensely colored than octahedral complexes.

CHE509: Inorganic Chemistry – II Page 112


SOLVED PROBLEMS 03
1] Explain the Laporte selection rules determination of term symbol
Solution: Transitions will be allowed only when there is a change in parity. That is,
from gerade to ungerade( g→u) and vice versa ( u→g); s→p , p →dtransitions are
allowed, but( g →g ) and(u→u);d→d ,p→p are forbidden.
According to this selection rule ΔL= 1for an allowed transitions.
In centrosymmetric molecules, that is, molecules with center of symmetry, the d
orbitalshave gerade symmetry (g symmetry) and hence, d-d transitions are forbidden
(not allowed).In other words, if an octahedral complex has a center of symmetry, th e
d-d transitions in thatcomplex are forbidden.
However, in complexes, d-d transitions are taking place in electronic
spectraandtheynotonlygiveimportantcluesaboutthestructureofthecomplexbutalsoproof
thetheoriesofbondingin complexes.
D-P MIXINGAND THEBREAKDOWN OF LAPORTESELECTIONRULE

In tetrahedral complexes, there is no center of symmetry and hence, this


selection rule isnot applicable. Moreover, the MOs formed have the character of both
d and p orbitals fromwhich they are formed. This is calledd-p mixing because the p
orbitals have (u) characterand thedorbitalshave (g) character.
In a molecule which is noncentro symmetric (e.g. tetrahedral), p -d mixing
canoccurtoagreaterextentandsotheprobabilityofd-
dtransitionsisgreaterthaninacentrosymmetriccomplex.Thisleadstotetrahedralcomplex
esbeingmoreintenselycolored than octahedralcomplexes.
2] Explain the Spin Selection rule for determination of term symbol
Solution: According to this selection rule, (no change in total spin) or S = 0
for an allowed transition (there is no changein the number of unpaired electrons
before and after the transition).

S = S 1 – S 2 =(+1/2)– (+1/2)

Total change in spin =0


ΔS=0

CHE509: Inorganic Chemistry – II Page 113


In other words, there should not be any change in the spin of the states and
their multiplicities should be the same during the transition of electrons.That is,
singlet-singletand triplet-triplet transitions are allowed like 3A2g →3T2g , while singlet-triplet
and vice-versatransitions arenotallowedlike 1 A 1g →3 T 1g .
SHORT ANSWER QUESTIONS WITH MODEL ANSWER 03
1] Explain in short d-p mixing
Solution: In tetrahedral complexes, there is no center of symmetry and hence,
this selection rule isnot applicable. Moreover, the MOs formed have the character of
both d and p orbitals fromwhich they are formed. This is calledd-p mixing because
the p orbitals have (u) characterand thedorbitalshave (g) character.
01-04: TERM SYMBOL
Absorptions result in the excitation from lower to higher states energy. Because
thesestates are quantized, we observe absorption in bands. These bands can have
determined from interactions or coupling of electrons interms of their orbital angular
momenta (their m l value) and spin angular momenta (their msvalues). This is called
Russell-Saunders coupling (LScoupling). In a free atom (one in absence of a ligand
field) these interactions produce atomic states called microstates which are the
detailed electronic configuration of atom or ion.

Term is denoted by symbol, n L j , where n=(2S+1) and called the spin


multiplicity.
S is the total spin of the electrons.

0 1 2 3 4 5 6
L
Terms S P D F G H I
symbol

Were L be the angular momentum


In the interpretation of spectra of coordination compounds, it is often important
to identify the lowest-energy term.
Example: [Cr (NH 3 ) 6 ] 3+ Cr 3+ [ 18 Ar] 3d 3 4s 0
L =+2+1+0 =3; therefore, F term and S = 3/2

CHE509: Inorganic Chemistry – II Page 114


Spin multiplicity =2S+1 = 4Therefore,the ground term is 4 F
The relation ship between the number of unpaired electrons and
multiplicity is shown in Table below:

Absorption spectra of coordination compounds in most cases involve the d


orbitals of the metal, and it is consequently important to know the free-ion terms for
the possible d configurations. For reference, therefore, these are listed for the
possible d electron configurations infollowing Table:

Arranging the terms in the increasing order of energy


Hund ’ srules
Rule1: Maximum multiplicity will have minimum energy, when we have 1 S, 1 D and
3
P states, 3 P state should be the ground state.
Rule 2: If two states have the same multiplicity, then the one with higher L value will have
lower energy.
Example: 3P and 3F
L = 1 for P and 3 for F. Hence 3F, 3P
For d2 configuration, these two rules allow us to order the terms according to
increasing energy: 3F< 3P<1G <1D<1S

CHE509: Inorganic Chemistry – II Page 115


We need a new way of writing electron configurations that allows us to not only
describe the ground state, but any possible excited states as well. We can do this by
describing the electronic state according to its orbital and spin degeneracy.
Splitting of dn Terms
Each of these free ion terms will be affected by the ligands in a complex and this will
depend upon the geometry of the complex. As an example, d orbitals will be split into t2g and
eg orbitals and in the same way, 2D terms will also be split into 2T2g and 2Eg terms in an
octahedral complex. In the same way, other terms arising from the free ion terms will be
transformed and split as follows in an octahedral field.
If the ligands cause large splitting between the terms, it is called strong field case. If
they cause small splitting between the terms, it is called weak field case. In strong field case,
electrons will try to pair whenever possible and in weak field case, the electrons will try to
remain unpaired.
The ground state terms obtained from d1 and d2 configurations are split by octahedral
field and the resulting energy diagrams are shownbelow:

SOLVED PROBLEMS 04
1] Describe the process of determination of term symbol with suitable example
Solution: Absorptions result in the excitation from lower to higher states
energy. Because thesestates are quantized, we observe absorption in bands. These
bands can have determined from interactions or coupling of electrons interms of
their orbital angular momenta (their m l value) and spin angular momenta (their msvalues).
This is called Russell-Saunders coupling (LScoupling). In a free atom (one in
absence of a ligand field) these interactions produce atomic states called microstates
which are the detailed electronic configuration of atom or ion.

Term is denoted by symbol, n L j , where n=(2S+1) and called the spin


multiplicity.

CHE509: Inorganic Chemistry – II Page 116


S is the total spin of the electrons.

0 1 2 3 4 5 6
L
Terms S P D F G H I
symbol

Were L be the angular momentum

In the interpretation of spectra of coordination compounds, it is often important


to identify the lowest-energy term.
Example: [Cr(NH 3 ) 6 ] 3+ Cr 3+ [ 18 Ar] 3d 3 4s 0
L =+2+1+0 =3; therefore, F term and S = 3/2
Spin multiplicity =2S+1 = 4Therefore, the ground term is 4 F
SHORT ANSWER QUESTIONS WITH MODEL ANSWER 04
1] Explain the Hunds rules for the multiplicity to determine ground state term
Solution:
Rule1: Maximum multiplicity will have minimum energy, when we have 1S,
1D and 3P states, 3P state should be the ground state.
Rule 2: If two states have the same multiplicity, then the one with higher L value will
have lower energy.
Example: 3P and 3F
L = 1 for P and 3 for F. Hence 3F, 3P
For d2 configuration, these two rules allow us to order the terms according to
increasing energy: 3F< 3P<1G <1D<1S
We need a new way of writing electron configurations that allows us to not only
describe the ground state, but any possible excited states as well. We can do this by
describing the electronic state according to its orbital and spin degeneracy.

CHECK POINT 01-04


1] Electronic excitations are observed in
1) IR Radiation
2) UV-Visible Radiations
3) Microwave
4) X-rays

CHE509: Inorganic Chemistry – II Page 117


Solution: a

2] Calculate term symbol for following complexes


Soluion: [Fe(H 2 O) 6 ] +3
Fe +3 [ 18 Ar] 3d 3 4s 0

L =+2+1+0 =3; therefore, F term and S = 3/2


Spin multiplicity =2S+1 = 4 Therefore, the ground term is 4 F

3] Term symbol is an abbreviated description of____


a) Total spin only
b) orbital angular momentum
c) Total spin and orbital angular momentum
d) None of these
Solution: c

4] Equation is used for meaurment of


a) Magnetic Moment
b) Term Symbol
c) Total Spin
d) Anguler Momentum
Solution: a

CHE509: Inorganic Chemistry – II Page 118


SUMMARY
A substance appeared colored when the emitted light falls in this visible region.
This spectroscopy is used to find out the geometry of complexes because the number
of bands seen in the visible region due to d-d transitions depends on the geometry of
the complex.Electronic spectroscopy relies on the quantized nature of energy st ates.
With sufficient energy, an electron can be excited from its initial ground state or
initial excited state (hot band) and briefly exist in a higher energy excited state.
Electronic transitions involve the excitation of anelectron from one principle
quantum state to another. Without a stimulus, an electron will not move to a higher
level. Only by absorbing energy can an electron be excited.
KEY WORDS
Spectra, Transitions, Selection rule, Term symbols
REFERENCES

MOOCS
______
YOUTUBE VIDEOS
1) https://www.youtube.com/watch?v=9YoRq0s_ZI0
2) https://www.youtube.com/watch?v=8MP5Mfue4Q4&list=PL0KRvN5K
p6y8YALa3i4q5joGrm2oYYzUY
3) https://www.youtube.com/watch?v=bu5CMsNtnAU
4) https://www.youtube.com/watch?v=Exm7LLZ38zs
WIKIPEDIA
1) https://en.wikipedia.org/wiki/Term_symbol
2) https://en.wikipedia.org/wiki/Hund's_rule_of_maximum_multiplicity
OER
1) https://uomustansiriyah.edu.iq/media/lectures/6/6_2021_09_15!08_19_16_PM
.pdf
2) https://chem.libretexts.org/Courses/Saint_Marys_College_Notre_Dame_IN/C
HEM_431%3A_Inorganic_Chemistry_(Haas)/CHEM_431_Readings/13%3A_Sp
ectroscopic_and_Magnetic_Properties/13.02%3A_Electronic_Spectra_of_Coordi
nation_Compounds

CHE509: Inorganic Chemistry – II Page 119


3) https://www.dalalinstitute.com/wp-content/uploads/Books/A-Textbook-of-
Inorganic-Chemistry-Volume-1/ATOICV1-8-0-Electronic-Spectra-of-Transition-
Metal-Complexes.pdf
REFERENCE BOOKS
1) Inorganic Chemistry Principles of Structure and Reactivity , 2006 by J
Huheey , Medhi
2) Electronic Spectra of Transitions Metal Complexes, 2015by Vinod Jena

CHE509: Inorganic Chemistry – II Page 120


CREDIT 02 -UNIT 02: ORGEL DIAGRAM AND TANABE – SUGANO
ENERGY LEVEL DIAGRAMS
LEARNING OBJECTIVES
After successful completion of this unit, you will be able to
 Understand the use of Orgel diagram and Tanabe Sugano diagram
 Understand the concept of Jahn-Teller effect for distortion
 Understand the spin orbit coupling concept
 Understand the concept of nephelauxetic effect
 Understand process of photoluminescence
INTRODUCTION
Orgel diagrams are correlation diagrams that show the relative energies of
electronic terms in transition metal complexes, similar to Tanabe–Sugano diagrams.
They are named after their creator Leslie Orgel. Organ diagrams are limited to
showing weak-field (ie high-spin) cases and offer no information on strong-field
(low-spin) cases. Since Orgel diagrams are qualitative, no energy calculations can be
made from these diagrams; also, Orgel diagrams show only the symmetry states with
the highest spin multiplicity instead of all possible terms, unli ke the Tanabe-Sugano
diagram. However, Orgel diagrams will show the number of rotation transitions
allowed, along with the appropriate symmetry label. In the Orgel diagram, the parent
member (P, D, or F) in the presence of no ligand field is located in the center of the
diagram, with members due to this electronic configuration in the ligand field on
each side. There are two Orgel diagrams, one for configurations d 1 , d 4 , d 6 , and d 9 .,
and another with configurations d 2 , d 3 , d 7 , and d 8 .

02-01: ORGEL DIAGRAM


Covered on the left are d 1 ,d 6 tetrahedral and d 4 ,d 9 octahedral complexes, and on
the right, d 4 ,d 9 tetrahedral and d 1 ,d 6 octahedral. For simplicity, the g indices required
for octahedral complexes are not shown. For complexes with F base members, three
electron transitions are expected and D may not directly correspond to the transition
energy. These are the following configurations: d 2 , d 3 , high spin d 7 and d 8 .

CHE509: Inorganic Chemistry – II Page 121


D Orgel diagram

F Orgel diagram
Orgel diagrams are useful for representing the energy levels of both octahedral
and tetrahedral high-spin transition metal ions. They only display rotation enabled
transitions. For complexes with D basis members, only one electron transition is
expected, and the energy of the transition corresponds directly to D. Therefore, the
following high-spin configurations are involved: d 1 , d 4 , d 6 , and d 9 .
On the left, d 2 , d 7 tetrahedral and d 3 , d 8 octahedral complexes are covered, and on
the right, d 3 , d 8 tetrahedral and d 2 and high spin d 7 octahedral. Again, for simplicity,
the g indices required for octahedral complexes are not given. On the left side, the
first transition corresponds to D, the equation for calculating the second contains
expressions with both D and C.I. (configurational interactions from repulsion of like
terms) and the third has terms that contain D, C.I. and Racah parameter B.
4
 T 2g <--- 4 A 2g transition energy = D

CHE509: Inorganic Chemistry – II Page 122


4
 T 1g (F) <--- 4 A 2g transition energy = 9/5 *D - C.I.
4
 T 1g (P) <--- 4 A 2g transition energy = 6/5 *D + 15B' + C.I.

On the right hand side, the first transition can be unambiguously assigned as:
3
T 2g <--- 3 T 1g transition energy = 4/5 *D + C.I.

But, depending on the size of the ligand field (D) the second transition may be due
to:
3
A 2g <--- 3 T 1g transition energy = 9/5 *D + C.I.

for a weak field or


3
T 1g (P) <--- 3 T 1g transition energy = 3/5 *D + 15B' + 2 * C.I.

for a strong field.


TANABE-SUGANO DIAGRAMS

An alternative method is to use Tanabe Sugano diagrams, which are able to


predict transition energies for both spin-allowed and spin-forbidden transitions, as
well as strong-field (low-spin) and weak-field (high-spin) complexes. In this
method, the energy of the electronic states is listed on the vertical axis and the
ligand field strength increases on the horizontal axis from left to right. Linear lines
are found when there are no other members of the same type, and curved lines are
found when 2 or more members are repeated. It is a consequence of the "no -crossing
rule". The baseline in the Tanabe-Sugan diagram represents the lowest energy or
ground state.The electronic spectrum of the V 3+ ion, where V(III) is doped
into alumina (Al 2 O 3 ), shows three major peaks with frequencies of: ν1ν1=17400 cm -
1
, ν2ν2=25400 cm -1 and ν3ν3=34500 cm -1 .

These have been assigned to the following spin-allowed transitions.


3
 T 2g <--- 3 T 1g
3
 T 1g (P)<--- 3 T 1g
3
 A 2g <--- 3 T 1g
The ratio between the first two transitions is calculated as ν1ν2ν1ν2 which is
equal to 25400 / 17400 = 1.448.To calculate the Racah parameter, B, the position on
the horizontal axis where the ratio between the lines representing ν2ν2 and ν1ν1 is
equal to 1.448, has to be determined. On the diagram below, this occurs at D/B=30.9.
Having found this value, a vertical line is drawn at this position.

CHE509: Inorganic Chemistry – II Page 123


Tanabe-Sugano diagram for d2 octahedral complexes
Moving along the line from the base member to where it is crossed by the
lines from the other members, we are able to identify both spin forbidden and spin
allowed, and therefore the total number of transitions that are possible in the
electronic spectrum.
Next, find the values on the vertical axis that correspond to the transitions
allowed by the rotation to determine the values of n1/B, n2/B, and n3/B. From the
diagram above, they are 28.78, 41.67 and 59.68.
Knowing the values of n1, n2, and n3, we can now calculate the value of B. Since
n1/B=28.78 and n1 is equal to 17,400 cm-1, then
B=n1/28.78
=17400/28.78
or
B=604.5cm −1
Then it is possible to calculate the value of D. Since D/B=30.9, then: D=B X 30.9
and hence:
D = 604.5 X 30.9 = 18680 cm -1

Calculate the value of B and D for the Cr 3+ ion in [Cr(H 2 O) 6 )] 3+ if n1=17000


cm 1 , n2=24000 cm -1 and n3=37000 cm -1 .

CHE509: Inorganic Chemistry – II Page 124


Solution
These values have been assigned to the following spin -allowed transitions.
4
T 2g ← 4 A 2g
4
T 1g ← 4 A 2g
4
T 1g (P)← 4 A 2g
From the information given, the ratio \(n_2 / n_1 = 24000 / 17000 = 1.412\). Using
a Tanabe-Sugano diagram for a d 3 system this ratio is found at D/B=24.00

Tanabe-Sugano diagram for d 3 octahedral complexes

It is observed that the value of Racah parameter BB in the complex is 708.3 cm -


1
, while the value of B in the free Cr3+ ion is 1030 cm -1. This shows a 31% reduction in
the Racah parameter indicating a strong Nephelauxetic effect.
The Nephelauxetic Series is as follows:
F->H2O>urea>NH3 >en~C2O42- >NCS- >Cl-~CN->Br- >S2- ~I-.
Ionic ligands such as F -give small reduction in B, while covalently bonded ligands such
as I- give a large reduction in B.
SOLVED PROBLEMS 01
1] What is orgel diagram? Give orgel diagram for for D and F terms
Solution: Covered on the left are d 1 ,d 6 tetrahedral and d 4 ,d 9 octahedral complexes,
and on the right, d 4 ,d 9 tetrahedral and d 1 ,d 6 octahedral. For simplicity, the g indices
required for octahedral complexes are not shown. For complexes with F base
members, three electron transitions are expected and D may not directly correspond

CHE509: Inorganic Chemistry – II Page 125


to the transition energy. These are the following configurations: d 2 , d 3 , high spin d 7
and d 8 .

D Orgel diagram

F Orgel diagram

Orgel diagrams are useful for representing the energy levels of both octahedral
and tetrahedral high-spin transition metal ions. They only display rotation enabled
transitions. For complexes with D basis members, only one electron transition is
expected, and the energy of the transition corresponds directly to D. Therefore, the
following high-spin configurations are involved: d 1 , d 4 , d 6 , and d 9 .

On the left, d 2 , d 7 tetrahedral and d 3 , d 8 octahedral complexes are covered, and on


the right, d 3 , d 8 tetrahedral and d 2 and high spin d 7 octahedral. Again, for simplicity,
the g indices required for octahedral complexes are not given. On the left side, the
first transition corresponds to D, the equation for calculating the second contains

CHE509: Inorganic Chemistry – II Page 126


expressions with both D and C.I. (configurational interactions from repulsion of like
terms) and the third has terms that contain D, C.I. and Racah parameter B.
4
 T 2g <--- 4 A 2g transition energy = D
4
 T 1g (F) <--- 4 A 2g transition energy = 9/5 *D - C.I.
4
 T 1g (P) <--- 4 A 2g transition energy = 6/5 *D + 15B' + C.I.
On the right hand side, the first transition can be unambiguously assigned as:
3
T 2g <--- 3 T 1g transition energy = 4/5 *D + C.I.
But, depending on the size of the ligand field (D) the second transition may be due
to:
3
A 2g <--- 3 T 1g transition energy = 9/5 *D + C.I.
for a weak field or
3
T 1g (P) <--- 3 T 1g transition energy = 3/5 *D + 15B' + 2 * C.I.
for a strong field.
2] Discribe in detail about tanabe Sugano diagrams for prediction of electronic
transitions.
Solution: An alternative method is to use Tanabe Sugano diagrams, which are
able to predict transition energies for both spin -allowed and spin-forbidden
transitions, as well as strong-field (low-spin) and weak-field (high-spin) complexes.
In this method, the energy of the electronic states is listed on the vertical axis and
the ligand field strength increases on the horizontal axis from left to right. Linear
lines are found when there are no other members of the same type, and curved lines
are found when 2 or more members are repeated. It is a consequence of the "no -
crossing rule". The baseline in the Tanabe-Sugan diagram represents the lowest
energy or ground state.

The electronic spectrum of the V 3+ ion, where V(III) is doped into


alumina (Al 2 O 3 ), shows three major peaks with frequencies of: ν1ν1=17400 cm -
1
, ν2ν2=25400 cm -1 and ν3ν3=34500 cm -1 .
These have been assigned to the following spin-allowed transitions.
3
 T 2g <--- 3 T 1g
3
 T 1g (P)<--- 3 T 1g
3
 A 2g <--- 3 T 1g
The ratio between the first two transitions is calculated as ν1ν2ν1ν2 which is
equal to 25400 / 17400 = 1.448.To calculate the Racah parameter, B the position on

CHE509: Inorganic Chemistry – II Page 127


the horizontal axis where the ratio between the lines representing ν2ν2 and ν1ν1 is
equal to 1.448, has to be determined. On the diagram below, this occ urs at D/B=30.9.
Having found this value, a vertical line is drawn at this position.

Tanabe-Sugano diagram for d2 octahedral complexes

Moving along the line from the base member to where it is crossed by the
lines from the other members, we are able to identify both spin forbidden and spin
allowed, and therefore the total number of transitions that are possible in the
electronic spectrum.

Next, find the values on the vertical axis that correspond to the transitions
allowed by the rotation to determine the values of n1/B, n2/B, and n3/B. From the
diagram above, they are 28.78, 41.67 and 59.68.
Knowing the values of n1, n2, and n3, we can now calculate the value of B. Since
n1/B=28.78 and n1 is equal to 17,400 cm-1, then
B=n1/28.78
=17400/28.78
or

B=604.5cm −1

CHE509: Inorganic Chemistry – II Page 128


SHORT ANSWER QUESTIONS WITH MODEL ANSWER 01

1] Give Orgal diagram for D term

Solution:

D Orgel diagram

2] Give Orgal diagram for F term

Solution:

F Orgel diagram

02-02: SPECTROCHEMICAL SERIES


The spectrochemical series is a list of ligands ordered by ligand "strength"
and a list of metal ions based on oxidation number, group, and element. For metal
ions, ligands modify the energy difference Δ between the d orbitals, called the ligand
field splitting parameter in ligand field theory o r the crystal field splitting parameter
in crystal field theory. The splitting parameters are reflected in the ion's electronic

CHE509: Inorganic Chemistry – II Page 129


and magnetic properties, such as spin states, and optical properties, such as color
and absorption spectra.
A partial spectrochemical series listing of ligands from small Δ to large Δ is
given below.
I− < Br − < S 2− < SCN − (S–bonded) < Cl − < N 3 − < F − < NCO − < OH − < C 2 O 4 2− <
O 2− < H 2 O < acac − (acetylacetonate) < NCS − (N–bonded) < CH 3 CN < gly (glycine)
< py (pyridine) < NH 3 < en (ethylenediamine) < bipy (2,2'-bipyridine) < phen (1,10-
phenanthroline) < NO 2 − (N–bonded) < PPh 3 < CN − < CO
Weak field ligands: H 2 O, F − , Cl − , OH −
Strong field ligands: CO, CN − , NH 3 , PPh 3

The ligands on the far left of this spectrochemical series are generally
considered to be weak ligands, unable to induce forced electron pairing within the 3d
level, thus giving rise to high-spin octahedra in external orbitals. forms a complex.
On the other hand, the rightmost ligand is a stronger ligand, which forms an internal
orbital octahedral complex within the 3d level after forced electron pairing, and
therefore, called. However, it is known that ``spectrochemica l sequences are
inherently set back from what they should be for reasonable predictions based on the
assumptions of crystal field theory''. This departure from crystal field theory
highlights a weakness in the assumption of crystal field theory: a purely i onic bond
between metal and ligand. The order of the spectrochemical series can be derived
from the recognition that ligands are often classified according to their donor or
acceptor potency. Like NH 3 , it lacks orbitals with good symmetry for π-bond
interactions and may be only σ-bond donors. Binding of these ligands to metals is
relatively straightforward as only σ bonds are used to create relatively weak
interactions. Another example of a σ-bonded ligand is ethylenediamine. However,
ethylenediamine has a stronger effect than ammonia; producing a larger ligand field
splitting Δ. Ligands with occupied p-orbitals are potential π-donors. These types of
ligands tend to donate these electrons along with the σ bonding electrons to the
metal, exhibiting stronger metal-ligand interactions and effective reduction of Δ.
Most halide ligands and OH- are prime examples of π-donor ligands.

If a ligand has vacant π* and d orbitals of suitable energy, there is a


possibility of π bonding and the ligand can be a π acceptor. T his addition to the

CHE509: Inorganic Chemistry – II Page 130


binding scheme increases Δ. Ligands that do this very effectively include CN -, CO,
and many other ligands

The metal ions can also be arranged in order of increasing Δ, and this order is
largely independent of the identity of the ligand. Mn 2+ < Ni 2+ < Co 2+ < Fe 2+ < V 2+ <
Fe 3+ < Cr 3+ < V 3+ < Co 3+ In general, it is not possible to say whether a given ligand
will exert a strong field or a weak field on a given metal ion. However, when we
consider the metal ion, the following two useful trends are observed.

JAHN–TELLER EFFECT
The Jahn-Teller effect, also called the Jahn-Teller distortion, describes geometric
distortions in molecules and ions associated with specific electronic configurations. This
electronic effect is named after Hermann Arthur Jahn and Edward Teller. He used group
theory to prove that orbital degenerate molecules are not stable. The Jahn-Teller theorem
essentially states that a nonlinear molecule with a spatially degenerate electronic ground state
undergoes a geometric distortion that removes this degeneracy. This is because strain lowers
the overall energy of the molecule.

Jahn-Teller distortion of a d 9 octahedral transition metal complex. The


tetragonal distortion lengthens the bonds along the z -axis as the bonds in the x-y

CHE509: Inorganic Chemistry – II Page 131


plane become shorter. This change lowers the overall energy, because the two
electrons in the d z2 orbital go down in energy as the one electron in the d x2-y2 orbital
goes up.
This effect can be understood in the context of octahedral metal complexes,
for example, by considering d-electron configurations where the orbital set contains
one or three electrons. The most common of these are high -spin d 4 (such as CrF 2 ),
low-spin d 7 (such as NaNiO2), and d9 (such as Cu 2+ ). For example, if the complex
can distort and break symmetry, the energy of one of the (previously) degenerate
orbitals will decrease and the energy of another will increase. There is an overall
decrease in electron energy because more electrons occupy the lower orbitals than
the upper orbitals. A similar distortion could occur in the tetrahedral complex if the
t2 orbitals were partially filled. Such geometric distortions that reduce electron
energy are said to be electronically driven. Similar electronically driven strains
occur in one-dimensional chain connections, called Peierls strain, and two -
dimensional connected layers, called charge density waves.
The Jahn-Teller effect occurs most frequently in octahedral complexes,
especially hexacoordinated copper (II) complexes. The d 9 electron configuration of
this ion gives him three electrons in two degenerate orbitals and so on, resulting in a
doubly degenerate electronic ground state. Such complexes are distorted along one
of the four axes of the molecule (always called the z -axis), resulting in degeneracy
of orbitals and electrons and lower overall energy. Strain usually takes the form of
elongation of bonds to the ligand along the z-axis, but may instead occur as
shortening of these bonds (Jahn-Teller theorem states that the direction of strain
(unpredictable and unstable geometry). When such elongation occurs, the
electrostatic repulsion between the electron pair of the basic Lewis ligand and the
electrons of the orbital with the z component is reduced, thereby reducing the ener gy
of the complex. If the undistorted complex is expected to have an inversion center,
this remains after the distortion
SOLVED PROBLEMS 02
1] What is spectrochemical series? Explain in detail.
Solution: The spectrochemical series is a list of ligands ordered by ligand
"strength" and a list of metal ions based on oxidation number, group, and element.
For metal ions, ligands modify the energy difference Δ between the d orbitals, called
the ligand field splitting parameter in ligand field theory or the cr ystal field splitting

CHE509: Inorganic Chemistry – II Page 132


parameter in crystal field theory. The splitting parameters are reflected in the ion's
electronic and magnetic properties, such as spin states, and optical properties, such
as color and absorption spectra.
A partial spectrochemical series listing of ligands from small Δ to large Δ is
given below.
I− < Br − < S 2− < SCN − (S–bonded) < Cl − < N 3 − < F − < NCO − < OH − < C 2 O 4 2− <
O 2− < H 2 O < acac − (acetylacetonate) < NCS − (N–bonded) < CH 3 CN < gly (glycine)
< py (pyridine) < NH 3 < en (ethylenediamine) < bipy (2,2'-bipyridine) < phen (1,10-
phenanthroline) < NO 2 − (N–bonded) < PPh 3 < CN − < CO
Weak field ligands: H 2 O, F − , Cl − , OH −
Strong field ligands: CO, CN − , NH 3 , PPh 3

The ligands on the far left of this spectrochemical series are generally
considered to be weak ligands, unable to induce forced electron pairing within the 3d
level, thus giving rise to high-spin octahedra in external orbitals. forms a complex.
On the other hand, the rightmost ligand is a stronger ligand, which forms an internal
orbital octahedral complex within the 3d level after forced electron pairing, and
therefore, called. However, it is known that ``spectrochemical s equences are
inherently set back from what they should be for reasonable predictions based on the
assumptions of crystal field theory''. This departure from crystal field theory
highlights a weakness in the assumption of crystal field theory: a purely ioni c bond
between metal and ligand. The order of the spectrochemical series can be derived
from the recognition that ligands are often classified according to their donor or
acceptor potency. Like NH 3 , it lacks orbitals with good symmetry for π-bond
interactions and may be only σ-bond donors. Binding of these ligands to metals is
relatively straightforward as only σ bonds are used to create relatively weak
interactions. Another example of a σ-bonded ligand is ethylenediamine. However,
ethylenediamine has a stronger effect than ammonia, producing a larger ligand field
splitting Δ. Ligands with occupied p-orbitals are potential π-donors. These types of
ligands tend to donate these electrons along with the σ bonding electrons to the
metal, exhibiting stronger metal-ligand interactions and effective reduction of Δ.
Most halide ligands and OH- are prime examples of π-donor ligands.
If a ligand has vacant π* and d orbitals of suitable energy, there is a
possibility of π bonding and the ligand can be a π acceptor. This addition to the

CHE509: Inorganic Chemistry – II Page 133


binding scheme increases Δ. Ligands that do this very effectively include CN -, CO,
and many other ligands
The metal ions can also be arranged in order of increasing Δ, and this order is
largely independent of the identity of the ligand. Mn 2+ < Ni 2+ < Co 2+ < Fe 2+ < V 2+ <
Fe 3+ < Cr 3+ < V 3+ < Co 3+ In general, it is not possible to say whether a given ligand
will exert a strong field or a weak field on a given metal ion. However, when we
consider the metal ion, the following two useful trends are observed.

2] What is Jahn-Teller effect? Explain in detail with examples.


SOLUTION : The Jahn-Teller effect, also called the Jahn-Teller distortion, describes geometric
distortions in molecules and ions associated with specific electronic configurations. This
electronic effect is named after Hermann Arthur Jahn and Edward Teller. He used group
theory to prove that orbital degenerate molecules are not stable. The Jahn-Teller theorem
essentially states that a nonlinear molecule with a spatially degenerate

These have been assigned to the following spin-allowed transitions.

CHE509: Inorganic Chemistry – II Page 134


Electronic ground state undergoes a geometric distortion that removes this degeneracy. This
is because strain lowers the overall energy of the molecule.

Jahn-Teller distortion of a d 9 octahedral transition metal complex. The


tetragonal distortion lengthens the bonds along the z -axis as the bonds in the x-y
plane become shorter. This change lowers the overall energy, because the two
electrons in the d z2 orbital go down in energy as the one electron in the d x2-y2 orbital
goes up.

This effect can be understood in the context of octahedral metal complexes,


for example, by considering d-electron configurations where the orbital set contains
one or three electrons. The most common of these are high-spin d 4 (such as CrF 2 ),
low-spin d 7 (such as NaNiO2), and d9 (such as Cu 2+ ). For example, if the complex
can distort and break symmetry, the energy of one of the (previously) degenerate
orbitals will decrease and the energy of another will increase. There is an overall
decrease in electron energy because more electrons occupy the lower orbitals than
the upper orbitals. A similar distortion could occur in the tetrahedral complex if the
t2 orbitals were partially filled. Such geometric distortions that reduce electron
energy are said to be electronically driven. Similar electronically driven strains
occur in one-dimensional chain connections, called Peierls strain, and two -
dimensional connected layers, called charge density waves.
The Jahn-Teller effect occurs most frequently in octahedral complexes,
especially hexacoordinated copper(II) complexes . The d 9 electron configuration of
this ion gives him three electrons in two degenerate orbitals and so on, resulting in a
doubly degenerate electronic ground state. Such complexes are distorted along one
of the four axes of the molecule (always called the z -axis), resulting in degeneracy
of orbitals and electrons and lower overall energy. Strain usually takes the form of
elongation of bonds to the ligand along the z-axis, but may instead occur as
shortening of these bonds (Jahn-Teller theorem states that the direction of strain
(unpredictable and unstable geometry). When such elongation occurs, the
electrostatic repulsion between the electron pair of the basic Lewis ligand and the
electrons of the orbital with the z component is reduced, thereby reducing the energy
of the complex. If the undistorted complex is expected to have an inversion center,
this remains after the distortion

CHE509: Inorganic Chemistry – II Page 135


SHORT ANSWER QUESTIONS WITH MODEL ANSWER 02
1] Define Jahn-Teller effect
Solution: The Jahn-Teller effect is a geometric distortion of a non -linear molecular
system that reduces its symmetry and energy. This distortion is typically observed
among octahedral complexes where the two axial bonds can be shorter or longer than
those of the equatorial bonds. This effect can also be observed in tetrahedral
compounds. This effect is dependent on the electronic state of the system.

2] Give correct order of spectrochemical series of legands from small Δ to large Δ.


Solution: I− < Br − < S 2− < SCN − (S–bonded) < Cl − < N3− < F−< NCO − <
OH − < C 2 O 4 2− < O 2− < H 2 O < acac − (acetylacetonate) < NCS − (N–bonded) < CH 3 CN
< gly (glycine) < py (pyridine) < NH 3 < en (ethylenediamine) < bipy (2,2'-
bipyridine) < phen (1,10-phenanthroline) < NO 2 − (N–bonded) < PPh 3 < CN − < CO
02-03: SPIN-ORBIT COUPLING (RUSSELL-SAUNDERS COUPLING)

The magnetic fields produced by S and L are not isolated from each other.
They interact via spin-orbit coupling (also called Russell-Saunders coupling). Only
this simple join form will be considered in this text. Its application is limited to
elements with z < 40, including the first row of transition elements. For heavier
elements, jj joins should also be considered. However, the latter will not be
discussed here.

In the Russell-Saunders spin-orbit coupling scheme, the interaction between


S and L is expressed by an additional quantum number, the total angular
momentum quantum number ( J ). The possible values of J are values between L+S
and |L−S| .

J=L+S,L+S−1,L+S−2,...,|L−S|

For multi-electron systems, the value of J must be positive or zero. J values


can fall in the series 1/2,3/2,5/2,... or 0,1,2,... 0,1,2,.... The quantum number J is
added to the term symbol as a subscript to the right of the letter describing the term.

The full conceptual symbol is: (2S+1) L J

Spin-orbit coupling results in the splitting of the free ion terms into states of
different energies. For example, the 3P state of a carbon atom with p2 electron

CHE509: Inorganic Chemistry – II Page 136


configuration is split into three different energy states (corresponding to three
possible J values of 0, 1, and 2): 3 P 0 , 3 P 1 , 3 P 2 3 P 0 , 3 P 1 , 3 P 2 .

Hund's third Rule:

 For subshells that are less than half-filled, the lowest energy state has
the lowest J value.

 For subshells that are exactly half-filled, there is only one J value, thus
it is the lowest energy.

 For subshells that are more than half-filled, the lowest energy state has
the largest J value.

Therefore, in this case with less than half of the p-subshell, the lowest energy
states from the 3P free ion term are those followed by J = 0, 3 P 0 , J = 1, J = 2. The
splitting energies and relative energies are shown in Figure below Spin-orbit
coupling and the splitting of the free ion term have important implications for
electronic spectra, as they affect the energies of electronic transitions.

SOLVED PROBLEMS 03

1] Describe the Russell-Saunders couplingscheme in detail

Solution: The magnetic fields produced by S and L are not isolated from each
other. They interact via spin-orbit coupling (also called Russell-Saunders coupling).

CHE509: Inorganic Chemistry – II Page 137


Only this simple join form will be considered in this text. Its application is limited to
elements with z < 40, including the first row of transition elements. For heavier
elements, jj joins should also be considered. However, the latter will not be
discussed here.

In the Russell-Saunders spin-orbit coupling scheme, the interaction between S


and L is expressed by an additional quantum number, the total angular momentum
quantum number ( J ). The possible values of J are values between L+S and |L−S|.

J=L+S,L+S−1,L+S−2,...,|L−S|

For multi-electron systems, the value of J must be positive or zero. J values


can fall in the series 1/2,3/2,5/2,... or 0,1,2,... 0,1,2,.... The quantum number J is
added to the term symbol as a subscript to the right of the letter describing the term.

(2S+1)
The full conceptual symbol is: LJ
Spin-orbit coupling results in the splitting of the free ion terms into states of
different energies. For example, the 3P state of a carbon atom with p2 electron
configuration is split into three different energy states (corresponding to three
possible J values of 0, 1, and 2): 3 P 0 , 3 P 1 , 3 P 2 3 P 0 , 3 P 1 , 3 P 2 .
Hund's third Rule:
 For subshells that are less than half-filled, the lowest energy state has
the lowest J value.
 For subshells that are exactly half-filled, there is only one J value, thus
it is the lowest energy.
 For subshells that are more than half-filled, the lowest energy state has
the largest J value.
Therefore, in this case with less than half of the p-subshell, the lowest energy
states from the 3P free ion term are those followed by J = 0, 3 P 0 , J = 1, J = 2. The
splitting energies and relative energies are shown in Figure below Spin-orbit
coupling and the splitting of the free ion term have important implications for
electronic spectra, as they affect the energies of electronic transitions.

CHE509: Inorganic Chemistry – II Page 138


SHORT ANSWER QUESTIONS WITH MODEL ANSWER 03
1] Give Hunds Rule for spin orbit coupling

Solution:
Hund's Rulefor spin orbit coupling

 For subshells that are less than half-filled, the lowest energy state has
the lowest J value.

 For subshells that are exactly half-filled, there is only one J value, thus
it is the lowest energy.

 For subshells that are more than half-filled, the lowest energy state has
the largest J value.
02-04: NEPHELAUXETIC EFFECT

The nephelauxetic effect is a term from transition metal inorganic chemistry.


This refers to the decrease in the Racah electron-electron repulsion parameter, given
the symbol B, that occurs when a transition metal-free ion forms a complex with a
ligand. The name "nephelauxetic" comes from the Greek word meaning cloud spread
and was proposed by the Danish inorganic chemist C.K. Jorgensen. The existence of

CHE509: Inorganic Chemistry – II Page 139


this effect highlights the shortcomings of crystal field theory, which treats metal -
ligand interactions as purely electrostatic, as the nepherotic effect reveals the
covalent character of metal-ligand interactions increase.

A decrease in the Racah parameter B indicates that there is less repulsion


between the two electrons of certain doubly occupied metal d -orbitals in the complex
than in the corresponding gaseous Mn+ metal ion complex. This electron cloud
expansion effect can occur for either (or both) of two reasons. One is that the net
positive charge of the metal has decreased. The d orbital expands slightly because
the positive charge of the metal is reduced by the negative charge of the ligand. The
second is the act of overlap with ligand orbitals, where the resulting molecular
orbital is formed from two atomic orbitals, thus increasing the orbital size through
covalent bond formation. The reduction from the free ion value of B is usually given
by the nepherotic parameter β.

Β = (B complex form) / (B free ion)

It has been experimentally observed that the magnitude of the nepherotic


parameter always follows a certain trend related to the nature of the ligands present.

The list shown below enlists some common ligands (showi ng increasing
nephelauxetic effect)

F − < H2O < NH3 < en < [NCS - N] − < Cl − < [CN] − < Br − < N3 − < I−

Some of this series of spectrochemical series of ligands appear very similar,


for example cyanide, ethylenediamine, and fluoride appear to occupy similar
positions in the two. However, others such as chloride, iodide, and bromide (among
others) appear to occupy very different positions. position. This order roughly
reflects the ability of the ligand to form good covalent bonds with the metal. Low
potency is at the beginning of the line and high potency is at the bottom of the line.

The nephelauxetic effect does not only depend upon the ligand type, but also
upon the central metal ion. These too can be arranged in order of increasing
nephelauxetic effect as follows.

Mn(II) < Ni(II) ≈ Co(II) < Mo(II) < Re(IV) < Fe(III) < Ir(III) < Co(III) <
Mn(IV)

CHE509: Inorganic Chemistry – II Page 140


PHOTOLUMINESCENCE
Photoluminescence is a process in which a molecule absorbs a photon in the
visible region, exciting one of its electrons to a higher electronic exci ted state, and
then radiates a photon as the electron returns to a lower energy state.

Photoluminescence spectroscopy is a non-contact, non-destructive method for


studying the electronic structure of materials. Light is directed at the sample where it
is absorbed, adding extra energy to the material in a process called photoexcitation.
One way this extra energy can be removed from the sample is by emitting light or
luminescence. In the case of photoexcitation, this emission is called
photoluminescence. Photoexcitation causes electrons to transition to allowed excited
states within the material. When these electrons return to their equilibrium state,
they release excess energy, which may or may not be accompanied by the emission
of light (radiative processes) (nonradiative processes). The energy of the emitted
light (photoluminescence) refers to the energy level difference between the two
electronic states involved in the transition between the excited and equilibrium
states. The amount of emitted light is related to the relative contributions of the
radiation processes. In most photoluminescence systems, aggregation of the
chromophores typically results in emission quenching by aggregation -induced
quenching (ACQ). This means that fluorophores must be used and studied in dilute
solutions or as isolated molecules. This reduces the sensitivity of devices that use
fluorescence. B. Biosensors and bioassays. Recently, however, examples have been
reported in which luminogen aggregation played a constructive rather th an a
destructive role in the luminescence process. This aggregated stimulated emission
(AIE) is potentially very important, especially for solid-state devices.
SOLVED PROBLEMS 04
1] What is nephelauxetic effect? Explain in detail.

Solution: The nephelauxetic effect is a term from transition metal inorganic


chemistry. This refers to the decrease in the Racah electron -electron repulsion
parameter, given the symbol B, that occurs when a transition metal -free ion forms a
complex with a ligand. The name "nephelauxetic" comes from the Greek word
meaning cloud spread and was proposed by the Danish inorganic chemist C.K.
Jorgensen. The existence of this effect highlights the shortcomings of crystal field

CHE509: Inorganic Chemistry – II Page 141


theory, which treats metal-ligand interactions as purely electrostatic, as the
nepherotic effect reveals the covalent character of metal-ligand interactions increase.

A decrease in the Racah parameter B indicates that there is less repulsion


between the two electrons of certain doubly occupied metal d -orbitals in the complex
than in the corresponding gaseous Mn+ metal ion. Complex. This electron cloud
expansion effect can occur for either (or both) of two reasons. One is that the net
positive charge of the metal has decreased. The d orbital expands slightly because
the positive charge of the metal is reduced by the negative charge of the ligand. The
second is the act of overlap with ligand orbitals, where the resulting molecular
orbital is formed from two atomic orbitals, thus increasing the orbital size through
covalent bond formation. The reduction from the free ion value of B is usually given
by the nepherotic parameter β.

Β = (B complex form) / (B free ion)

It has been experimentally observed that the magnitude of the nepherotic


parameter always follows a certain trend related to the nature of the ligands present.
The list shown below enlists some common ligands (showing incr easing
nephelauxetic effect)
F − < H2O < NH3 < en < [NCS - N] − < Cl − < [CN] − < Br − < N3 − < I−
Some of this series of spectrochemical series of ligands appear very similar,
for example cyanide, ethylenediamine, and fluoride appear to occupy similar
positions in the two. However, others such as chloride, iodide, and bromide (among
others) appear to occupy very different positions. position. This or der roughly
reflects the ability of the ligand to form good covalent bonds with the metal. Low
potency is at the beginning of the line and high potency is at the bottom of the line.
The nephelauxetic effect does not only depend upon the ligand type, but al so
upon the central metal ion. These too can be arranged in order of increasing
nephelauxetic effect as follows.
Mn(II) < Ni(II) ≈ Co(II) < Mo(II) < Re(IV) < Fe(III) < Ir(III) < Co(III) < Mn(IV)
2] Write a note on Photoluminescence.
Solution: Photoluminescence spectroscopy is a non-contact, non-destructive method
for studying the electronic structure of materials. Light is directed at the sample
where it is absorbed, adding extra energy to the material in a process called
photoexcitation. One way this extra energy can be removed from the sample is by

CHE509: Inorganic Chemistry – II Page 142


emitting light or luminescence. In the case of photoexcitation, this emission is called
photoluminescence. Photoexcitation causes electrons to transition to allowed excited
states within the material. When these electrons return to their equilibrium state,
they release excess energy, which may or may not be accompanied by the emission
of light (radiative processes) (nonradiative processes). The energy of the emitted
light (photoluminescence) refers to the energy level difference between the two
electronic states involved in the transition between the excited and equilibrium
states. The amount of emitted light is related to the relative contributions of the
radiation processes. In most photoluminescence systems, ag gregation of the
chromophores typically results in emission quenching by aggregation -induced
quenching (ACQ). This means that fluorophores must be used and studied in dilute
solutions or as isolated molecules. This reduces the sensitivity of devices that u se
fluorescence. B. Biosensors and bioassays. Recently, however, examples have been
reported in which luminogen aggregation played a constructive rather than a
destructive role in the luminescence process. This aggregated stimulated emission
(AIE) is potentially very important, especially for solid-state devices.
SHORT ANSWER QUESTIONS WITH MODEL ANSWER 04
1] Define nephelauxetic effect.
Solution: This effect is the result of ligands expanding the d-electron cloud. This
effect of ligands in expanding the d-electron cloud is called the nephelauxetic effect.
2] Define Photoluminescence effect.
Solution: Photoluminescence is a process in which a molecule absorbs a photon in
the visible region, exciting one of its electrons to a higher electronic excited state,
and then radiates a photon as the electron returns to a lower energy state.
CHECK POINT 01-04
1] Which one of the following octahedral complexes will be distorted?
a) [Cr(H 2 O) 6 ] 2+
b) [Cr(H 2 O) 6 ] 3+
c)[Mn(H 2 O) 6 ] 2+
d [Fe(H 2 O) 6 ] 3+
Solution: a

2] What is the currect order of legands for Nephelauxetic effect


a) F − < Cl − < Br − < I−

CHE509: Inorganic Chemistry – II Page 143


b) F − < I− < Br − < Cl −
c) F − < Br − < Cl − < I−
d) I− < Br − < Cl − < F −
Solution: a

3] This effect is the result of ligands expanding the d -electron cloud is __


a) Total anguler momentum
b) Nephelauxetic effect
c) Tyndal effect
d) Spin effect
Solution: b

4] The compound which exhibit Jahn-teller distortion is


a) [Mn(H 2 O) 6 ] 2+
b) [Mn(H 2 O) 6 ] 3+
c) [Cr(H 2 O) 6 ] 3+
d) [Fe(CN) 6 ] 4-
Solution: b

SUMMARY
Orgel diagrams are correlation diagrams which show the relative energies of
electronic terms in transition metal complexes, much like Tanabe –Sugano diagrams.
They are named after their creator, Leslie Orgel. Orgel diagrams are restricted to
only show weak field (i.e. high spin) cases, and offer no information about strong
field (low spin) cases. Because Orgel diagrams are qualitative, no energy
calculations can be performed from these diagrams; also, Orgel diagrams only show
the symmetry states of the highest spin multiplicity instead of all possible terms,
unlike a Tanabe–Sugano diagram. Orgel diagrams will, however, show the number of
spin allowed transitions, along with their respective symmetry designations. In an
Orgel diagram, the parent term (P, D, or F) in the presence of no ligand field is
located in the center of the diagram, with the terms due to that electronic

CHE509: Inorganic Chemistry – II Page 144


configuration in a ligand field at each side. There are two Orgel diagrams, one for
d 1 , d 4 , d 6 , and d 9 configurations and the other with d 2 , d 3 , d 7 , and d 8 configurations.
KEY WORDS
Spin orbit coupling, Photoluminescence, Jhan-teller effect, Orgal diagram, Tanabe
Sugano diagrams.
REFERENCES

MOOCS
--
YOUTUBE VIDEOS
1) https://www.youtube.com/watch?v=25Tj_0VENQo
2) https://www.youtube.com/watch?v=-Y58uH5zP-o
3) https://www.youtube.com/watch?v=U_M59sIobZY
4) https://www.youtube.com/watch?v=h7jRcvU6vek
5) https://www.youtube.com/watch?v=h7jRcvU6vek
WIKIPEDIA
1) https://en.wikipedia.org/wiki/Orgel_diagram
2) https://en.wikipedia.org/wiki/Orgel_diagram
3) https://en.wikipedia.org/wiki/Spin%E2%80%93orbit_interaction
4) https://en.wikipedia.org/wiki/Jahn%E2%80%93Teller_effect
OER
1) https://chem.libretexts.org/Bookshelves/Inorganic_Chemistry/Supplement
al_Modules_and_Websites_(Inorganic_Chemistry)/Crystal_Field_Theory/
Orgel_Diagrams
2) https://en.wikipedia.org/wiki/Tanabe%E2%80%93Sugano_diagram
3) https://chem.libretexts.org/Bookshelves/Inorganic_Chemistry/Supplement
al_Modules_and_Websites_(Inorganic_Chemistry)/Coordination_Chemist
ry/Structure_and_Nomenclature_of_Coordination_Compounds/Co ordinat
ion_Numbers_and_Geometry/Jahn-Teller_Distortions
REFERENCE BOOKS
3) Inorganic Chemistry Fourth Edition By Pearson: Principles of Structure and
Reactivity , January 2006 by J Huheey and Medhi
4) Spectroscopy of Inorganic Compounds, Principles, Problems a nd their
Solutions , December 2020 by Jagdamba Singh , Jaya Singh, Pandey M D.

CHE509: Inorganic Chemistry – II Page 145


CREDIT 02 -UNIT 03: METAL CARBONYLS

LEARNING OBJECTIVES
After successful completion of this unit, you will be able to
 Understand the structure of metal carbonyl complexes
 Understand the bonding in metal carbonyl complexes
 Understand the Vaska’s complex with preparation
 Understand the IR spectroscopy of carbonyl group in complexes
INTRODUCTION

Ludwig Mond (1839-1909)

Ludwig Mond with whom the German chemists Carl Langer and Friedrich
(ruin&e discovered the first binary metal carbonyl Ni(CO), one hundred years
ago.Metal carbonyls are coordination complexes between transition
metals and carbon monoxide ligands. Metal carbonyls are useful as catalysts or
catalyst precursors in organic synthesis and homogeneous catalysis such as
hydroformylation and Reppe chemistry. The Mondo process uses nickel
tetracarbonyl to produce pure nickel. In organometallic chemistry, metal carbonyls
serve as precursors to prepare other organometallic complexes.
Metal carbonyls are toxic by skin contact, inhalation, or ingestion due to their ability

CHE509: Inorganic Chemistry – II Page 146


to carbonylate hemoglobin to carboxyhemoglobin and interfere with oxygen
removal.

03-01: PREPARATION OF METAL CARBONYL COMPLEXES


Metal carbonyls are an important class of organometallic compounds that have
been studied for a long time. As long ago as 1884, when Ludwig Mond observed
nickel valves being corroded by his CO gas in the nickel smelting industry, he heated
nickel powder in his CO gas stream, produced the first known metal carbonyl
compound in the form of Ni(CO) 4 . Thus, the famous lunar smelting process was
born, based on the premise that volatile Ni(CO) 4 compounds could decompose into
pure metals at high temperatures. Mondo subsequently establish ed Mondo His
Nickel His Company Limited to refine nickel from ore using this method. Carbonyl
ligands (CO) differ from other ligands in several ways. For example, unlike alkyl
ligands, carbonyl (CO) ligands are unsaturated so that the ligand not only donat es σ−
but also accepts electrons from metal dπ orbitals on its π* orbitals. and the CO
ligand can acquire π-. sour. Another difference is that CO is a soft ligand compared
to other common σ- and π-base ligands such as HO and alkoxides (RO - ), which are
considered hard ligands. There is something.
The common methods of the preparation of the metal carbonyl compounds are,
1) Directly using CO

The main requirement of this method is that the metal center must be in a
reduced, low oxidation state to facilitate the binding of CO to the metal center by π
backdonation from the metal to the ligand.
1) Using CO and a reducing agent

2) By reaction: One of the most commonly used methods for synthesizing metal
carbonyls is the reduction of the corresponding metal salts in the presence of carbon
monoxide.

CHE509: Inorganic Chemistry – II Page 147


This process is commonly called reductive carbonylation and is mainly used for
compounds in which the metal center is in a higher oxidation state. The reducing
agent first reduces the metal center to a lower oxidation state before combining CO
to form a metal carbonyl compound.
3) From iron pentacarbonyl: Carbon monoxide ligands in Fe(CO) 5 are labile and
therefore can be used to synthesize other metal carbonyls.
110°C
MoCl6+3Fe(CO)5→−−−−−−−−−−−−−−→Mo(CO)6+3FeCl2+9CO
Ether
110°C
WCl6+ 3Fe(CO)5→−−−−−−−−−−−−−−→ W(CO)6+3FeCl2+9CO

4) From metathesis reaction:Mixed-metal carbonylscan successfully be prepared via a


metathesis reaction routeas
KCo (CO)4+[Ru(CO)3Cl2]2→−−−−−−−−−−→2RuCo 2 (CO) 11 +4KCl
SOLVED PROBLEMS 01
1] Explain the methods of preparations of metal carbonyls
Solution: The common methods of the preparation of the metal carbonyl
compounds are,
1) Directly using CO

The main requirement of this method is that the metal center must be in a
reduced, low oxidation state to facilitate the binding of CO to the metal center by π
backdonation from the metal to the ligand.
2) Using CO and a reducing agent

CHE509: Inorganic Chemistry – II Page 148


3) By reaction: One of the most commonly used methods for synthesizing metal
carbonyls is the reduction of the corresponding metal salts in the presence of carbon
monoxide.

This process is commonly called reductive carbonylation and is mainly used for
compounds in which the metal center is in a higher oxidation state. The reducing
agent first reduces the metal center to a lower oxidation state before combining CO
to form a metal carbonyl compound.
4) From iron pentacarbonyl: Carbon monoxide ligands in Fe(CO) 5 are labile and
therefore can be used to synthesize other metal carbonyls.
110°C
MoCl6+3Fe(CO)5→−−−−−−−−−−−−−−→Mo(CO)6+3FeCl2+9CO
Ether
110°C
WCl6+ 3Fe(CO)5→−−−−−−−−−−−−−−→ W(CO)6+3FeCl2+9CO

5) From metathesis reaction: Mixed-metal carbonylscan successfully be prepared via


ametathesis reaction routeas

KCo (CO)4+[Ru(CO)3Cl2]2→−−−−−−−−−−→2RuCo 2 (CO) 11 +4KCl

2] What are the metal carbonyls? Explain.

Solution: Metal carbonyls are an important class of organometallic compounds


that has been studied for a long time. As long ago as 1884, when Ludwig Mond

CHE509: Inorganic Chemistry – II Page 149


observed nickel valves being corroded by his CO gas in the nickel smelting industry,
he heated nickel powder in his CO gas stream, produced the first known metal
carbonyl compound in the form of Ni(CO) 4 . Thus, the famous lunar smelting process
was born, based on the premise that volatile Ni(CO) 4 compounds could decompose
into pure metals at high temperatures. Mondo subsequently established Mondo His
Nickel His Company Limited to refine nickel from ore using this method. Carbonyl
ligands (CO) differ from other ligands in several ways. For example, unlike alkyl
ligands, carbonyl (CO) ligands are unsaturated so that the ligand not only donates σ−
but also accepts electrons from metal dπ orbitals on its π* orbitals. and the CO
ligand can acquire π- sour. Another difference is that CO is a soft ligand compared to
other common σ- and π-base ligands such as HO and alkoxides (RO - ), which are
considered hard ligands. There is something.

SHORT ANSWER QUESTIONS WITH MODEL ANSWER 01


1] Define metal carbonyls.

Solution: Metal carbonyls are coordination complexes of transition metals with


carbon monoxide ligands. Metal carbonyls are useful in organic synthesis and as
catalysts or catalyst precursors in homogeneous catalysis, such as hydroformylation
and Reppe chemistry. In the Mond process, nickel tetracarbonyl is used to produce
pure nickel. In organometallic chemistry, metal carbonyls serve as precursors for the
preparation of other organometallic complexes.

03-02: STRUCTURE, BONDING AND REACTIONS


The structures of metal carbonyls can be divided into three main categories.
The first as a mononuclear system containing only one metal atom, the second as a
binuclear system with or without a bridging carbonyl, and the last as a with two or
more metal centers, all terminal, all bridging, or as a polynuclear system c ontaining
a mixture of the two carbonyl group.

1) Mononuclear metal carbonyls


The structures of mononuclear metal carbonyls are fairly simple and easy to
visualize. This is undoubtedly because there is only one metal center. Common
examples of mononuclear metal carbonyls include tetrahedral Ni(CO) 4 and Pd(CO) 4 .

CHE509: Inorganic Chemistry – II Page 150


For trigonal bipyramids of Fe(CO) 5 , Ru(CO) 5 , and Os(CO) 5 . Octahedral shape of
V(CO) 6 , Cr(CO) 6 , Mo(CO) 5 , W(CO) 6

Structures of some mononuclear metal carbonyls

1) BINUCLEAR METAL CARBONYL


The structure of a binuclear metal carbonyl consists of two metal centers and
contains an intermetallic bond or a bridging CO group, or both. For example,
Co 2 (CO) 8 is known to have two isomers. The first has the D 3 d symmetry of the
intermetallic bond without bridging carbonyls. The second is C2v symmetric, with
two bridging CO ligands and an intermetallic bond. The structure of Fe 2 (CO) 9 exists
in D 3 h symmetry and contains six terminal CO groups bonded with three bridging
CO ligands. Moreover, M 2 CO 10 (M = Mn, Tc, Re) exists in D 4 d symmetry with four
CO ligands at each metal center with intermetallic bonds.

The structures of some binuclear metal carbonyls

CHE509: Inorganic Chemistry – II Page 151


2) POLYNUCLEAR METAL CARBONATES :
The structures of polynuclear metal carbonyls consist of three or more metal
centers, including all bridges, all terminals, or a mixture of two types of CO groups.
For example, the Ru 3 (CO) 12 cluster has D 3 h symmetry and consists of a Ru-centered
equilateral triangle with two axial and two equatorial CO ligands, respectively.
Os 3 (CO) 12 has the same structure, but Fe 3 (CO) 12 is different, with two bridging CO
ligands leading to C 2 v symmetry. M 4 CO 12 (M=Co,Rh) is composed of a tetrahedral
M4 core, but with C 3 v molecular symmetry. Three carbonyl ligands are bridging and
nine are terminal. However, Ir 4 (CO) 12 has perfect Td symmetry and no bridging CO
ligand groups. The Rh 4 and Ir 4 clusters are more thermally robust than the Co 4
compounds, reflecting the normal trend of intermetallic bond strength of second - and
third-row metals to first-row metals. Furthermore, [Re 4 (CO) 16 ] 2− has D 2 h symmetry
and no bridging carbonyl. Moreover, the structures of Os 4 (CO) 16 , Os 4 (CO) 15 , and
Os 4 (CO) 14 are somewhat complicated due to their nonrigidity. Tetranuclear
Os 4 (CO) 16 is a folded cyclobutane analogue. X-ray diffraction analysis of Os 4 (CO) 14
revealed an irregular tetrahedral Os 4 framework with four weakly semi-bridging CO
groups and four different Os–Os bond lengths. The experimental structure of
Os 4 (CO) 15 was found to have a planar butterfly-like shape composed of two edge-
sharing triangles. Hexanuclear M 6 (CO) 16 (M = Rh, Co) exists with an octahedral
core with alternating faces involved in bridging. with 4 triple bridges and 12
terminal carbonyls

CHE509: Inorganic Chemistry – II Page 152


BONDING IN METAL CARBONYLS COMPOUNDS
To explain the nature of the bond between the metal center and the carbonyl
ligand, we first need to understand the bond of the carbonyl ligand itself. It is a
fairly well-known fact that the CO group acts as both a good sigma donor and a good
π-acceptor ligand. His two general approaches for studying the bonding of carbon
monoxide and metal carbonyls are described below.

1) bonding of metal carbonyls according to Valence Bond Theory:

According to this model, the bonds in the CO molecule are best described as:

The half-filled spx hybrid orbital of the carbon atom overlaps with the half -
filled spx hybrid orbital of the oxygen atom to form a σ bond, while the spx
hybridized lone pair on either atom remains essentially unbonded. Moreover, the
lateral overlap forms two π bonds. one between the half-filled 2py orbitals and the
other as a dative or coordinative interaction between the fully filled 2pz orbitals of
oxygen and the empty 2pz orbitals of carbon.

CHE509: Inorganic Chemistry – II Page 153


Valence bond theory deals with the bonding modes of carbonyls with metal
centers in terms of hybridization and resonance phenomena. The central metal atom
or ion provides the required number of vacant hybrid orbitals in the correct direction
to accept electron pairs from the surrounding ligands. For example, the chromium
atom in Cr(CO) 6 undergoes d 2 sp 3 hybridization, producing six empty hybrid orbitals
with equivalent geometries and the same energies. When one of the carbonyl ligands
approaches this metal ion with the core axis along the x -axis, the filled hybrid lone
pair of electrons of the carbon atom forms two empty hybrids pointing in oppos ite
directions along the x-axis. Overlaps one of the trajectories direction. Metal-carbon
multiple bonds are explained by different resonance structures, resulting in reduced
bond strength of carbon-oxygen bonds. Note also that there are two hybrid lone
pairs, one on carbon and one on oxygen. Bonding of carbonyl groups to metals
always occurs by carbon-terminal mediated donation. This can be explained by the
higher energy of hybrid lone carbon pairs than oxygen.

2) Bonding of metal carbonyls according to molecular orbital theory

This is the best model for describing bonding within CO ligands and within
metal carbonyl complexes. There are a total of three molecular diagrams of carbonyl

CHE509: Inorganic Chemistry – II Page 154


ligands that have been proposed from time to time. However, all three molecu lar
orbital (MO) diagrams can explain the nature of the metal -carbonyl π bond. The first
treatment was not very effective in explaining σ donation.The second treatment also
suffers from some minor anomalies.The third molecular orbital diagram is the most
widely accepted in the scientific community which provides a logical explanation for
the mystery of metal-carbonyl chemistry. Examine these MO diagrams in the order
suggested.

The formation of molecular orbitals above is actually an oversimplification of a


delicate problem. This statement is based on two facts. The first is that the removal
of electrons from CO to form CO+ actually increases bonding order. This is the
opposite of what would be expected when an electron is lost from the highest
occupied molecular orbital (HOMO) of bonding. Its bond order should be reduced by
removing one electron from π2px² or π2py², suggesting that his HOMO of carbon
monoxide is antibonding rather than bonding in nature. A second anomaly is also
evident from the MO diagrams of the CO ligands, with the molecular orbital z-axis
assigned to the π-bonded central atom. This describes how empty π*2px and π*2py
are used to accept electron densities from the filled d orbitals of the central metal
atom or ion.

CHE509: Inorganic Chemistry – II Page 155


SOLVED PROBLEMS 02
1] Describe the structure of metal carbonyls

Solution: The structures of metal carbonyls can be divided into three main
categories. The first as a mononuclear system containing only one metal atom, the
second as a binuclear system with or without a bridging carbonyl, and the last as a
with two or more metal centers, all terminal, all bridging, or as a polynuclear system
containing a mixture of the two carbonyl group.

3) Mononuclear metal carbonyls


The structures of mononuclear metal carbonyls are fairly simple and easy to
visualize. This is undoubtedly because there is only one metal center. Common
examples of mononuclear metal carbonyls include tetrahedral Ni(CO) 4 and Pd(CO) 4 .
For trigonal bipyramids of Fe(CO) 5 , Ru(CO) 5 , and Os(CO) 5 . Octahedral shape of
V(CO) 6 , Cr(CO) 6 , Mo(CO) 5 , W(CO) 6

Structures of some mononuclear metal carbonyls

CHE509: Inorganic Chemistry – II Page 156


4) BINUCLEAR METAL CARBONYL
The structure of a binuclear metal carbonyl consists of two metal centers and
contains an intermetallic bond or a bridging CO group, or both. For example,
Co 2 (CO) 8 is known to have two isomers. The first has the D 3 d symmetry of the
intermetallic bond without bridging carbonyls. The second is C2v symmetric, with
two bridging CO ligands and an intermetallic bond. The structure of Fe 2 (CO) 9 exists
in D 3 h symmetry and contains six terminal CO groups bonded with three bridging
CO ligands. Moreover, M 2 CO 10 (M = Mn, Tc, Re) exists in D 4 d symmetry with four
CO ligands at each metal center with intermetallic bonds.

The structures of some binuclear metal carbonyls

5) POLYNUCLEAR METAL CARBONATES:


The structures of polynuclear metal carbonyls consist of three or more metal
centers, including all bridges, all terminals, or a mixture of two types of CO groups.
For example, the Ru 3 (CO) 12 cluster has D 3 h symmetry and consists of a Ru-centered
equilateral triangle with two axial and two equatorial CO ligands, respectively.
Os 3 (CO) 12 has the same structure, but Fe 3 (CO) 12 is different, with two bridging CO
ligands leading to C 2 v symmetry. M 4 CO 12 (M=Co,Rh) is composed of a tetrahedral
M4 core, but with C 3 v molecular symmetry. Three carbonyl ligands are bridging and
nine are terminal. However, Ir 4 (CO) 12 has perfect Td symmetry and no bridging CO
ligand groups. The Rh 4 and Ir 4 clusters are more thermally robust than the Co 4

CHE509: Inorganic Chemistry – II Page 157


compounds, reflecting the normal trend of intermetallic bond strength of second - and
third-row metals to first-row metals. Furthermore, [Re 4 (CO) 16 ] 2− has D 2 h symmetry
and no bridging carbonyl. Moreover, the structures of Os 4 (CO) 16 , Os 4 (CO) 15 , and
Os 4 (CO) 14 are somewhat complicated due to their nonrigidity. Tetranuclear
Os 4 (CO) 16 is a folded cyclobutane analogue. X-ray diffraction analysis of Os 4 (CO) 14
revealed an irregular tetrahedral Os 4 framework with four weakly semi-bridging CO
groups and four different Os–Os bond lengths. The experimental structure of
Os 4 (CO) 15 was found to have a planar butterfly-like shape composed of two edge-
sharing triangles. Hexanuclear M 6 (CO) 16 (M = Rh, Co) exists with an octahedral
core with alternating faces involved in bridging. with 4 triple bridges and 12
terminal carbonyls

2] Explain the Bonding in metal carbonyls according to VBT and MOT

Solution:To explain the nature of the bond between the metal center and the
carbonyl ligand, we first need to understand the bond of t he carbonyl ligand itself. It
is a fairly well-known fact that the CO group acts as both a good sigma donor and a
good π-acceptor ligand. His two general approaches for studying the bonding of
carbon monoxide and metal carbonyls are described below.

CHE509: Inorganic Chemistry – II Page 158


6) bonding of metal carbonyls according to Valence Bond Theory:

According to this model, the bonds in the CO molecule are best described as:

The half-filled spx hybrid orbital of the carbon atom overlaps with the half -
filled spx hybrid orbital of the oxygen atom to form a σ bond, while the spx
hybridized lone pair on either atom remains essentially unbonded. Moreover, the
lateral overlap forms two π bonds. one between the half-filled 2py orbitals and the
other as a dative or coordinative interaction between the fully filled 2pz orbitals of
oxygen and the empty 2pz orbitals of carbon.

valence bond theory deals with the bonding modes of carbonyls with metal
centers in terms of hybridization and resonance phenomena. The central metal atom
or ion provides the required number of vacant hybrid orbitals in the correct direction
to accept electron pairs from the surrounding ligands. For example, the chromium
atom in Cr(CO) 6 undergoes d 2 sp 3 hybridization, producing six empty hybrid orbitals
with equivalent geometries and the same energies. When one of the carbonyl ligands

CHE509: Inorganic Chemistry – II Page 159


approaches this metal ion with the core axis along the x -axis, the filled hybrid lone
pair of electrons of the carbon atom forms two empty hybrids pointing in opposite
directions along the x-axis. Overlaps one of the trajectories. direction. Metal -carbon
multiple bonds are explained by different resonance structures, resulting in reduced
bond strength of carbon-oxygen bonds. Note also that there are two hybrid lone
pairs, one on carbon and one on oxygen. Bonding of carbonyl groups to metals
always occurs by carbon-terminal mediated donation. This can be explained by the
higher energy of hybrid lone carbon pairs than oxygen.

7) Bonding of metal carbonyls according to molecular orbital theory

This is the best model for describing bonding within CO ligands and within
metal carbonyl complexes. There are a total of three molecular diagrams of carbonyl
ligands that have been proposed from time to time. However, all three molecular
orbital (MO) diagrams can explain the nature of the metal-carbonyl π bond. The first
treatment was not very effective in explaining σ donation.The second treatment also
suffers from some minor anomalies.The third molecular orbital diagram is the most
widely accepted in the scientific community which provides a logical explanation for
the mystery of metal-carbonyl chemistry. Examine these MO diagrams in the order
suggested.

The formation of molecular orbitals above is actually an oversimplification of a


delicate problem. This statement is based on two facts. The first is that the removal
of electrons from CO to form CO+ actually increases bonding order. This is the
opposite of what would be expected when an electron is lost from the highest
occupied molecular orbital (HOMO) of bonding. It s bond order should be reduced by
removing one electron from π2px² or π2py², suggesting that his HOMO of carbon
monoxide is antibonding rather than bonding in nature. A second anomaly is also
evident from the MO diagrams of the CO ligands, with the molecul ar orbital z-axis
assigned to the π-bonded central atom. This describes how empty π*2px and π*2py
are used to accept electron densities from the filled d orbitals of the central metal
atom or ion.

CHE509: Inorganic Chemistry – II Page 160


SHORT ANSWER QUESTIONS
1] Define mononuclear cluster and give examples

Solution:The structures of mononuclear metal carbonyls are fairly simple and


easy to visualize. This is undoubtedly because there is only one metal center.
Common examples of mononuclear metal carbonyls include tetrahedral Ni(CO) 4 and
Pd(CO) 4 . For trigonal bipyramids of Fe(CO) 5 , Ru(CO) 5 , and Os(CO) 5 . Octahedral
shape of V(CO) 6 , Cr(CO) 6 , Mo(CO) 5 , W(CO) 6

2] Define binuclear cluster and give examples

Solution:The structure of a binuclear metal carbonyl consists of two metal


centers and contains an intermetallic bond or a bridging CO group, or both. For
example, Co 2 (CO) 8 is known to have two isomers.

03-03: VASKA'S COMPLEX


Vaska complex is the generic name for the compound trans -carbonyl
chlorobis(triphenylphosphine)iridium(I) with the formula IrCl(CO)[P(C6H5)3]2.
This square planar diamagnetic organometallic complex consists of a central iridium
atom bound to two trans-triphenylphosphine ligands, carbon monoxide, and chloride
ions. This complex was first described by J. W. DiLuzio and Lauri Va ska in 1961.
The Vaska complex can be oxidatively added and is characterized by its ability to
reversibly bind O2. It is a pale yellow crystalline solid

CHE509: Inorganic Chemistry – II Page 161


Vaska’s complex

Preparation

In this synthesis, almost all iridium chloride salts are heated along with
triphenylphosphine and a carbon monoxide source. The most common method is to
use dimethylformamide (DMF) as solvent and sometimes add aniline to accelerate
the reaction. Another common solvent is 2-methoxyethanol. The reaction is typically
run under nitrogen. In the synthesis, triphenylphosphine serves as both ligand and
reducing agent, and the carbonyl ligand is induced by decomposition of
dimethylformamide, presumably by deinsertion of the intermediate Ir-C(O)H
species. Below is a balanced equation for this complex reaction.

IrCl 3 (H 2 O) 3 + 3P(C 6 H 5 ) 3 + HCON(CH 3 ) 2 + C 6 H 5 NH 2 → IrCl(CO)[P(C 6 H 5 ) 3 ] 2 +


[(CH 3 ) 2 NH 2 ]Cl + OP(C 6 H 5 ) 3 + [C 6 H 5 NH 3 ]Cl + 2 H 2 O

Reactions

Studies on Vaska complexes have helped establish a conceptual framework for


homogeneous catalysis. Vaska complexes with 16 valence electrons are considered to
be "coordinatively unsaturated", so that they can bond to two -electron or two one-
electron ligands to form an electronically complex with 18 valence electrons. can be
saturated with The addition of two one-electron ligands is called oxidative addition.
Oxidative addition increases the oxidation state of iridium from Ir(I) to Ir(III). The
4-coordinate square planar arrangement of the first complex is transformed into the
6-coordinate product of the octahedron. Vaska complexes undergo oxidative addition
with common oxidants such as halogens, strong acids such as HCl, and other
molecules known to react as electrophiles such as iodomethane (CH 3 I).

Vaska's complex binds O2 reversibly

IrCl(CO)[P(C 6 H 5 ) 3 ] 2 + O 2 ⇌ IrCl(CO)[P(C 6 H 5 ) 3 ]2O 2


The dioxygen ligand binds to Ir through both oxygen atoms, called side
bonds. On the other hand, in myoglobin and hemoglobin, O2 is end -on bound and
binds to the metal only through one of the two oxygen atoms. The resulting dio xygen
adduct reverts to the initial complex upon heating or purging the solution with an
inert gas indicated by a color change from orange back to yellow.
SOLVED PROBLEMS 03
1] Dive synthesis and reactions of Vaska complex

CHE509: Inorganic Chemistry – II Page 162


Solution:Vaska complex is the generic name for the compound trans-
carbonylchlorobis(triphenylphosphine)iridium(I) with the formula
IrCl(CO)[P(C6H5)3]2. This square planar diamagnetic organometallic complex
consists of a central iridium atom bound to two trans -triphenylphosphine ligands,
carbon monoxide, and chloride ions. This complex was first described by J. W.
DiLuzio and Lauri Vaska in 1961. The Vaska complex can be oxidatively added and
is characterized by its ability to reversibly bind O2. It is a pale yellow crystalline
solid

Preparation

In this synthesis, almost all iridium chloride salts are heated along with
triphenylphosphine and a carbon monoxide source. The most common method is to
use dimethylformamide (DMF) as solvent and sometimes add aniline to accelerate
the reaction. Another common solvent is 2-methoxyethanol. The reaction is typically
run under nitrogen. In the synthesis, triphenylphosphine serves as both ligand and
reducing agent, and the carbonyl ligand is induced by decomposition of
dimethylformamide, presumably by deinsertion of the intermediate Ir-C(O)H
species. Below is a balanced equation for this complex reaction.

IrCl 3 (H 2 O) 3 + 3P(C 6 H 5 ) 3 + HCON(CH 3 ) 2 + C 6 H 5 NH 2 → IrCl(CO)[P(C 6 H 5 ) 3 ] 2 +


[(CH 3 ) 2 NH 2 ]Cl + OP(C 6 H 5 ) 3 + [C 6 H 5 NH 3 ]Cl + 2 H 2 O
Reactions
Studies on Vaska complexes have helped establish a conceptual framework for
homogeneous catalysis. Vaska complexes with 16 valence electrons are considered to
be "coordinatively unsaturated", so that they can bond to two -electron or two one-
electron ligands to form an electronically complex with 18 valence electrons. can be
saturated with The addition of two one-electron ligands is called oxidative addition.
Oxidative addition increases the oxidation state of iridium from Ir(I) to Ir(III). The
4-coordinate square planar arrangement of the first complex is transformed into the
6-coordinate product of the octahedron. Vaska complexes undergo oxidative addition
with common oxidants such as halogens, strong acids such as HCl, and other
molecules known to react as electrophiles such as iodomethane (CH 3 I).
Vaska's complex binds O 2 reversibly
IrCl(CO)[P(C 6 H 5 ) 3 ] 2 + O 2 ⇌ IrCl(CO)[P(C 6 H 5 ) 3 ] 2 O 2

CHE509: Inorganic Chemistry – II Page 163


The dioxygen ligand binds to Ir through both oxygen atoms, called side bonds.
On the other hand, in myoglobin and hemoglobin, O2 is end-on bound and binds to
the metal only through one of the two oxygen atoms. The resulting dioxygen adduct
reverts to the

SHORT ANSWER QUESTIONS WITH MODEL ANSWER 03


1] Give structure of vaska’s complexe

Solution:

03-04: IR SPECTROSCOPY OF CARBONYL COMPLEXES


Since these two modes lead to changes in the dipole moment, two bands are
expected in the infrared spectrum of terminally bound carbon monoxide. Using
infrared spectroscopy together with Raman spectroscopy, the shape of the metal
carbonyl can be determined. Mononuclear pentacarbonyl can exist in both square
pyramidal and trigonal bipyramidal forms. Performing an infrared spectrum after
calculating the IR and Raman activity bands for both possible shapes of can provide
information about the actual shape of the molecule.

The C–O bond order and frequency of their absorption are directly
proportional. Therefore, we can expect the absorption frequencies to be in the order:
free CO > metal carbonyl cation > neutral metal carbonyl > metal carbonyl anion

CHE509: Inorganic Chemistry – II Page 164


The C-O bond of the terminal carbonyl group is stronger than that of the
bridging carbonyl group. It is therefore possible to distinguish between terminal
carbonyls, which absorb in the 2050–1900 cm -1 region, and bridging carbonyls,
which absorb below 1900 cm -1 . Changes in band intensity for the carbonyl group can
provide information for kinetic studies of substitution reactions involving carbonyl
substitution.
SOLVED PROBLEMS 04
1] Explain the IR absorption of carbonyl compounds
Solution: Since these two modes lead to changes in the dipole moment, two
bands are expected in the infrared spectrum of terminally bound carbon monoxide.
Using infrared spectroscopy together with Raman spectroscopy, the shape of the
metal carbonyl can be determined. Mononuclear pentacarbonyl can exist in both
square pyramidal and trigonal bipyramidal forms. Performing an infrared spectrum
after calculating the IR and Raman activity bands for both possible shapes of can
provide information about the actual shape of the molecule.

CHE509: Inorganic Chemistry – II Page 165


The C–O bond order and frequency of their absorption are directly
proportional. Therefore, we can expect the absorption frequencies to be in the order:
free CO > metal carbonyl cation > neutral metal carbonyl > metal carbonyl anion

The C-O bond of the terminal carbonyl group is stronger than that of the
bridging carbonyl group. It is therefore possible to distinguish between terminal
carbonyls, which absorb in the 2050–1900 cm -1 region, and bridging carbonyls,
which absorb below 1900 cm -1 . Changes in band intensity for the carbonyl group can
provide information for kinetic studies of substitution reactions involving carbonyl
substitution.

SHORT ANSWER QUESTIONS WITH MODEL ANSWER 04


1] How to determine the shape of carbonyl complexes?
Solution: Since these two modes lead to changes in the dipole moment, two
bands are expected in the infrared spectrum of terminally bound carbon monoxide.
Using infrared spectroscopy together with Raman spectroscopy, the shape of the
metal carbonyl can be determined. Mononuclear pentacarbonyl can exist in both
square pyramidal and trigonal bipyramidal forms.
CHECK POINT 01-04
1] What is the geometry of pentacarbonyliron (0)?
a) Square planar
b) Tetrahedral
c) Trigonal bipyramidal
d) Octahedral
Solution: c Explanation: The coordination number of pentacarbonyliron (0) is 5
as CO is a unidentate ligand and hence its geometry id trigonal bipyramidal.
2] The metal-carbon bond in metal carbonyls possesses only sigma character.
a) True

CHE509: Inorganic Chemistry – II Page 166


b) False
Solution: b Explanation: The M-C bond in metal carbonyls has both sigma and
pi bonds and this helps in creating a synergic effect and, hence strengthening the
bond.
3] How is the M-C pi bond formed?
a) Donation of electron pair of half-filled metal d orbital to empty bonding pi
orbital of CO
b) Donation of electron pair of filled metal d orbital to empty bonding pi
orbital of CO
c) Donation of electron pair of filled metal d orbital to empty antibon ding pi
orbital of CO
d) Donation of electron pair of half-filled metal d orbital to empty antibonding
pi orbital of CO

Solution: c. The pi bond involves donation of electrons from filled metal d


orbitals into empty antibonding pi orbitals of CO. This is also called a back bond.

4] Vaska complexes undergo oxidative addition with __

a) NaOH

b) KOH

c) KBr

d) HCl

Solution: d

SUMMARY
Metal carbonyls are coordination complexes between transition metals and
carbon monoxide ligands. Metal carbonyls are useful as catalysts or catalyst
precursors in organic synthesis and homogeneous catalysis such as hydroformylation
and Reppe chemistry. The Mondo process uses nickel tetracarbonyl to produce pure
nickel. In organometallic chemistry, metal carbonyls serve as precurso rs to prepare
other organometallic complexes. Metal carbonyls are toxic by skin contact,
inhalation, or ingestion due to their ability to carbonylate hemoglobin, which
interferes with the binding of oxygen, to carboxyhemoglobin.

CHE509: Inorganic Chemistry – II Page 167


KEY WORDS
Metal carbonyls, Carbonyl-metal Bonding, Vaskas Complexes, IR of Metal carbonyls
REFERENCES

MOOCS
YOUTUBE VIDEOS
5) https://www.youtube.com/watch?v=T-PgkrGFfDA
6) https://www.youtube.com/watch?v=WITqeSRKuTg
7) https://www.youtube.com/watch?v=SX3d4XURsh4
8) https://www.youtube.com/watch?v=UHs7mO-iy_c
WIKIPEDIA
3) https://en.wikipedia.org/wiki/Metal_carbonyl
4) https://en.wikipedia.org/wiki/Organometallic_chemistry
5) https://en.wikipedia.org/wiki/Vaska%27s_complex
6) https://en.wikipedia.org/wiki/Metal_carbonyl
OER
4) https://www.dalalinstitute.com/wp-content/uploads/Books/A-Textbook-of-
Inorganic-Chemistry-Volume-1/ATOICV1-11-1-Metal-Carbonyls-Structure-
and-Bonding.pdf
5) https://chem.libretexts.org/Bookshelves/Inorganic_Chemistry/Inorganic_Coor
dination_Chemistry_(Landskron)/10%3A_Organometallic_Chemistry/10.02
%3A_Ligands_in_Organometallic_Chemistry
6) https://chem.libretexts.org/Bookshelves/Inorganic_Chemistry/Book%3A_Intro
duction_to_Organometallic_Chemistry_(Ghosh_and_Balakrishna)/08%3A_C
arbonyls_and_Phosphine_Complexes/8.01%3A_Metal_Carbonyls
REFERENCE BOOKS
5) Inorganic Chemistry Principles of Structure and Reactivity by Huheey Medhi
2006
6) Coordination Chemistry 2009 by D. Banerjea

CHE509: Inorganic Chemistry – II Page 168


CREDIT 02 -UNIT 04: NITROSYL COMPLEXES AND DINITROGEN
COMPLEXES
LEARNING OBJECTIVES
After successful completion of this unit, you will be able to
 Understand the reactions of nitrosyl complexes
 Understand the preparation of dinitrogen compounds
 Understand the preparation of dinitrogen compounds
 Understand the reactions of dinitrogen compounds
INTRODUCTION

Valence bond and hybrid orbital theory provided an opportunity to rationalize


the geometries of simple molecules. The idea behind bond theory begins with
writing a simple Lewis electron dot structure, and then visualizing atomic orbitals to
provide a valid explanation for the obvious question, "Why do atoms bond?" started.
However, while these theories were able to explain the bond formation of simple
molecules like methane and, in part, the shape of these molecules, the relationship
between molecular properties and bonding aspects was For example, they could not
explain the phenomenon of light absorption followed by molecule excitation and
emission. For example, MO theory can explain "why is O 2 paramagnetic?" Molecular
orbital theory (MO) describes electrons in molecules using specific wavefunctions
called molecular orbitals. A molecular orbital is formed by the linear combination of
two atomic orbitals. Two atomic orbitals (AO) combine to form two molecular
orbitals: bonding and antibonding molecular orbitals. MOs have many properties
similar to atomic orbitals. Similar to AO, MO can accept up to two electrons and has

CHE509: Inorganic Chemistry – II Page 169


a defined energy. Unlike AOs, MOs are associated with entire molecules rather than
single atoms. Two s orbitals and two orbitals combine in various ways to form
bonding MOs (σ1s) and antibonding (σ*1s) MOs.
04-01: METAL NITROSYL COMPLEXES
Metal nitrosyl complexes are complexes containing nitric oxide NO bound to a
transition metal. Many types of nitrosyl conjugates are known, differing in both structure and
co-ligand
Bonding and structure
Most complexes containing NO ligands can be viewed as derivatives of the
nitrosyl cation NO+. Nitrosyl cations are isoelectronic with carbon monoxi de, so the
bonding between nitrosyl ligands and metals follows the same principle as the
bonding of carbonyl complexes. Nitrosyl cations act as two -electron donors for
metals, accepting electrons from metals via back bonding. The compounds
Co(NO)(CO) 3 and Ni(CO) 4 show an analogy for NO + and CO. In terms of electron
counts, two linear NO ligands correspond to three CO groups. This trend is
illustrated by the isoelectronic pair of Fe(CO) 2 (NO) 2 and [Ni(CO) 4 ]. These
complexes are isoelectronic and coincidentally both obey the 18-electron rule. A
formal description of nitric oxide as NO + is inconsistent with certain measurable and
calculated properties. Another explanation is that nitric oxide acts as a three -electron
donor and the metal-nitrogen interaction is a triple bond.
Linear vs bent nitrosyl ligands

The M-N-O units of nitrosyl conjugates are typically linear or within 15° of
linear. However, in some complexes, especially when backdonation is less important,
the M-N-O angle can deviate significantly from 180°. Linear and curvilinear NO
ligands can be distinguished by infrared spectroscopy. Linear M -N-O groups absorb
between 1650 and 1900 cm-1, while curved nitrosyl groups absorb between 1525 and
1690 cm-1. Different vibrational frequencies reflect different N-O bond orders for
linear (triple bond) and bent NO (double bond). Bent NO ligands are sometimes

CHE509: Inorganic Chemistry – II Page 170


called anionic NO - . Prototypes for such compounds are organic nitroso compounds
such as nitrosobenzene. Complexes with bent NO ligands ar e trans-
[Co(en)2(NO)Cl] + . NO - is also common in the NO molecules of alkali metals or
alkaline earth metals. for example. The ionic forms of LiNO and BeNO are Li + NO –
and Be + NO – .

The use of linear and curved binding can be analyzed usi ng Enemark-Feltham
notation. In their framework, the determining factor for bent pairwise NO ligands is
the sum of electrons with π symmetry. Complexes with more than six “pi electrons”
tend to have bent NO ligands. For example, [Co(en) 2 (NO)Cl] + has eight pi-
symmetric electrons (six in the t 2 g orbital, two in NO, {CoNO} 8 ) and the
2−
[Fe(CN) 5 (NO)] has 6 electrons, and {FeNO} 6 ), which is π symmetric, adopts a
linear nitrosyl. Another figure shows the {MNO} d electron number of the anion
[Cr(CN) 5 NO] 3− . In this example, the cyanide ligand is "harmless". H. Each charge is
-1 and the sum is -5. Therefore, the charge of {CrNO} is +2 (-3 = -5 + 2) to balance
the overall charge of the fragment. Using the neutral electron counting scheme, Cr
has 6 d electrons and NO has 1 electron, for a total of 7. Subtract two electrons to
get 5 considering the total charge +2 on this fragment. Written in Enemark -Feltham
notation, the d-electron number is {CrNO} 5 . The result is the same if the nitrosyl
ligand is considered to be NO + or NO - .

One of the earliest examples of nitrosyl complexes to be synthesized is


Roussin's red salt, the sodium salt of the anion [Fe 2 (NO) 4 S 2 ] 2− . The anion structure
can be viewed as consisting of two edge-sharing tetrahedra. Each iron atom is
linearly bonded to two NO + ligands and shares two bridging sulfide ligands with
other iron atoms. Lusan black salt has a more complex cluster structure. The formula
for this kind of anion is [Fe 4 (NO) 7 S 3 ] − . It has C 3 v symmetry. It consists of a
tetrahedron of iron atoms with sulfide ions on his three faces of the tetrahedron.
Three iron atoms are attached to two nitrosyl groups. An iron atom on the three -fold
axis of symmetry also has a single nitrosyl group on that axis.

SOLVED PROBLEMS 01
1] Write a note on bonding and structure of Metal nitrosyl

CHE509: Inorganic Chemistry – II Page 171


SOLUTION: Metal nitrosyl complexes are complexes containing nitric oxide NO bound
to a transition metal. Many types of nitrosyl conjugates are known, differing in both structure
and co-ligand

Bonding and structure

Most complexes containing NO ligands can be viewed as derivatives of the


nitrosyl cation NO+. Nitrosyl cations are isoelectronic with carbon monoxide, so the
bonding between nitrosyl ligands and metals follows the same principle as the
bonding of carbonyl complexes. Nitrosyl cations act as two-electron donors for
metals, accepting electrons from metals via back bonding. The compounds
Co(NO)(CO) 3 and Ni(CO) 4 show an analogy for NO + and CO. In terms of electron
counts, two linear NO ligands correspond to three CO groups. This trend is
illustrated by the isoelectronic pair of Fe(CO) 2 (NO) 2 and [Ni(CO) 4 ]. These
complexes are isoelectronic and coincidentally both obey the 18 -electron rule. A
formal description of nitric oxide as NO + is inconsistent with certain measurable and
calculated properties. Another explanation is that nitric oxide acts as a three -electron
donor and the metal-nitrogen interaction is a triple bond.

2] Write a note on Linear vs bent nitrosyl ligands

Solution: Linear vs bent nitrosyl ligands

The M-N-O units of nitrosyl conjugates are typically linear or within 15° of
linear. However, in some complexes, especially when backdonation is less important,
the M-N-O angle can deviate significantly from 180°. Linear and curvilinear NO
ligands can be distinguished by infrared spectroscopy. Linear M-N-O groups absorb
between 1650 and 1900 cm-1, while curved nitrosyl groups absorb between 1525 and
1690 cm-1. Different vibrational frequencies reflect different N-O bond orders for
linear (triple bond) and bent NO (double bond). Bent N O ligands are sometimes
called anionic NO - . Prototypes for such compounds are organic nitroso compounds
such as nitrosobenzene. Complexes with bent NO ligands are trans -
[Co(en)2(NO)Cl] + . NO - is also common in the NO molecules of alkali metals or

CHE509: Inorganic Chemistry – II Page 172


alkaline earth metals. for example. The ionic forms of LiNO and BeNO are Li + NO –
and Be + NO – .
The use of linear and curved binding can be analyzed usi ng Enemark-Feltham
notation. In their framework, the determining factor for bent pairwise NO ligands is
the sum of electrons with π symmetry. Complexes with more than six “pi electrons”
tend to have bent NO ligands. For example, [Co(en) 2 (NO)Cl] + has eight pi-
symmetric electrons (six in the t 2 g orbital, two in NO, {CoNO} 8 ) and the
2−
[Fe(CN) 5 (NO)] has 6 electrons, and {FeNO} 6 ), which is π symmetric, adopts a
linear nitrosyl. Another figure shows the {MNO} d electron number of the anion
[Cr(CN) 5 NO] 3− . In this example, the cyanide ligand is "harmless". H. Each charge is
-1 and the sum is -5. Therefore, the charge of {CrNO} is +2 (-3 = -5 + 2) to balance
the overall charge of the fragment. Using the neutral electron counting scheme, Cr
has 6 d electrons and NO has 1 electron, for a total of 7. Subtract two electrons to
get 5 considering the total charge +2 on this fragment. Written in Enemark-Feltham
notation, the d-electron number is {CrNO} 5 . The result is the same if the nitrosyl
ligand is considered to be NO + or NO - .
One of the earliest examples of nitrosyl complexes to be synthesized is
Roussin's red salt, the sodium salt of the anion [Fe 2 (NO) 4 S 2 ] 2− . The anion structure
can be viewed as consisting of two edge-sharing tetrahedra. Each iron atom is
linearly bonded to two NO + ligands and shares two bridging sulfide ligands with
other iron atoms. Lusan black salt has a more complex cluster structure. The formula
for this kind of anion is [Fe 4 (NO) 7 S 3 ] − . It has C 3 v symmetry. It consists of a
tetrahedron of iron atoms with sulfide ions on his three faces of the tetrahedron.
Three iron atoms are attached to two nitrosyl groups. An iron atom on the three-fold
axis of symmetry also has a single nitrosyl group on that axis.
SHORT ANSWER QUESTIONS WITH MODEL ANSWER 01
1] Define Metal nitrosyl complexes
Solution: Metal nitrosyl complexes are complexes containing nitric oxide NO
bound to transition metal. Many types of nitrosyl conjugates are known, differing in
both structure and co-ligand
2] Give example of Metal nitrosyl complexes.
Solution: One of the earliest examples of nitrosyl complexes to be synthesized
is Roussin's red salt, the sodium salt of the anion [Fe 2 (NO) 4 S 2 ] 2− . The anion
structure can be viewed as consisting of two edge-sharing tetrahedra. Each iron atom

CHE509: Inorganic Chemistry – II Page 173


is linearly bonded to two NO + ligands and shares two bridging sulfide ligands with
other iron atoms.
04-02: REACTIONS AND APPLICATIONS OF NITROSYL COMPLEXES
An important reaction is the acid/base equilibrium, yielding transition metal
nitrite complexes:
[Ln MNO] 2+ + 2OH − ⇌ Ln MNO 2 + H 2 O
This equilibrium serves to confirm that the linear nitrosyl ligand is, formally,
NO+, with nitrogen in the oxidation state
NO + + 2 OH − ⇌ NO 2 − + H 2 O
Since nitrogen is more electronegative than carbon, metal -nitrosyl complexes
tend to be more electrophilic than related metal carbonyl complexes. Nucleophiles
often add to the nitrogen. The nitrogen atom in bent metal nitrosyls is basic, thus can
be oxidized, alkylated, and protonated,
(Ph 3 P) 2 (CO)ClOsNO + HCl → (Ph 3 P) 2 (CO)ClOsN(H)O
In rare cases, NO is cleaved by metal centers:
Cp 2 NbMe 2 + NO → Cp 2 (Me)Nb(O)NMe
2 Cp 2 (Me)Nb(O)NMe → 2 Cp 2 Nb(O)Me + ½MeN=NMe

Applications
Metal nitrosyl is believed to be an intermediate in catalytic converters to
reduce NOx emissions from internal combustion engines. The application has been
described as "one of the most successful stories in catalyst development.
Metal-catalyzed reactions of NO are not very useful in organic chemistry.
However, in biology and medicine, nitric oxide is an essential signaling molecule
and this fact underlies the most important applications of metal nitrosyl. The
nitroprusside anion, [Fe(CN) 5 NO] 2- , a mixed nitrosyl-cyano complex, has
pharmaceutical uses as a slow-release agent for NO. The signaling function of NO
occurs through complex formation to heme proteins, where it binds curvilinearly.
Nitric oxide also attacks iron-sulfur proteins to produce dinitrosyl-iron complexes.

CHE509: Inorganic Chemistry – II Page 174


SOLVED PROBLEMS 02

1] Give reactions of metal nitrite complexes

Solution: An important reaction is the acid/base equilibrium, yielding


transition metal nitrite complexes:

[Ln MNO] 2+ + 2OH − ⇌ Ln MNO 2 + H 2 O

This equilibrium serves to confirm that the linear nitrosyl ligand is, formally,
NO+, with nitrogen in the oxidation state

NO + + 2 OH − ⇌ NO 2 − + H 2 O

Since nitrogen is more electronegative than carbon, metal -nitrosyl complexes


tend to be more electrophilic than related metal carbonyl complexes. Nucleophil es
often add to the nitrogen. The nitrogen atom in bent metal nitrosyls is basic, thus can
be oxidized, alkylated, and protonated,

(Ph 3 P) 2 (CO)ClOsNO + HCl → (Ph 3 P) 2 (CO)ClOsN(H)O

In rare cases, NO is cleaved by metal centers:

Cp 2 NbMe 2 + NO → Cp 2 (Me)Nb(O)NMe

2 Cp 2 (Me)Nb(O)NMe → 2 Cp 2 Nb(O)Me + ½MeN=NMe


2] Give applications of metal nitrite complexes

Solution: Metal nitrosyl is believed to be an intermediate in catalytic


converters to reduce NOx emissions from internal combustion engines. The
application has been described as "one of the most successful s tories in catalyst
development.
Metal-catalyzed reactions of NO are not very useful in organic chemistry.
However, in biology and medicine, nitric oxide is an essential signaling molecule
and this fact underlies the most important applications of metal nitrosyl. The
nitroprusside anion, [Fe(CN) 5 NO] 2- , a mixed nitrosyl-cyano complex, has
pharmaceutical uses as a slow-release agent for NO. The signaling function of NO
occurs through complex formation to heme proteins, where it binds curvilinearly.
Nitric oxide also attacks iron-sulfur proteins to produce dinitrosyl-iron complexes.

CHE509: Inorganic Chemistry – II Page 175


SHORT ANSWER QUESTIONS WITH MODEL ANSWER 02
1] (Ph 3 P) 2 (CO)ClOsNO + HCl → ?
Solution :(Ph 3 P) 2 (CO)ClOsNO + HCl → (Ph 3 P) 2 (CO)ClOsN(H)O
04-03: TRANSITION METAL DINITROGEN COMPLEX
Transition metal dinitrogen complexes are coordination compounds that contain
transition metals as ion centers the dinitrogen molecules (N 2 ) as ligands.

Structure of [Ru(NH 3 ) 5 (N 2 )] 2+.


In terms of binding to transition metals, N 2 is related to CO and acetylene. This
is because all three species have triple bonds. Various binding modes have been
characterized. Based on whether the N 2 molecule is shared by two other metal
centers, complexes can be divided into mononuclear and bridged complexes. Based
on the geometrical relationship between the N 2 molecule and the metal center, the
complexes can be classified into end-on or side-on modes. In the end-on binding
mode of transition metal dinitrogen complexes, the N–N vector can be viewed as
coincident with the metal ion center, whereas in the side-on mode the metal-ligand
bond is found to be perpendicular to the N–N vector is known.

As a ligand, N 2 typically binds to metals as an "end-on" ligand, as exemplified


by [Ru(NH 3 ) 5 N 2 ] 2+ . Such complexes typically resemble related CO derivatives. This
relationship is illustrated by the complex pair of IrCl(CO)(PPh3) 2 and
IrCl(N 2 )(PPh 3 ) 2 . For these mononuclei, N 2 acts as both a σ donor and a π acceptor.
The M-N-N bond angle is close to 180°. N 2 is a weaker pi acceptor than CO,
reflecting the π* orbital nature of CO with respect to N 2 . For this reason, there are

CHE509: Inorganic Chemistry – II Page 176


few examples of complexes containing both CO and N 2 ligands. Transition metal
dinitrogen complexes can contain multiple N 2 as 'end-on' ligands. B. mer-
[Mo(N 2 ) 3 (PPrn 2 Ph) 3 ] octahedral geometry In another example, the dinitrogen ligand
of Mo(N2) 2 (Ph 2 PCH 2 CH 2 PPh 2 ) 2 can be reduced to ammonia. The Mo-N 2 complex is
of particular interest because many nitrogenases contain Mo.

N 2 can also act as a bridging ligand with “end-on” bonding to two metal
centers, exemplified by {[Ru(NH 3 ) 5 ] 2 (μ-N 2 )} 4+ . These complexes are also called
polynuclear dinitrogen complexes. In contrast to their mononuclear counterparts,
they can be prepared for both early and late transition metals, Holland and
coworkers Studies on nitrogen complexes have shown that the N –N bond is greatly
weakened when complexed with low-coordination number iron atoms. This complex
contains a bidentate chelating ligand bound to the iron atoms of the Fe-N-N-Fe core,
with N 2 he acting as a bridging ligand between the two iron atoms. We show that
increasing the coordination number of iron by changing the chelating ligand and
adding one more ligand per iron atom increases the strength of the His NÀN bond of
the resulting complex. it was done. Therefore, it is hypothesized that Fe in a low -
coordination environment is a key factor in nitrogen fixation by nitrogenase
enzymes. This is because its Fe-Mo cofactor also contains Fe with a low
coordination number. The average bond length of these endon-bridged dinitrogen
complexes is about 1.2 Å. In some cases, bond lengths can reach 1.4 Å, which is
similar to N-N single bonds.

SOLVED PROBLEMS 03

1] Discribe the transition bonding in metal dinitrogen complexes with examples

Solution: Transition metal dinitrogen complexes are coordination compounds


that contain transition metals as ion centers the dinitrogen molecules (N 2 ) as ligands.

CHE509: Inorganic Chemistry – II Page 177


Structure of [Ru(NH 3 ) 5 (N 2 )] 2+.

In terms of binding to transition metals, N 2 is related to CO and acetylene. This


is because all three species have triple bonds. Various binding modes have been
characterized. Based on whether the N 2 molecule is shared by two other metal
centers, complexes can be divided into mononuclear and bridged complexes. Based
on the geometrical relationship between the N 2 molecule and the metal center, the
complexes can be classified into end-on or side-on modes. In the end-on binding
mode of transition metal dinitrogen complexes, the N–N vector can be viewed as
coincident with the metal ion center, whereas in the side-on mode the metal-ligand
bond is found to be perpendicular to the N–N vector is known.

As a ligand, N 2 typically binds to metals as an "end-on" ligand, as exemplified


by [Ru(NH 3 ) 5 N 2 ] 2+ . Such complexes typically resemble related CO derivatives. This
relationship is illustrated by the complex pair of IrCl(CO)(PPh 3 ) 2 and
IrCl(N 2 )(PPh 3 ) 2 . For these mononuclei, N 2 acts as both a σ donor and a π acceptor.
The M-N-N bond angle is close to 180°. N 2 is a weaker pi acceptor than CO,
reflecting the π* orbital nature of CO with respect to N 2 . For this reason, there are
few examples of complexes containing both CO and N 2 ligands. Transition metal
dinitrogen complexes can contain multiple N 2 as 'end-on' ligands. B. mer-
[Mo(N 2 ) 3 (PPrn 2 Ph) 3 ] octahedral geometry In another example, the dinitrogen ligand
of Mo(N2) 2 (Ph 2 PCH 2 CH 2 PPh 2 ) 2 can be reduced to ammonia. The Mo-N 2 complex is
of particular interest because many nitrogenases contain Mo.

SHORT ANSWER QUESTIONS WITH MODEL ANSWER 03


1] Definetransition metal dinitrogen complexes
Solution: Transition metal dinitrogen complexes are coordination compounds
that contain transition metals as ion centers the dinitrogen molecules (N 2 ) as ligands.
04-04: PREPRATION OF TRANSITION METAL DINITROGEN COMPLEXES
The innovative Haber-Bosch method has long been thought to involve the
adsorption of dinitrogen as the nitrogen atom onto an active iron catalyst at high
pressure and temperature. In, a nitrogen metal complex, numerous ligands can
coexist with the dinitrogen on the metal atom. Other ligands compatible with
nitrogen in metal complexes include phosphines, halides, hydrides, ammonia, carbon
monoxide, and even water. In addition, nitrogen-fixing bacteria (metalloenzyme
nitrogenase) catalytically convert nitrogen to ammonium ions. The active centers of

CHE509: Inorganic Chemistry – II Page 178


these bacteria contain Fe, Mo, and S. Inspired by this, a number of Fe and Mo have
been synthesized and research is ongoing to develop effici ent artificial catalysts.

The reaction is exciting because it may provide a model for the first step in the
"fixation" of dinitrogen by nitrogenase. The enzymic reaction is competitively
inhibited by dihydrogen, which is consistent with a rapid equilibri um reaction
between a polyhydride and N2 to give a dinitrogen complex.
[MHn (PR 3 ) 3 ] + N 2 → [MHn- 2 (N 2 )(PR 3 ) 3 ] + H 2
This reaction is exciting because it can provide a model for the first step in the
'fixation' of dinitrogen by nitrogenase. The enzymatic reaction is competitively
inhibited by dihydrogen, consistent with a rapid equilibrium reaction between
polyhydrides and N 2 to form dinitrogen complexes.
[(RuL5 H 2 O] 2+ + N 2 → [(RuL5 (N 2 )] 2+ + [(L5 RuN 2 RuL5 ] 4+
+ H2O
Isopropylmagnesium chloride reacts with [FeC1 3 (PPh 3 ) 2 ] under N 2 to form a
completely uncharacterizedcomplex containing 1 mol N2 per 2 mol
iron. The Grignard
reagent has also been used in the preparation of his N2 complexes of molybdenum
and titanium.
SOLVED PROBLEMS 04
1] Preparation of dinitrogen metal complex
Solution:The reaction is exciting because it may provide a model for the first
step in the "fixation" of dinitrogen by nitrogenase. The enzymic reaction is
competitively inhibited by dihydrogen, which is consistent with a rapid equilibrium
reaction between a polyhydride and N2 to give a dinitrogen complex.
[MHn(PR 3 ) 3 ] + N 2 → [MHn- 2 (N 2 )(PR 3 ) 3 ] + H 2

CHE509: Inorganic Chemistry – II Page 179


This reaction is exciting because it can provide a model for the first step in the
'fixation' of dinitrogen by nitrogenase. The enzymatic reaction is competitively
inhibited by dihydrogen, consistent with a rapid equilibrium reaction between
polyhydrides and N 2 to form dinitrogen complexes.
[(RuL5 H 2 O] 2+ + N 2 → [(RuL5 (N 2 )] 2+ + [(L5 RuN 2 RuL5 ] 4+
+ H2O
Isopropylmagnesium chloride reacts with [FeC1 3 (PPh 3 ) 2 ] under N 2 to form a
completely uncharacterizedcomplex containing 1 mol N2 per 2 mol
iron. The Grignard
reagent has also been used in the preparation of his N2 complexes of molybdenum
and titanium.
CHECK POINT 01-04
1] Linear and curvilinear NO ligands can be distinguished by infrared
spectroscopy.
a) Linear M-N-O groups absorb between 1650 and 1900 cm-1, while curved
nitrosyl groups absorb between 1525 and 1690 cm-1.
b) Linear M-N-O groups absorb between 1359 and 900 cm-1, while curved
nitrosyl groups absorb between 1550 and 1690 cm-1.
c) Linear M-N-O groups absorb between 1525 and 1900 cm-1, while curved
nitrosyl groups absorb between 1525 and 1500 cm-1.
d) Linear M-N-O groups absorb between 1250 and 1700 cm-1, while curved
nitrosyl groups absorb between 17505 and 1600 cm-1.
Solution: a

2] side-on mode of the metal-ligand bond is found to be _____to the N–N


vector is known.
a) perpendicular
b) Parallel
Solution: a

3] The use of linear and curved binding can be analyzed usi ng Enemark-
Feltham notation.
a) True
b) False
Soluton: a

CHE509: Inorganic Chemistry – II Page 180


4] Metal-nitrosyl complexes tend to be more electrophilic than related metal
carbonyl complexes because __
a) Carbon is more electronegative than Nitrogen
b) Nitrogen is more electropositive than carbon
c) Nitrogen is more electronegative than Carbon
d) Nitrogen is electron donor
Solution: c

SUMMARY

Most complexes containing the NO ligand can be viewed as derivatives of the


nitrosyl cation, NO+. The nitrosyl cation is isoelectronic with carbon monoxide, thus
the bonding between a nitrosyl ligand and a metal follows the same principles as the
bonding in carbonyl complexes. The nitrosyl cation serves as a two -electron donor to
the metal and accepts electrons from the metal via back -bonding.The chemical
conversion of dinitrogen to commercially useful raw materials is one of the most
important reactions in the chemical industry and has long been the subject of
extensive research1. The strong triple bond activation of N 2 is a challenge for
ingenious chemists who have tried various transition metals in the past to bind
molecules as ligands for molecular complexes. Dinitrogen is a weak binding donor
due to relatively weak donor-acceptor interactions in transition metal complexes.

KEY WORDS

Nitrosyl complexes, dinitrogen complexes

REFERENCES

MOOCS

______

YOUTUBE VIDEOS

1) https://www.youtube.com/watch?v=7T4-ohp5ZgA
2) https://www.youtube.com/watch?v=kcxD8VXgpE8
3) https://www.youtube.com/watch?v=kcxD8VXgpE8

CHE509: Inorganic Chemistry – II Page 181


4) https://www.youtube.com/watch?v=NbxofuhhxCU

WIKIPEDIA
1) https://en.wikipedia.org/wiki/Metal_nitrosyl_complex#:~:text=Metal%20nitros
yl%20complexes%20are%20complexes,in%20structure%20and%20coligand .
2) https://en.wikipedia.org/wiki/Transition_metal_dinitrogen_complex#:~:text=Tr
ansition%20metal%20dinitrogen%20complexes%20are,(N2)%20as%20ligands.
3) https://en.wikipedia.org/wiki/Transition_metal_dinitrogen_complex#:~:text=Tr
ansition%20metal%20dinitrogen%20complexes%20are,(N2)%20as%20ligands

OER
1) https://pubs.acs.org/doi/10.1021/jacs.9b08053

REFERENCE BOOKS
1) Inorganic Chemistry Principles of Structure and Reactivity 2006 by James E
Huheey and Medhi
2) Concise Inorganic Chemistry Wiley's J.D. Lee

CHE509: Inorganic Chemistry – II Page 182


C REDIT 03

CHE509: Inorganic Chemistry – II Page 183


CREDIT 03 -UNIT 01: CARBOCYCLIC PI COMPLEXES AND REACTION OF
COORDINATION COMPOUNDS

LEARNING OBJECTIVES

After successful completion of this unit, you will be able to


 Understand the concept of metallocene
 Understand the nature of cyclopentadienyl legands
 Understand the acylation of metallocene
 Understand the acylation of alkylation of metallocene
 Understand the reactions of metallocene
INTRODUCTION

The term metallocene refers to bis(cyclopentadiene) metal complexes


(“sandwiches”), but now cyclopentadienyl complexes (“half -sandwiches”) and
multicyclopentadienyl complexes (“multidecker sandwi ches”) A wide range of
usages is accepted, including Additional substitutions at the metal center. Metal
prefixes cannot be used when considering the evolution of complexes with non -
metallic elements such as boron, silicon, or arsenic. In fact, metallocene -like
complexes are now known for many of the elements of the periodic table.

01-01: THE CYCLOPENTADIENE LIGAND

The cyclopentadienyl moiety serves as the key 'spectator' ligand and has the
ubiquitous organometallic chemistry. It remains inert to most nucle ophiles and
electrophiles and is used only to stabilize organometallic complexes.
Cyclopentadienyl Ligands Form Various Organometallic Compounds used for These
compounds have varying compositions, starting with the so -called 'piano stool'
CpMLn type (n = 2, 3, or 4), the most commonly observed 'metallocene' Cp2M type,
and the 'bent metallocene' Cp2MXn (n = 1, 2 or 3) type. , "piano stool" CpMLn
structure, cyclopentadienyl (Cp) ligands are considered his "seats" of the piano
stool, the remaining L ligands are called "legs" of his piano stool increase.
Cyclopentadienyl ligands often bind to metals in the η5 (pentahapto) fashion. Like
G. ferrocene, other forms of binding to metals with low tactile sensitivity such as in
the η3 (trihapto) bond. B. For (η 5 −Cp)(η 3 −Cp)W(CO) 2 and η1 (monohapto) bonds. is
also rarely observed, like (η 5 −Cp)(η1−Cp)Fe(CO) 2 . The binding mode of
cyclopentadienyl ligands in metal complexes can be determined to some extent by

CHE509: Inorganic Chemistry – II Page 184


1H NMR in diamagnetic metal complexes, where the Cp proton appears as a singlet
between 5-5.

The interaction of cyclopentadiene with metal

The interaction of cyclopentadiene and metals can be understood in the molecular


orbital diagram. The frontier orbitals of cyclopentadienyl ligands include five
orbitals (Ψ1−Ψ5) in three energy levels. The lowest energy orbital Ψ1 contains no
nodes and is represented by doubly degenerate e1 states consisting of the a1 state
followed by Ψ2 and Ψ3 orbitals. Another doubly degenerate e2 states consist of the
Ψ4 and Ψ5 orbitals.

The frontier molecular orbital diagram above becomes more interesting as we


progress through to metallocenes containing two cyclopentadienyl ligands. In
particular, in the Cp 2 M system (such as ferrocene), each of these five molecular
orbitals on top of two cyclopentadienyl ligands combine to form ten ligands at three
energy levels. You can get the molecular orbital.

Common Cp 2 M-type metallocene complexes are formed in many of the


transition metal series in the th row along Sc to Zn. Therefore, the numbers of
unpaired electrons in correlates with the number of unpaired electrons present in the
metal's valence orbitals. Manganocene exists in two different forms. Like Cp2Mn,
other with low spin forms with unpaired electrons. This is similar to that of Cp*2Mn
due to the higher ligand field strength of Cp* ligands. Cobaltocene, Cp 2 Co, has 19

CHE509: Inorganic Chemistry – II Page 185


valence electrons (VE) and is therefore readily oxidized to the diamagnetic 18 -
valence electron species, Cp 2 Co + .

Bent metallocenes are Cp2MXn type complexes formed from group 4 an d the
heavier group 5-7 elements. In these complexes, the doubly degenerate e2g frontier
orbitals of the Cp2M fragment interact with the filled lone pair orbitals of the ligand.

SOLVED PROBLEMS 01

1] Write a note on cyclopentadienyl legands in 'metallocene'

Solution: The cyclopentadienyl moiety serves as the key 'spectator' ligand and
has the ubiquitous organometallic chemistry. It remains inert to most nucleophiles
and electrophiles and is used only to stabilize organometallic complexes.
Cyclopentadienyl Ligands Form Various Organometallic Compounds used for These
compounds have varying compositions, starting with the so -called 'piano stool'
CpMLn type (n = 2, 3, or 4), the most commonly observed 'metallocene' Cp2M type,
and the 'bent metallocene' Cp2MXn (n = 1, 2 or 3) type. , "piano stool" CpMLn
structure, cyclopentadienyl (Cp) ligands are considered his "seats" of the piano
stool, the remaining L ligands are called "legs" of his piano stool increase.
Cyclopentadienyl ligands often bind to metals in the η5 (pentahapto) fashion. Like
G. ferrocene, other forms of binding to metals with low tactile sensitivity such as in

CHE509: Inorganic Chemistry – II Page 186


the η3 (trihapto) bond. B. For (η 5 −Cp)(η 3 −Cp)W(CO) 2 and η1 (monohapto) bonds. is
also rarely observed, like (η 5 −Cp)(η1−Cp)Fe(CO) 2 . The binding mode of
cyclopentadienyl ligands in metal complexes can be determined to some extent by
1H NMR in diamagnetic metal complexes, where the Cp proton appears as a singlet
between 5-5.

2] Describe the interaction of cyclopentadiene with metal according to MOT.

Solution: The interaction of cyclopentadiene with metal

The interaction of cyclopentadiene and metals can be understood in the


molecular orbital diagram. The frontier orbitals of cyclopentadienyl ligands include
five orbitals (Ψ1−Ψ5) in three energy levels. The lowest energy orbital Ψ1 contains
no nodes and is represented by doubly degenerate e1 states consisting of the a1 state
followed by Ψ2 and Ψ3 orbitals. Another doubly degenerate e2 states consist of the
Ψ4 and Ψ5 orbitals.

The frontier molecular orbital diagram above becomes more interesting as we


progress through to metallocenes containing two cyclopentadienyl ligands. In
particular, in the Cp 2 M system (such as ferrocene), each of these five molecular
orbitals on top of two cyclopentadienyl ligands combine to form ten ligands at three
energy levels. You can get the molecular orbital.

Common Cp 2 M-type metallocene complexes are formed in many of the


transition metal series in the th row along Sc to Zn. Therefore, the numbers of

CHE509: Inorganic Chemistry – II Page 187


unpaired electrons in correlates with the number of unpaired electrons present in the
metal's valence orbitals. Manganocene exists in two different forms. Like Cp2Mn,
other with low spin forms with unpaired electrons. This is similar to that of Cp*2Mn
due to the higher ligand field strength of Cp* ligands. Cobaltocene, Cp 2 Co, has 19
valence electrons (VE) and is therefore readily oxidized to the diamagnetic 18 -
valence electron species, Cp 2 Co + .

Bent metallocenes are Cp2MXn type complexes formed from group 4 and th e
heavier group 5-7 elements. In these complexes, the doubly degenerate e2g frontier
orbitals of the Cp2M fragment interact with the filled lone pair orbitals of the ligand.

SHORT ANSWER QUESTIONS WITH MODEL ANSWER 02

1] What is sandwich complex? Give examples

Solution: In organometallic chemistry, a sandwich compound is a chemical


compound featuring a metal bound by haptic, covalent bonds to two arene (ring)
ligands. E.g. ferrocene Cp 2 Fe

01-02 SYNTHESIS OF METALLOCENES

Metathesis reactions are widely used in the synthesis of metallocenes. Alkali


metal cyclopentadienide, magnesium dicyclopentadienide, and thallium
cyclopentadienide are important synthetic sources of Cp' ligands. These and other

CHE509: Inorganic Chemistry – II Page 188


commonly applied methods and specific reactions (used for specific conjugates) are
grouped under Key Examples and Synthetic Procedures for specific conjugates.

Synthesis by reaction using Cyclopentadienes:


1. 2E + 2Cp'H →2ECp' + H2 or E + 2Cp'H ECp'2 + H2
2. ER + Cp’H → ECp’ + RH
3. EH + Cp'H → ECp' + H2
4. ENH2 + Cp'H = ECp’ + NH3

Cyclopentadiene has a relatively acidic proton and readily reacts with many
more reactive main group elements such as sodium and magnesium, and certain
metallocenes can be prepared by cocondensation.

Cyclopentadiene with elements. Equation (1) has been extrapolated to other His
Cp′ derivatives and is valid for other organometallic reagents such as MgMe2 and
Al2Me6. Metallocene compounds of sodium or potassium metals can be obtained by
reaction of elemental hydrides with cyclopentadiene. (3), or the reaction of
elemental amides with cyclopentadiene in liquid ammonia (4). Similar amide
substitution reactions are used for chalcosene and barocene derivatives.

Synthesis by Salt Metathesis Reactions

nMCp' + EXn → Cp'nE + nMX (M = Li, Na)

or

n/2 MgCp'2 + EXn → Cp'nE + n/2MgXm

Salt metathesis reactions are the most commonly used method for the
preparation of metallocenes containing Groups 2, 13, 14, and 15 elements.
Dicyclopentadienide magnesium salt. Complexes of aluminum and gallium
containing lesser known lower oxidation state elements are obtained from the in situ
formed metastable halides 'AlCl' and 'GaCl'.

Salt metathesis reactions are the most commonly used method to produce
metallocenes containing group 2, 13, 14, and 15 element dicyclopentadienide
magnesium salts. Complexes of aluminum and gallium containing lesser known
lower oxidation state elements are obtained from the in situ formed metastable
halides 'AlCl' and 'GaCl'.

CHE509: Inorganic Chemistry – II Page 189


Disproportionation, Comproportionation and Decomposition

2Cp'EX → Cp'2E + EX 2

and

2Cp'EX → Cp'2E + EX 2

This method is useful for the synthesis of magnesocene and its methylated
derivatives by thermal decomposition of Cp'MgBr. Proportionation reactions were
used to obtain substituted derivatives of the semi-sandwich complexes of
magnesium, germanium, and tin. The reductive elimination of the Cp ligand from
Cp3In and the phenyl group from Cp 2 PbPh 2 is also thermally induced.

SOLVED PROBLEMS 02

1 ] Give synthetic methods for synthesis of metallocenes

Solution: Metathesis reactions are widely used in the synthesis of metallocenes.


Alkali metal cyclopentadienide, magnesium dicyclopentadienide, and thallium
cyclopentadienide are important synthetic sources of Cp' ligands. These and other
commonly applied methods and specific reactions (used for specific conjugates) are
grouped under Key Examples and Synthetic Procedures for specific conjugates.

Synthesis by reaction using Cyclopentadienes:


1. 2E + 2Cp'H →2ECp' + H2 or E + 2Cp'H ECp'2 + H2
2. ER + Cp'H → ECp' + RH
3. EH + Cp'H → ECp' + H2
4. ENH2 + Cp'H = ECp' + NH3

Cyclopentadiene has a relatively acidic proton and readily reacts with many
more reactive main group elements such as sodium and magnesium, and certain
metallocenes can be prepared by cocondensation.

Cyclopentadiene with elements. Equation (1) has been extrapolated to other His
Cp′ derivatives and is valid for other organometallic reagents such as MgMe2 and
Al2Me6. Metallocene compounds of sodium or potassium metals can be obtained by
reaction of elemental hydrides with cyclopentadiene. (3), or the reaction of
elemental amides with cyclopentadiene in liquid ammonia (4). Similar amide
substitution reactions are used for chalcosene and barocene derivatives.

CHE509: Inorganic Chemistry – II Page 190


Synthesis by Salt Metathesis Reactions

nMCp' + EXn → Cp'nE + nMX (M = Li, Na)

or

n/2 MgCp'2 + EXn → Cp'nE + n/2MgXm

Salt metathesis reactions are the most commonly used method for the
preparation of metallocenes containing Groups 2, 13, 14, and 15 elements.
Dicyclopentadienide magnesium salt. Complexes of aluminum and gallium
containing lesser known lower oxidation state elements are obtained from the in situ
formed metastable halides 'AlCl' and 'GaCl'.

Salt metathesis reactions are the most commonly used method to produce
metallocenes containing group 2, 13, 14, and 15 element dicyclopentadienide
magnesium salts. Complexes of aluminum and gallium containing lesser known
lower oxidation state elements are obtained from the in situ formed metastable
halides 'AlCl' and 'GaCl'.

Synthesis of magnesoceneby Decomposition

2Cp'EX → Cp'2E + EX 2

and

2Cp'EX → Cp'2E + EX 2

This method is useful for the synthesis of magnesocene and its methylated
derivatives by thermal decomposition of Cp'MgBr. Proportionation reactions were
used to obtain substituted derivatives of the semi -sandwich complexes of
magnesium, germanium, and tin. The reductive elimination of the Cp ligand from
Cp3In and the phenyl group from Cp 2 PbPh 2 is also thermally induced.

SHORT ANSWER QUESTIONS WITH MODEL ANSWER 01

1] Describe the Synthesis of metallocenes by Salt Metathesis Reactions

Solution: Synthesis by Salt Metathesis Reactions


nMCp' + EXn → Cp'nE + nMX (M = Li, Na)

or

n/2 MgCp'2 + EXn → Cp'nE + n/2MgXm

CHE509: Inorganic Chemistry – II Page 191


Salt metathesis reactions are the most commonly used method for the
preparation of metallocenes containing Groups 2, 13, 14, and 15 elements.
Dicyclopentadienide magnesium salt. Complexes of aluminum and gallium
containing lesser known lower oxidation state elements are obtained from the in situ
formed metastable halides 'AlCl' and 'GaCl'.

2] DescribeSynthesis of magnesoceneby Decomposition

Solution : Synthesis of magnesoceneby Decomposition

2Cp'EX → Cp'2E + EX 2

and

2Cp'EX → Cp'2E + EX 2

This method is useful for the synthesis of magnesocene and its methylated
derivatives by thermal decomposition of Cp'MgBr. Proportionation reactions were
used to obtain substituted derivatives of the semi-sandwich complexes of
magnesium, germanium, and tin. The reductive elimination of the Cp ligand from
Cp3In and the phenyl group from Cp 2 PbPh 2 is also thermally induced.
01-03: STRUCTURE OF METALLOCENE
The structure of the main group metallocene chemistry consists of the position
of the element relative to the Cp' ligands, the number of Cp' ligands relative to the
elemental center and their relative orientations, and the involvement
of ancillary ligands (other than Cp' derivatives). changes depending on ) element and
the association of the dimer-forming complex, oligomer or polymer. The general
cyclic fractionality of Cp' ligands indicates that all five carbon atoms within the Cp'
framework participate in π-interactions of the Cp' elements, making the system less
tactile. Means Non-equivalent element-carbon distances obtained from X-ray
crystallographic data often show closer interactions with some framework carbon
atoms than with others. This can be defined by the similarity or difference between
the Cp' centroid E-distance and the Cp' plane E-distance.
Sandwich Complexes

CHE509: Inorganic Chemistry – II Page 192


Staggered conformations and Bent ‘sandwich’ structure

Most sandwich complexes involve equivalent interactions with each Cp' ligand.
LiCp5Bz is a rare example of asymmetric coordination of two Cp ligands to the
lithium center.

Half Sandwich Complexes

Many elements can be complexed with a single π-coordinated Cp ligand, as


shown at in Figure 8. Where Y is attached with a non-bonding electron pair, covalent
bond (substituent R), or coordinative covalent bond (auxiliary bond) ligands) are
involved. The formation of molecular half-sandwich complexes of elements depends
on the valence and size of the elements. The broadest set of complexes of lithium
(e.g. Cp3TLi THF) and beryllium (e.g. CpBeCl) are known and cation systems of
boron (e.g. [Cp*BBr][AlBr4]) are available.

Elemental center association via π-bonded µ-Cp' ligands is a common feature


of polymer structures. We also show a neutral ketalocene bound by an additional
cyclopentadiene anion in the molecular anion system. As shown in Figure 9, the
general formula [E2Cp3]– is observed as a non-parallel multi-decker sandwich of
cesium, and the thallium and plumcene sandwich unit Cp2Pb is linked to the unusual
anions [Cp5Pb2] and [Cp9Pb4] Related to Each lead center has three
cyclopentadienyl linkages, like the tris(cyclopentadienyl) anion known for both tin
and lead in the salt [Mg(THF)6][Cp3E]2. There are ranks.

CHE509: Inorganic Chemistry – II Page 193


Dimers sandwich complexes

A dimeric arrangement is observed in the solid state of Cp5BzIn and Cp5BzTl


(one of two crystalline forms). Magnesium, calcium, aluminum, and bismuth
compounds adopt the more familiar dimeric structure involving halide bridges with
terminal Cp' sandwich complexes (X = halogen; Y = halogen or donor). increase.

SOLVED PROBLEMS 03

1] Describe the Sandwich and Half Sandwich Complexes.

Solution: The structure of the main group metallocene chemistry consists


of the position of the element relative to the Cp' ligands, the number of Cp'
ligands relative to the elemental center and their relative orientations, and the
involvement of ancillary ligands (other than Cp' derivatives). changes depending on
) element and the association of the dimer-forming complex, oligomer or polymer.
The general cyclic fractionality of Cp' ligands indicates that all five carbon atoms
within the Cp' framework participate in π-interactions of the Cp' elements, making
the system less tactile. Means Non-equivalent element-carbon distances obtained
from X-ray crystallographic data often show closer interactions with some
framework carbon atoms than with others. This can be defined by the similarity or
difference between the Cp' centroid E-distance and the Cp' plane E-distance.

Sandwich Complexes

CHE509: Inorganic Chemistry – II Page 194


Staggered conformations and Bent ‘sandwich’ structure

Most sandwich complexes involve equivalent interactions with each Cp' ligand.
LiCp5Bz is a rare example of asymmetric coordination of two Cp ligands to t he
lithium center.

Half Sandwich Complexes

Many elements can be complexed with a single π-coordinated Cp ligand, as


shown at in Figure 8. Where Y is attached with a non-bonding electron pair, covalent
bond (substituent R), or coordinative covalent bond (auxiliary bond) ligands) are
involved. The formation of molecular half-sandwich complexes of elements depends
on the valence and size of the elements. The broadest set of complexes of lithium
(e.g. Cp3TLi THF) and beryllium (e.g. CpBeCl) are known and cation systems of
boron (e.g. [Cp*BBr][AlBr4]) are available.

Elemental center association via π-bonded µ -Cp' ligands is a common feature


of polymer structures. We also show a neutral ketalocene bound by an additional
cyclopentadiene anion in the molecular anion system. As shown in Figure 9, the
general formula [E2Cp3]– is observed as a non-parallel multi-decker sandwich of
cesium, and the thallium and plumcene sandwich unit Cp2Pb is linked to the unusual
anions [Cp5Pb2] and [Cp9Pb4] Related to Each lead center has three
cyclopentadienyl linkages, like the tris(cyclopentadienyl) anion known for both tin
and lead in the salt [Mg(THF)6][Cp3E]2. There are ranks.

CHE509: Inorganic Chemistry – II Page 195


2] Write a note on Half Sandwich and Dimers sandwich complexes

Solution: Half Sandwich Complexes

Many elements can be complexed with a single π-coordinated Cp ligand, as


shown at in Figure 8. where Y is attached with a non-bonding electron pair, covalent
bond (substituent R), or coordinative covalent bond (auxiliary bond). ligands) are
involved. The formation of molecular half-sandwich complexes of elements depends
on the valence and size of the elements. The broadest set of complexes of lithium
(e.g. Cp3TLi THF) and beryllium (e.g. CpBeCl) are known and cation systems of
boron (e.g. [Cp*BBr][AlBr4]) are available.

Elemental center association via π-bonded µ -Cp' ligands is a common feature


of polymer structures. We also show a neutral ketalocene bound by an additional
cyclopentadiene anion in the molecular anion system. As shown in Figure 9, the
general formula [E2Cp3]– is observed as a non-parallel multi-decker sandwich of
cesium, and the thallium and plumcene sandwich unit Cp2Pb is linked to the unusual
anions [Cp5Pb2] and [Cp9Pb4] Related to Each lead center has three
cyclopentadienyl linkages, like the tris(cyclopentadienyl) anion known for both tin
and lead in the salt [Mg(THF)6][Cp3E]2. There are ranks.

Dimers sandwich complexes

A dimeric arrangement is observed in the solid state of Cp5BzIn and Cp5BzTl


(one of two crystalline forms). Magnesium, calcium, aluminum, and bismuth
compounds adopt the more familiar dimeric structure involving halide bridges with
terminal Cp' sandwich complexes (X = halogen; Y = halogen or donor). increase.

CHE509: Inorganic Chemistry – II Page 196


SHORT ANSWER QUESTIONS WITH MODEL ANSWER 03
1] Explain the structure of Half Sandwich Complexes.

Solution: Many elements can be complexed with a single π-coordinated Cp


ligand, as shown at in Figure 8. Where Y is attached with a non-bonding electron
pair, covalent bond (substituent R), or coordinative covalent bond (auxiliary bond)
ligands) are involved. The formation of molecular half-sandwich complexes of
elements depends on the valence and size of the elements. The broadest set of
complexes of lithium (e.g. Cp3TLi THF) and beryllium (e.g. CpBeCl) are known and
cation systems of boron (e.g. [Cp*BBr][AlBr4]) are available.

01-04: REACTIONS OF METALLOCENES

Acylation

Acylation of metallocenes has been studied more extensively than any other
substitution reaction. A good example is the Friedel -Crafts reaction between
ferrocene and acetyl chloride in the presence of aluminum chloride. Using these
reaction partners in equimolar amounts of leads to almost exclusively formation of
the monoacetylferrocene. Excess use of acetyl chloride and aluminum chloride gives
, a mixture of two isomeric diacetylferrocenes. It can be separated by careful
chromatography on activated aluminum.

CHE509: Inorganic Chemistry – II Page 197


The main product is the heterocyclic disubstituted derivative 1,1' -
diacetylferrocene , but a very small amount of the angular cyclic isomer 1,2 -
diacetylferrocene is also obtained. The first acetyl group appears to deactivate the
substituted ring toward further electrophilic substitution of , while the second acetyl
group preferentially enters the opposite ring of .

The structure of isomer major substitution product was confirmed by evidence


of decomposition. Under high temperature and pressure impingement this product
was catalytically hydrogenated to ethylcyclopentane to eliminate cyclopentane and
diethylcyclopentane. The isomeric minor substitution product was revealed by
haloform oxidation to the diacid followed by facile formation of the anhydride.

Alkylation

Ferrocene is readily alkylated by alkyl halides, alcohols, or olefins


to yield alkylated derivative. The very high reactivity of ferrocene under these
conditions results in the formation of mixtures of mono -, di-, tri- and poly-alkylated
products, with usually low yields of alkylated products. Like benzenoid chemistry.

Introduction of an alkyl group to the ferrocene core facilitates subsequent


alkylation on the same ring. Aralkyl ferrocenes can be conveniently prepared by
reacting ferrocene with a substituted carbinol such as triphenylcarbinol in the
presence of a Lewis acid and carrying out the reaction in the melt phase.

CHE509: Inorganic Chemistry – II Page 198


SOLVED PROBLEMS 04

1] Explain the reactions of ferrocene.

Solution: Acylation

Acylation of metallocenes has been studied more extensively than any other
substitution reaction. A good example is the Friedel -Crafts reaction between
ferrocene and acetyl chloride in the presence of aluminum chloride. Using these
reaction partners in equimolar amounts of leads to almost exclusively formation of
the monoacetylferrocene. Excess use of acetyl chloride and aluminum chloride gives
, a mixture of two isomeric diacetylferrocenes. It can be separated by careful
chromatography on activated aluminum.

The main product is the heterocyclic disubstituted derivative 1,1' -


diacetylferrocene , but a very small amount of the angular cyclic isomer 1,2 -
diacetylferrocene is also obtained. The first acetyl group appears to deactivate the
substituted ring toward further electrophilic substitution of , while the second acetyl
group preferentially enters the opposite ring of .

The structure of isomer major substitution product was confirmed by evidence


of decomposition. Under high temperature and pressure impingement this product
was catalytically hydrogenated to ethylcyclopentane to eliminate cyclopentane and
diethylcyclopentane. The isomeric minor substitution product was revealed by
haloform oxidation to the diacid followed by facile formation of the anhydride.

Alkylation

Ferrocene is readily alkylated by alkyl halides, alcohols, or olefins


to yield alkylated
derivative. The very high reactivity of ferrocene under these conditions results in the

CHE509: Inorganic Chemistry – II Page 199


formation of mixtures of mono-, di-, tri- and poly-alkylated products, with usually
low yields of alkylated products. Like benzenoid chemistry.

Introduction of an alkyl group to the ferrocene core facilitates subsequent


alkylation on the same ring. Aralkylferrocenes can be conveniently prepared by
reacting ferrocene with a substituted carbinol such as triphenylcarbinol in the
presence of a Lewis acid and carrying out the reaction in the melt phase.

2] Write a note on Acylation of metallocenes.

Solution:

Acylation of metallocenes has been studied more extensively than any other
substitution reaction. A good example is the Friedel -Crafts reaction between
ferrocene and acetyl chloride in the presence of aluminum chloride. Using these
reaction partners in equimolar amounts of leads to almost exclusively formation of
the monoacetylferrocene. Excess use of acetyl chloride and aluminum chloride gives
, a mixture of two isomeric diacetylferrocenes. It can be separated by careful
chromatography on activated aluminum.

The main product is the heterocyclic disubstituted derivative 1,1' -


diacetylferrocene , but a very small amount of the angular cyclic isomer 1,2 -
diacetylferrocene is also obtained. The first acetyl group appears to deactivate the
substituted ring toward further electrophilic substitution of , while the second acetyl
group preferentially enters the opposite ring of .

CHE509: Inorganic Chemistry – II Page 200


The structure of isomer major substitution product was confirmed by evidence
of decomposition. Under high temperature and pressure impin gement this product
was catalytically hydrogenated to ethylcyclopentane to eliminate cyclopentane and
diethylcyclopentane. The isomeric minor substitution product was revealed by
haloform oxidation to the diacid followed by facile formation of the anhydride.

SHORT ANSWER QUESTIONS WITH MODEL ANSWER 04

1]

Solution:

CHE509: Inorganic Chemistry – II Page 201


CHECK POINT 01-04

1] The bonding of a hydrogen atom to a metal atom is realized through:

a) π-backbonding only

b) σ-bonding only

c) a combination of σ- bonding and π-backbonding

d) None of these

Solution: b, The bonding of a hydrogen atom to a metal atom is realized through σ -


bonding only.

2] Backbonding from a metal atom to a benzene ligand is possible only as a

a) s-interaction between a1 benzene MO and the dz2 metal orbital in correct

b) p-interaction between e 1 , e 2 benzene MO and the d zx , d yz metal orbitals

c) d-interaction between the antibonding e2 benzene orbitals and the dx2 -y2, dxy
metal orbitals

d) None of these

Solution: c, Backbonding from a metal atom to a benzene ligand is possible only as


a d-interaction between the antibonding e2 benzene orbitals and the dx2 - y2, dxy
metal orbitals

3] Many metallocenes exhibit fluxionality and undergo internal rotation because

a) the energy barrier to the interconversion is nonexistent

b) the energy barrier to the interconversion is low

c) the energy barrier to the interconversion is infinite

d) None of these

Solution: b, Many metallocenes exhibit fluxionality and undergo internal rotation


because the energy barrier to the interconversion is low.

4] Which statement is correct?

CHE509: Inorganic Chemistry – II Page 202


a) CO is very good at stabilizing low oxidation states; many complexes can have
the metal in its zero oxidation state.

b) CO is very good at stabilizing low oxidation states; many complexes can have
the metal in its highest oxidation state

c) CO is very good at stabilizing low oxidation states; however, the metal is


never present in zero oxidation state

Solution: a, Carbon monoxide is very good at stabilizing low oxidation states; many
complexes containing CO ligand can have the central metal in its zero oxidation
state.

SUMMARY

The chemistry of organometallic compounds has fascinated chemists since the field began
more than a century ago. Many great discoveries have been made in recent years, and
these discoveries have had a great impact on the science and technology of
chemistry. Despite extensive research in organometallic chemistry from about 1830
to his 1950s, the preparation and study of organic derivatives of transition metals
remained largely a mystery. A number of main group metallocene complexes have
now been identified and have been extensively studied. Most group 1, 2, 13, 14, and
15 elements participate in π interactions with cp' ligands. A large structural diversity
due to the relatively weak cp'-e bond is observed in metallocenes. Flexible cp'-e
interactions and the ability of cp' to act as a leaving group allow for a wide range of
substitution chemistries. Further diversification with isolobal analogues of cp' has
already begun and represents the future of main-group metallocene chemistry.

KEY WORDS

Metallocene, Sandwich complexes, Ferrocenes, Cyclopentadienyl


REFERENCES

MOOCS

YOUTUBE VIDEOS
1) https://www.youtube.com/watch?v=ER2jiuYTokk

CHE509: Inorganic Chemistry – II Page 203


2) https://www.youtube.com/watch?v=gJUIICGWANY
3) https://www.youtube.com/watch?v=N8IXAcewALc
4) https://www.youtube.com/watch?v=hAfgYrFan-w

WIKIPEDIA
1) https://en.wikipedia.org/wiki/Ferrocene
2) https://en.wikipedia.org/wiki/Cyclopentadienyl_anion
3) https://en.wikipedia.org/wiki/Acetylferrocene

OER

1) file:///C:/Users/PC/OneDrive/Desktop/v63-182.pdf

2) https://www.britannica.com/science/ferrocene

REFERENCE BOOKS
1) Inorganic and organometallic reaction mechanisms: derivation of simple and
complex rate expressions kindle edition by esther iornumbe, kayode Ebenezer
2) Inorganic and Organometallic Transition Metal Complexes with Biological
Molecules and Living Cells , 2017 by Kenneth Kam -Wing Lo

CHE509: Inorganic Chemistry – II Page 204


C REDIT 03 -U NIT 02: THE S TRUCTURE AND R EACTIVITY OF
MOLECULES
LEARNING OBJECTIVES

After successful completion of this unit, you will be able to


 Understand the VSEPR theory
 Apply the VSEPR theory to for shape determination
 Understand the shape of SF 4 molecule using VSEPR
 Understand the effect of lone pairs on shape of molecule
INTRODUCTION

Reactivity is the term used to describe the ability of a structure to undergo


chemical change. Some structures are more fragile than others. Structures that are
likely to change chemically are called reactive. A structure that are resistant to
chemical changes are said to not react.This is the golden age of musical instruments.
Many of the techniques you used in the organic lab were the only methods of
structure determination 100 years ago. Recall that Kernel has been known for over
100 years. From the 1920s and his 1930s to the present da y, we have witnessed the
invention and development of many analytical techniques for st ructure determination
by people infrared, NMR, UV-visible (electron), and ESR spectroscopy,
electrochemistry, and X-ray crystallography are the most important. Sixty yea rs ago,
X-ray crystallography could have provided his doctorate paper. Today students can
often solve the same structure in just afternoons. Some of these methods are
described later in this chapter.

02-01: VSEPR THEORY

The valence shell electron-pair repulsion theory, abbreviated as VSEPR theory,


is based on the premise that there is repulsion between the valence pairs of all
atoms, and the atom always minimizes this electron-pair repulsion. This arrangement
of atoms determines the shape of the resulting molecule.

Postulates of VSEPR Theory:

 In polyatomic molecules (that is, molecules composed of three or more


atoms), one of the constituent atoms is identified as the central atom to
which all other atoms of the molecule are attached.

CHE509: Inorganic Chemistry – II Page 205


 The total number of electron pairs in the valence shell determines the shape of
the molecule.

 Electron pairs tend to align in a way that minimizes electron -to-electron


repulsion between them and maximizes the distance between them.

 A valence shell can be thought of as a sphere in which electron pairs are


localized on the surface such that the distance between them is maximized.

 Asymmetrically shaped molecules are expected when the central atom of the
molecule is surrounded by bonding pairs of electrons.

 When the central atom is surrounded by both lone and bonding electron pairs,
the molecule tends to have a distorted shape.

 VSEPR theory can be applied to any resonance structure of a molecule.

 The strength of repulsion is strongest for two isolated pairs and weakest for
two bonded pairs.

 If pairs of electrons are close to each other around the central atom, they repel
each other. This increases the energy of the molecule.

 The farther apart the electron pairs are, the less repulsion there is between them
and ultimately the lower the energy of the molecule.

Limitations of VSEPR Theory

 This theory does not account for isoelectronic species (that is, elements with
the same number of electrons). Species can have the same number of electrons
but different shapes.

 VSEPR theory does not shed light on compounds of transition metals. The
structures of some such compounds cannot be adequately described by this
theory. This is because the VSEPR theory does not take into account the
relative sizes of substituents and inactive lone pairs.

 Another limitation of the VSEPR theory is that it predicts that group 2


halides have a linear conformation while the real conformation has a
bent conformation.

CHE509: Inorganic Chemistry – II Page 206


SOLVED PROBLEMS 01

1] Describe the postulates of VSEPR theory and give Limitations of it.

 In polyatomic molecules (that is, molecules composed of three or more atoms),


one of the constituent atoms is identified as the central atom to which all other
atoms of the molecule are attached.

 The total number of electron pairs in the valence shell determines the shape of
the molecule.

 Electron pairs tend to align in a way that minimizes electron -to-electron


repulsion between them and maximizes the distance between them.

 A valence shell can be thought of as a sphere in which electron pairs are


localized on the surface such that the distance between them is maximized.

 Asymmetrically shaped molecules are expected when the central atom of the
molecule is surrounded by bonding pairs of electrons.

 When the central atom is surrounded by both lone and bondin g electron pairs,
the molecule tends to have a distorted shape.

 VSEPR theory can be applied to any resonance structure of a molecule.

 The strength of repulsion is strongest for two isolated pairs and weakest for
two bonded pairs.

 If pairs of electrons are close to each other around the central atom, they repel
each other. This increases the energy of the molecule.

 The farther apart the electron pairs are, the less repulsion there is between them
and ultimately the lower the energy of the molecule.

Limitations of VSEPR Theory

 This theory does not account for isoelectronic species (that is, elements with
the same number of electrons). Species can have the same number of electrons
but different shapes.

 VSEPR theory does not shed light on compounds of transition metals. The
structures of some such compounds cannot be adequately described by this theory.

CHE509: Inorganic Chemistry – II Page 207


This is because the VSEPR theory does not take into account the relative sizes of
substituents and inactive lone pairs.

 Another limitation of the VSEPR theory is that it predicts that group 2 halides
have a linear conformation while the real conformation has a bent conformation.
SHORT ANSWER QUESTIONS WITH MODEL ANSWER 01

1] Explain the important limitations of VSEPR theory.

Solution: Limitations of VSEPR Theory

 This theory does not account for isoelectronic species (that is, elements with
the same number of electrons). Species can have the same number of electrons but
different shapes.

 VSEPR theory does not shed light on compounds of transition metals. The
structures of some such compounds cannot be adequately described by this theory.
This is because the VSEPR theory does not take into account the relative sizes of
substituents and inactive lone pairs.

 Another limitation of the VSEPR theory is that it predi cts that group 2 halides
have a linear conformation while the real conformation has a bent conformation.
02-02: PREDICT THE SHAPES OF MOLECULES

The strength of the repulsive force between a lone pair and a bond pair is the
repulsive force between two lone pairs and two bond pairs. The order of repulsion
between electron pairs is:

Lone Pair- lone pair > Lone Pair- bond- pair >Bond Pair- bond pair.

Linear Shape of Molecule:

In this type of molecule there are two sites in the valence shell of the central
atom.They should be placed in such a way that repulsion can be minimized (pointing
in opposite directions).

For e.g. BeF 2

Trigonal Planar Shape of Molecule:

In this type of molecule you will find 3 molecules attached to the central atom. They
are arranged in such a way that repulsion between electrons can be minimized
(towards the corners of the equilateral triangle).

CHE509: Inorganic Chemistry – II Page 208


For e.g. BF 3

Tetrahedral Shape of Molecule:

In a two-dimensional molecule, the atoms lie in the same plane, and applying these
conditions to methane gives a square planar geometry with a bond angle of 90 0
between H-C-H. Applying all these conditions to a 3D molecule, we get a tetrahedral
molecule in which the bond angle between H-C-H is 109 0 28' (towards the corners of
the equilateral triangle) CH 4 .

Trigonal Bipyramid Shape of Molecule:

Let's take an example from PF5. Here, repulsion can be minimized by distributing
the electrons evenly towards the corners of the triangular pyramid. In a triangular
bipyramid, he has three positions along the equat or of the molecule. The two
locations are along the axis perpendicular to the equatorial plane.
SOLVED PROBLEMS 02

1] How to predict the shape of molecule based on VSEPR theory?

Solution: The strength of the repulsive force between a lone pair and a bond pair is
the repulsive force between two lone pairs and two bond pairs. The order of
repulsion between electron pairs is:

Lone Pair- lone pair > Lone Pair- bond- pair >Bond Pair- bond pair.

Linear Shape of Molecule:

In this type of molecule there are two sites in the valence shell of the central
atom.They should be placed in such a way that repulsion can be minimized (pointing
in opposite directions).

For e.g. BeF 2

Trigonal Planar Shape of Molecule:


In this type of molecule you will find 3 molecules attach ed to the central atom.
They are arranged in such a way that repulsion between electrons can be minimized
(towards the corners of the equilateral triangle).
For e.g. BF 3

Tetrahedral Shape of Molecule:

CHE509: Inorganic Chemistry – II Page 209


In a two-dimensional molecule, the atoms lie in the same plane, and applying these
conditions to methane gives a square planar geometry with a bond angle of 90 0
between H-C-H. Applying all these conditions to a 3D molecule, we get a tetrahedral
molecule in which the bond angle between H-C-H is 109 0 28' (towards the corners of
the equilateral triangle) CH 4 .

Trigonal Bipyramid Shape of Molecule:

Let's take an example from PF 5 . Here, repulsion can be minimized by


distributing the electrons evenly towards the corners of the triangular pyramid. In a
triangular bipyramid, he has three positions along the equator of the molecule. The
two locations are along the axis perpendicular to the equatorial plane.

SHORT ANSWER QUESTIONS WITH MODEL ANSWER 02

1] How to attain the shape of liner molecules by VSEPR theory?

Solution: In this type of molecule there are two sites in the valence shell of the
central atom.They should be placed in such a way that repulsion can be minimized
(pointing in opposite directions).

For e.g. BeF 2

02-03: THE EFFECT OF LONE PAIRS


The Lewis electron pair approach can be used to predict the number and type of
bonds between atoms in materials, showing atoms with lone pairs. However, this
approach gives no information about the actual arrangement of atoms in space. We
continue our discussion of structure and bonding by introducing the valence shell
electron pair repulsion (VSEPR) model, which can be used to predict the shape of
many molecules and polyatomic ions. However, remember that the VSEPR model,
like any model, is a limited representation of reality. The model does not provide
information about bond lengths or the existence of multiple bonds.

VSEPR models can predict the structure of almost any molecule or polyatomic
ion with a non-metallic central atom, and many molecules and polyatomic ion s
containing a central metal atom. The premise of VSEPR theory is to adopt a
geometry that causes electron pairs in bonds and lone pairs to repel each other, thus
placing electron pairs as far apart as possible. This theory is very simplistic and does
not take into account the complex orbital interactions that affect the shape of

CHE509: Inorganic Chemistry – II Page 210


molecules. However, the simple VSEPR counting method accurately predicts the
conformation of many compounds that cannot be predicted by the Lewis electron
pair approach.

VSEPR models can be used to predict the shape of most polyatomic molecules
and ions simply by focusing on the number of electron pairs around the central atom
and ignoring other valence electrons present. According to this model, the valence
electrons in the Lewis structure form groups composed of single bonds, double
bonds, triple bonds, lone electron pairs, or even single unpaired electrons counted as
unpaired electron pairs . VSEPR model. Electrons electrostatically repel each other,
so the most stable (i.e., lowest energy) configuration of electron groups is the one
that minimizes repulsion. The groups are arranged around the central atom to form
the lowest energy molecular structure.

In the VSEPR model, molecules or polyatomic ions are given the designation
AXmEn. where A is the central atom, X is the bonding atom, E is the non -bonding
valence group (usually a lone pair), and m and n are integers. Each group around the
central atom is called a bonded pair (BP) or lone (non-bonded) pair (LP). Both the
relative positions of the atoms and the angles between the bonds, the so -called bond
angles, can be predicted from the interaction of BP and LP. This information can be
used to describe molecular structure, the arrangement of bonded atoms within a
molecule, or polyatomic ions.

SOLVED PROBLEMS 03

1] What is effect of lone pair on shape of molecules

Solution: The Lewis electron pair approach can be used to predict the number
and type of bonds between atoms in materials, showing atoms with lone pairs.
However, this approach gives no information about the actual arrangement of atoms
in space. We continue our discussion of structure and bonding by introducing the
valence shell electron pair repulsion (VSEPR) model, which can be used to predict

CHE509: Inorganic Chemistry – II Page 211


the shape of many molecules and polyatomic ions. However, remember that the
VSEPR model, like any model, is a limited representation of reality. The model does
not provide information about bond lengths or the existence of multiple bonds.

VSEPR models can predict the structure of almost any molecule or polyatomic
ion with a non-metallic central atom, and many molecules and polyatomic ions
containing a central metal atom. The premise of VSEPR theory is to adopt a
geometry that causes electron pairs in bonds and lone pairs to repel eac h other, thus
placing electron pairs as far apart as possible. This theory is very simplistic and does
not take into account the complex orbital interactions that affect the shape of
molecules. However, the simple VSEPR counting method accurately predicts the
conformation of many compounds that cannot be predicted by the Lewis electron
pair approach.

VSEPR models can be used to predict the shape of most polyatomic molecules
and ions simply by focusing on the number of electron pairs around the central ato m
and ignoring other valence electrons present. According to this model, the valence
electrons in the Lewis structure form groups composed of single bonds, double
bonds, triple bonds, lone electron pairs, or even single unpaired electrons counted as
unpaired electron pairs . VSEPR model. Electrons electrostatically repel each other,
so the most stable (i.e., lowest energy) configuration of electron groups is the one
that minimizes repulsion. The groups are arranged around the central atom to form
the lowest energy molecular structure.

In the VSEPR model, molecules or polyatomic ions are given the designation
AXmEn. Where A is the central atom, X is the bonding atom, E is the non -bonding
valence group (usually a lone pair), and m and n are integers. Each gro up around the
central atom is called a bonded pair (BP) or lone (non-bonded) pair (LP). Both the
relative positions of the atoms and the angles between the bonds, the so -called bond
angles, can be predicted from the interaction of BP and LP. This informati on can be

CHE509: Inorganic Chemistry – II Page 212


used to describe molecular structure, the arrangement of bonded atoms within a
molecule, or polyatomic ions.

02-04: STRUCTURE OF SULPHUR TETRAFLUORIDE

Sulfur in SF4 has a formal oxidation state of +4. Of the six valence electrons of
sulfur, two form lone pairs. Therefore, the structure of SF 4 can be predicted using
the principles of VSEPR theory. This is a seesaw shape with an S in the middle. One
of the three equatorial positions is occupied by an unbonded lone pair. As a result,
there are two different types of F ligands in the molecule: two axial and two
equatorial. The relevant bond distances are S–Fax = 164.3 pm and S–Feq = 154.2
pm. The axial ligands of hypervalent molecules are usually not very tightly bound. A
related molecule, SF6, differs from SF4 in that the sulfur is in the 6+ state, leaving
no valence electrons unbonded to the sulfur, giving the molecule a highly
symmetrical octahedral geometry. In contrast to SF 4 , SF 6 is very chemically inert.
The 19F NMR spectrum of SF4 shows only one signal, indicating that the axial and
equatorial F atom positions are rapidly interconverted by pseudorotation.

To determine the hybridization of sulfur tetrafluoride, we first need to


understand its Lewis structure and the number of valence electrons p resent. The SF4
molecule consists of a total of 34 valence electrons. Here 6 comes from sulfur and
the 4 fluorine atoms each have 7 electrons.

During the formation of SF 4 , the sulfur atom uses eight valence electrons to
form bonds with each fluorine atom. On the other hand, the four fluorine atoms have
three lone pairs in their octets and still use 24 valence electrons. Also, the sulfur
atom holds two electrons as a lone pair. Sulfur hybridization can now be determined
by counting the number of regions of electron density. When bonding occurs, four

CHE509: Inorganic Chemistry – II Page 213


single bonds are formed in sulfur and it has one lone pair. Looking at this, we can
say that there are five regions of high electron density.

The central S atom, containing five valence orbitals, essentially hybri dizes to
form five sp3d hybrid orbitals. In the 2P orbital, four hybrid orbitals overlap and the
fifth orbital contains a lone pair. Knowing the stereo number also helps determine
how many hybrid orbitals an atom uses. Sulfur uses five orbitals, including a 3s
orbital, three his 3p orbitals, and a 3d orbital.

SOLVED PROBLEMS 04

1] Describe the structure of SF 4 with respect toVSEPR

Solution: Sulfur in SF4 has a formal oxidation state of +4. Of the six valence
electrons of sulfur, two form lone pairs. Therefore, the structure of SF 4 can be
predicted using the principles of VSEPR theory. This is a seesaw shape with an S in
the middle. One of the three equatorial positions is occupied by an unbonded lone
pair. As a result, there are two different types of F ligan ds in the molecule: two axial
and two equatorial. The relevant bond distances are S–Fax = 164.3 pm and S–Feq =
154.2 pm. The axial ligands of hypervalent molecules are usually not very tightly
bound. A related molecule, SF6, differs from SF4 in that the su lfur is in the 6+ state,
leaving no valence electrons unbonded to the sulfur, giving the molecule a highly
symmetrical octahedral geometry. In contrast to SF 4 , SF 6 is very chemically inert.
The 19F NMR spectrum of SF4 shows only one signal, indicating that the axial and
equatorial F atom positions are rapidly interconverted by pseudorotation.

To determine the hybridization of sulfur tetrafluoride, we first need to


understand its Lewis structure and the number of valence electrons present. The SF4

CHE509: Inorganic Chemistry – II Page 214


molecule consists of a total of 34 valence electrons. Here 6 comes from sulfur and
the 4 fluorine atoms each have 7 electrons.

During the formation of SF 4 , the sulfur atom uses eight valence electrons to
form bonds with each fluorine atom. On the other hand, the four fluorine atoms have
three lone pairs in their octets and still use 24 valence electrons. Also, the sulfur
atom holds two electrons as a lone pair. Sulfur hybridization can now be determined
by counting the number of regions of electron density. When b onding occurs, four
single bonds are formed in sulfur and it has one lone pair. Looking at this, we can
say that there are five regions of high electron density.

The central S atom, containing five valence orbitals, essentially hybridizes to


form five sp3d hybrid orbitals. In the 2P orbital, four hybrid orbitals overlap and the
fifth orbital contains a lone pair. Knowing the stereo number also helps determine
how many hybrid orbitals an atom uses. Sulfur uses five orbitals, including a 3s
orbital, three his 3p orbitals, and a 3d orbital.

SF4 molecular geometry is see-saw with one pair of valence electrons. The
nature of the molecule is polar. These atoms form a trigonal bipyramidal shape.

SHORT ANSWER QUESTIONS WITH MODEL ANSWER 04

1] What is geometry of SF 4 molecule?

Solution: SF4 molecular geometry is see-saw with one pair of valence electrons. The
nature of the molecule is polar. These atoms form a trigonal bipyramidal shape.

CHECK POINT 01-04

1] Which of the following species has 120° bond angles?

a) NCl3
b) PH3
c) ClF3
d) BCl3
Solution: d (Boron atom is having no lone pair of electrons)

2] Number of bond pair and lone pair electrons present on N atom in NO 3– are

a) 3, 1

b) 4, 0

CHE509: Inorganic Chemistry – II Page 215


c) 2, 2
d) 1, 3
Solution: b
3] XeF6 is
a) Distorted octahedral
b) Tetrahedral
c) Octahedral
d) Planar
Solution: a
4] Find the molecule with the least bond angle
a) BeF2
b) CH4
c) NH3
d) H2O
Solution: d
SUMMARY

To determine the shape of the molecule, we need to become familiar with the dot
structure of Lewis electrons. Lewis theory does not determine the shape of molecules, but it
is the first step towards predicting the shape of molecules. Lewis structures help identify
bond pairs and lone pairs. The valence shell electron pair repulsion theory (VSPER) is then
applied to the Lewis structure to determine the shape of the molecule and the shape of the
electron group. To specify the three-dimensional shape of the molecule and get a complete
description, we also need to know the bond angles. The Lewis electron point structure plays
an important role in determining the shape of the molecule as it helps identify the valence
electrons. Click the link above for instructions on how to draw the Lewis electron dot
structure.
KEY WORDS

VSEPR theory, Shapes of molecules, Electrons. Click the link above for instructions
on how to draw the Lewis electron dot structure.

CHE509: Inorganic Chemistry – II Page 216


REFERENCES

MOOCS

______
YOUTUBE VIDEOS
1) https://www.youtube.com/watch?v=x2-nP7i6T34
2) https://www.youtube.com/watch?v=DWS5HlyndAI
3) https://www.youtube.com/watch?v=wj4u5hM6QhI
4) https://www.youtube.com/watch?v=DBrq31w8vC4

WIKIPEDIA
5) https://en.wikipedia.org/wiki/VSEPR_theory
6) https://en.wikipedia.org/wiki/VSEPR_theory#AXE_method
7) https://en.wikipedia.org/wiki/Sulfur_tetrafluorid E

OER

8) https://chem.libretexts.org/Courses/Purdue/Purdue_Chem_26100%3A_Organi
c_Chemistry_I_(Wenthold)/Chapter_02._Structures_and_Properties_of_Organic_
Molecules/2.2_Molecular_Shapes_and_Hybridization/2.2.1._VSEPR#:~:text=The
%20valence%2Dshell%20electron%2Dpair%20repulsion%20(VSEPR)%20theory,
as%20possible%20to%20minimize%20repulsion.
9) https://unacademy.com/content/jee/study-material/chemistry/vsepr-theory/

REFERENCE BOOKS
1) A Textbook of Inorganic Chemistry by B.R. Puri , L.R. Sharma, K.C. Kalia

2) The VSEPR Model of Molecular Geometry (Dover Books on Chemistry) by Ronald


J Gillespie

CHE509: Inorganic Chemistry – II Page 217


CREDIT 03 -UNIT 03: APPLICATIONS OF VSEPR THEORY

LEARNING OBJECTIVES

After successful completion of this unit, you will be able to


 Understand the concept of shape determination by VSEPR theory
 Apply VSEPR theory to understand shape of Phosphorus pentoxide
 Apply VSEPR theory to understand shape of Nitrogen dioxide
 Apply VSEPR theory to understand shape of Xenon hexa -fluoride
 Apply VSEPR theory to understand shape of Silicon tetrachloride

INTRODUCTION

The valence shell electron-pair repulsion theory, abbreviated as VSEPR theory,


is based on the premise that there is repulsion between the valence pairs of all
atoms, and the atom always minimizes this electron-pair repulsion. This arrangement
of atoms determines the shape of the resulting molecule. The different shap es that
molecules can adopt according to VSEPR theory are shown in the figure below. The
overall geometry is further refined by distinguishing between bonding and non -
bonding electron pairs. Bonded pairs of electrons that are shared in sigma bonds
with neighboring atoms are farther from the central atom than unbonded (lone) pairs
of that atom that are held close to its positively charged nucleus. Therefore, VSEPR
theory assumes that the repulsion due to lone pairs is greater than the repulsion due
to bonding pairs. Therefore, if a molecule has two interactions with different degrees
of repulsion, VSEPR theory predicts a structure in which the lone pair occupies a
less repulsive position. Lone pair-lone pair repulsion (lp-lp) is considered stronger
than lone pair bonded pair repulsion (lp-bp), which is considered stronger than
bonded pair bonded pair repulsion (bp-bp). A decision about the total geometry when
two or more unequal positions are possible. For example, if five pairs of valence
electrons surround a central atom, it adopts a trigonal bipyramidal molecular
geometry with two collinear axis positions and three equatorial positions. Axial pairs
of electrons have three neighboring pairs at 90° and a fourth pair at 180° on the
equator, while pairs of electrons at the equator have two adjacent pairs at 90° and a
fourth pair at 180°. There are only two neighboring pairs of electrons at 120°.
Repulsion from neighbors at 90° is more significant, so axial locations receive more
repulsion than equatorial locations. Therefore, if there are lone pairs, they tend to

CHE509: Inorganic Chemistry – II Page 218


occupy positions on the equator, as shown in the stereo number 5 diagram in the next
section.

03-01: BROMINE TRIFLUORIDE

BrF 3 is a diagram of the AX 5 molecule because it has two lone pairs and three
bond pairs. Each fluorine atom contains nine electrons, but the outer shell of the
bromine molecule has seven valence electrons, three of which are bonded to three
fluorine atoms. As a result, three pairs of electrons are bound and two are lonely. Br
is a primary atom. There are three bonding pairs because three fluorine atoms
surround the central Br atom. The valence electrons of fluorine and bromine atoms
are both 7 according to the Lewis structure or the concept of valence electrons.

Now that we have seen the molecular structure of BrF 3 , let's look at the
electronic structure of BrF 3 . BrF 3 is an excellent example of an AX 5 molecule. Each
fluorine atom has nine electrons, but the outer shell of the bromine molecule has
seven valence electrons, three of which are bonded to three fluorine atoms.
According to the VSEPR theory (Valence Shell Electron Pair Repulsion Theory), the
shape of the BrF 3 molecule is triangular pyramidal. To reduce the repulsion between
lone pairs, the structure of the molecule bends into a T shape.

BrF 3 is classified as a polar molecule because the electronegativity values of


the fluorine and bromine particles are significantly different. These particles have
lone electron pairs formed in the triangular planes, resulting in a random distribution
of negative charges across the central bromine atom and the formation of highly
polar bonds. Bromine trifluoride is therefore a polar molecule.

BrF 3 has a bond angle of 86.2° and a T-shaped or triangular pyramid shape.
This angle occurs because the electron pair repulsion is tighter than in the Br -F
bond. MO theory deals with the energy and spatial properties of e lectron pairs. It
also addresses linear combinations of atomic orbitals to create molecular orbitals.
Shape distortion is caused by electrical repulsion between lone electron pairs and
bond pairs. As a result of the lone pair being more spatially spread, t he curved
geometry has narrower bond angles compared to the perfect trigonal bipyramid.

Take a bromine atom and study its electron configuration and D orbitals to
determine the hybridization of bromine trifluoride. For hybridization purposes, BrF 3
has seven electrons in its outermost shell. Br and F form a bond, forming two lone

CHE509: Inorganic Chemistry – II Page 219


pairs of electrons and three covalent bonds. Br-F binding and hybridization score
answer is 5. Fluorine has a higher oxidizing capacity than bromine, allowing
bromine to promote electrons. Bromine can then use the D orbital for hybridization.
This is important for evaluating the geometry of bond pairs.

DICHLOROIODATE (I) ANION

Using this formula for ICl 2 - , we can find the number of lone pairs and the
number of bond pairs and predict its shape. Alternatively, this formula allows us to
find the number of lone pairs and the number of bond pairs to predict their
shape.where in ICl2-, iodine has seven valence electrons, so V = 7; two monovalent
chlorine atoms are bound to iodine, so M = 2; and the charge of the compound is -1,
so C = 0 and A = -1.From here we can see that there are 2 bonded pairs (I-Cl) and 3
lone pairs, and the compound's hybridization is sp 3 d.Therefore, the shape is a
triangular bipyramid.However, Cl and I leave a linear molecular geometry due to the
presence of three lone iodine atom pairs present at the equatorial
positions.Therefore, the shape of the compound is a distorted triangular pyramid.

SOLVED PROBLEMS 01

1] Explain the molecular shape of BrF 3 molecule.

CHE509: Inorganic Chemistry – II Page 220


Solution: BrF 3 is a diagram of the AX 5 molecule because it has two lone pairs
and three bond pairs. Each fluorine atom contains nine electrons, but the outer shell
of the bromine molecule has seven valence electrons, three of which are bonded to
three fluorine atoms. As a result, three pairs of electrons are bound and two are
lonely. Br is a primary atom. There are three bonding pairs because three fluorine
atoms surround the central Br atom. The valence electrons of fluorine and bromine
atoms are both 7 according to the Lewis structure or the concept of valence
electrons.

Now that we have seen the molecular structure of BrF 3 , let's look at the
electronic structure of BrF 3 . BrF3 is an excellent example of an AX 5 molecule. Each
fluorine atom has nine electrons, but the outer shell of the bromine molecule has
seven valence electrons, three of which are bonded to three fluorine atoms.
According to the VSEPR theory (Valence Shell Electron Pair Repulsion Theory), the
shape of the BrF3 molecule is triangular pyramidal. To reduce the repulsion between
lone pairs, the structure of the molecule bends into a T shape.

BrF 3 is classified as a polar molecule because the electronegativity values of


the fluorine and bromine particles are significantly different. These particles have
lone electron pairs formed in the triangular planes, resulting in a random distribution
of negative charges across the central bromine atom and the formation of polar
bonds. Bromine trifluoride is therefore a polar molecule.

BrF 3 has a bond angle of 86.2° and a T-shaped or triangular pyramid shape.
This angle occurs because the electron pair repulsion is tighter than in the Br -F
bond. MO theory deals with the energy and spatial properties of electron pairs. It
also addresses linear combinations of atomic orbitals to create molecular orbitals.
Shape distortion is caused by electrical repulsion between lone electron pairs and
bond pairs. As a result of the lone pair being more spatially spread, the curved
geometry has narrower bond angles compared to the perfect trigonal bipyramid.

Take a bromine atom and study its electron configuration and D orbitals to
determine the hybridization of bromine trifluoride. For hybridization purposes, BrF 3
has seven electrons in its outermost shell. Br and F form a bond, forming two lone
pairs of electrons and three covalent bonds. Br-F binding and hybridization score
answer is 5. Fluorine has a higher oxidizing capacity than bromine, allowing

CHE509: Inorganic Chemistry – II Page 221


bromine to promote electrons. Bromine can then use the D orbital for hybridization.
This is important for evaluating the geometry of bond pairs.

2] Describe the shape of dichloroiodate (I) anion

Solution:

Using this formula for ICl 2 - , we can find the number of lone pairs and the
number of bond pairs and predict its shape. Alternatively, this formula allows us to
find the number of lone pairs and the number of bond pairs to predict their
shape.where in ICl2-, iodine has seven valence electrons, so V = 7; two monovalent
chlorine atoms are bound to iodine, so M = 2; and the charge of the compound is -1,
so C = 0 and A = -1.From here we can see that there are 2 bonded pairs (I-Cl) and 3
lone pairs, and the compound's hybridization is sp 3 d.Therefore, the shape is a
triangular bipyramid.However, Cl and I leave a linear molecular geometry due to the
presence of three lone iodine atom pairs present at the equatorial
positions.Therefore, the shape of the compound is a distorted triangular pyramid.

SHORT ANSWER QUESTIONS WITH MODEL ANSWER 01

1] What is shape of BrF 3 Molecule?

Solution: BrF3 molecular geometry is said to be T-shaped or Trigonal


Bipyramidal with a bond angle of 86.2 o , which is slightly smaller than the usual 90°.

CHE509: Inorganic Chemistry – II Page 222


2] What is shape of ICl 2 - Molecule?

Solution: ICl 2 - Moleculeis having distorted triangular pyramid.


03-02: PHOSPHOROUS PENTACHLORIDE

Elements present in the third period include d orbitals along with s and p
orbitals. The energies of the 3d orbitals are close to the energies of the 3s and 3p
orbitals. The energy of the 3d orbital corresponds to the 4s orbital as well as the 4p
orbital. As a result, hybridization can be performed in either 3s, 3p and 3d or 3d, 4s
and 4p. Hybridization with the 3p, 3d, and 4s orbitals is not possible due to the
energy difference between the 3p and 4s orbitals.Note that not all bond angles are
identical in trigonal bipyramidal geometry. In PCl5, his 5sp3d orbital of phosphorus
overlaps the p-orbital of the chlorine atom. The p-orbitals are singly occupied.
Together, they form five P-Cl sigma bonds.

Equatorial Bonds: Three P-Cl bonds are in the plane and form an angle with
each other. The angle formed between them is 120°.

Axial Bonds: Two P-Cl bonds. One above the equatorial plane and one below
the plane, forming an angle with the plane. The angle with the plane is 90°.

Axial bonds tend to be slightly longer because axial bond pairs receive more
repulsive interactions from equatorial bond pairs. Thus, it is slightly weaker than the
equatorial bond, resulting in a more reactive PCl 5 molecule.

According to the VESPR theory, the shape of PCl5 is triangular bipyramidal.


where P is the central atom with five Cl atoms around it. The compound PCl5 has
five pairs of electrons around the central atom. Also, there are 5 bonded pairs and no
lone pairs.

NITROGEN DIOXIDE
Another problem arises when unpaired electrons are present. This is well
described for nitrogen dioxide. So for NO 2 , the number of electrons is an integer, but

CHE509: Inorganic Chemistry – II Page 223


the number of electron pairs is not. Each orbital can hold 0, 1, or 2 electrons, so 2.5
pairs of electrons he has to place in 3 orbitals. The geometry is therefore based on
the trigonal planar arrangement of electron pairs. The lone-pair orbitals are only
half-filled, so they take up less space and the E-N-E angle opens slightly (to 134.1°)
from the ideal triangular angle of 120°.NO 2 has an extra electron in the orbital of the
nitrogen atom, which makes it more repulsive. However, using one lone electron or
one-electron range results in less repulsion at the two bonding oxyg en atoms.
Therefore, the repulsions are not identical. As a result, the oxygen atoms are widely
scattered.

Nitrogen, on the other hand, requires three hybrid orbitals that are used to host
two sigma bonds and one electron. This leads to sp 2 hybridization. The three sp 2 -
hybridized orbitals of nitrogen contain one electron, and the p -orbital contains one
electron. However, only one sp 2 hybrid orbital and one p orbital contain one electron
each when forming two sigma bonds. The p-orbital forms a pi bond with the oxygen
atom.

In the structure of nitrate, there are three bonding pairs and lone or odd
electrons around the nitrogen atom. By calculating the number of lone electron pairs
and bond pairs around the central atom, the geometry or shape of the molecule can
be determined. Around nitrogen, there are two sigma bonds and one electron
according to the Lewis structure above. According to the VSEPR assumption, the
sum of lone pairs and binding pairs is the hybridization of the molecule. In nitra te
ion, the lone pair of electrons on nitrogen is zero. The number is 2 (bond pairs) + 1
(single electron) = 3. A hybridizing number of 3 means that the central atom is sp2
hybridized. Therefore, for sp2 hybridization, the expected geometry is a trigonal
planar structure with bond angles of 120 degrees between atoms. However, the
nitrite ion is unexpected in both shape and bond angles. The actual shape observed
for the nitrite ion is a bent shape or V-shape, with a bond angle of 134 degrees
instead of 120 degrees. Nitrogen has an extra or odd number of electrons in its
molecule. In molecules containing lone electron pairs and bond pairs, repulsive

CHE509: Inorganic Chemistry – II Page 224


forces occur between the lone electron pairs and bond pairs, resulting in large
structural differences.

For nitrate ions, the molecule experiences a greater repulsive force due to the
presence of odd electrons. If there is a lone pair of electrons, the repulsive forces can
be equal on both sides, but due to the extra or odd electrons, the repulsive forces
between the two oxygen atoms are unequal and they move further. This bond -pair
atom spread increases the bond angles in the molecule. Additionally, the shape or
structure changes into a curved shape or V-shape similar to water molecules.

SOLVED PROBLEMS 02

1] Explain the shape of PCl 5 molecule according to VSEPR theory

Solution: Elements present in the third period include d orbitals along with s
and p orbitals. The energies of the 3d orbitals are close to the energies of the 3s and
3p orbitals. The energy of the 3d orbital corresponds to the 4s orbital as well as the
4p orbital. As a result, hybridization can be performed in either 3s, 3p and 3d or 3d,
4s and 4p. Hybridization with the 3p, 3d, and 4s orbitals is not possible due to the
energy difference between the 3p and 4s orbitals.Note that not all bond angles are
identical in trigonal bipyramidal geometry. In PCl5, his 5sp3d orbital of phosphorus
overlaps the p-orbital of the chlorine atom. The p-orbitals are singly occupied.
Together, they form five P-Cl sigma bonds.

Equatorial Bonds: Three P-Cl bonds are in the plane and form an angle with each
other. The angle formed between them is 120°.
Axial Bonds: Two P-Cl bonds. One above the equatorial plane and one below the
plane, forming an angle with the plane. The angle with the plane is 90°.
Axial bonds tend to be slightly longer because axial bond pairs receive more
repulsive interactions from equatorial bond pairs. Thus, it is slightly weaker than the
equatorial bond, resulting in a more reactive PCl 5 molecule.

CHE509: Inorganic Chemistry – II Page 225


According to the VESPR theory, the shape of PCl 5 is triangular bipyramidal. where P
is the central atom with five Cl atoms around it. The compound PCl 5 has five pairs
of electrons around the central atom so there are 5 bonded pairs and no lone pairs.
2] Explain the shape of nitrogen dioxide molecule in detain by VSEPR
Solution: Another problem arises when unpaired electrons are present. This is
well described for nitrogen dioxide. So for NO 2 , the number of electrons is an
integer, but the number of electron pairs is not. Each orbital can hold 0, 1, or 2
electrons, so 2.5 pairs of electrons he has to place in 3 orbitals. The geometry is
therefore based on the trigonal planar arrangement of electron pairs. The lone -pair
orbitals are only half-filled, so they take up less space and the E-N-E angle opens
slightly (to 134.1°) from the ideal triangular angle of 120°. NO 2 has an extra electron
in the orbital of the nitrogen atom, which makes it more repulsive. However, using
one lone electron or one-electron range results in less repulsion at the two bonding
oxygen atoms. Therefore, the repulsions are not identical. As a result, the oxygen
atoms are widely scattered.

Nitrogen, on the other hand, requires three hybrid orbitals that are used to host
two sigma bonds and one electron. This leads to sp 2 hybridization. The three sp 2 -
hybridized orbitals of nitrogen contain one electron, and the p -orbital contains one
electron. However, only one sp 2 hybrid orbital and one p orbital contain one electron
each when forming two sigma bonds. The p-orbital forms a pi bond with the oxygen
atom.

In the structure of nitrate, there are three bonding pairs and lone or odd
electrons around the nitrogen atom. By calculating the number of lone electron pairs
and bond pairs around the central atom, the geometry or shape of the molecule can
be determined. Around nitrogen, there are two sigma bonds and one electron
according to the Lewis structure above. According to the VSEPR assumption, the
sum of lone pairs and binding pairs is the hybridization of the molecule. In nitrate
ion, the lone pair of electrons on nitrogen is zero. The number is 2 (bond pairs) + 1
(single electron) = 3. A hybridizing number of 3 means that the central atom is sp2
hybridized. Therefore, for sp2 hybridization, the expected geometry is a trigonal

CHE509: Inorganic Chemistry – II Page 226


planar structure with bond angles of 120 degrees between atoms. However, the
nitrite ion is unexpected in both shape and bond angles. The actual shape observed
for the nitrite ion is a bent shape or V-shape, with a bond angle of 134 degrees
instead of 120 degrees. Nitrogen has an extra or odd number of electrons in its
molecule. In molecules containing lone electron pairs and bond pairs, repulsive
forces occur between the lone electron pairs and bond pairs, resulting in large
structural differences.
For nitrate ions, the molecule experiences a greater repulsive force due to the
presence of odd electrons. If there is a lone pair of electrons, the repulsive forces can
be equal on both sides, but due to the extra or odd electrons, the repulsive forces
between the two oxygen atoms are unequal and they move further. This bond-pair
atom spread increases the bond angles in the molecule. Additionally, the shape or
structure changes into a curved shape or V-shape similar to water molecules.

SHORT ANSWER QUESTIONS WITH MODEL ANSWER 02


1] What is shape of PCl5 molecule and how many lone pairs and bond pairs are
there on central P atom?
Solution: According to the VESPR theory, the shape of PCl 5 is triangular
bipyramidal. Where P is the central atom with five Cl atoms around it. The
compound PCl 5 has five pairs of electrons around the central atom so there are 5
bonded pairs and no lone pairs.
2] What is shape of NO 2 molecule?

Solution: Shape or structure of nitrogen dioxide molecule is curved shape or V-


shape similar to water molecules.

03-03: XENON HEXA FLORIDE

XeF 6 has seven electron pairs. It consists of 6 bonded pairs and 1 free pair.
Xenon has 8 electrons in its valence shell and forms 6 bonds with fluorine atoms.
When xenon fluoride is formed, the electrons in the valence shell of xenon become
unpaired and the empty he is promoted to the 5d orbital.

The structure of XeF 6 took several years to establish, in contrast to that of XeF 2
and XeF 4 . In the gas phase the compound is a monomer. VSEPR theory predicts that
the structure lacks perfect octahedral symmetry due to the presence of six fluoride

CHE509: Inorganic Chemistry – II Page 227


ligands and lone pairs.Since XeF 6 has sp 3 d 3 hybridization, the shape of XeF 6
becomes distorted octahedron.After hybridization, the molecular structure of XeF 6
becomes a distorted octahedron or square bipyramid. What happens here is that the
fluorine atoms are placed at the vertices of the octahedron, and the lone pair of
electrons moves in space to avoid or reduce repulsion.

SILICON TETRACHLORIDE
The geometry of a silicon tetrachloride (SiCl 4 ) molecule is tetrahedral. The
molecule consists of a central Si atom, which has a coordination number of 4. Four
chlorides (Cl) atoms can bond to Si. When they do, they assume a geometry around
the Si, in such a way as to put maximum distance between themselves/each other,
and minimize repulsion. This results in a tetrahedral configuration. Therefore, the
molecule has no net dipole moment.

SOLVED PROBLEMS 03
1] Describe distorted shape of XeF 6 molecule

Solution: XeF 6 has seven electron pairs. It consists of 6 bonded pairs and 1
free pair. Xenon has 8 electrons in its valence shell and forms 6 bonds with fluorine
atoms. When xenon fluoride is formed, the electrons in the valence shell of xenon
become unpaired and the empty he is promoted to the 5d orbital.

The structure of XeF 6 took several years to establish, in contrast to that of XeF 2
and XeF 4 . In the gas phase the compound is a monomer. VSEPR theory predicts that
the structure lacks perfect octahedral symmetry due to the presence of six fluoride
ligands and lone pairs.Since XeF 6 has sp 3 d 3 hybridization, the shape of XeF 6
becomes distorted octahedron.After hybridization, the molecular structure of XeF 6

CHE509: Inorganic Chemistry – II Page 228


becomes a distorted octahedron or square bipyramid. What happens here is that the
fluorine atoms are placed at the vertices of the octahedron, and the lone pair of
electrons moves in space to avoid or reduce repulsion.

2] Explain the structure of SiCl 4 molecule according to VSEPR theory.

Solution: The geometry of a silicon tetrachloride (SiCl 4 ) molecule is


tetrahedral. The molecule consists of a central Si atom, which has a coordination
number of 4. Four chlorides (Cl) atoms can bond to Si. When they do, they assume
geometry around the Si, in such a way as to put maximum distance between
themselves/each other, and minimize repulsion. This results in a tetrahedral
configuration. Therefore, the molecule has no net dipole moment. Because of
absence of lone pair on Si atom SiCl 4 has tetrahedral geometry instead of square
pyramidal.

SHORT ANSWER QUESTIONS WITH MODEL ANSWER 03

1] Why SiCl 4 is having tetrahedral instead of square pyramidal?

Solution: Because of absence of lone pair on Si atom SiCl 4 has Tetrahedral


geometry instead of square pyramidal.

2] Why XeF 6 is having distorted octahedral geometry instead of perfect octahedral


geometry?

Solution: Because of presence of lone pair on Xe atom XeF 6 has distorted


octahedral geometry instead of square perfect octahedral geometry.

CHE509: Inorganic Chemistry – II Page 229


03-04: CHLORINE TRIFLUORIDE
When we talk about the hybridization of chlorine trifluoride, we must consider
its central atom, Cl. Although this atom contains seven valence electrons, ClF3
should consist of three bond pairs and two lone pairs. A closer look at the valence
configuration of Cl is represented as 3s 2 , 3px 2 , 3py2 , 3pz 1 , 3d.

Now, if Cl required to bond with a fluorine atom to form ClF3, it required three
unpaired electrons to bond with the three F atoms. Here, one of the paired Cl
electrons remains in the 3p subshell as a lone pair or unpaired. Overall, the
hybridization involves one 3s, three 3p and one 3d orbital in the process, forming
five sp3d hybrid orbitals. Here, two hybrid orbitals contain electron pairs and three
hybrid orbitals contain unpaired electrons, which overlap with the 2p orbitals of F to
form a single bond.

The central atom Cl required three unpaired electrons to bond with the three F
atoms. ClF 3 should consist of 3 bound pairs and 2 lone pairs. One 3s, three 3p, and
one of the 3d orbitals of Cl participate in hybridization, forming five sp 3 d hybrid
orbitals.

The molecular structure of ClF 3 is said to be T-shaped. There are two isolated
pairs occupying positions on the equator, and there is a large repulsive force,
resulting in this shape. It also has an asymmetric charge distribution around t he

CHE509: Inorganic Chemistry – II Page 230


central atom. The electronic structure of chlorine trifluoride is trigonal bipyramidal
with an F-Cl-F bond angle of 175°.

Fluorine is the most electronegative element in the periodic table and is highly
reactive to get one electron to get the electronic configuration of noble family or it
shows only -1 oxidation state And due to high electronegativity it can't donate or
share its 3–3 electrons with chlorine and it can also form only one bond due to the
absence of d- orbitals, while chlorine have d orbitals and can donate its 3 electrons
to fluorine,. Therefore ClF3 exist but FCl3 doesn't exist.Cl is a large atom and F is a
small atom. So Cl can hold three F atoms, but not F. Cl has a d orbital, but F does
not. Note that all halogens have the p-5 configuration. So, in order for the valence to
increase, each atom must detach its pair electron and promote them to the d orbitals.
Otherwise, F cannot have a valence greater than 1 and FCl3 is not formed.

SOLVED PROBLEMS 04

1] Describe shape of ClF 3 molecule

Solution: When we talk about the hybridization of chlorine trifluoride, we


must consider its central atom, Cl. Although this atom contains seven valence
electrons, ClF3 should consist of three bond pairs and two lone pairs. A closer look
at the valence configuration of Cl is represented as 3s 2 , 3px 2 , 3py2 , 3pz 1 , 3d.

Now, if Cl required to bond with a fluorine atom to form ClF3, it required three
unpaired electrons to bond with the three F atoms. Here, one of the paired Cl
electrons remains in the 3p subshell as a lone pair or unpaired. Overall, the
hybridization involves one 3s, three 3p and one 3d orbital in the process, forming
five sp3d hybrid orbitals. Here, two hybrid orbitals contain electron pairs and three

CHE509: Inorganic Chemistry – II Page 231


hybrid orbitals contain unpaired electrons, which overlap with the 2p orbitals of F to
form a single bond.

The central atom Cl required three unpaired electrons to bond with the three F
atoms. ClF 3 should consist of 3 bound pairs and 2 lone pairs. One 3s, three 3p, and
one of the 3d orbitals of Cl participate in hybridization, forming five sp 3 d hybrid
orbitals.

The molecular structure of ClF 3 is said to be T-shaped. There are two isolated
pairs occupying positions on the equator, and there is a large repulsive force,
resulting in this shape. It also has an asymmetric charge distribution around the
central atom. The electronic structure of chlorine trifluoride is trigonal bipyramidal
with an F-Cl-F bond angle of 175°.

Fluorine is the most electronagitive element in the periodic table and is highly
reactive to get one electron to get the electronic configuration of noble family or it
shows only -1 oxidation state And due to high electronegativity it can't donate or
share its 3–3 electrons with chlorine and it can also form only one bond due to the
absence of d- orbitals, while chlorine have d orbitals and can donate its 3 electrons
to fluorine,. Therefore ClF3 exist but FCl 3 doesn't exist.Cl is a large atom and F is a
small atom. So Cl can hold three F atoms, but not F. Cl has a d orbital, but F does
not. Note that all halogens have the p 5 configuration. So, in order for the valence to
increase, each atom must detach its pair electron and promote them to the d orbitals.
Otherwise, F cannot have a valence greater than 1 and FCl 3 is not formed.
SHORT ANSWER QUESTIONS WITH MODEL ANSWER 04
1] Why FCl 3 molecule cannot exist but ClF 3 exist explain.
Solution: Fluorine is the most electronagitive element in the periodic table and
is highly reactive to get one electron to get the electronic configuration of noble

CHE509: Inorganic Chemistry – II Page 232


family or it shows only -1 oxidation state And due to high electronegativity it can't
donate or share its 3–3 electrons with chlorine and it can also form only one bond
due to the absence of d- orbitals, while chlorine have d orbitals and can donate its 3
electrons to fluorine,. Therefore ClF3 exist but FCl 3 doesn't exist.Cl is a large atom
and F is a small atom. So Cl can hold three F atoms, but not F. Cl has a d orbital, but
F does not. Note that all halogens have the p 5 configuration. So, in order for the
valence to increase, each atom must detach its pair electron and promote them to the
d orbitals. Otherwise, F cannot have a valence greater than 1 and FCl 3 is not formed.

CHECK POINT 01-04


1] In BrF 3 , lone pairs are present at the equatorial posi tions. This is to minimize
a) bp-bp repulsion only
b) lp-lp repulsion only
c) lp-bp repulsion only
d) both (b) and (c)
Solution: b
2] XeF6 is
a) Distorted octahedral
b) Tetrahedral
c) Octahedral
d) Planar
Solution: a
3] Which of the following species has 120° bond angles?
(a) NCl3
(b) PH3
(c) ClF3
(d) BCl3
Solution: d
4] Which of the following species has a trigonal planar shape?
a) NO3 –
b) CO2
c) N3
d) NO2 –
Solution: a

CHE509: Inorganic Chemistry – II Page 233


SUMMARY
VSEPR theory is used to predict the shape of a molecule from the electron pairs
surrounding the central atom of the molecule. This theory was first put forward by Sidgwick
and Powell in 1940. VSEPR theory is based on the assumption that molecules adopt
geometries such that electron repulsion in the valence shells of atoms is minimized.The
valence shell electron-pair repulsion theory, abbreviated as VSEPR theory, is based on the
premise that there is repulsion between the valence pairs of all atoms, and the atom always
minimizes this electron-pair repulsion. This arrangement of atoms determines the shape of
the resulting molecule.
KEY WORDS
VSEPR theory, dichloroiodate(I)anion, Nitrogen dioxide, Silicon tetroxide,
Chlorine Trifloride
REFERENCES

MOOCS
--
YOUTUBE VIDEOS
5) https://www.youtube.com/watch?v=x2-nP7i6T34
6) https://www.youtube.com/watch?v=wj4u5hM6QhI
7) https://www.youtube.com/watch?v=1TWeEu_UGwQ
8) https://www.youtube.com/watch?v=yFLTE52koCc
9) https://www.youtube.com/watch?v=oBPQsOrhbuc

WIKIPEDIA
4) https://en.wikipedia.org/wiki/VSEPR_theory
5) https://en.wikipedia.org/wiki/Molecular_geometry
6) https://en.wikipedia.org/wiki/Molecular_geometry#VSEPR_table
OER
3) https://byjus.com/jee/vsepr-theory/
4) https://www.sigmaaldrich.com/IN/en/technical -documents/technical-
article/chemistry-and-synthesis/organic-reaction-toolbox/vsepr-chart-
valence-shell-electron-pair-repulsion-theory
5) https://chemed.chem.purdue.edu/genchem/topicreview/bp/ch8/v sepr.
html

CHE509: Inorganic Chemistry – II Page 234


6) https://chem.libretexts.org/Bookshelves/Inorganic_Chemistry/Map%3
A_Inorganic_Chemistry_(Miessler_Fischer_Tarr)/03%3A_Simple_Bo
nding_Theory/3.02%3A_Valence_Shell_Electron-Pair_Repulsion
REFERENCE BOOKS
3) The VSEPR Model of Molecular Geometry (Dover Books on Chemistry) by
Ronald J Gillespie
4) Inorganic Chemistry | Fourth Edition | By Pearson: Principles of Structure
and , 2006 by James E Huheey and Medhi

CHE509: Inorganic Chemistry – II Page 235


CREDIT 03 -UNIT 04: APPLICATIONS OF MOT

LEARNING OBJECTIVES

After successful completion of this unit, you will be able to


 Understand the concept of LCAO method of MOT
 Understand the formation of O 2 molecule according to MOT
 Understand the formation of H 2 O molecule according to MOT
 Understand the formation of CO 2 molecule according to MOT
 Understand the formation of NH 3 molecule according to MOT

INTRODUCTION

Molecular orbital theory (often abbreviated to MOT) is a theory of chemical


bonding developed by F. Hund and R.S. Mulliken in the early 20th century to
explain the structure and properties of various molecules. Valence bond theory fails
to adequately explain how certain molecules contain two or more equivalent bonds
whose bond order is between single and double bonds, such as those in resonance -
stabilized molecules. bottom. This is where molecular orbital theory proved to be
stronger than valence bond theory (because the orbitals described by MOT reflect
the shape of the molecule to which it is applied).Simply put, molecular orbital theory
states that all atoms tend to bond to form molecular orbitals. As a result of such
arrangement, the electrons are in different atomic orbitals and are usually associated
with different nuclei. That means electrons in a molecule can exist anywhere in the
molecule.

04-01: FORMATION OF O2 MOLECULE ACCORDING TO MOT

The bond length of oxygen species can be explained by the position of


electrons in molecular orbital theory. To obtain the molecular orbital energy level
diagram of O2, we need to place 12 valence electrons (6 from each O atom) in the
energy level diagram. Refill orbitals starting with the lowest energy according to
Hund's Law and Pauli's Principle. Two electrons each are required to fill the σ2s and
σ2s* orbitals, two more to fill the σ2pz orbitals, and four electrons to fill the
degenerate π⋆2px and π⋆2py orbitals. Is required. According to Hund's first rule, the
last two electrons must be placed in separate parallel spin π* orbitals, giving a

CHE509: Inorganic Chemistry – II Page 236


multiplicity of 3 (triplet state) with two unpaired electrons. This leads to the
expected bond order (8 - 4) / 2 = 2.

Molecular orbital energy level diagram of O2. Since there are 12 valence
electrons (6 from each O atom), only 2 electrons need to be placed in the
(π⋆nPx,π⋆nPy) orbital pair. Hund's first rule states that one electron occupies each
orbital and that their spins are parallel, giving the O2 molecule two unpaired
electrons. The figure shows 8 electrons in bonding orbitals and 4 electrons in
antibonding orbitals, giving a predicted bond order of 2.

Molecular orbital theory describes the formation of molecules by the overlap of


two atomic orbitals. Represent molecular orbitals using the Linear Combination of
Atomic Orbitals (LCAO) method. Molecular orbitals are divided into bonding, anti -
bonding and non-bonding.

Bonding: In bonding orbitals, electrons are dense and concentrated be tween pairs of
atoms.

Antibonding: In antibonding orbitals, there is an electron density behind the nucleus


that tends to pull the two nuclei away from each other, actually weakening the bond
between the two nuclei.

Non-bonding - Electrons in non-bonding orbitals are associated with atomic orbitals


that do not interact with each other and do not participate in bond formation.

Electronic configuration of oxygen (Z=8): 1S 2 , 2S 2 , 2P x 2 , 2P y1 , 2Pz 1

When two oxygen atom combine total electron present in the orbital is 16. MOT
combination results in the formation of molecular orbitals

Bond orders are determined from the electronic configuration via Equation for the
electronic configurations of the four species are contrasted below.

O 2 : KK ' (σ 2s ) 2 <(σ ∗ 2s ) 2 <(σ 2pz ) 2 < (π 2px ) 2 = (π 2py) 2 < (π ∗ 2 px ) 1 =(π ∗ 2 py ) 1

KK' represents the combination of 1s orbitals of both oxygen atoms (σ 1s ) 2 , (σ ∗ 1s ) 2

The molecular orbital of the O 2 bond. Coincidentally, the molecular orbital


description of this molecule provided an explanation for a longstanding puzzle that
other binding models could not. Other bond models (such as valence bond theory
and Lewis bond) cannot predict the presence of two unpaired electrons in O 2 .

CHE509: Inorganic Chemistry – II Page 237


Chemists have long wondered why liquid O 2 is attracted to magnetic fields, unlike
most other substances. The liquid actually remains stuck between the poles of the
magnet until it evaporates. The only way to explain this behavior was that O 2 has an
unpaired electron, making it paramagnetic. This result was one of the early victories
of molecular orbital theory over other bonding approaches.

Energy level diagram for O 2 molecule

SOLVED PROBLEMS 01

1] Explain the formation of O 2 molecule with energy level diagram

Solution : The bond length of oxygen species can be explained by the position
of electrons in molecular orbital theory. To obtain the molecular orbital energy level
diagram of O2, we need to place 12 valence electrons (6 from each O atom) in the
energy level diagram. Refill orbitals starting with the lowest energy according to
Hund's Law and Pauli's Principle. Two electrons each are required to fill the σ2s and
σ2s* orbitals, two more to fill the σ2pz orbitals, and four el ectrons to fill the
degenerate π⋆2px and π⋆2py orbitals. Is required. According to Hund's first rule, the
last two electrons must be placed in separate parallel spin π* orbitals, giving a
multiplicity of 3 (triplet state) with two unpaired electrons. This leads to the
expected bond order (8 - 4) / 2 = 2.

CHE509: Inorganic Chemistry – II Page 238


Molecular orbital energy level diagram of O2. Since there are 12 valence
electrons (6 from each O atom), only 2 electrons need to be placed in the
(π⋆nPx,π⋆nPy) orbital pair. Hund's first rule states that one electron occupies each
orbital and that their spins are parallel, giving the O2 molecule two unpaired
electrons. The figure shows 8 electrons in bonding orbitals and 4 electrons in
antibonding orbitals, giving a predicted bond order of 2.

Molecular orbital theory describes the formation of molecules by the overlap of two
atomic orbitals. Represent molecular orbitals using the Linear Combination of
Atomic Orbitals (LCAO) method. Molecular orbitals are divided into bonding, anti -
bonding and non-bonding.

Bonding: In bonding orbitals, electrons are dense and concentrated between pairs of
atoms.

Antibonding: In antibonding orbitals, there is an electron density behind the nucleus


that tends to pull the two nuclei away from each other, actually weakening the bond
between the two nuclei.

Non-bonding - Electrons in non-bonding orbitals are associated with atomic orbitals


that do not interact with each other and do not participate in bond formation.

Electronic configuration of oxygen (Z=8): 1S 2 , 2S 2 , 2P x 2 , 2P y1 , 2Pz 1

When two oxygen atom combine total electron present in the orbital is 16. MOT
combination results in the formation of molecular orbitals

Bond orders are determined from the electronic configuration via Equation for
the electronic configurations of the four species are contrasted below.

O 2 : KK ' (σ 2s ) 2 <(σ ∗ 2s ) 2 <(σ 2pz ) 2 < (π 2px ) 2 = (π 2py) 2 < (π ∗ 2 px ) 1 =(π ∗ 2 py ) 1

KK' represents the combination of 1s orbitals of both oxygen atoms (σ 1s ) 2 ,


(σ ∗ 1s ) 2

The molecular orbital of the O 2 bond. Coincidentally, the molecular orbital


description of this molecule provided an explanation for a longstanding puzzle that
other binding models could not. Other bond models (such as valence bond theory
and Lewis bond) cannot predict the presence of two unpaired electrons in O 2 .
Chemists have long wondered why liquid O 2 is attracted to magnetic fields, unlike
most other substances. The liquid actually remains stuck between the poles of the

CHE509: Inorganic Chemistry – II Page 239


magnet until it evaporates. The only way to explain this behavior was that O 2 has an
unpaired electron, making it paramagnetic. This result was one of the early victories
of molecular orbital theory over other bonding approaches.

Energy level diagram for O 2 molecule

SHORT ANSWER QUESTIONS WITH MODEL ANSWER 01

1] Give molecular electronic configuration of O 2 .

Solution : O 2 : KK ' (σ 2s ) 2 <(σ ∗ 2s ) 2 <(σ 2pz ) 2 < (π 2px ) 2 = (π 2py) 2 < (π ∗ 2 px ) 1 =(π ∗ 2 py ) 1

04-02: FORMATION OF H2O MOLECULE ACCORDING TO MOT

The localization of electrons within atomic orbitals in valence bond theory,


electrons are considered to be delocalized throughout the molecule in the molecular
orbital approach. A simple MO diagram for H 2 O is shown on the right. After a
simple symmetry treatment, his 1s orbitals of hydrogen atoms are premixed as a1
and b1. Orbitals with the same symmetry and similar energy levels can then be
mixed to form a new set of molecular orbitals with bonding, non -bonding, and anti-
bonding properties. In the simple MO diagram of HO, the 2s orbitals of oxygen mix
with the premixed hydrogen orbitals to form new bonding (2a1) and antibonding
(4a1) orbitals. Similarly, the 2p orbitals (b1) and other premixed hydrogen 1s

CHE509: Inorganic Chemistry – II Page 240


orbitals (b1) mix to form 1b1 bonding and 2b1 antibonding orbitals. The remaining
two 2p orbitals are unmixed. Although this simple MO diagram does not provide the
four different energy levels experimentally determined by PES, nevertheless the two
bonding orbitals are clearly different and the energy levels of the bonding electrons
can be distinguished.

The atomic orbital size of the final molecular orbital is different from the
original atomic orbital size. This is due to the different mixing ratios of oxygen and
hydrogen orbitals due to different initial atomic orbital energies. That is, if two
orbitals are mixed, the amount by which the orbitals are mixed is inversely
proportional to the initial energy difference of the orbitals. Thus, orbitals that are
initially energetically closer mix (ie, interact) mo re than initially energetically
distant orbitals. When two orbitals of different energies mix (that is, interact), the
lower energy combination is closer to the first lower energy orbital. Higher energy
combinations are similar to the first high energy orbitals. If two orbitals interact and
have the same initial energy, the resulting two bonding orbitals are equally derived
from the two initial orbitals. (Secondary perturbation theory).Valence bond theory
predicts that H 2 O is sp 3 hybridized, but the prediction from MO theory is more
complex. Some would argue that HO is sp2 hybridized, since the 2pz orbital does not
participate in any interaction with the hydrogen atom, resulting in an
unhydrogenated lone pair (nO(π)). This holds true under the idealized assu mption
that the s and p properties are evenly distributed between the two O –H bonds and the
O lone pair (nO(σ)). However, this prediction (120° bond angle) is inconsistent with
the 104.5° bond angle of H 2 O.

In water molecules, the highest occupied orbital (1b 1 ) is non-bonding and


strongly localized to the oxygen atom, similar to the non-bonding orbital of
hydrogen fluoride. The next lowest orbital (2a 1 ) can be considered a non-bonding
orbital because it has lobes pointing away from the two hydrogen atoms. T he lower
energy bonding orbitals show that oxygen also makes up more than a "fair fraction"
of the total electron density.
(1a 1 ) 2 (2a 1 ) 2 (1b 2 ) 2 (3a 1 ) 2 (1b 1 ) 2 (4a 1 ) 0 (2b 2 ) 0 (3b 2 ) 0

CHE509: Inorganic Chemistry – II Page 241


SOLVED PROBLEMS 02
1] Explain the formation of H 2 Omolecule with energy level diagram
Solution: The localization of electrons within atomic orbitals in valence bond
theory, electrons are considered to be delocalized throughout the molecule in the
molecular orbital approach. A simple MO diagram for H 2 O is shown on the right.
After a simple symmetry treatment, his 1s orbitals of hydrogen atoms are premixed
as a1 and b1. Orbitals with the same symmetry and similar energy levels can then be
mixed to form a new set of molecular orbitals with bonding, non -bonding, and anti-
bonding properties. In the simple MO diagram of HO, the 2s orbitals of oxygen mix
with the premixed hydrogen orbitals to form new bonding (2a1) and antibonding
(4a1) orbitals. Similarly, the 2p orbitals (b1) and other premixed hydrogen 1s
orbitals (b1) mix to form 1b1 bonding and 2b1 antibonding orbitals. The remaining
two 2p orbitals are unmixed. Although this simple MO diagram does not provide the
four different energy levels experimentally determined by PES, nevertheless the two
bonding orbitals are clearly different and the energy levels of the bonding e lectrons
can be distinguished.
The atomic orbital size of the final molecular orbital is different from the
original atomic orbital size. This is due to the different mixing ratios of oxygen and
hydrogen orbitals due to different initial atomic orbital energies. That is, if two

CHE509: Inorganic Chemistry – II Page 242


orbitals are mixed, the amount by which the orbitals are mixed is inversely
proportional to the initial energy difference of the orbitals. Thus, orbitals that are
initially energetically closer mix (ie, interact) more than initially energetically
distant orbitals. When two orbitals of different energies mix (that is, interact), the
lower energy combination is closer to the first lower energy orbital. Higher energy
combinations are similar to the first high energy orbitals. If two orbitals interact and
have the same initial energy, the resulting two bonding orbitals are equally derived
from the two initial orbitals. (Secondary perturbation theory).Valence bond theory
predicts that H 2 O is sp 3 hybridized, but the prediction from MO theory is more
complex. Some would argue that HO is sp2 hybridized, since the 2pz orbital does not
participate in any interaction with the hydrogen atom, resulting in an
unhydrogenated lone pair (nO(π)). This holds true under the idealized assumption
that the s and p properties are evenly distributed between the two O –H bonds and the
O lone pair (nO(σ)). However, this prediction (120° bond angle) is inconsistent with
the 104.5° bond angle of H 2 O.

In water molecules, the highest occupied orbital (1b 1 ) is non-bonding and


strongly localized to the oxygen atom, similar to the non-bonding orbital of
hydrogen fluoride. The next lowest orbital (2a 1 ) can be considered a non-bonding
orbital because it has lobes pointing away from the two hydrogen atoms. The lower

CHE509: Inorganic Chemistry – II Page 243


energy bonding orbitals show that oxygen also makes up more than a "fair fraction"
of the total electron density.
(1a 1 ) 2 (2a 1 ) 2 (1b 2 ) 2 (3a 1 ) 2 (1b 1 ) 2 (4a 1 ) 0 (2b 2 ) 0 (3b 2 ) 0

SHORT ANSWER QUESTIONS WITH MODEL ANSWER 02


1] Give molecular electronic configuration of O 2 .
Solution : O 2 : KK ' (2a 1 ) 2 (1b 2 ) 2 (3a 1 ) 2 (1b 1 ) 2 (4a 1 ) 0 (2b 2 ) 0 (3b 2 ) 0

04-03: FORMATION OF CO2 MOLECULE ACCORDING TO MOT

Carbon dioxide is a well-studied molecule. Carbon dioxide is a covalent


compound consisting of three atoms in which carbon is surrounded by two oxygen
atoms. Both carbon and oxygen contain p-orbitals that can interact due to symmetry
suitability. Valence bond theory (VB) predicts four bonds to carbon and two to each
oxygen. Each carbon-oxygen bond has a bond order of 2. Carbon dioxide contains
two double bonds. Each double bond consists of a sigma bond and a pi bond. Carbon
dioxide is an important reagent in industry, a central aspect of the global carbon
cycle, and the basis of climate change. Finally, molecular orbital (MO) theory is a
better tool than VB theory because it makes no assumptions about electron
localization. MO theory is based on linear combinations of atomic orbitals (LCAO).

The MO diagram of carbon dioxide is based on C atoms and OO ligand


fragments. Carbon has 2S and 2Px,y,z orbitals, and OO fragments have 2S and
2Px,y,z orbitals that participate in molecular orbital formation. Since CO 2 has D∞h
symmetry, the orbital symmetry identifiers of the central atoms ca n be obtained from
the corresponding point group tables: 2S=σg, 2Pz=σu, and 2Px,y=πu. Although the
diagram below does not show orbital symmetry, LGO 2Px,y is involved in the
formation of the π double bond. The 2πg orbitals are non -bonding because the C
2Px,y atomic orbitals are πu. The LGO 2Pz orbitals are involved in σ bonding.

After σ bond formation resulting from C 2S and 2Pz electrons with O 2S and
2Pz orbitals, the remaining C 2Px and 2Py orbitals interact with the O LGO
fragment. Qualitatively, πu(2Px,y) is represented in MO diagrams as in-phase
overlapping dumbbells, and πg(2Px,y)* as anti-phase overlapping.

CHE509: Inorganic Chemistry – II Page 244


SOLVED PROBLEMS 03

1] Explain the formation of CO 2 molecule with energy level diagram

Solution: Carbon dioxide is a well-studied molecule. Carbon dioxide is a


covalent compound consisting of three atoms in which carbon is surrounded by two
oxygen atoms. Both carbon and oxygen contain p-orbitals that can interact due to
symmetry suitability. Valence bond theory (VB) predicts four bonds to carbon a nd
two to each oxygen. Each carbon-oxygen bond has a bond order of 2. Carbon dioxide
contains two double bonds. Each double bond consists of a sigma bond and a pi
bond. Carbon dioxide is an important reagent in industry, a central aspect of the
global carbon cycle, and the basis of climate change. Finally, molecular orbital (MO)
theory is a better tool than VB theory because it makes no assumptions about
electron localization. MO theory is based on linear combinations of atomic orbitals
(LCAO).
The MO diagram of carbon dioxide is based on C atoms and OO ligand
fragments. Carbon has 2S and 2Px,y,z orbitals, and OO fragments have 2S and
2Px,y,z orbitals that participate in molecular orbital formation. Since CO 2 has D∞h
symmetry, the orbital symmetry identifiers of the central atoms can be obtained from
the corresponding point group tables: 2S=σg, 2Pz=σu, and 2Px,y=πu. Although the
diagram below does not show orbital symmetry, 2Px,y is involved in the formation

CHE509: Inorganic Chemistry – II Page 245


of the π double bond. The 2πg orbitals are non-bonding because the C 2Px,y atomic
orbitals are πu. The 2Pz orbitals are involved in σ bonding.
After σ bond formation resulting from C 2S and 2Pz electrons with O 2S and
2Pz orbitals, the remaining C 2Px and 2Py orbitals interact with the O fragment.
Qualitatively, πu(2Px,y) is represented in MO diagrams as in-phase overlapping
dumbbells, and πg(2Px,y)* as anti-phase overlapping.
The valence electron configuration of C is 2S 2 2P 2 , and for O it is 2S 2 2P 4 ,
therefore there is a total of 4+2(4)=12 valence electrons. The energy of the 2s and 2p
orbitals in C are higher than the the same orbitals in O because O has smaller radius,
therefore the valence electrons are closer to the nucleus. Atomic orbitals are shown
as duplicates because of the two O atoms hence 2. Constructing MOs The 2S orbitals
in O do not have match in C, therefore they don't change, they will have non -
bonding electrons, which will be two lone pairs. The 2S orbital in carbon can
overlap with the 2Pz orbital from the O atoms (which is aligned along the bond) and
form a bonding ( σ ) and anti-bonding (σ∗) orbital. The 2Pz orbital of C and the
same orbital from the second O form a bonding (σ) and anti-bonding (σ∗) orbital.
The 2Py orbital on C and the same orbital on O can form a π -bond and also a π∗
anti-bonding orbital. C is out of electrons, therefore the 2Px orbital on the O atoms
remain unchanged and form non-bonding orbitals (two lone pairs of electrons).

CHE509: Inorganic Chemistry – II Page 246


SHORT ANSWER QUESTIONS WITH MODEL ANSWER 03

1] Explain the π bond formation in carbon dioxide molecule

Solution: After σ bond formation resulting from C 2S and 2Pz electrons with O
2S and 2Pz orbitals, the remaining C 2Px and 2Py orbitals interact with the O
fragment. Qualitatively, πu(2Px,y) is represented in MO diagrams as i n-phase
overlapping dumbbells, and πg(2Px,y)* as anti-phase overlapping.

04-04: FORMATION OF NH3 MOLECULE ACCORDING TO MOT


Molecular orbital theory (MO) is used to predict the electronic structure of
molecules. A molecular orbital is formed from the interaction of two or more atomic
orbitals, and the interaction between atomic orbitals can be bonding, anti -bonding,
or non-bonding. Bonding orbitals are interactions of two atom/group orbitals in
phase, and antibonding orbitals are formed by out-of-phase combinations. In general,
the energy levels of molecular orbitals increase from bonding to non -bonding and
anti-bonding molecular orbitals. Since pi interactions are less effective than sigma
interactions, pi bonding molecular orbitals generally have greater energy than sigma
bonding molecular orbitals. Increasing the number of nodes increases the molecular
orbital energy, and increasing the number of nodes increases the molecular orbital
energy. Within a bond molecular orbital of the same symmetry, the lowest energy
comes from a perfectly symmetric sigma bond molecular orbital.

NH 3 MO Diagrams Molecular orbital diagrams are useful for representing and


explaining the chemical bonds of molecules in the context of molecular orbital
theory. The formation of molecular orbitals involves interactions between atomic
orbitals when the symmetries fit each other based on group theory. Linear
combinations of atomic orbitals occur only in orbitals of similar symmetry.
Overlapping orbitals of similar symmetry contribute to the formation of molecular
orbitals. If the overlap is large enough, the resulting molecular orbitals are split. For
NH 3 , the a1 in the 3H(1s) orbital can only bond with the nitrogen a1 orbitals (such as
2s and 2pz). The 2py nitrogen orbital can bond with the e1 SALC and the 2px with
e2. The linear combination of atomic orbitals is drawn in the MO diagram of NH 3
and shown in detail.

Ammonia or NH 3 has 8 valence electrons, consisting of a lone pair on nitrogen


and 3 N-H sigma bonds. A molecular orbital diagram of NH 3 is shown in Figure to

CHE509: Inorganic Chemistry – II Page 247


detail its interactions. The three hydrogen s orbitals are used to construct the sigma
and antibond combinations of the N sp3 and H 1s orbitals.

The three H 1s orbital forms a combination of a1 and e. Bonding and


antibonding interactions are established at N-orbitals of similar symmetry. The
remaining a1 orbitals on N are higher in energy than the atomic orbitals because they
interact with lower energy a1 bonding orbitals. Therefore, this orbital is considered
non-bonded.

Note that the molecular orbital is non-degenerate and symmetrical about the x-,
y-, and z-axis rotations about z. However, the bonding e orbital is a doubly
degenerate orbital of px and py.

The two frontier molecular orbitals of interest are the highest occupied
molecular orbital (HOMO) and the lowest occupied molecular orbital (LUMO). If
you look at the molecular orbital diagram of NH 3 , notice that it has two electrons in
the 2a1 orbital, making it a HOMO. On the other hand, 3a1 is an anti -bonding
molecular orbital and does not contain an electron, so it is a LUMO. Therefore, these
two orbitals are on the boundary of the NH3 molecule.

CHE509: Inorganic Chemistry – II Page 248


SOLVED PROBLEMS 04

1] Explain the formation of NH 3 molecule with energy level diagram

Solution: Molecular orbital theory (MO) is used to predict the electronic


structure of molecules. A molecular orbital is formed from the interaction of two or
more atomic orbitals, and the interaction between atomic orbitals can be bonding,
anti-bonding, or non-bonding. Bonding orbitals are interactions of two atom/group
orbitals in phase, and antibonding orbitals are formed by out -of-phase combinations.
In general, the energy levels of molecular orbitals increase from bonding to non -
bonding and anti-bonding molecular orbitals. Since pi interactions are less effective
than sigma interactions, pi bonding molecular orbitals generally have greater energy
than sigma bonding molecular orbitals. Increasing the number of nodes increases the
molecular orbital energy, and increasing the number of nodes increases the
molecular orbital energy . Within a bond molecular orbital of the same symmetry, the
lowest energy comes from a perfectly symmetric sigma bond molecular orbital.

NH 3 MO Diagrams Molecular orbital diagrams are useful for representing and


explaining the chemical bonds of molecules in the context of molecular orbital
theory. The formation of molecular orbitals involves interactions between atomic
orbitals when the symmetries fit each other based on group theory. Linear
combinations of atomic orbitals occur only in orbitals of similar symmetry.
Overlapping orbitals of similar symmetry contribute to the formation of molecular
orbitals. If the overlap is large enough, the resulting molecular orbitals are split. For
NH 3 , the a1 in the 3H (1s) orbital can only bond with the nitrogen a1 orbitals (such
as 2s and 2pz). The 2py nitrogen orbital can bond with the e1 SALC and the 2px
with e2. The linear combination of atomic orbitals is drawn in the MO diagram of
NH 3 and shown in detail.

Ammonia or NH 3 has 8 valence electrons, consisting of a lone pair on nitrogen


and 3 N-H sigma bonds. A molecular orbital diagram of NH 3 is shown in Figure to
detail its interactions. The three hydrogen s orbitals are used to construct the sigma
and antibond combinations of the N sp3 and H 1s orbitals.

The three H 1s orbital forms a combination of a1 and e. Bonding and


antibonding interactions are established at N-orbitals of similar symmetry. The
remaining a1 orbitals on N are higher in energy than the atomic orbitals because they

CHE509: Inorganic Chemistry – II Page 249


interact with lower energy a1 bonding orbitals. Therefore, this orbital is considered
non-bonded.

Note that the molecular orbital is non-degenerate and symmetrical about the x-,
y-, and z-axis rotations about z. However, the bonding e orbital is a doubly
degenerate orbital of px and py.

The two frontier molecular orbitals of interest are the highest occupied
molecular orbital (HOMO) and the lowest occupied molecular orbital (LUMO). If
you look at the molecular orbital diagram of NH 3 , notice that it has two electrons in
the 2a1 orbital, making it a HOMO. On the other hand, 3a1 is an anti -bonding
molecular orbital and does not contain an electron, so it is a LUMO. Therefore, these
two orbitals are on the boundary of the NH3 molecule.

SHORT ANSWER QUESTIONS WITH MODEL ANSWER 04

1] Explain the Sigmabond formation in NH 3 molecule

CHE509: Inorganic Chemistry – II Page 250


Solution: Ammonia or NH 3 has 8 valence electrons, consisting of a lone pair on
nitrogen and 3 N-H sigma bonds. A molecular orbital diagram of NH 3 is shown in
Figure to detail its interactions. The three hydrogen s orbitals are used to construct
the sigma and antibond combinations of the N sp3 and H 1s orbitals.

CHECK POINT 01-04

1] The oxygen molecule is paramagnetic. It can be explained by

a) Resonance

b) Hybridisation

c) Valence bond theory

d) Molecular orbital theory

Solution: d

2] O-O bond length is minimum in

a) O2 –

b) O2

c) O2 +

d) O 2 2-

Solution: c

3] Find the molecule having the highest bond order

a) O 2 +

b) O 2 –

c) O 2 2-

d) O 2

Solution: a

4] The molecule in which the bond order increases upon addition of an electron
is

a) O 2

b) B 2

CHE509: Inorganic Chemistry – II Page 251


c) P 2

d) N 2

Solution: b

SUMMARY

Molecular orbital (MO) theory provides an explanation for the chemical bonds
responsible for the paramagnetism of the oxygen molecule. We also describe many other
molecular bonds that are difficult to describe in Lewis structures, such as octet rule violations
and many more molecules with more complex bonds (beyond the scope of this text). In
addition, we provide a model that describes the energies of electrons in molecules and the
putative positions of those electrons. Unlike valence bond theory, which uses hybrid orbitals
assigned to specific atoms, MO theory uses combinations of atomic orbitals to be delocalized
throughout the molecule rather than being localized to constituent atoms. give the molecular
orbitals. MO theory also helps us understand why some materials are conductors, some
semiconductors, and some insulators. summarizes the main points of two complementary
attachment theories. Both theories offer different and useful ways to explain molecular
structure.

KEY WORDS

MOT, Shape of molecule, Energy profile diagram for Molecules

REFERENCES

MOOCS

______

YOUTUBE VIDEOS

1) https://www.youtube.com/watch?v=AmedDfE0UNI

2) https://www.youtube.com/watch?v=TdGy6a1L7kI

3) https://www.youtube.com/watch?v=2hkCj7cUpTg

4) https://www.youtube.com/watch?v=fvGrdPkXxg0

5) https://www.youtube.com/watch?v=PLbtN_m04fw

CHE509: Inorganic Chemistry – II Page 252


WIKIPEDIA

1) https://en.wikipedia.org/wiki/Molecular_orbital_theory#:~:text=In%2
0molecular%20orbital%20theory%2C%20electrons,nuclei%20in%20th
e%20whole%20molecule.

2) https://en.wikibooks.org/wiki/Introduction_to_Inorganic_Chemistry/
Molecular_Orbital_Theory

OER

1) https://chem.libretexts.org/Courses/University_of_Florida/CHM2047
%3A_One-
Semester_General_Chemistry_(Kleiman)/06%3A_Understanding_Mol
ecules_with_QM/6.09%3A_Summary_of_Molecular_Orbital_Theory

REFERENCE BOOKS

1) Principles of Inorganic Chemistry by B.R. Puri, L.R. Sharma

2) Inorganic Chemistry Principles of Structure and Reactivity by James E


Huheey, Medhi

CHE509: Inorganic Chemistry – II Page 253


C REDIT 04

CHE509: Inorganic Chemistry – II Page 254


CREDIT 04 -UNIT 01: SYMMETRY ELEMENTS AND SYMMETRY OPERATIONS

LEARNING OBJECTIVES

After successful completion of this unit, you will be able to


 Apply the concept of center of symmetry
 Apply the concept of axis of symmetry
 Apply the concept of order of axis of symmetry
 Apply the concept of plane of symmetry

INTRODUCTION

Molecular symmetry in chemistry describes the symmetry present in molecules and the
classification of these molecules according to their symmetry. Molecular symmetry is a
fundamental concept in chemistry, as it can be used to predict or explain many of a
molecule's chemical properties, such as whether or not it has a dipole moment, as well as its
allowed spectroscopic transitions. To do this it is necessary to use group theory. This involves
classifying the states of the molecule using the irreducible representations from the character
table of the symmetry group of the molecule. Symmetry is useful in the study of molecular
orbitals, with applications to the Hückel method, to ligand field theory, and to the
Woodward-Hoffmann rules. Many university level textbooks on physical chemistry, quantum
chemistry, spectroscopy and inorganic chemistry discuss symmetry. Another framework on a
larger scale is the use of crystal systems to describe crystallographic symmetry in bulk
materials.

01-01: CENTER OF SYMMETRY

Any point in space that can be used as a mirror to reflect any group on a
molecule back through it in the opposite direction and obtain an equivalent group is
known as a center of symmetry.

CHE509: Inorganic Chemistry – II Page 255


In their crystal lattices, covalent molecules and ionic compounds have a certain
arrangement that gives them symmetry. A point is nothing mo re than one atom from
a certain molecule or an ion that is present in an ionic lattice in a crystal.
Represented by ‘i’

A molecule has a center of symmetry when an identical atom is located


diametrically opposite and equally away from the center.

Inversion i through the center of symmetry leaves the numerator unchanged. An


inversion consists of passing each point the same distance through the center of the
inversion and the opposite side of the molecule. If inversion symmetry exists, a line
drawn through the center by any atom connects to equivalent atoms equidistant from
the center. An example of a molecule with an inversion center is shown in following
figure The center of inversion is indicated by a dot, which may or may not overlap
an atom. The center of inversion is always at the center of the molecule, and there
can only be one center of inversion in any system. The inversion center in the
following example does not overlap the atom.

CHE509: Inorganic Chemistry – II Page 256


Examples of Center of Inversion Symmetry. (left) Benzene and (right)
staggard ethane have centers of inversion (green balls).

SOLVED PROBLEMS 02

1] Describe the concept of center of symmetry with suitable examples.

Solution: Any point in space that can be used as a mirror to reflect any group
on a molecule back through it in the opposite direction and obtain an equivalent
group is known as a center of symmetry.

In their crystal lattices, covalent molecules and ionic compounds hav e a certain
arrangement that gives them symmetry. A point is nothing more than one atom from
a certain molecule or an ion that is present in an ionic lattice in a crystal.
Represented by ‘i’

A molecule has a center of symmetry when an identical atom is loc ated


diametrically opposite and equally away from the center.

Inversion i through the center of symmetry leaves the numerator unchanged. An


inversion consists of passing each point the same distance through the center of the
inversion and the opposite side of the molecule. If inversion symmetry exists, a line
drawn through the center by any atom connects to equivalent atoms equidistant from
the center. An example of a molecule with an inversion center is shown in following
figure the center of inversion is indicated by a dot, which may or may not overlap an
atom. The center of inversion is always at the center of the molecule, and there can
only be one center of inversion in any system. The inversion center in the following
example does not overlap the atom.

CHE509: Inorganic Chemistry – II Page 257


SHORT ANSWER QUESTIONS WITH MODEL ANSWER 02

1] Define Center of symmetry

Solution: Any point in space that can be used as a mirror to reflect any group
on a molecule back through it in the opposite direction and obtain an equivalent
group is known as a center of symmetry.

01-02: AXIS OF SYMMETRY

It is the imaginary axis around which when molecule rotate we will get same or
original geomentry of molecule is knows as axis of symmetry.

Or in other words, the axis of symmetry is an imaginary line that divides a figure
into two identical parts such that each part is a mirror reflection of one another.

CHE509: Inorganic Chemistry – II Page 258


When the figure is folded along the axis of symmetry, the two identical parts
superimpose.

An axis of symmetry is an imaginary straight line that divides a shape into two
identical parts, with one part being the mirror image of the other. When folded along
the axis of symmetry, the two parts overlap. A straight line is called a line of
symmetry or a mirror line. This line can be vertical, horizontal, or diag onal. This
axis of symmetry is also found in nature in flowers, riverbanks, buildings and
foliage. This can be seen at India's iconic marble structure, the Taj Mahal

The axis of symmetry is the straight line that makes the shape of an object
symmetrical. An axis of symmetry produces accurate reflections on both sides of it.
Either horizontal, vertical, or horizontal. When you fold or unfold an object along its
axis of symmetry, the two sides are the same. Different shapes have different lines of
symmetry. A square has 4 lines of symmetry, a rectangle has 2 lines of symmetry, a
circle has an infinite number of lines of symmetry, and a parallelogram has no lines
of symmetry. A regular polygon with 'n' sides has 'n' axes of symmetry.

n -fold Axis of Rotation, Cn


The rotation operation (also called proper rotation) Cn rotates an object
about its axis by 2π/n radians, or 360∘/n. Rotation around Cn leaves the molecule
unchanged. The H 2 O molecule has a C2 axis. A molecule can have multiple Cn axes.
In this case, the one with the largest value of n is called the major axis. Some highly
symmetrical systems may have multiple principal axes. Note that rotation about an
axis is counterclockwise. The Cn rotations are indicated by the
labeled vectors shown below.

CHE509: Inorganic Chemistry – II Page 259


SOLVED PROBLEMS 02

1] Explain the concept of axis of symmetry with example.

Solution: It is the imaginary axis around which when molecule rotate we will
get same or original geomentry of molecule is knows as axis of symmetry.

Or in other words, the axis of symmetry is an imaginary line that divides a


figure into two identical parts such that each part is a mirror reflection of one
another. When the figure is folded along the axis of symmetry, the two identical
parts superimpose.
An axis of symmetry is an imaginary straight line that divides a shape into two
identical parts, with one part being the mirror image of the other. When folded along
the axis of symmetry, the two parts overlap. A straight line is called a line of
symmetry or a mirror line. This line can be vertical, horizontal, or diagonal. This
axis of symmetry is also found in nature in flowers, riverbanks, buildings and
foliage. This can be seen at India's iconic marble structure, the Taj Mahal

The axis of symmetry is the straight line that makes the shape of an object
symmetrical. An axis of symmetry produces accurate reflections on both sides of it.
Either horizontal, vertical, or horizontal. When you fold or unfold an object along its
axis of symmetry, the two sides are the same. Different shapes have different lines of
symmetry. A square has 4 lines of symmetry, a rectangle has 2 lines of symmetry, a

CHE509: Inorganic Chemistry – II Page 260


circle has an infinite number of lines of symmetry, and a parallelogram has no lines
of symmetry. A regular polygon with 'n' sides has 'n' axes of symmetry.

n -fold Axis of Rotation, Cn


The rotation operation (also called proper rotation) Cn rotates an object
about its axis by 2π/n radians, or 360∘/n. Rotation around Cn leaves the molecule
unchanged. The H 2 O molecule has a C2 axis. A molecule can have multiple Cn axes.
In this case, the one with the largest value of n is called the major axis. Some highly
symmetrical systems may have multiple principal axes. Note that rotation about an
axis is counterclockwise. The Cn rotations are indicated by the
labeled vectors shown below.

SHORT ANSWER QUESTIONS WITH MODEL ANSWER 02

1] Define axis of symmetry.

Solution: It is the imaginary axis around which when molecule rotate we will
get same or equivalent configuration of molecule is knows as axis of symmetry.

CHE509: Inorganic Chemistry – II Page 261


01-03: ORDER OF AXIS
The order of axis is nothing but ratio of 360/degree of ration required to get
original configuration of molecule.

e.g 1) H2O molecule has 2 fold axis of rotation. i.e. we require two times
rotation to get original configuration.

After rotation about 180 ° structures (ii) is obtained and it is completely


superimposable on structure (i) i.e. we get equivalent configuration after 180 °
rotation

And we get same configuration after two time totation by 180 ° hence it is two
fold axis of symmentry for H 2 O molecule

Hence order of axis = 360/(degree of ration required to get original


configuration)

= 360/180= 2

= 2 fold axis of symmetry

Hence corresponding axis is known as is known as C 2 axis

e.g. 2) Ammonia molecule NH 3

The order of axis is nothing but ratio of 360/degree of ration required to get
original configuration of molecule.

NH 3 molecule has 3 fold axis of rotation. i.e. we require three times rotation to
get original configuration.

After rotation about 120 ° structures (ii) is obtained and it is completely


superimposable on structure (i) i.e. we get equivalent configuration after 180 °
rotation

CHE509: Inorganic Chemistry – II Page 262


And we get same or original configuration after three time totation by 120 °
hence it is two fold axis of symmentry for H 2 O molecule

Hence order of axis = 360/(degree of ration required to get original


configuration)

= 360/120= 3

= 3 fold axis of symmetry

Hence corresponding axis is known as is known as C 3 axis

SOLVED PROBLEMS 03

1] Explain the concept of order of axis with examples

Solution: The order of axis is nothing but ratio of 360/degree of ration required
to get equivalent configuration of molecule.

e.g 1) H2O molecule has 2 fold axis of rotation. i.e. we require two times
rotation to get original configuration.

After rotation about 180 ° structures (ii) is obtained and it is completely


superimposable on structure (i) i.e. we get equivalent configuration after 180 °
rotation

And we get same configuration after two time totation by 180 ° hence it is two
fold axis of symmentry for H 2 O molecule

Hence order of axis = 360/ (degree of ration required to get original


configuration)

= 360/180= 2

= 2 fold axis of symmetry

Hence corresponding axis is known as is known as C 2 axis

CHE509: Inorganic Chemistry – II Page 263


e.g. 2) Ammonia molecule NH 3

The order of axis is nothing but ratio of 360/degree of ration required to get
original configuration of molecule.

NH 3 molecule has 3 fold axis of rotation. i.e. we require three times rotation to
get original configuration.

After rotation about 120 ° structures (ii) is obtained and it is completely


superimposable on structure (i) i.e. we get equivalent configuration after 180 °
rotation

And we get same or original configuration after three time totation by 120 °
hence it is two fold axis of symmentry for H 2 O molecule

Hence order of axis = 360/ (degree of ration required to get original


configuration)

= 360/120= 3

= 3 fold axis of symmetry

Hence corresponding axis is known as is known as C 3 axis

SHORT ANSWER QUESTIONS WITH MODEL ANSWER 03


1] Define order of axis.
Solution: The order of axis is nothing but ratio of 360/degree of ration required
to get equivalent configuration of molecule.

CHE509: Inorganic Chemistry – II Page 264


01-04: PLANE OF SYMMETRY
If reflection of all parts of a molecule through a plane produces an
indistinguishable configuration, the plane is a plane of symmetry; the symmetry
operation is one of reflection and the symmetry element is the mirror plane (denoted
by σ). For e.g. BF 3 ,
The plane containing the molecular framework (the yellow plane shown in
Fig.) is a mirror plane.The plane lies perpendicular to the vertical principal axis and
is denoted by the symbol σh .Molecules which are in a linear, bent or planar can
always be drawn in a plane, but this plane can be labelled σh only if the molecule
possesses a Cn axis perpendicular to the plane. If the plane contains the pri ncipal
axis, it is labelled σv. A special type of σ plane which contains the principal rotation
axis, but which bisects the angle between two adjacent 2-fold axes, is labelled σd . A
square planar molecule such as XeF 4 provides an example.XeF 4 contains a C 4 axis
(the principal axis) and perpendicular to this is the σh pl ane in which the molecule
lies. Reflection planes may be vertical, horizontal or dihedral (more on σ d
later).Two successive reflections are equivalent to the identity operation (nothing is
moved) σ n = σ (n = even)

CHE509: Inorganic Chemistry – II Page 265


SOLVED PROBLEMS 01

1] Describe the plane of symmetry with suitable example

Solution: If reflection of all parts of a molecule through a plane produces an


indistinguishable configuration, the plane is a plane of symmetry; the symmetry
operation is one of reflection and the symmetry element is the mirror plane (denoted
by σ). For e.g. BF 3 ,

The plane containing the molecular framework (the yellow plane shown in
Fig.) is a mirror plane.The plane lies perpendicular to the vertical principal axis and
is denoted by the symbol σh .Molecules which are in a linear, bent or planar can
always be drawn in a plane, but this plane can be labelled σh only if the molecule
possesses a Cn axis perpendicular to the plane. If the plane contains the pri ncipal
axis, it is labelled σv. A special type of σ plane which contains the principal rotation
axis, but which bisects the angle between two adjacent 2-fold axes, is labelled σd . A
square planar molecule such as XeF 4 provides an example.XeF 4 contains a C 4 axis
(the principal axis) and perpendicular to this is the σh pl ane in which the molecule
lies. Reflection planes may be vertical, horizontal or dihedral (more on σ d
later).Two successive reflections are equivalent to the identity operation (nothing is
moved) σ n = σ (n = even)
Types of plane of symmetry:
the plane of symmetry can be divided into three types
1) Vertical plane of symmetry (σv) - The plane passing through the principal axis
and one of the subsidiary axis (If present) is called vertical plane of symmetry.
2) Horizontal plane of symmetry (σh) - The plane perpendicular to the axis is called
horizontal plane of symmetry.
3) Dihedral plane of symmetry (σd) – The plane passing through principal axis but
passing in between two subsidiary axes is called dihedral plane.

CHE509: Inorganic Chemistry – II Page 266


As this plane passes through C2 axis which is principal axis in case of H2O
molecule so this plane is a vertical plane and it is represented as σvyz. The other
isthe molecular plane passing through O and 2 H atoms in XZ plane. This plane also

SHORT ANSWER QUESTIONS WITH MODEL ANSWER 01

1] find out plane of symmetries in following molecules H 2 O

CHE509: Inorganic Chemistry – II Page 267


Solution: Two Symmetry plane of symmetries are present in water molecule
one is passing through all atoms of water molecule and one plane passing through
central oxygen atom. Both planes are parallel to principle axis so both planes are
vertical planes

2] find out plane of symmetries in the following molecules NH 3

Solution: Three Symmetry planes of symmetries are present in NH 3

Molecule each plane passing through one H atom and central N atom as shown
in figure both planes are parallel to principle axis so both planes are vertical planes

CHECK POINT 01-04


1] What is point group of H 2 O molecule

a) D 3 h

CHE509: Inorganic Chemistry – II Page 268


b) C 3 h

c) C 2 h

d) C 2 V

Solution: d

2] How meny verticle symmentry planes are possible for ammonia molecule

a) 1

b) 2

c) 3

d) 4

Solution:c

3] what is poin group of PCl 5

a) D 3 h

b) C 3 h

c) C 2 h

d) C 2 V

Solution:a

4] what is poin group of XeF4

a) D 4 h

b) C 3 h

c) C 4 h

d) C 4 V

Solution:a

SUMMARY

Use of molecular symmetry in geometry optimization has its good and bad points. The
use of symmetry reduces the number of coordinates that must be optimized. The number of
coordinates to be optimized can be determined by looking at the sets of symmetry equivalent
atoms and the symmetry elements. The problem with the use of molecular symmetry is that if

CHE509: Inorganic Chemistry – II Page 269


we start an optimization in a certain point group. Molecular symmetry in chemistry describes
the symmetry present in molecules and the classification of these molecules according to
their symmetry. Molecular symmetry is a fundamental concept in chemistry, as it can be used
to predict or explain many of a molecule's chemical properties, such as whether or not it has a
dipole moment, as well as its allowed spectroscopic transitions. To do this it is necessary to
use group theory.

KEY WORDS

Centre of symmetry, Plane of symmetry, axis of symmetry, order of axis.

REFERENCES

MOOCS

_______

YOUTUBE VIDEOS

1) https://www.youtube.com/watch?v=kgLYwxhceKM

2) https://www.youtube.com/watch?v=4HhdfXObDFM

3) https://www.youtube.com/watch?v=3vT3vsxGC9g

4) https://www.youtube.com/watch?v=R317wEBWQx8

5) https://www.youtube.com/watch?v=svDql--yD5U

WIKIPEDIA

1) https://en.wikipedia.org/wiki/Molecular_symmetry

2) https://en.wikipedia.org/wiki/Axial_symmetry

3) https://en.wikipedia.org/wiki/Symmetry

4) https://en.wikipedia.org/wiki/Molecular_symmetry#Operations

OER

1) https://chem.libretexts.org/Bookshelves/Physical_and_Theoretical_Chemistry
_Textbook_Maps/Book%3A_Symmetry_(Vallance)/01%3A_Chapters/1.02%3
A_Symmetry_Operations_and_Symmetry_Elements

CHE509: Inorganic Chemistry – II Page 270


2) https://serc.carleton.edu/NAGTWorkshops/mineralogy/xtlsymmetry/elements.
html

3) https://www2.chemistry.msu.edu/faculty/reusch/virttxtjml/symmetry/symmtry.
htm

REFERENCE BOOKS

1) Simple Approach To Group Theory In Chemistry , 2008 by Swarnalakshmi

2) Molecular Symmetry & Group Theory - A Programmed Introduction to Chemical


Applications , 2000 by A Vincent

CHE509: Inorganic Chemistry – II Page 271


CREDIT 04 -UNIT 02: PRINCIPALAXISANDSUBSIDIARYAXES
LEARNING OBJECTIVES

After successful completion of this unit, you will be able to


 Understand Symmetry of molecule
 Understand the concept of point group
 Understand the Concept of principal axis of symmetry
 Understand the Concept of subsidiary axis of symmetry

INTRODUCTION

Molecular symmetry consists of symmetry operations and symmetry elements.


Symmetry operations are operations performed on molecules that are indistinguishable in
their original position and are not superimposable. Symmetry operations are performed on
symmetry elements (points, lines, or planes). An example of a symmetric operation is her
180° rotation of a water molecule. In this case, the resulting position of the molecule is
indistinguishable from the original position. In this example, the symmetry operation is
rotation and the symmetry element is the axis of rotation.

02-01: PRINCIPAL AXIS AND SUBSIDIARY AXIS

The principle axis of a molecule is the highest order proper rotation axis. For
example, if a molecule had C 2 and C 4 axes, the C 4 is the principle axis.

For e.g. BF 3 molecule

BF 3 has trigonal planer geometry. It has one C 3 axis which is passing through B
atom as shown in following figure. (three times rotation is required to get original
geometry)

CHE509: Inorganic Chemistry – II Page 272


It also has three C 2 axis passing through B and F atom. As shown in following
figure

So BF 3 molecule has three C 2 and one C 3 axis hence it has total four axis. Out
of four axis C 3 axis has highest order i.e. 3. So C 3 axis becomes principal axis. The
C 2 axis becomes secondary axis and secondary axis is also known as subsidiary
axis.

So highest order axis is principal axis and lower order axis is secondary axis or
subsidiary axis. Secondary axis or subsidiary axis is perpendicular to the principal
axis.

The plane parallel to principal axis is known as vertical plane represented by


‘v’and the plane parallel to secondary or subsidiary axis is known as horizontal
planerepresented by ‘h’.

Both principal and subsidiary axis is also known as proper axis of symmetry

SOLVED PROBLEMS 01

1] Explain the concept of principal axis and secondary axis with suitable example.

Solution:The principle axis of a molecule is the highest order proper rotation


axis. For example, if a molecule had C 2 and C 4 axes, the C 4 is the principle axis.

For e.g. BF 3 molecule

CHE509: Inorganic Chemistry – II Page 273


BF 3 has trigonal planer geometry. It has one C 3 axis which is passing through B
atom as shown in following figure. (three times rotation is required to get original
geometry)

It also has three C 2 axis passing through B and F atom. As shown in following
figure

So BF 3 molecule has three C 2 and one C 3 axis hence it has total four axis. Out
of four axis C 3 axis has highest order i.e. 3. So C 3 axis becomes principal axis. The
C 2 axis becomes secondary axis and secondary axis is also known as subsidiary
axis.

So highest order axis is principal axis and lower order axis is secondary axis or
subsidiary axis. Secondary axis or subsidiary axis is perpendicular to the principal
axis.

The plane parallel to principal axis is known as vertical plane represented by


‘v’and the plane parallel to secondary or subsidiary axis is known as horizontal
planerepresented by ‘h’.

SHORT ANSWER QUESTIONS WITH MODEL ANSWER 01

1] Define principal axis and subsidiary axis

CHE509: Inorganic Chemistry – II Page 274


Solution: The principle axis of a molecule is the highest order proper rotation
axis. And the axis having less order than principal axis is known as secondary axis
or subsidiary axis. Secondary axis or subsidiary axis is perpendicular to the
principal axis.

2] Define Vertical and Horizontal plane of symmetry

Solution: The plane parallel to principal axis is known as vertical plane


represented by ‘v’and the plane parallel to secondary or subsidiary axis is known
as horizontal planerepresented by ‘h’.

02-02: IMPROPER AXIS OF SYMMETRY

The improper rotational axis is a proper rotation followed by a reflection in a


mirror plane perpendicular to the proper rotation axis to get original configuration is
known as improper axis of rotation. Represented by ‘Sn’

First molecule rotates about imaginary axis and through some angle and we
will get nonequivalent configuration from original configuration.

Then reflect molecule (nonequivalent configuration) plane perpendicular to the


rotational axis to get equivalent geometry this phenomenon of reflection after
rotation is known as improper axis of rotation.

e.g. Some molecules has improper axis of symmetry for e.g. C 2 Br 2 Cl 2

SOLVED PROBLEMS 02

1] Describe the improper axis of symmetry with suitable example.

CHE509: Inorganic Chemistry – II Page 275


Solution:The improper rotational axis is a proper rotation followed by a
reflection in a mirror plane perpendicular to the proper rotation axis to get original
configuration is known as improper axis of rotation. represented by ‘ Sn’

First molecule rotate about imaginary axis and through some angle and we will
get nonequivalent configuration from original configuration.

Then reflect molecule (nonequivalent configuration) plane perpendicular to the


rotational axis to get equivalent geometry this phenomenon of reflection after
rotation is known as improper axis of rotation.

e.g. Some molecules has improper axis of symmetry for e.g. C 2 Br 2 Cl 2

SHORT ANSWER QUESTIONS WITH MODEL ANSWER 02

1] Define improper axis of symmetry

Solution: The improper rotational axis is a proper rotation followed by a


reflection in a mirror plane perpendicular to the proper rotation axis to get original
configuration is known as improper axis of rotation. Represented by ‘ Sn’

02-03: ASSIGNING POINTGROUPS


A point cloud represents all the symmetry operations that can be performed on
a molecule that result in a conformation that is indistinguishable from the original
conformation. Point groups are used in group theory (mathematical analysis of
groups) to determine properties such as the molecular orbitals of molecules.

CHE509: Inorganic Chemistry – II Page 276


A point cloud contains all the symmetry operations that can be performed on a
particular molecule, but it is not necessary to identify all of these operations to
determine the molecule's overall point cloud. Instead, the molecu lar point cloud can
be determined by following a series of steps that analyze for the presence (or
absence) of specific symmetry elements.

Steps for assigning a molecule's point group:

1) Determine if the molecule is of high or low symmetry

2) If not, find the highest order rotation axis, Cn.

3) Determine whether the molecule has any C 2 axes perpendicular to the


principal Cn axis. If so, then there are n such C 2 axes, and the molecule is in
the D set of point groups. If not, it is in either the C set of point groups.

4) Determine whether the molecule has a horizontal mirror plane (σh)


perpendicular to the principal Cn axis. If so, the molecule is either in the Cnh
or Dnh set of point groups.

5) Determine whether the molecule has a vertical mirror plane (σv) containing
the principal Cn axis. If so, the molecule is either in the Cnv or Dnd set of
point groups. If not, and if the molecule has n perpendicular C2 axes, then it
is part of the Dn set of point groups.

6) Determine whether there is an improper rotation axis, S2n, coll inear with the
principal Cn axis. If so, the molecule is in the S2n point group. If not, the
molecule is in the Cn point group.

CHE509: Inorganic Chemistry – II Page 277


SOLVED PROBLEMS 03

1] Describe the steps to determine the point group of H 2 O molecule.

Solution: Determine if the molecule is of high or low symmetry

1) If not, find the highest order rotation axis, Cn.

2) Determine whether the molecule has any C 2 axes perpendicular to the


principal Cn axis. If so, then there are n such C 2 axes, and the molecule is in the D
set of point groups. If not, it is in either the C set of point groups.

3) Determine whether the molecule has a horizontal mirror plane (σh)


perpendicular to the principal Cn axis. If so, the molecule is either in the Cnh or
Dnh set of point groups.

4) Determine whether the molecule has a vertical mirror plane (σv) containing
the principal Cn axis. If so, the molecule is either in the Cnv or Dnd set of point
groups. If not, and if the molecule has n perpendicular C 2 axes, then it is part of
the Dn set of point groups.

CHE509: Inorganic Chemistry – II Page 278


5) Determine whether there is an improper rotation axis, S 2 n, collinear with the
principal Cn axis. If so, the molecule is in the S 2 n point group. If not, the molecule
is in the Cn point group.
SHORT ANSWER QUESTIONS WITH MODEL ANSWER 03
1] Give schematic flowsheet of determination of point group
Solution:

Abbreviations: σh = horizontal plane, σv = vertical plane, d = diagonal plane,


S = improper axis. C = only principal axis, D = presence of secondary axis with
principal axis.

02-04: POINT GROUP DETERMINATION EXAMPLES

Point group of H 2 O
For the point group determination, we have to follow the steps
1) The geometry of molecule
For H 2 O geometry is angular

2) Find out Principal axis (Cn)

CHE509: Inorganic Chemistry – II Page 279


For H 2 O molecule axis of symmetry is present and it has only one axis of
symmetry (imaginary axis passing through oxygen atom) having order 2 i.e. C 2

So from above data point group becomes C 2

3) Find out subsidiary axis (D)

For H 2 O molecule there is no any secondary or subsidiary axis found


perpendicular to the principal axis.

So from above data point group remains C 2

4) Find out vertical plane of symmetry ( σ v)

For H 2 O molecule, there is a plane of symmetry observed and this plane is parallel
to principal axis i.e. vertical plane

Two planes are observed as follows

One plane passing through only Oxygen atom ( σ 2 )

Other plane passing through plane of molecule i.e. passing through all atoms ( σ 1 )

Both planes are parallel to principal axis C 2 hence all are vertical planes. (V)

CHE509: Inorganic Chemistry – II Page 280


5) Find out horizontal plane of symmetry ( σ v)

Hence, from all above data overall point group of H 2 O is C 2 V

Point group of PCl 5

For the point group determination, we have to follow the steps

1) The geometry of molecule

For PCl 5 geometry is trigonal bipyramidal

2) Find out Principal axis (Cn)

3) For PCl 5 molecule axis of symmetry is present and it has total four axis, one
C 3 having order 3 passing through P and two axial Cl atoms and there are three C 2
passing through Cl and P atom having order 2 so C 3 is principal axis

Therefore, from above data point group becomes C 3

CHE509: Inorganic Chemistry – II Page 281


4) Find out subsidiary axis (D)

For PCl 5 molecule there are three secondary or subsidiary axis found perpendicular
to the principal axis. So C is converted in to D

from above data point group of PCl 5 is D 3

5) Find out vertical plane of symmetry ( σ v)

For PCl 5 molecule, there are three vertical planes passes through P, Two axial Cl
and one equatorial Cl atom.

Hence, above data point group becomes D 3 V

6) Find out horizontal plane of symmetry

There is one horizontal plane is present passing through three equatoria l C atoms
and P atom. As shown in following figure

We know that if both vertical and horizontal planes are present we have to use h

Hence, from all above data overall point group of PCl 5 is D 3 h

Point group of NH 3

For the point group determination, we have to follow the steps

1) The geometry of molecule

For NH 3 geometry is trigonal pyramidal

CHE509: Inorganic Chemistry – II Page 282


2) Find out Principal axis (Cn)

For NH 3 molecule axis of symmetry is present and it only one axis C 3 having order
3 passing through Natom

Therefore, from above data point group becomes C 3

3) Find out subsidiary axis (D)

For NH 3 molecule there is no any axis perpendicular to principal axis

From above data point group of NH 3 is C 3

4) Find out vertical plane of symmetry ( σ v)

For NH 3 molecule, there are three vertical planes passes through N, and
equatorial H atoms. As shown in figure σ va, σ vb, and σ vc

Hence, above data point group becomes C 3V

5) Find out horizontal plane of symmetry

For ammonia molecule there is no horizontal plane possible which divide


ammonia in to two equal parts so we cannot use term h here

Hence, from all above data overall point group of NH 3 is C 3 V.

SOLVED PROBLEMS 04

1] Find out the symmetry elements and point group for H 2 O

CHE509: Inorganic Chemistry – II Page 283


Solution: Point group of H 2 O

For the point group determination, we have to follow the steps

1) The geometry of molecule

For H 2 O geometry is angular

2) Find out Principal axis (Cn)

For H 2 O molecule axis of symmetry is present and it has only one axis of
symmetry (imaginary axis passing through oxygen atom) having order 2 i.e. C 2

So from above data point group becomes C 2

3) Find out subsidiary axis (D)

For H 2 O molecule there is no any secondary or subsidiary axis found


perpendicular to the principal axis.

So from above data point group remains C 2

4) Find out vertical plane of symmetry ( σ v)

For H 2 O molecule, there is a plane of symmetry observed and this plane is


parallel to principal axis i.e. vertical plane

Two planes are observed as follows

CHE509: Inorganic Chemistry – II Page 284


One plane passing through only Oxygen atom ( σ 2 )

Other plane passing through plane of molecule i.e. passing through all atoms ( σ 1 )

Both planes are parallel to principal axis C 2 hence all are vertical planes. (V)

5) Find out horizontal plane of symmetry ( σ v)

Hence, from all above data overall point group of H 2 O is C 2 V

2] Find out the symmetry elements and point group for PCl 5

Solution: Point group of PCl 5

For the point group determination, we have to follow the steps

1) The geometry of molecule

For PCl 5 geometry is trigonal bipyramidal

2) Find out Principal axis (Cn)

CHE509: Inorganic Chemistry – II Page 285


3) For PCl 5 molecule axis of symmetry is present and it has total four axis, one
C 3 having order 3 passing through P and two axial Cl atoms and there are three
C 2 passing through Cl and P atom having order 2 so C 3 is principal axis

Therefore, from above data point group becomes C 3

4) Find out subsidiary axis (D)

For PCl 5 molecule there are three secondary or subsidiary axis found
perpendicular to the principal axis. So C is converted in to D

from above data point group of PCl 5 is D 3

5) Find out vertical plane of symmetry ( σ v)

For PCl 5 molecule, there are three vertical planes passes through P, Two axial Cl
and one equatorial Cl atom.

Hence, above data point group becomes D 3 V

3] Find out the symmetry elements and point group for NH 3

CHE509: Inorganic Chemistry – II Page 286


Solution: Point group of NH 3

For the point group determination, we have to follow the steps

1) The geometry of molecule

For NH 3 geometry is trigonal pyramidal

2) Find out Principal axis (Cn)

For NH 3 molecule axis of symmetry is present and it only one axis C 3 having
order 3 passing through Natom

Therefore, from above data point group becomes C 3

3) Find out subsidiary axis (D)

For NH 3 molecule there is no any axis perpendicular to principal axis

from above data point group of NH 3 is C 3

4) Find out vertical plane of symmetry ( σ v)

For NH 3 molecule, there are three vertical planes passes through N, and
equatorial H atoms. As shown in figure σ va, σ vb, and σ vc

Hence, above data point group becomes C 3V

5) Find out horizontal plane of symmetry

For ammonia molecule there is no horizontal plane possible which divide


ammonia in to two equal parts so we cannot use term h here

Hence, from all above data overall point group of NH 3 is C 3 V.

CHE509: Inorganic Chemistry – II Page 287


CHECK POINT 01-04

1] What is point group for the BF 3 ?

a) C 3 V

b) C 3 h

c) D 3 V

d) D 3 h

Ans: d (trigonal planer geomentry)

2] What is point group for the H 2 O?

a) C 2 V

b) C 2 h

c) D 2 V

d) D 2 h

Ans: a (angular geometry)

3] What is point group for the CO 2 ?

a) C ∞ V

b) C ∞ h

c) D ∞ V

d) D 2 h

Ans: c (liner geometry and for liner geometry ∞ time rotation is possible to get
equivalent configuration)

4] What is point group for the H 2 O 2 ?

a) C 3

CHE509: Inorganic Chemistry – II Page 288


b) C 2

c) C2V

d) C3V

Ans: b (open book like geometry, no any plane of symmetry hence h and v is not
possible, ony c 2 axis is there)

SUMMARY

Each molecule has a set of symmetry operations that describe the overall
symmetry of the molecule. This sequence of operations defines a group of
molecules. A group is a finite or infinite set of elements with binary operations
(called group operations) that together satisfy four fundamental properties: closure,
associativity, identity, and inverse. The operation in which the group is defined is
often called the "group operation", and the set is called the group "under" that
operation.

KEY WORDS
Point group, Symmetry planes, Symmetry axis, Centre of symmetry, B

REFERENCES

MOOCS

______

YOUTUBE VIDEOS

1) https://www.youtube.com/watch?v=ggyB8cP0mv8

CHE509: Inorganic Chemistry – II Page 289


2) https://www.youtube.com/watch?v=4ENNnuJz9Kc

3) https://www.youtube.com/watch?v=6oi8pCEGUWA

4) https://www.youtube.com/watch?v=yO8v0nszUz8

5) https://www.youtube.com/watch?v=yOsHtE1tvxU

WIKIPEDIA

1) https://en.wikipedia.org/wiki/Group_theory

2) https://en.wikipedia.org/wiki/Group_theory#Main_classes_of_groups

OER

1) chem.libretexts.org/Bookshelves/Physical_and_Theoretical_Chemistry_Textb
ook_Maps/Quantum_Tutorials_(Rioux)/06%3A_Group_Theory_with_Mathca
d/6.01%3A_Group_Theory_Principles_Applied_to_H2O

2) https://www.staff.ncl.ac.uk/j.p.goss/symmetry/Water/water0.html

3) http://mpbou.edu.in/slm/mscche1p4.pdf

REFERENCE BOOKS

1) Molecular Symmetry & Group Theory - A Programmed Introduction to


Chemical Applications 2e by A Vincent

2) Group Theory in Chemistry and Spectroscopy: A Simple Guide to Advanced


Usage (Dover Books on Chemistry) Illustrated Edition by Boris S. Tsukerblat

CHE509: Inorganic Chemistry – II Page 290


CREDIT 04 -UNIT 03: IONIC COMPOUNDS
LEARNING OBJECTIVES
After successful completion of this unit, you will be able to
 Understand Concept of radius ratio
 Understand the concept of close packing
 Apply the concept of radius ratio rules
 Understand the structure of ionic solids NaCl and CsCl

INTRODUCTION

Gilbert Newton Lewis (1875–1946)

The subject of chemical bonding is at the heart of chemistry. In 1916 Gilbert


Newton Lewis published his seminal paper suggesting that a chemical bond is a pair
of electrons shared by two atoms. Ionic compounds can form many types of crystal
structures and tend to be more complex than the close-packed schemes presented in
the previous section. There are multiple reasons for this. First, the charges of the
ions should not be equal, but should be in a neutral relationship forming a lattice that
maximizes the attraction between opposite charges while minimizing repulsion
between like charges. Second, anions and cations are different entities and can have
completely different volumes. In this case, the structure is often based on large ions
(usually anions) and smaller ions that fit into "holes or cavities". Ultimately, many

CHE509: Inorganic Chemistry – II Page 291


ionic compounds form hydrated salts, trapping water in the pores and cavities of the
crystal lattice.
03-01: PROPERTIES OF IONIC COMPOUNDS

1] Physical properties of ionic compounds

Due to the strong attraction between positive and negative ions, ionic
compounds are solid and not easily broken. They are considered brittle because they
usually break under pressure.

2] Melting and boiling points of ionic compounds

A large amount of energy is required to break the ionic bond between atoms
due to the presence of electrostatic attraction between ions. Therefore, ionic
compounds have high melting and boiling points.

3] The solubility of ionic compounds

Ionic compounds are generally soluble in polar solvents such as water, but tend
to be less soluble in non-polar solvents such as gasoline.

4] Conduction of Electricity

Ionic compounds do not conduct electricity when they are solid, but they
conduct electricity well when dissolved. Conduction involves the flow of charge
from one point to another. Ionic compounds do not conduct electricity because the
movement of ions in solids is impossible. On the other hand, in the molten state,
ionic compounds conduct electricity. This is because the electrostatic attra ction
between ions is overcome by the released heat.

5] They are good insulators

Ionic solids conduct electricity in the molten state or in aqueous solution, but
they are poor conductors of electricity because the ions are so strongly bound to
each other.

6] They have higher enthalpies of fusion and vaporization than molecular


compounds

Just as ionic compounds have high melting and boiling points, they typically
have enthalpies of melting and enthalpies of vaporization 10 to 100 times higher
than most molecular compounds. The enthalpy of fusion is the heat required to melt

CHE509: Inorganic Chemistry – II Page 292


1 mole of solid under constant pressure. The enthalpy of vaporization is the heat
required to evaporate one mole of a liquid compound under constant pressure.
SOLVED PROBLEMS 01

1] Give important properties of ionic compounds

Solution: 1] Physical properties of ionic compounds

Due to the strong attraction between positive and negative ions, ionic
compounds are solid and not easily broken. They are considered brittle because they
usually break under pressure.

2] Melting and boiling points of ionic compounds

A large amount of energy is required to break the ionic bond between atoms
due to the presence of electrostatic attraction between ions. Therefore, ionic
compounds have high melting and boiling points.

3] The solubility of ionic compounds

Ionic compounds are generally soluble in polar solvents such as water, but tend
to be less soluble in non-polar solvents such as gasoline.

4] Conduction of Electricity

Ionic compounds do not conduct electricity when they are solid, but they
conduct electricity well when dissolved. Conduction involves the flow of charge
from one point to another. Ionic compounds do not conduct electricity because the
movement of ions in solids is impossible. On the other h and, in the molten state,
ionic compounds conduct electricity. This is because the electrostatic attraction
between ions is overcome by the released heat.

5] They are good insulators

Ionic solids conduct electricity in the molten state or in aqueous solution, but
they are poor conductors of electricity because the ions are so strongly bound to
each other.

6] They have higher enthalpies of fusion and vaporization than molecular


compounds

Just as ionic compounds have high melting and boiling points, the y typically
have enthalpies of melting and enthalpies of vaporization 10 to 100 times higher

CHE509: Inorganic Chemistry – II Page 293


than most molecular compounds. The enthalpy of fusion is the heat required to melt
1 mole of solid under constant pressure. The enthalpy of vaporization is the hea t
required to evaporate one mole of a liquid compound under constant pressure.
SHORT ANSWER QUESTIONS WITH MODEL ANSWER 01
1] Which is an ionic compound?
Solution: Ionic compounds are ion compounds. These ions are atoms that gain
or lose electrons, resulting in a net positive or negative charge. Metals tend to lose
electrons, so they have a net positive charge and become cations. Non -metals tend to
gain electrons, creating a net negative charge of anions.
03-02: THE RADIUS RATIO RULE

The cubic close-packed arrangements of anions are having tetrahedral and


octahedral holes of different sizes. Therefore, cations will only occupy cavities if
there is enough space to accommodate them. This prediction of whether an ion can
hold cations can be based on the radius ratio.

Ionic crystals are composed of many cations and anions. It is found that the
anions are larger and surround the smaller cations. They are arranged in space so
that the anions and cations are in contact to create maximum stabili ty.

This stability of the ionic crystals can be explained on the basis of radius ratio.
Therefore, radius ratio is the ratio of cation to the ratio of an anion. Here, Ratio of
cation= r, Ratio of anion = R. Thus, Radius ratio = (r/R). Limiting radius ratio helps
in expressing the range of radius ratio.

The radius ratio refers to the ratio of the smaller ionic radius (cation) to
the larger ionic radius (anion). Hence the radius ratio ρ = rs/rl.

This rule helps determine the arrangement of ions in various types of crystal
structures. It also helps determine the stability of ionic crystal structures. For
example, large cations fill large cavities such as cubic sites, and small cations fill
small cavities such as tetrahedral sites. It is also possible to pred ict the coordination
number of each compound. The radius ratio rule is therefore useful in determining
the structure of ionic solids.
Examples of Radius Ratio Rule
The ratio of ion radii can affect the arrangement of ions within the crystal.
Furthermore, the boundary ratio must be greater than 0.414 (radius ratio greater than

CHE509: Inorganic Chemistry – II Page 294


0.414) to match the octahedral arrangement of the anions. In this configuration, the
cation can accommodate 6 anions.
However, when the radius ratio is between 0.225 and 0.414, it fit s into the
tetrahedral cavities of the crystal lattice and favors tetrahedral coordination, and
above 0.414 favors octahedral coordination. For example, considering the zinc
sulfide ion, the radius ratio is

Therefore, zinc ions favor tetrahedral cavities within the dense lattice of sulfide
ions. However, for larger cations such as cesium, the radius ratio is greater than the
coordination number limit of 6. Therefore, the cesium ion fits in a cubic site,
increasing the coordination number of the chloride i on lattice to eight.

The table below shows the relationship between the radius ratio (boundary
ratio) and the coordination number.

Effect of Difference in Size of Ions on Arrangement


The three figures below illustrate how differences in ion size affect ion placement
and stability.

CHE509: Inorganic Chemistry – II Page 295


Figure 1 shows that the M+ ions are in contact with the X - ions, and the fit of
the first layer is "just right." However, Figure 2 shows that the M+ ions are in
contact with the X- ions. The fit of the first device layer is therefore poor.
Furthermore, as the cations move closer to the anions, the array compaction
increases, leading to mutual repulsion.
This will further increase the energy of the system. Furthermore, if the ions try
to hold each other apart then it will not be near to the opposite M + ion. Figure 3
represents the arrangement of M + ion and X – ions where M + ion is too large.
Therefore, the X – will be far away and not feel the X – – X – repulsion.
Therefore, in cases like Fig. 2 and Fig. 3, the crystalline packing of MX is
different. So, as an example of a compound, think of Figure 1 as a lattice of NaCl.
We can fit Figure 3 to the structure of CsCl, where the M+ ions are large and can
accommodate more X- ions.
Similarly, the M + ions in Figure 2 are so small that they can only accommodate
four negative ions, and compounds like ZnS are an example of such a case.

SOLVED PROBLEMS 02

1] If a solid “A + B – ” has a structure similar to NaCl. Consider the radius of anion as


250 pm. Find the ideal radius of the cation in the structure. Is it possible to fit a
cation C + of radius 180 pm in the tetrahedral site of the structure “A + B – ”? Explain
your answer

CHE509: Inorganic Chemistry – II Page 296


Solution: If the A+B– structure resembles a Na+Cl– ion, we find that six Cl–
ions surround Na+ and vice versa. Therefore, Na + ions fit in the octahedral gap.
Therefore, the limiting ratio of octahedral sites is 0.414.

Thus, limiting radius ratio= r/R= 0.414

From the question, R=250 pm

Therefore, r=0.414R= 0.414 × 250pm

Hence, r= 103.5pm

So the ideal radius ration of cation will be 103.5pm or A+ = 103.5 pm

From the above table, we know r/R for a tetrahedral site is 0.225

Therefore, r/R= 0.225 Or, r= 0.225R = 0.225 × 250pm = 56.25 pm

Therefore, the ideal radius of a cation in the given st ructure is 56.25 pm at the
tetrahedral site. However, we know that C + has a radius of 180 pm. This means that
the radius of C + is much larger than 56.25 o'clock. Therefore, the cation C + cannot
be aligned with the tetrahedral position.

2] What are the radius ratio rules? Explain.

Solution: The cubic close-packed arrangement of anions are having tetrahedral


and octahedral holes of different sizes. Therefore, cations will only occupy cavities
if there is enough space to accommodate them. This prediction of w hether an ion can
hold cations can be based on the radius ratio.

Ionic crystals are composed of many cations and anions. It is found that the
anions are larger and surround the smaller cations. They are arranged in space so
that the anions and cations are in contact to create maximum stability.

This stability of the ionic crystals can be explained on the basis of radius ratio.
Therefore, radius ratio is the ratio of cation to the ratio of an anion. Here, Ratio of
cation= r, Ratio of anion = R. Thus, Radius ratio = (r/R). Limiting radius ratio helps
in expressing the range of radius ratio.

The radius ratio refers to the ratio of the smaller ionic radius (cation) to
the larger ionic radius (anion). Hence the radius ratio ρ = rs/rl.
This rule helps determine the arrangement of ions in various types of crystal
structures. It also helps determine the stability of ionic crystal structures. For

CHE509: Inorganic Chemistry – II Page 297


example, large cations fill large cavities such as cubic sites, and small cations fill
small cavities such as tetrahedral sites. It is also possible to predict the coordination
number of each compound. The radius ratio rule is therefore useful in determining
the structure of ionic solids.
Examples of Radius Ratio Rule
The ratio of ion radii can affect the arrangement of ions w ithin the crystal.
Furthermore, the boundary ratio must be greater than 0.414 (radius ratio greater than
0.414) to match the octahedral arrangement of the anions. In this configuration, the
cation can accommodate 6 anions.
However, when the radius ratio is between 0.225 and 0.414, it fits into the
tetrahedral cavities of the crystal lattice and favors tetrahedral coordination, and
above 0.414 favors octahedral coordination. For example, considering the zinc
sulfide ion, the radius ratio is

Therefore, zinc ions favor tetrahedral cavities within the dense lattice of sulfide
ions. However, for larger cations such as cesium, the radius ratio is greater than the
coordination number limit of 6. Therefore, the cesium ion fits in a cubic site,
increasing the coordination number of the chloride ion lattice to eight.

The table below shows the relationship between the radius ratio (boundary
ratio) and the coordination number.
The three figures below illustrate how differences in ion size affect ion placement
and stability.

CHE509: Inorganic Chemistry – II Page 298


Figure 1 shows that the M+ ions are in contact with the X - ions, and the fit of
the first layer is "just right." However, Figure 2 shows that the M+ ions are in
contact with the X- ions. The fit of the first device layer is therefore poor.
Furthermore, as the cations move closer to the anions, the array compaction
increases, leading to mutual repulsion.
This will further increase the energy of the system. Furthermore, if the ions try
to hold each other apart then it will not be near to the opp osite M + ion. Figure 3
represents the arrangement of M + ion and X – ions where M + ion is too large.
Therefore, the X – will be far away and not feel the X – – X – repulsion.
Therefore, in cases like Fig. 2 and Fig. 3, the crystalline packing of MX is
different. So, as an example of a compound, think of Figure 1 as a lattice of NaCl.
We can fit Figure 3 to the structure of CsCl, where the M+ ions are large and can
accommodate more X- ions.
Similarly, the M + ions in Figure 2 are so small that they can only accomm odate
four negative ions, and compounds like ZnS are an example of such a case.

SHORT ANSWER QUESTIONS WITH MODEL ANSWER 02


1] Define radius ratio.
Solution: The radius ratio refers to the ratio of the smaller ionic radius (cation)
to the larger ionic radius (anion). Hence the radius ratio ρ = rs/rl.

03-03: CLOSE PACKING STRUCTURES OF IONIC SOLID SODIUM CHLORIDE


The structure of ionic solids is determined by how cations and anions are
bound together. In general, one of the ions adopts a standard packing structure, like
metal atoms in metallic solids. The counterion fits into the hole or space between
these ions. It is very common for anions to form densely packed structures and for
cations to find space in the resulting holes, but the opposite is also tru e.

In solid sodium chloride, each ion is surrounded by six ions of opposite charge,
as expected from electrostatic considerations. This can be represented as a face
centered cubic (fcc) lattice where the base is his two atoms or two. The first atom is
at each lattice point and the second atom is halfway between the lattice points along
the edge of the fcc unit cell.

CHE509: Inorganic Chemistry – II Page 299


The NaCl unit cell consists of Na+ and Cl- ions. There are four types of sites,
unique central sites, front sites, edge sites, and corner sit es, which are used to
determine the number of Na+ and Cl ions in the NaCl unit cell. Counting the number
of ions, the corner site is shared by seven other unit cells. So one corner is 1/8 th of
an ion. Similar events occur on the front and edge sides. For f ace sites, it is shared
by one other unit cell, and for edge sites, the ion is shared by three other unit cells.
NaCl is a face-centered cubic unit cell containing four cations and four anions. This
can be shown by counting the number of ions and multiplyi ng their positions.

Na + : (1 at center) + (12edge×1/4)= 4 sodium ions total per cell


-
Cl : (4 at face×1/2) + (8corner×1/8) =4chloride ions total per cell

Each ion in this lattice has 6 ions of the opposite species as its nearest
neighbors and 12 ions of the same species as its second closest neighbors. There are
many ionic compounds that adopt this structure, including all other halides of Na,
Li, K, and Rb.

SOLVED PROBLEMS 03

1] Describe structure of NaCl and find out number of atoms per unit cell.

Solution: The structure of ionic solids is determined by how cations and anions
are bound together. In general, one of the ions adopts a standard packing structure,
like metal atoms in metallic solids. The counterion fits into the hole or space
between these ions. It is very common for anions to form densely packed structures
and for cations to find space in the resulting holes, but the opposite is also true.

In solid sodium chloride, each ion is surrounded by six ions of opposite charge,
as expected from electrostatic considerations. This can be represented as a face
centered cubic (fcc) lattice where the base is his two atoms or two. The first atom is

CHE509: Inorganic Chemistry – II Page 300


at each lattice point and the second atom is halfway between the lattice points along
the edge of the fcc unit cell.

The NaCl unit cell consists of Na+ and Cl- ions. There are four types of sites,
unique central sites, front sites, edge sites, and corner sites, which are used to
determine the number of Na+ and Cl ions in the NaCl unit cell. Counting the number
of ions, the corner site is shared by seven other unit cells. So one corner is 1/8 th of
an ion. Similar events occur on the front and edge sides. For face sites, it is shared
by one other unit cell, and for edge sites, the ion is shared by three other unit cells.
NaCl is a face-centered cubic unit cell containing four cations and four anions. This
can be shown by counting the number of ions and multiplying their positions.

Na + : (1 at center) + (12edge×1/4)= 4 sodium ions total per cell


Cl - : (4 at face×1/2) + (8corner×1/8) =4chloride ions total per cell

Each ion in this lattice has 6 ions of the opposite species as its nearest
neighbors and 12 ions of the same species as its second closest neighbors. There are
many ionic compounds that has structure like NaCl, including all other halides of
Na, Li, K, and Rb.

SHORT ANSWER QUESTIONS WITH MODEL ANSWER 03

1] How many Chlorine atoms are in NaCl Cubic cell ?

Solution: Cl - : (4 at face×1/2) + (8corner×1/8) =4chloride ions total per


cell

2] How many Sodium atoms are in NaCl Cubic cell ?

Solution: Na + : (1 at center) + (12edge×1/4)= 4 sodium ions total per cell

CHE509: Inorganic Chemistry – II Page 301


03-04: CLOSE PACKING STRUCTURES OF IONIC SOLID CAESIUM CHLORIDE
This page describes the molecular structure of cesium chloride (CsCl), a white
hygroscopic solid with a mass of 168.36 g/mol. Cesium chloride is a type of unit
cell, often erroneously called body-centered cubic. This misconception is likely
because the unit cell has a central atom, but CsCl is actually a loosely packed
structural type.

CsCl crystallizes in a basic cubic lattice. This means that a cubic unit cell has
nodes only at its corners. The structure of CsCl can be viewed as two
interpenetrating cubes, one of Cs + and one of Cl - . Do not touch ions. When touched,
a repulsion occurs between anions and cations. Some may confus e the structural
types of CsCl and NaCl, but the two are actually different. CsCl is more stable than
NaCl because it produces more stable crystals and releases more energy.

Anions and cations are similar in size. Each Cs + is surrounded by 8 Cl - at its


cube corners, and each Cl - is also surrounded by 8 Cs + at its cube corners. The cation
is in the center of the anion cube and the anion is in the center of the cation cube.
CsCl has one atom. To determine this, we are given the following formula.

8 Corners of a given atom x 1/8 of the given atom's unit cell = 1 atom

Cesium chloride is used in centrifugation, a process that uses centrifugal force


to separate mixtures based on molecular density. It is also used in the production of
conductive glass. Radioactive CsCl is used in some types of radiation therapy in
cancer patients.

SOLVED PROBLEMS 04

1] Explain the crystal arrengment in CsCl molecule

Solution: This page describes the molecular structure of cesium chloride (CsCl), a
white hygroscopic solid with a mass of 168.36 g/mol. Cesium chloride is a type of

CHE509: Inorganic Chemistry – II Page 302


unit cell, often erroneously called body-centered cubic. This misconception is likely
because the unit cell has a central atom, but CsCl is actually a loosely packed
structural type.

CsCl crystallizes in a basic cubic lattice. This means that a cubic unit cell has
nodes only at its corners. The structure of CsCl can be viewed as two
interpenetrating cubes, one of Cs + and one of Cl - . Do not touch ions. When touched,
repulsion occurs between anions and cations. Some may confuse the structural types
of CsCl and NaCl, but the two are actually different. CsCl is more stable than NaCl
because it produces more stable crystals and releases more energy.

Anions and cations are similar in size. Each Cs + is surrounded by 8 Cl - at its


cube corners, and each Cl - is also surrounded by 8 Cs + at its cube corners. The cation
is in the center of the anion cube and the anion is in the center of the cation cube.
CsCl has one atom. To determine this, we are given the following formula.
8 Corners of a given atom x 1/8 of the given atom's unit cell = 1 atom
Cesium chloride is used in centrifugation, a process that uses centrifugal force
to separate mixtures based on molecular density. It is also used in the production of
conductive glass. Radioactive CsCl is used in some types of radiation therapy in
cancer patients.
2] How many Sodium atoms are in CsCl Cubic cell?
Solution: 8 Corners of a given atom x 1/8 of the given atom's unit cell = 1 atom
CHECK POINT 01-04
1] How many Sodium atoms are in NaCl Cubic cell?

a) 1

b) 2

c) 3

d) 4

CHE509: Inorganic Chemistry – II Page 303


Solution: d

2] How many Sodium atoms are surrounded by Cl atoms NaCl Cubic cell?

a) 2

b) 4

c) 6

d) 8

Solution: c

3] What is coordination number for radious ratio 0.74?

a) 2

b) 4

c) 6

d) 8

Solution: d

4] What is coordination number for radious ratio 0.155?

a) 2

b) 4

c) 6

d) 8

Solution: a

_______
SUMMARY
It should be remembered from molecular dynamics theory that molecules in
solids do not move in the same way as molecules in liquids and gases. Rather than
moving, solid molecules simply vibrate and rotate in place. Solids are generally held
together by ionic or strong covalent bonds, and the attractive forces between atoms,
ions, or molecules within solids are very strong. In fact, these forces a re so strong
that particles within solids are held in fixed positions with little freedom of
movement. A solid has a definite shape and a definite volume and is never

CHE509: Inorganic Chemistry – II Page 304


compressible. There are two main categories of solids: crystalline solids and
amorphous solids. A crystalline solid is one in which the atoms, ions, or molecules
that make up the solid are arranged in a regular, well -defined arrangement. The
smallest repeating pattern in a crystalline solid is known as a unit cell, and a unit
cell is like the bricks in a wall - they are all identical and repeating. called a solid.
Amorphous solids are less regular in their structure. Their molecules are close
together and have little freedom of movement, but they are not arranged in a regular
order as in crystalline solids.Common examples of this type of solid are glass and
plastic.

KEY WORDS
Radius ratio, Close packs solids, Structure of NaCl, Structure of CsCl.
REFERENCES
MOOCS
--
YOUTUBE VIDEOS
1) https://www.youtube.com/watch?v=4BDRuFiUetQ
2) https://www.youtube.com/watch?v=scIG1ffzXiI
3) https://www.youtube.com/watch?v=xE2q8tjH764
4) https://www.youtube.com/watch?v=z8w2eYbsgU8
5) https://www.youtube.com/watch?v=lYwX-lv6rBc

WIKIPEDIA
1) https://en.wikipedia.org/wiki/Cation-anion_radius_ratio
2) https://en.wikipedia.org/wiki/Crystal_structure

OER
1) https://chem.libretexts.org/Bookshelves/Inorganic_Chemistry/Book%3A
_Introduction_to_Inorganic_Chemistry_(Wikibook)/09%3A_Ionic_and_
Covalent_Solids_-
_Energetics/9.01%3A_Ionic_Radii_and_Radius_Ratios
2) https://www.toppr.com/guides/chemistry/the-solid-state/radius-ratio-
rule/
3) https://chemistnotes.com/inorganic/radius-ratio-the-radius-ratio-rule/

CHE509: Inorganic Chemistry – II Page 305


REFERENCE BOOKS
1) Solid State Chemistry by M.G. Arora
2) Solid State Chemistry: An Introduction, 2nd Edition by Lesley E. Smart and
Elaine A. Moore
3) Inorganic and Solid State Chemistry by Glen E. Rodgers

CHE509: Inorganic Chemistry – II Page 306


CREDIT 04 -UNIT 04: LATTICE ENERGY OF IONIC SOLIDS

LEARNING OBJECTIVES

After successful completion of this unit, you will be able to


 Understand the concept of lattice energy
 Understand the concept of Born-Haber cycle
 Apply the Born-Haber cycle to determine lattice energy
 Understand the concept of Hydrogen bonding
 Understand the hydrogen bonding in water

INTRODUCTION

Most of the rocks and minerals that surround us are made up of ions held
together by ionic bonds, the electrical attraction between oppositely charged ions.
Look closely at the salt crystals. what does it look like? take a closer look. Table salt
is composed of Na + and Cl-, which combine in ionic bonds to form NaCl. But can
you see NaCl? You can see that the numbers Na and Cl have a clean lattice structure.
Consider the Lewis point structures of Na and Cl. They are both originally neutral
with 1 and 7 valence electrons. Sodium, being a metal, easily loses its electrons and
becomes a positively charged cation. Non-metallic chlorine willingly accepts
electrons and becomes negatively charged. Sodium transfers its electrons to chlorine ,
making both happy. The strength of ionic compounds is measured by the so-called
lattice energy. It is the energy released when one mole of ionic compound is formed.
That is, when individual ions of a compound come together to form a crystal lattice,
they need less energy to stay together, so they give it up, and the energy released is
called lattice energy The binding force between oppositely charged ions is strongest
when the ions are small. This is because the valence electrons are closest to the
nucleus, which has a strong force or pull across them. Apply the same force to
adjacent atoms. As the ion charge increases, so does the bond. Therefore, the binding
force between +1 cation and -1 anion is not as strong as that between +3 cation and -
2 anion.

CHE509: Inorganic Chemistry – II Page 307


04-01: LATTICE ENERGY

Definition: Lattice energy can be defined as the energy required to convert one mole of
an ionic solid into gaseous ionic constituents.

Alternatively, it can be defined as the energy that must be supplied to one mole
of an ionic crystal in order to break it into gaseous ions in vacuum via an
endothermic process.Therefore; this quantity will always be positive.Some sources
define lattice energy in the opposite way. That is, the amount of energy released
when an ionic solid is formed from gaseous ionic components by an exothermic
process.According to this definition, lattice energy must always hold a negative
value.

The two main factors that influence the lattice energy of ionic compounds are
the magnitude of the charges associated with the constituent ions and the spacing
between the ions.

Charge held by Ions

Individual ions in the ion lattice are attracted to each other due to electrostatic
forces between them. The strength of the electrostatic attraction is directly
proportional to the amount of charge carried by the constituent ions. H. The greater
the charge, the stronger the attractive force and the stronger the grid. For example,
the lattice energy of calcium chloride is greater than that of potassium chloride,
despite the similarity of the crystallographic configurations of these compounds.
This is because the calcium cation (+2) has a greater positive charge than the
potassium cation (+1). As a result, the electrostatic attraction is stronger with
calcium chloride (than potassium chloride). Therefore, the lattice energy of CaCl2 is
larger than that of KCl.

Distance between the Ions

The lattice energy of ionic compounds is inversely proportional to the distance


between ions. The greater the distance between ions in the lattice, the weaker the
electrostatic force holding them and the lower the lattice energy. The smaller the
atoms, the smaller the interatomic distance in the ion lattice and the stronger the
bonding forces. Therefore, the smaller the size of the constituent ions, the higher the
lattice energy of an ionic solid.

CHE509: Inorganic Chemistry – II Page 308


SOLVED PROBLEMS 01

1] Define lattice energy and explain factors affecting on it.

Solution: Definition: Lattice energy can be defined as the energy required to convert
one mole of an ionic solid into gaseous ionic constituents.

Alternatively, it can be defined as the energy that must be supplied to one mole
of an ionic crystal in order to break it into gaseous ions in vacuum via an
endothermic process. Therefore, this quantity will always be positive. Some sources
define lattice energy in the opposite way. That is, the amount of energy released
when an ionic solid is formed from gaseous ionic components by an exothermic
process. According to this definition, lattice energy must always hold a negative
value.

The two main factors that influence the lattice energy of ionic compounds are
the magnitude of the charges associated with the constituent ions and the spacing
between the ions.

Charge held by Ions

Individual ions in the ion lattice are attracted to each other due to electrostatic
forces between them. The strength of the electrostatic attraction is directly
proportional to the amount of charge carried by the constituent ions. H. The greater
the charge, the stronger the attractive force and the stronger the grid. For example,
the lattice energy of calcium chloride is greater than that of potassium chloride,
despite the similarity of the crystallographic configurations of these compounds.
This is because the calcium cation (+2) has a greater positive charge than the
potassium cation (+1). As a result, the electrostatic attraction is stronger with
calcium chloride (than potassium chloride). Therefore, the lat tice energy of CaCl2 is
larger than that of KCl.

Distance between the Ions

The lattice energy of ionic compounds is inversely proportional to the distance


between ions. The greater the distance between ions in the lattice, the weaker the
electrostatic force holding them and the lower the lattice energy. The smaller the
atoms, the smaller the interatomic distance in the ion lattice and the stronger the

CHE509: Inorganic Chemistry – II Page 309


bonding forces. Therefore, the smaller the size of the constituent ions, the higher the
lattice energy of an ionic solid.

SHORT ANSWER QUESTIONS WITH MODEL ANSWER 01

1] Define lattice energy

Solution: Lattice energy can be defined as the energy required to convert one mole of
an ionic solid into gaseous ionic constituents.

Lattice energy is a type of potential energy that can be defined in two ways. In
one definition, lattice energy is the energy required to break up an ionic solid and
convert its atoms into gaseous ions. This definition means that the lattice energy
value is always positive, since the reaction is always endothermic. Another
definition is that lattice energy is the reverse process, the energy released when
gaseous ions combine to form an ionic solid. As indicated by the definition, this
process is always exothermic, so the value of the lattice energy is negative. Its value
is usually expressed in units of kJ/mol.

04-02: BORN-HABER CYCLE

All values used in the Born-Haber cycle are given enthalpy changes for the
processes described in the section above. By Hess' law, these values can be
added or subtracted to determine the lattice energy.

Hess's Law states that the overall change in energy of a process can be
determined by breaking the process down into steps, then adding the changes in
energy of each step. The Born-Haber Cycle is essentially Hess's Law applied to an
ionic solid.

Step 1

Determine the energy of the metal and nonmetal in their elemental forms.
(Elements in their natural state have an energy level of zero.) Subtract from this the
heat of formation of the ionic solid that would be formed from combining these
elements in the appropriate ration. This is the energy of the ionic solid, and will be
used at the end of the process to determine the lattice energy.

Step 2

CHE509: Inorganic Chemistry – II Page 310


The Born-Haber cycle requires the elements participating in the r eaction to be
in gaseous form. Add the change in enthalpy to put one of the elements into the gas
state, and do the same for the others.

Step 3

Metals exist in nature as single atoms, so there is no need to add dissociation


energy to this element. However, many nonmetals exist as polyatomic species. For
example, Cl exists as Cl2 in its elemental state. The energy required to convert Cl2
to 2 Cl atoms must be added to the value obtained in step 2.

Step 4

Both metals and non-metals must be converted to ionic forms as they exist in
ionic solids. To do this, add the ionization energy of the metal to the value from step
3. Then the electron affinities of non-metals are subtracted from the previous values.
It is subtracted because it is the energy release associat ed with the addition of
electrons.

Step 5

Here the metal and non-metal are combined to form an ionic solid. This
releases energy called lattice energy. The lattice energy value is the difference
between the step 1 value and the step 4 value.

The Born-Haber cycle can be reduced to one equation:

Heat of formation= Heat of atomization+ Dissociation energy+ (sum of


Ionization energies)+ (sum of Electron affinities)+ Lattice energy

CHE509: Inorganic Chemistry – II Page 311


e.g. formation of NaCl molecule

Bourne-Harbor cycle of sodium chloride NaCl (or any AB-type monovalent ion
solid).

Na (s) + (1/2)Cl 2 (g) → NaCl(s) ΔH f 0 = -411kJ/mol

The formation of ionic solid sodium chloride from solid sodium metal and
gaseous chlorine is not a single-step process, but a multi-process. Thermal changes
in all processes except lattice energy can be measured experimentally.

The process or step in the formation of sodium chloride is

1. Solid sodium atom sublimes to gaseous atom by absorbing heat energy


(∆Hsub).

Na (s) → Na (g),

Sublimation energy ΔHsub = + 107kJ/mol

2. Gaseous sodium atoms absorb ionization energy and release electrons to form
gaseous sodium ions.

Na(g) → Na + (g) + 1e – ,

Ionization energy ∆H IE = +502kJ/mol

3. Diatomic chlorine gas splits into two single atoms by absorbing the binding
energy, so each chlorine atom absorbs half of the binding energy of the chlorine
molecule.

Cl 2 (g) → 2Cl(g) (1/2)

Bond dissociation energy of chlorine = (1/2) ∆H diss = (1/2) 242 = +121kJ/mol

4. Chlorine atom accepts an electron to form chloride ion and releases energy
equivalent to electron affinity.

Cl(g) + 1e – → Cl – (g)

Electron affinity = ∆H EA = -355kJ/mol

5. Gaseous sodium ions and gaseous chloride ions combine to form solid sodium
chloride molecules, releasing energy equal to the lattice energy.

Na + (g) + Cl – (g) → Na + Cl – (s)

CHE509: Inorganic Chemistry – II Page 312


Lattice energy = ∆H LE = U = ?

Summing the enthalpies of all processes from step 1 to step

Net enthalpy of formation of solid crystalline sodium chloride from sodium and
chlorine under standard solid and gas conditions, respectively. This should equal
the experimentally measured enthalpy of formation of solid sodium chloride.

Enthalpy is shown as a cycle in the diagram.

= -411 - 107 - 502 - 121 + 355

= – 786 kJ/mol

SOLVED PROBLEMS 02

1] Describe the Born Haber cycle in detail

Solution: All values used in the Born-Haber cycle are given enthalpy changes
for the processes described in the section above. By Hess' law, these values can be
added or subtracted to determine the lattice energy.

Hess's Law states that the overall change in energy of a process can be
determined by breaking the process down into steps, then adding the changes in
energy of each step. The Born-Haber Cycle is essentially Hess's Law applied to an
ionic solid.

CHE509: Inorganic Chemistry – II Page 313


energy of each step. The Born-Haber Cycle is essentially Hess's Law applied
Step 1

Determine the energy of the metal and nonmetal in their elemental forms.
(Elements in their natural state have an energy level of zero.) Subtract from this the
heat of formation of the ionic solid that would be formed from combining these
elements in the appropriate ration. This is the energy of the ionic solid, and will be
used at the end of the process to determine the lattice energy.

Step 2

The Born-Haber cycle requires the elements participating in the reaction to be


in gaseous form. Add the change in enthalpy to put one of the elements into the gas
state, and do the same for the others.

Step 3

Metals exist in nature as single atoms, so there is no need to add dissociation


energy to this element. However, many nonmetals exist as polyatomi c species. For
example, Cl exists as Cl2 in its elemental state. The energy required to convert Cl2
to 2 Cl atoms must be added to the value obtained in step 2.

Step 4

Both metals and non-metals must be converted to ionic forms as they exist in
ionic solids. To do this, add the ionization energy of the metal to the value from step
3. Then the electron affinities of non-metals are subtracted from the previous values.
It is subtracted because it is the energy release associated with the addition of
electrons.

Step 5

Here the metal and non-metal are combined to form an ionic solid. This
releases energy called lattice energy. The lattice energy value is the difference
between the step 1 value and the step 4 value.

CHE509: Inorganic Chemistry – II Page 314


The Born-Haber cycle can be reduced to one equation:

Heat of formation= Heat of atomization+ Dissociation energy+ (sum of


Ionization energies)+ (sum of Electron affinities)+ Lattice energy

2] Explain the formation of NaCl molecule with the help of Born -Haber
cycle

Solution: Bourne-Harbor cycle of sodium chloride NaCl (or any AB-type


monovalent ion solid).

Na (s) + (1/2)Cl 2 (g) → NaCl(s) ΔH f 0 = -411kJ/mol

The formation of ionic solid sodium chloride from solid sodium metal and
gaseous chlorine is not a single-step process, but a multi-process. Thermal changes
in all processes except lattice energy can be measured experimentally.

The process or step in the formation of sodium chloride is

6. Solid sodium atom sublimes to gaseous atom by absorbing heat


energy (∆Hsub).

Na (s) → Na (g),

Sublimation energy ΔHsub = + 107kJ/mol

7. Gaseous sodium atoms absorb ionization energy and release


electrons to form gaseous sodium ions.

Na(g) → Na + (g) + 1e – ,

Ionization energy ∆H IE = +502kJ/mol

CHE509: Inorganic Chemistry – II Page 315


8. Diatomic chlorine gas splits into two single atoms by absorbing the
binding energy, so each chlorine atom absorbs half of the binding energy of the
chlorine molecule.

Cl 2 (g) → 2Cl(g) (1/2)

Bond dissociation energy of chlorine = (1/2) ∆H diss = (1/2) 242 = +121kJ/mol

9. Chlorine atom accepts an electron to form chloride ion and releases


energy equivalent to electron affinity.

Cl(g) + 1e – → Cl – (g)

Electron affinity = ∆H EA = -355kJ/mol

10. Gaseous sodium ions and gaseous chloride ions combine to form solid
sodium chloride molecules, releasing an energy equal to the lattice energy.

Na + (g) + Cl – (g) → Na + Cl – (s)

Lattice energy = ∆H LE = U = ?

Summing the enthalpies of all processes from step 1 to step

Net enthalpy of formation of solid crystalline sodium chloride from sodium and
chlorine under standard solid and gas conditions, respectively. This should equal the
experimentally measured enthalpy of formation of solid sodium chloride.

Enthalpy is shown as a cycle in the diagram.

= -411 - 107 - 502 - 121 + 355

= – 786 kJ/mol

CHE509: Inorganic Chemistry – II Page 316


SHORT ANSWER QUESTIONS WITH MODEL ANSWER 02

1] Describe Hess's Law in short

Solution: Hess's Law states that the overall change in energy of a process can
be determined by breaking the process down into steps, then adding the changes in
energy of each step. The Born-Haber Cycle is essentially Hess's Law applied to an
ionic solid.

04-03: FORMAL CHARGE


Formal charge is also called fake charge. This is the theoretical single-atom
charge of the ion, since the actual charge of a polyatomic molecule or ion is
distributed throughout the ion rather than a single atom. The formal charges of the
atoms of polyatomic molecules or ions are defined below.

Definition: The formal charge over an atom of a polyatomic molecule or ion is


the difference between the valence electron of that atom in the elemental state and
the number of electrons assigned to that atom in Lewis structure.

Mathematically, it can be expressed by the following formula

F.C. = [Total no. of valence e – in the free state] – [total no. of e –


assigned in
molecular structure]

F.C. = [Total no. of valence e – in the free state] – [total no. of non-bonding pair
e – (lone pair)] – 1/2 [total no. of bonding e – ]

The factor of ½ is used to the no. of bonding e – because bonding e – is shared


between two atoms.

CHE509: Inorganic Chemistry – II Page 317


No. of non- No. of
Valence e – in bonding e – in bonding pairs Formal
Atom
free state Molecular in Molecular Charge
structure structure

= 6 – 0 – 12/2
Sulphur (S) 6 0 12 =6-6
=0

= 6 – 4 – 4/2
–1
Oxygen (O) 6 4 4 = 6 -6
=0

= 6 -6 -2/2
– 2
Oxygen (O) 6 6 2 = 6 -7
= -1

= 6 – 4 – 4/2
– 3
Oxygen(O) 6 4 4 = 6 -6
=0

= 6 -6 -2/2
– 4
Oxygen(O) 6 6 2 = 6 -7
= -1

Importance of Formal charge

Meaning of Formal Charge: Formal charge as theoretical charge does not


indicate the actual charge separation within the molecule. The formal charge helps
select the lowest energy structure from the range of possible Lewis structures for a
given species. Knowledge of the lowest energy structure is useful for predicting the
main products of reactions and explains many phenomena. In general, the lowest -
energy structure is the structure with the smallest formal atomic charge an d the
widest distribution of charge.

SOLVED PROBLEMS 03

1] Define formal charge and give importance of formal charge

Solution: Formal charge is also called fake charge. This is the theoretical
single-atom charge of the ion, since the actual charge of a polyatomic molecule or

CHE509: Inorganic Chemistry – II Page 318


ion is distributed throughout the ion rather than a single atom. The formal charges of
the atoms of polyatomic molecules or ions are defined below.

Definition: The formal charge over an atom of a polyatomic molecule or ion is


the difference between the valence electron of that atom in the elemental state and
the number of electrons assigned to that atom in Lewis structure.

Mathematically, it can be expressed by the following formula

F.C. = [Total no. of valence e – in the Free State] – [total no. of e – assigned in
molecular structure]

F.C. = [Total no. of valence e – in the Free State] – [total no. of non-bonding
pair e – (lone pair)] – 1/2 [total no. of bonding e – ]

The factor of ½ is used to the no. of bonding e – because bonding e – is shared


between two atoms.

CHE509: Inorganic Chemistry – II Page 319


No. of
No. of non-
bonding
Valence e – in bonding e – in
Atom pairs in Formal Charge
free state Molecular
Molecular
structure
structure

= 6 – 0 – 12/2

Sulphur (S) 6 0 12 =6-6

=0

= 6 – 4 – 4/2
– 1
Oxygen (O) 6 4 4 = 6 -6

=0

= 6 -6 -2/2
– 2
Oxygen (O) 6 6 2 = 6 -7

= -1

= 6 – 4 – 4/2

Oxygen(O) – 3 6 4 4 = 6 -6

=0

= 6 -6 -2/2
– 4
Oxygen(O) 6 6 2 = 6 -7

= -1

Importance of Formal charge

Meaning of Formal Charge: Formal charge as theoretical charge does not


indicate the actual charge separation within the molecule. The formal charge helps
select the lowest energy structure from the range of possible Lewis struct ures for a
given species. Knowledge of the lowest energy structure is useful for predicting the
main products of reactions and explains many phenomena. In general, the lowest -
energy structure is the structure with the smallest formal atomic charge and the
widest distribution of charge.

CHE509: Inorganic Chemistry – II Page 320


SHORT ANSWER QUESTIONS WITH MODEL ANSWER 03

1] Define formal charge.

Solution: The formal charge over an atom of a polyatomic molecule or ion is


the difference between the valence electron of that atom in the elemental stat e and
the number of electrons assigned to that atom in Lewis structure.

2] What is importance of determination of formal charge?

Solution: Meaning of Formal Charge: Formal charge as theoretical charge does


not indicate the actual charge separation within the molecule. The formal charge
helps select the lowest energy structure from the range of possible Lewis structures
for a given species. Knowledge of the lowest energy structure is useful for predicting
the main products of reactions and explains many phe nomena. In general, the
lowest-energy structure is the structure with the smallest formal atomic charge and
the widest distribution of charge.

04-04: HYDROGEN BONDING


In water, each hydrogen nucleus is covalently bonded to a central oxygen atom
by a shared electron pair. In H2O, only two of his six outer-shell electrons of oxygen
are used for this purpose, four electrons he remains organized into two unbonded
pairs. The four electron pairs surrounding oxygen tend to be placed as far apart as
possible to minimize repulsion between these negative charge clouds. This typically
results in a tetrahedral geometry where the angle between electron pairs (and thus
the H-O-H bond angle) is 109.5°. However, when the two unbonded pairs stay close
to the oxygen atom, they exert a stronger repulsive force against the two covalent
bond pairs, effectively bringing the two hydrogen atoms closer together. The result is
a distorted tetrahedral configuration with an H-O-H angle of 104.5°.

CHE509: Inorganic Chemistry – II Page 321


The H 2 O molecule is electrically neutral, but the positive and negative charges
are not evenly distributed. This is illustrated by the color gradation in the schematic
here. The electronic (negative) charge is concentrated at the oxygen end of the
molecule, due in part to the non-bonded electrons (filled blue circles) and the high
nuclear charge of oxygen, exerting a stronger attraction force on the electrons. This
charge transfer forms an electric dipole represented by the bottom arrow. This dipole
can be thought of as an electrical "image" of the water molecule.

It is not surprising that the negative end of one water molecule tends to align
closer to the positive end of another molecule that happens to be nearby because of
the attraction of opposite charges. The strength of attraction bet ween this dipole is
less than that of normal chemical bonds and is therefore completely overwhelmed by
normal thermal motion in the gas phase. However, when HO molecules are pushed
together in a liquid, these attractive forces have a very pronounced effect . And at
temperatures low enough to turn off the perturbative effects of thermal motion, water
freezes into ice, in which hydrogen bonds form a tight and stable network.

Note that the hydrogen bonds (represented by the green dashed lines) are
slightly longer than the covalent OH bonds. It is also much weaker at about 23 kJ
mol -1 compared to the covalent OH bond strength of 492 kJ mol -1 . 41 anomalies in
the water - some of them are very arcane. Water has long been known to have many
physical properties that distinguish it from other small molecules of comparable
mass. Chemists have called these "unusual" properties of water, but they are by no
means mystical, and the full extent of how the oxygen atom's size and nuclear charge
conspire to distort the electronic charge cloud of other atoms. is a predictable result.
When chemically mixed, the elements bind to oxygen.

Effect of hydrogen bonding

Boiling Point

CHE509: Inorganic Chemistry – II Page 322


The most obvious feature of water is the very high boiling point of such light
molecules. Liquid methane CH 4 (molecular weight 16) boils at -161 °C.
extrapolating the boiling points of various Group 16 hydrides to HO, this substance
should be a gas under normal conditions.

Surface Tension

Water also has a high surface tension compared to most other liquids. Have you
ever seen an insect walking on the surface of a pond? Water striders take advantage
of the fact that the water surface acts like an elastic membrane that does not deform
even when a small weight is placed on it. This is all due to the surface tension of the
water. Most molecules in a liquid experience attractive force on their neighbors in
all directions, but this average to zero, so there is no net force on the molecules. The
situation is quite different for molecules on surfaces. The force can only be applied
laterally and downwards. This creates a stretched membrane effect.
Density
The most energetically favorable configuration of the H 2 O molecule is that in
which each molecule hydrogen bonds to four neighboring molecules. Due to the
aforementioned thermal motion, this ideal is never reached in liquids, but when
water freezes into ice, the molecules are stored in ice crystals in exactly this type of
arrangement. This arrangement requires the molecules to be slightly further apart
than they would otherwise be. As a result, ice, with maximum hydrogen bonding, has
a more open structure and is therefore less dense than water.
A hydrogen bond is formed when the electron cloud of a hyd rogen atom bound
to one of the more electronegative atoms is distorted by that atom, leaving the
hydrogen with a partial positive charge. Due to the very small size of the hydrogen
atom, the density of this partial charge is large enough to interact with l one pairs of
neighboring electronegative atoms. Hydrogen bonding is commonly described as a
form of attraction between dipoles, but it is now clear that it also involves some
degree of electron sharing (between external, non-bonding electrons and hydrogen),
so these bonds have covalent properties. .
SOLVED PROBLEMS 04
1] Describe hydrogen bonding with suitable example.
Solution: In water, each hydrogen nucleus is covalently bonded to a central
oxygen atom by a shared electron pair. In H2O, only two of his six outer-shell

CHE509: Inorganic Chemistry – II Page 323


electrons of oxygen are used for this purpose, four electrons he remains organized
into two unbounded pairs. The four electron pairs surrounding oxygen tend to be
placed as far apart as possible to minimize repulsion between these negative charge
clouds. This typically results in a tetrahedral geometry where the angle between
electron pairs (and thus the H-O-H bond angle) is 109.5°. However, when the two
unbonded pairs stay close to the oxygen atom, they exert a stronger repulsive force
against the two covalent bond pairs, effectively bringing the two hydrogen atoms
closer together. The result is a distorted tetrahedral configuration with an H -O-H
angle of 104.5°.

The H 2 O molecule is electrically neutral, but the positive and negative charg es
are not evenly distributed. This is illustrated by the color gradation in the schematic
here. The electronic (negative) charge is concentrated at the oxygen end of the
molecule, due in part to the non-bonded electrons (filled blue circles) and the high
nuclear charge of oxygen, exerting a stronger attraction force on the electrons. This
charge transfer forms an electric dipole represented by the bottom arrow. This dipole
can be thought of as an electrical "image" of the water molecule.
It is not surprising that the negative end of one water molecule tends to align
closer to the positive end of another molecule that happens to be nearby because of
the attraction of opposite charges. The strength of attraction between this dipole is
less than that of normal chemical bonds and is therefore completely overwhelmed by
normal thermal motion in the gas phase. However, when HO part of molecules are
pushed together in a liquid, these attractive forces have a very pronounced effect.
And at temperatures low enough to turn off the perturbative effects of thermal
motion, water freezes into ice, in which hydrogen bonds form a tight and stable
network.

CHE509: Inorganic Chemistry – II Page 324


Note that the hydrogen bonds (represented by the green dashed lines) are
slightly longer than the covalent OH bonds. It is also much weaker at about 23 kJ
mol -1 compared to the covalent OH bond strength of 492 kJ mol -1 . 41 anomalies in
the water - some of them are very arcane. Water has long been known to have many
physical properties that distinguish it from other small mo lecules of comparable
mass. Chemists have called these "unusual" properties of water, but they are by no
means mystical, and the full extent of how the oxygen atom's size and nuclear charge
conspire to distort the electronic charge cloud of other atoms. is a predictable result.
When chemically mixed, the elements bind to oxygen.
2] Explain the factors affected by hydrogen bonging.
Solution: following factors are affected by h-bonding
Boiling Point
The most obvious feature of water is the very high boiling point of such light
molecules. Liquid methane CH 4 (molecular weight 16) boils at -161 °C.
Extrapolating the boiling points of various Group 16 hydrides to HO, this substance
should be a gas under normal conditions.
Surface Tension
Water also has a high surface tension compared to most other liquids. Have you
ever seen an insect walking on the surface of a pond? Water striders take advantage
of the fact that the water surface acts like an elastic membrane that does not deform
even when a small weight is placed on it. This is all due to the surface tension of the
water. Most molecules in a liquid experience attractive forces on their neighbors in
all directions, but these average to zero, so there is no net force on the molecules.
The situation is quite different for molecules on surfaces. The force can only be
applied laterally and downwards. This creates a stretched membrane effect.
Density
The most energetically favorable configuration of the H2O molecule is that in
which each molecule hydrogen bonds to four neighboring molecules. Due to the
aforementioned thermal motion, this ideal is never reached in liquids, but when

CHE509: Inorganic Chemistry – II Page 325


water freezes into ice, the molecules are stored in ice crystals in exactly this type of
arrangement. This arrangement requires the molecul es to be slightly further apart
than they would otherwise be. As a result, ice, with maximum hydrogen bonding, has
a more open structure and is therefore less dense than water.
A hydrogen bond is formed when the electron cloud of a hydrogen atom bound
to one of the more electronegative atoms is distorted by that atom, leaving the
hydrogen with a partial positive charge. Due to the very small size of the hydrogen
atom, the density of this partial charge is large enough to interact with lone pairs of
neighboring electronegative atoms. Hydrogen bonding is commonly described as a
form of attraction between dipoles, but it is now clear that it also involves some
degree of electron sharing (between external, non-bonding electrons and hydrogen),
so these bonds have covalent properties.
SHORT ANSWER QUESTIONS WITH MODEL ANSWER 04
1] Which are the factors that contribute to lattice energy of an ionic compound?
Solution: The strength of the ionic bonds in an ionic compound is measured by
lattice energy. It reveals information about ionic materials’ volatility, solubility, and
hardness, among other things. The distance between the ions has an inverse
relationship with the lattice energy of an ionic compound. The weaker the
electrostatic forces binding ions together in a lattice, the lower the lattice energy.
The ionic lattice of smaller atoms has smaller interatomic distances and stronger
binding forces. As a result, the smaller the constituent ions are, the higher the ionic
solid’s lattice energy.

CHECK POINT 01-04


1] Many ionic compounds have some covalent ability due to
a) electron polarization
b) ion polarization
c) charge polarization
d) proton polarization
Solution: b
2] The value of lattice energy is affected by
a) size and charge of ions
b) size of ions only
c) charge of ions only

CHE509: Inorganic Chemistry – II Page 326


d) mass of ions
Solution: a
3] Lattice energy is inversely proportional to sum of radii of
a) cation only
b) anion only
c) anion and cation
d) ions
Solution: c
4] H bonding affects on boiling point of liquids
a) True
b) Fouls
Solution: a

SUMMARY
The lattice energy of ionic compounds is highly dependent on the charge of the
ions that make up the solid. The ions that make up a soli d must either attract or repel
each other according to Coulomb's law. In a more subtle way, the relative and
absolute sizes of ions affect. London dispersion forces also exist between ions and
contribute to the lattice energy via polarization effects. Ion -dipole and dipole-dipole
interactions can also occur in ionic compounds composed of molecular cations
and/or anions if one of the molecules exhibits a molecular dipole moment. The
theoretical treatment described below focuses on atomic cation and anion
compounds and ignores the contribution to lattice internal energy from thermalized
lattice vibrations.
KEY WORDS
Lattice energy, Born-haber cycle, Hydrogen bonding

REFERENCES

MOOCS
_______
YOUTUBE VIDEOS
1) https://www.youtube.com/watch?v=ch9HorGagHE
2) https://www.youtube.com/watch?v=xYprAuyxCL4
3) https://www.youtube.com/watch?v=DI-bwx8r3r8

CHE509: Inorganic Chemistry – II Page 327


4) https://www.youtube.com/watch?v=k8tYXDKb2yE

WIKIPEDIA
1) https://en.wikipedia.org/wiki/Lattice_energy
2) https://en.wikipedia.org/wiki/Lattice_energy#Lattice_energy_and
_lattice_enthalpy
3) https://en.wikipedia.org/wiki/Born%E2%80%93Haber_cycle
4) https://en.wikipedia.org/wiki/Born%E2%80%93Haber_cycle#Form
ation_of_NaBr
OER
1) https://chem.libretexts.org/Bookshelves/Inorganic_Chemistry/Sup
plemental_Modules_and_Websites_(Inorganic_Chemistry)/Crystal
_Lattices/Thermodynamics_of_Lattices/Lattice_Energy
2) http://chemed.chem.purdue.edu/genchem/topicreview/bp/ch7/latti
ce.html
REFERENCE BOOKS
1) Descriptive Inorganic, Coordination and Solid State Chemistry Hardcover
– , 2002 by Glen Rodgers
2) Solid State Chemistry: Synthesis, Structure, and Properties of Selected
Oxides and Sulfides , 1993 by Aaron Wold , Kirby Dwight

CHE509: Inorganic Chemistry – II Page 328


S CHOOL OF S CIENCES
(F ORMERLY S CHOOL OF A RCHITECTURE , S CIENCE & T ECHNOLOGY )
YASHWANTRAO C HAVAN MAHARASHTRA O PEN
U NIVERSITY

F EEDBACK S HEET FOR THE S TUDENT

Dear Student,
You have gone through this book, it is time for you to do some thinking for us.
Please answer the following questions sincerely. Your response will help us to
analyse our performance and make the future editions of this book more useful. Your
response will be completely confidential and will in no way affect your examination
results. Your suggestions will receive prompt attention from us.

Please submit your feedback online at this QR Code or at following link


https://forms.gle/rpDib9sy5b8JEisQ9

or email at: director.ast@ycmou.ac.in


or send this filled “Feedback Sheet” by post to above address.

(Please tick the appropriate box)

Pl. write your Program CodeCourse Code & Name

Style
01. Do you feel that this book enables you to learn the subject independently without
any help from others?

Yes No Not Sure


02. Do you feel the following sections in this book serve their purpose? Write the
appropriate code in the boxes.
Code 1 for “Serve the purpose fully”
Code 2 for “Serve the purpose partially”
Code 3 for “Do not serve any purpose”
Code 4 for “Purpose is not clear”
Warming up Check Point Answer to Check Points

To Begin With Summary References

CHE509: Inorganic Chemistry – II Page 329


Objectives Key Words

03. Do you feel the following sections or features, if included, will enhance self -
learning and reduce help from others?

Yes No Not Sure

Index
Glossary
List of “Important Terms Introduced”
Two Colour Printing

Content
04. How will you rate your understanding of the contents of this Book?

Very Bad Bad Average Good Excellent

05. How will you rate the language used in this Book?

Very Simple Simple AverageComplicated Extremely Complicated

06. Whether the Syllabus and content of book complement to each other?

Yes No Not Sure

07. Which Topics you find most easy to understand in this book?
Sr.No. Topic Name Page No.

CHE509: Inorganic Chemistry – II Page 330


08. Which Topics you find most difficult to understand in this book?
Sr.No. Topic Name Page No.

09. List the difficult topics you encountered in this Book. Also try to suggest how
they can be improved.
Use the following codes:
Code 1 for “Simplify Text”
Code 2 for “Add Illustrative Figures”
Code 3 for “Provide Audio-Vision (Audio Cassettes with companion Book)”
Code 4 for “Special emphasis on this topic in counseling”

Sr.No. Topic Name Page No. Required Action Code

10. List the errors which you might have encountered in this book.

Sr.No. Page Line Errors Possible Corrections


No. No.

CHE509: Inorganic Chemistry – II Page 331


11. Based on your experience, how would you place the components of distance
learning for their effectiveness?
Use the following code.
Code 1 for “Most Effective” Code 3 for “Average” Code 5 for “Least Effective”

Code 2 for “Effective” Code 4 for “less Effective”

Printed Book Counseling Lab Journal

Audio Lectures Home Assignment YouTube Videos

Video Lectures Lab-Experiment Online Counseling

12. Give your overall rating to this book?

1. 2. 3. 4. 5.

13. Any additional suggestions:

Thank you for your co-operation!

CHE509: Inorganic Chemistry – II Page 332

You might also like